Logo Passei Direto
Buscar
Material
páginas com resultados encontrados.
páginas com resultados encontrados.
left-side-bubbles-backgroundright-side-bubbles-background

Crie sua conta grátis para liberar esse material. 🤩

Já tem uma conta?

Ao continuar, você aceita os Termos de Uso e Política de Privacidade

left-side-bubbles-backgroundright-side-bubbles-background

Crie sua conta grátis para liberar esse material. 🤩

Já tem uma conta?

Ao continuar, você aceita os Termos de Uso e Política de Privacidade

left-side-bubbles-backgroundright-side-bubbles-background

Crie sua conta grátis para liberar esse material. 🤩

Já tem uma conta?

Ao continuar, você aceita os Termos de Uso e Política de Privacidade

left-side-bubbles-backgroundright-side-bubbles-background

Crie sua conta grátis para liberar esse material. 🤩

Já tem uma conta?

Ao continuar, você aceita os Termos de Uso e Política de Privacidade

left-side-bubbles-backgroundright-side-bubbles-background

Crie sua conta grátis para liberar esse material. 🤩

Já tem uma conta?

Ao continuar, você aceita os Termos de Uso e Política de Privacidade

left-side-bubbles-backgroundright-side-bubbles-background

Crie sua conta grátis para liberar esse material. 🤩

Já tem uma conta?

Ao continuar, você aceita os Termos de Uso e Política de Privacidade

left-side-bubbles-backgroundright-side-bubbles-background

Crie sua conta grátis para liberar esse material. 🤩

Já tem uma conta?

Ao continuar, você aceita os Termos de Uso e Política de Privacidade

left-side-bubbles-backgroundright-side-bubbles-background

Crie sua conta grátis para liberar esse material. 🤩

Já tem uma conta?

Ao continuar, você aceita os Termos de Uso e Política de Privacidade

left-side-bubbles-backgroundright-side-bubbles-background

Crie sua conta grátis para liberar esse material. 🤩

Já tem uma conta?

Ao continuar, você aceita os Termos de Uso e Política de Privacidade

left-side-bubbles-backgroundright-side-bubbles-background

Crie sua conta grátis para liberar esse material. 🤩

Já tem uma conta?

Ao continuar, você aceita os Termos de Uso e Política de Privacidade

Prévia do material em texto

1 
 
 
 
 
APOSTILA 01 DE FÍSICO-QUÍMICA – PROF. PEDRO MADEIRA (2022) 
 
 
 
 
 
 
 
 
 
 
 
 
 
 
APOSTILA 01 – FÍSICO-QUÍMICA 
SUMÁRIO 
 
 FOLHA DE DADOS 02 
01. O ESTUDO DOS GASES 03 
 Exercícios 19 
 Gabarito 51 
02. TERMODINÂMICA QUÍMICA 57 
 Exercícios 89 
 Gabarito 122 
 
 
 
 
 
 
 
 
2 
 
 
 
APOSTILA 01 DE FÍSICO-QUÍMICA – PROF. PEDRO MADEIRA (2022) 
 
 
 
 
CONSTANTES 
Constante de Avogadro = 6,02 x 1023 mol–1 
Constante de Faraday (F) = 9,65 x 104 C mol–1 = 9,65 x 104 A s mol–1 = 9,65 x 104 J V mol–1 
Volume molar de gás ideal = 22,4 L (CNTP) 
Carga elementar = 1,602 x 10–19 C 
Constante dos gases (R) = 8,21 x 10–2 atmLK–1mol–1 =8,31 J K–1mol–1 = 62,4 mmHgLK–1mol–1 = 1,98 calK–1mol–1 
Constante gravitacional (g) = 9,81 m.s–2 
Constante de Rydberg (R∞hc) = 2,18 x 10–18 J = 13,6 eV 
Constante de Planck(h) = 6,63 x 10–34 J.s 
 
DEFINIÇÕES 
Pressão de 1 atm = 760 mmHg = 101325 N.m–2 = 760 Torr 
1 J = 1 N.m = 1 kg.m2.s – 2 ; 1 pm = 1 x 10–12 m; 1 eV = 1,602 x 10–19 J 
Condições normais de temperatura e pressão (CNTP): 0°C e 760 mmHg. 
Condições ambientes: 25°C e 1 atm. 
Condições-padrão: 25°C, 1 atm, concentração das soluções: 1 mol L−1 (rigorosamente: atividade unitária das 
espécies), sólido com estrutura cristalina mais estável nas condições de pressão e temperatura em questão. 
(s) ou (c) = sólido cristalino; (l) ou (l) = líquido; (g) = gás; (aq) = aquoso; (graf) = grafite; (CM) = circuito metálico; 
(conc) = concentrado; (ua) = unidades arbitrárias; [A] = concentração da espécie química A em mol L−1. 
 
MASSAS MOLARES 
 
 
 
 
 
Elemento 
Químico 
Número 
Atômico 
Massa 
Molar (g.mol –1) 
Elemento 
Químico 
Número 
Atômico 
Massa 
Molar (g.mol –1) 
H 1 1,01 Mn 25 54,94 
He 2 4,00 Fe 26 55,85 
Li 3 6,94 Ni 28 58,71 
Be 4 9,01 Cu 29 63,54 
B 5 10,81 Zn 30 65,37 
C 6 12,01 Ge 32 72,63 
N 7 14,01 As 33 74,92 
O 8 16,00 Se 34 78,96 
F 9 19,00 Br 35 79,90 
Ne 10 20,18 Ag 47 107,87 
Na 11 22,99 Sn 50 118,69 
Mg 12 24,31 Te 52 127,60 
Al 13 26,98 I 53 126,90 
Si 14 28,09 Xe 54 131,29 
P 15 30,97 Ba 56 137,33 
S 16 32,06 Pt 78 195,08 
Cl 17 35,45 Au 79 196,97 
Ar 18 39,95 Hg 80 200,59 
K 19 39,10 Pb 82 207,19 
Ca 20 40,08 Bi 83 208,98 
Ti 22 47,87 U 92 238,00 
Cr 24 52,00 
3 
 
 
 
APOSTILA 01 DE FÍSICO-QUÍMICA – PROF. PEDRO MADEIRA (2022) 
 
 
 
 
 
 
 
CAPÍTULO 01 – O ESTUDO DOS GASES 
 
CONTEÚDO 
 
TÓPICO 01: GASES IDEAIS 
TÓPICO 02: TEORIA CINÉTICA DOS GASES IDEAIS 
TÓPICO 03: GASES REAIS 
 
TÓPICO 01 – GASES IDEAIS 
O gás enche qualquer recipiente que o contenha. 
 
A) Equações de Estado 
 
Forma Geral ! = #(%, ', () 
Gás Perfeito ! =
(*'
% 
van der Waals ! =
(*'
% − (, − -.
(
%/
!
 
Berthelot ! =
*'
%" − ,
−
-
'%"! 
Dieterici ! =
*'0#(% &'(!⁄ )
%" − , 
Virial ! =
*'
%"
12 +
4
%"
+
5
%"!
+⋯7
 
OBSERVAÇÕES 
 
 
 
 
 
 
 
 
 
 
 
 
 
 
 
 
 
 
PROF. PEDRO MADEIRA 
B) Variáveis de Estado 
 
B.1) Pressão 
SÍMBOLO VALOR 
1 Pa 1 N.m–2, 1 kg.m–1.s–2 
1 bar 105 Pa 
1 atm 101325 Pa 
1 Torr (101325/760) Pa 
1 mmHg (101325/760) Pa 
1 psi 6,894757... kPa 
 
Experiência de Torricelli (1643) 
 
OBSERVAÇÕES 
 
 
 
 
 
 
 
 
 
 
 
 
 
 
4 
 
 
 
APOSTILA 01 DE FÍSICO-QUÍMICA – PROF. PEDRO MADEIRA (2022) 
 
Manômetros de mercúrio 
 
 
 
OBSERVAÇÕES 
 
 
 
 
 
 
 
 
 
 
 
 
 
 
 
 
 
 
 
 
 
 
 
 
 
 
 
 
 
OBSERVAÇÕES 
 
 
 
 
 
 
 
 
 
 
 
 
 
 
 
 
 
 
 
 
 
 
 
 
 
 
 
 
 
 
 
 
 
 
 
 
 
 
 
 
 
 
 
 
 
 
 
 
 
 
 
 
 
 
 
 
 
 
 
 
Sistema de coleta de gases sobre líquidos 
5 
 
 
 
APOSTILA 01 DE FÍSICO-QUÍMICA – PROF. PEDRO MADEIRA (2022) 
 
B.2) Temperatura: Lei Zero da Termodinâmica 
 
 
 
 
 
 
 
 
 
 
 
 
 
Atenção: Escala Celsius x Escala Absoluta (Kelvin) 
q (oC) + 273,15 = T(K) 
 
Medição de temperaturas 
 
1) Termoscópio de Galileu (1593): termômetro de 
água/gás 
 
2) Termômetro de Galileu (1666): inventado por um 
grupo de pesquisadores que incluía Torricelli 
 
 
3) Fahrenheit (1709 e 1714): termômetros de álcool e 
Hg 
4) Medida da resistência elétrica de um material 
conhecido. Ex: sensor de platina metálica (acima) e 
sensor de RuO2 (abaixo). 
 
 
5) Medição da pressão de um gás em condições ideias 
 
6) Medição da pressão de vapor em equilíbrio com o 
liquido. Ex: He(l) em equilíbrio com seu vapor. 
 
 
 
 
 
6 
 
 
 
APOSTILA 01 DE FÍSICO-QUÍMICA – PROF. PEDRO MADEIRA (2022) 
 
7) Termômetro de infravermelho 
 
 
 
8) Termopar 
 
 
9) Carnot 
 
 
10) Estatística 
 
 
Crédito da imagem: LMU/MPQ Munich 
 
RESOLVA AGORA 01 (ITA 2015) 
Contribuiram de forma direta para o desenvolvimento do 
conceito de pressão atmosférica 
A ( ) Friedrich August Kekulé e John Dalton. 
B ( ) Michael Faraday e Fritz Haber. 
C ( ) Galileu Galilei e Evangelista Torricelli. 
D ( ) Jöns Jacob Berzelius e Eduard Büchner. 
E ( ) Robert Bunsen e Henry Louis Le Chatelier. 
 
B.3) Volume 
SÍMBOLO VALOR 
1 m3 1000 L 
1 dm3
 1 L 
1 cm3 1 mL 
PROF. PEDRO MADEIRA 
C) Leis Empíricas dos Gases Ideais 
Para que um gás se comporte como ideal, a pressão do 
gás precisa estar próxima de zero (gás rarefeito). 
O raciocínio atual para o entendimento das leis 
empíricas e confecção dos gráficos, deve se basear na 
equação PV = nRT. 
 
Boyle (1661) 
 
Charles – Gay-Lussac 
 
OBSERVAÇÕES 
 
 
 
 
 
 
 
 
 
 
 
 
 
 
 
 
 
 
 
 
 
 
 
 
 
 
 
 
 
 
Pr
es
sã
o 
Volume 
Pr
es
sã
o 
1/Volume 
Aumento da 
temperatura 
Aumento da 
temperatura 
Diminuição 
da pressão 
Vo
lu
m
e 
Temperatura (oC) -273 
0 
Temperatura (K) 
Diminuição 
do volume 
Pr
es
sã
o 
0 
0 
7 
 
 
 
APOSTILA 01 DE FÍSICO-QUÍMICA – PROF. PEDRO MADEIRA (2022) 
 
 
 
 
 
 
 
 
 
 
 
 
 
 
 
 
 
 
 
 
 
 
 
 
 
 
 
 
 
 
 
 
 
 
 
 
 
 
 
 
 
 
 
 
Experimento de Charles 
 
V = constante x (q + 273,15oC) 
V = constante x T; p = constante x T 
“Para uma massa fixa de gás sob pressão constante, o 
aumento relativo de volume por grau de aumento de 
temperatura é o mesmo para todos os gases” 
OBS: coeficiente de expansão térmica a 0oC = ao.
 
 
Escala gasosa de temperatura: è Se t é 
em oC, T = 273,15oC + t 
 
RESOLVA AGORA 02 
A lei de Charles também se escreve como V = V0 (1 + 
aq), onde q é a temperatura Celsius, a é uma constante 
e V0 o volume da amostra do gás a 0oC. Para o 
nitrogênio a 0oC, obtiveram-se os seguintes valores de 
a: 
 
p/Torr 749,7 599,6 333,1 98,6 
103 a/(oC)–1 3,6717 3,6697 3,6665 3,6643 
 
Escolha a melhor medição para estimar o valor do zero 
absoluto de temperatura na escala Celsius. Efetue este 
cálculo a partir deste ponto. 
 
 
 
 
 
 
 
 
 
 
 
 
 
 
 
 
 
 
 
 
 
 
 
o
o p
1 Vα =
V t
¶æ ö
ç ÷¶è ø
( )o o o o o
op
V 1V = V + t V = V 1+ α t V = V α + t
t α
æ ö¶æ ö Þ Þ ç ÷ç ÷¶è ø è ø
o
1T = + t
α
æ ö
ç ÷
è ø
8 
 
 
 
APOSTILA 01 DE FÍSICO-QUÍMICA – PROF. PEDRO MADEIRA (2022) 
 
RESOLVA AGORA 03 (IME 2010) 
As alternativas abaixo representam processos 
hipotéticos envolvendo 2 mols de um gás ideal, contidos 
em um conjunto cilindro-pistão. Assinale a alternativa 
que apresenta mais de três estados (V, T) nos quais a 
pressão é máxima: 
 
(A) 
 
 
(B) 
 
 (C) 
 
(D) 
 
 
 
 
 
 
(E) 
 
PROF. PEDRO MADEIRA 
D) Equação de Estado do gás ideal 
(Clapeyron): 
 
!!"!
#!$!
= !"""
#"$"
= & 
 
R = constante universal dos gases ideais. 
 
R 
8,31447 J.K–1.mol–1 
8,20574 x 10 – 2 L.atm.K–1.mol–1 
62,364 L.Torr.K–1.mol–1 
1,98721 cal.K–1.mol–1 
 
OBSERVAÇÕES 
 
 
 
 
 
 
 
 
 
 
 
 
 
 
 
 
 
 
 
 
 
 
 
 
 
 
 
 
 
 
 
 
 V(Litros) 
 
 
 15 
 
 12,5 
 
 10 
 300 390 480 T(K) 
 V(Litros) 
 
 
 12 
 
 
 
 9 
 300 330 450 480 T(K) 
 V(Litros) 
 
 
 15 
 
 
 
 10 
 320 480 T(K) 
 V(Litros) 
 
 
 12 
 
 
 
 9 
 300 330 450 480 T(K) 
 V(Litros)15 
 
 
 
 10 
300 450 T(K) 
9 
 
 
 
APOSTILA 01 DE FÍSICO-QUÍMICA – PROF. PEDRO MADEIRA (2022) 
 
RESOLVA AGORA 04 (IME 2008) 
Uma amostra de 0,512 g de uma liga metálica Al-Zn 
reage com HCl, recolhendo-se o gás formado. Após a 
total dissolução da amostra, o gás recolhido é seco, 
resfriado e submetido a um processo de compressão 
representado pela reta AB no diagrama P-V. Sabendo 
que a temperatura máxima ao longo do processo de 
compressão é 298K, determine o teor de alumínio nesta 
amostra. Considere que o gás se comporta idealmente. 
 
 
 
 
 
 
 
 
 
 
 
 
 
 
 
 
 
 
 
 
 
 
 
 
 
 
 
 
 
 
 
 
 
 
 
 
PROF. PEDRO MADEIRA 
E) Volumes molares 
Condições Vm (L/mol) 
CNTP (1atm; 273K) 22,4 
CNTP (1bar; 273K) 22,71 
CNATP (1atm; 298K) 24,4 
(1atm; 300K) 24,6 
PROF. PEDRO MADEIRA 
F) Densidades Gasosas 
F.1) Absoluta 
Observe a dedução: 
 
!% = (*' ⇒
!
*' =
(
%⟹
!
*' =
:
;%⟹
!;
*' =
:
% 
 
< =
=;
*' 
 
F.2) Relativa 
Observe a dedução: 
 
<+
<,
=
=+;+
*'+
=,;,
*',
⇒
<+
<,
=
=+;+',
=,;,'+
 
 
<+
<,
=
;+
;,
 
PROF. PEDRO MADEIRA 
G) Misturas Gasosas 
 
G.1) Fração Molar 
 
>+ =
(+
(&-&
; 		>, =
(,
(&-&
; 		A>. = 2
.
 
 
G.2) Fração Volumétrica 
Princípio de Avogadro: nas mesmas condições de 
pressão e temperatura, o volume dos gases é 
diretamente proporcional ao número de moléculas (mol). 
Assim, a fração molar de um gás em uma mistura é igual 
à sua fração volumétrica. 
Ex: Ar atmosférico seco: 21% em volume de O2 = fração 
molar de gás oxigênio na mistura. 
 
G.3) Massa Molar aparente 
Se 1 mol da mistura gasosa tiver sua massa medida, por 
exemplo, através do seu volume molar, a massa 
acusada será a média ponderada das massas molares 
das moléculas da mistura. Observe: 
 
;%/ =A>.;.
.
 
 
Ex: Ar atmosférico seco: 
Map = 0,21 x 32 + 0,79 x 28 = 28,8 g/mol
 
Prof. Pedro Madeira 
 
 
 
 
P(atm) 
 0,25 0,40 V (L) 
 0,90 
 
 
 
 0,60 
B 
A 
10 
 
 
 
APOSTILA 01 DE FÍSICO-QUÍMICA – PROF. PEDRO MADEIRA (2022) 
 
G.4) Fração em massa 
 
A fração em massa de cada gás deve ser calculada 
através da massa molar aparente. Observe: 
 
#+ =
:+
:010%2
=
>+;+
;%/
 
 
Ex: fração em massa do O2 no ar seco: 
 
#-" =
B, C2 ∙ EC
CF, F ≈ B, CE 
 
G.5) Pressões Parciais – Lei de Dalton 
Se um determinado gás da mistura ocupasse sozinho o 
volume total da mistura na mesma temperatura, a 
pressão deste gás seria menor do que a pressão total. 
Denomina-se de pressão parcial tal pressão. 
 
Como a pressão é diretamente proporcional ao número 
de moléculas, a Lei de Dalton estipula que a pressão 
parcial de um gás é dada pelo produto da pressão total 
pela sua fração molar: 
 
=+ = =&-& ∙ >+; 			A=. = =&-&
.
 
 
G.6) Volumes Parciais – Lei de Amagat 
 
De forma análoga, tem-se a lei dos volumes parciais: 
 
%+ = %&-& ∙ >+; 			A%. = %&-&
.
 
 
G.7) Equação de estado para misturas. 
 
Observe a dedução: 
 
=&-&%&-& = (&-&*'⟹ =&-& ∙ >+ ∙ %&-& = >+(&-&*' 
 
Assim, 
 
=+%&-& = (+*' ou =&-&%+ = (+*' 
 
Nunca utilize as duas grandezas parciais em misturas: 
PAVA = nART ERRADO 
 
RESOLVA AGORA 05 (ITA 2003) 
Determine a massa específica do ar úmido, a 25oC e 
pressão de 1 atm, quando a umidade relativa do ar for 
igual a 60%. Nessa temperatura, a pressão de vapor 
saturante da água é igual a 23,8 mmHg. Assuma que o 
ar seco é constituído por N2(g) e O2(g) e que as 
concentrações dessas espécies no ar seco são iguais a 
79 e 21% (v/v), respectivamente. 
 
 
 
 
 
 
 
 
 
 
 
 
 
 
 
 
 
 
 
 
G.8) Balanceamento de combustões com ar. 
 
Ex1: CH4 (completa) com ar estequiométrico (1 O2 : 4 N2) 
 
 
 
 
Ex2: TNT (completa) com ar estequiométrico (21 O2: 79 
N2) 
 
 
 
RESOLVA AGORA 06 (IME 2010 – Q32) 
Em um recipiente fechado queima-se propano com 80% 
da quantidade estequiométrica de ar. Admitindo que não 
haja hidrocarbonetos após a combustão, que todos os 
produtos da reação estejam na fase gasosa e que a 
composição volumétrica do ar seja de uma parte de O2 
para quatro partes de N2, calcule a porcentagem molar 
de CO2 no recipiente após a combustão (considere 
comportamento ideal para os gases). 
(A) 4,35% (B) 4,76% (C) 5,26% 
(D) 8,70% (E) 14,28% 
 
 
 
 
 
 
 
RESOLVA AGORA 07 (ITA 2020 – Q68) 
Considerando que o ar é composto aproximadamente 
de 21% de O2 e 79% de N2 em volume, tem-se que a 
razão molar ar/combustível da combustão completa de 
um determinado alcano é igual a 59,5. A partir desse 
dado, assinale a alternativa que corresponde à soma 
dos coeficientes estequiométricos de todas as 
substâncias presentes nessa reação. 
A ( ) 30,5 B ( ) 55,5 C ( ) 82,0 
D ( ) 112,0 E ( ) 124,5 
 
 
 
11 
 
 
 
APOSTILA 01 DE FÍSICO-QUÍMICA – PROF. PEDRO MADEIRA (2022) 
 
RESOLVA AGORA 08 – BASEADA NO IME 2022 
Forneça o balanceamento da combustão de C8H18 com 
ar (21% O2 e 79% N2) nas seguintes condições: 
a) considerando 20% de combustão incompleta dando 
CO e 80% de combustão completa. 
b) um motor de 5,7 L contendo 0,3g de combustível e ar 
a 100ºC e 1 atm, considerando 20% de combustão 
incompleta dando CO e 80% de combustão completa. 
Calcule a porcentagem molar final de cada componente. 
 
 
 
 
 
 
 
 
 
 
 
 
 
 
 
 
 
 
 
 
 
 
 
 
 
 
 
 
 
 
 
 
 
 
 
 
 
 
 
 
 
 
 
 
 
TÓPICO 02 – 
TEORIA CINÉTICA DOS GASES IDEAIS 
 
Postulados: 
• O gás ideal é uma coleção de partículas de 
massa “m” e volume desprezível que se 
deslocam caoticamente e por todo o volume do 
recipiente que o contém. 
• Não existem interações intermoleculares. 
• A única forma de interação entre as partículas é 
através das colisões perfeitamente elásticas 
entre elas e entre elas e a parede do recipiente. 
 
A) Relação entre a velocidade quadrática 
média e a temperatura. 
' = ()*+, 
 
OBSERVAÇÕES 
 
 
 
 
 
 
 
 
 
 
 
 
 
 
 
 
 
 
 
 
 
 
 
 
 
 
 
 
 
 
 
 
 
 
 
 
 
PROF. PEDRO MADEIRA 
12 
 
 
 
APOSTILA 01 DE FÍSICO-QUÍMICA – PROF. PEDRO MADEIRA (2022) 
 
B) Relação entre a energia cinética média e 
a temperatura. 
!! =
#
$%& 
 
PROF. PEDRO MADEIRA 
C) Distribuição das velocidades gasosas 
(Maxwell-Boltzmann) 
A distribuição das energias cinéticas de uma amostra 
gasosa segue a distribuição de Maxwell, ilustrada pela 
equação a seguir: 
 
#(H) = IJ1
;
CJ*'7
3 !⁄
H!0#45" !'&⁄ 
 
f(v) = fração de partículas com velocidade “v”. 
O gráfico a seguir é obtido a partir da equação da 
distribuição de Maxwell: 
 
 
 
 
 
 
 
 
 
 
 
 
 
 
 
Para uma mesma temperatura, o gás mais leve possui 
mais velocidade (curva V). Já o gás mais pesado possui 
menos velocidade média (curva I). Outra abordagem 
para o gráfico é de que um mesmo gás em temperaturas 
mais altas é mais veloz em média (curva V) do que em 
temperaturas mais baixas (curva I). 
PROF. PEDRO MADEIRA 
D) Velocidades Notáveis no gráfico da 
distribuição de Maxwell. 
vp = velocidade mais 
provável; 
vma = velocidade média 
aritmética; 
v = velocidade 
quadrática média 
(velocidade média). 
 
 
 
 
 
 
 
 
 
 
 
OBSERVAÇÕES 
 
 
 
 
 
 
 
 
 
 
 
 
 
 
 
 
 
 
 
 
 
 
 
 
 
 
 
 
 
 
 
 
 
 
 
 
 
 
 
 
 
 
 
 
 
 
 
 
 
PROF. PEDRO MADEIRA 
13 
 
 
 
APOSTILA 01 DE FÍSICO-QUÍMICA – PROF. PEDRO MADEIRA (2022) 
 
E) Velocidades relativas de dois gases (Lei 
de Graham). 
 
 
OBSERVAÇÕES 
 
 
 
 
 
 
 
 
 
 
 
 
 
 
 
 
 
 
 
 
 
 
 
 
 
 
 
 
 
 
 
RESOLVA AGORA 09 (ITA 2011) 
Considere dois cilindros idênticos (C1 e C2), de paredes 
rígidas e indeformáveis, inicialmente evacuados. Os 
cilindros C1 e C2 são preenchidos, respectivamente, 
com O2(g) e Ne(g) até atingirem a pressão de 0,5 atm e 
temperatura de 50ºC. Supondo comportamento ideal 
dos gases, são feitas as seguintes afirmações: 
I. O cilindro C1 contém maior quantidade de matéria 
que o cilindro C2. 
II. A velocidade média das moléculas no cilindro C1 é 
maior que no cilindro C2. 
III. A densidade do gás no cilindro C1 é maior que a 
densidade do gás no cilindro C2 . 
IV. A distribuição de velocidades das moléculascontidas 
no cilindro C1 é maior que a das contidas no cilindro 
C2. 
Assinale a opção que apresenta a(s) afirmação(ões) 
CORRETA(S). 
A ( ) Apenas I e III. B ( ) Apenas I e IV. 
C ( ) Apenas II. D ( ) Apenas II e IV. 
E ( ) Apenas III. 
PROF. PEDRO MADEIRA 
F) Frequência de colisões e livre caminho 
médio. 
; ; 
 
Onde Z = frequência de colisões e 
l = livre caminho médio 
OBS: o livre caminho médio é independente da T se o 
gás estiver confinado em um recipiente de volume 
constante, pois T/p é constante nesta situação. Já a 
freqüência de colisões aumenta pois a velocidade 
relativa é dependente da raiz de T. 
OBS2: o livre caminho médio só depende do volume. 
Esta dependência é linear. 
 
OBSERVAÇÕES 
 
 
 
 
 
 
 
 
 
 
 
 
 
 
 
 
 
 
 
 
 
 
 
 
 
 
 
 
 
 
 
 
 
 
 
 
 
 
 
 
A B A
B A B
v M T
=
v M T
relv p
Z = 
k T
s × ×
×
k T
 = 
2 p
l
s
×
× ×
reduzidamassa
mm
mm
ondekTv
BA
BA
rel =
+
== µ
pµ
8
14 
 
 
 
APOSTILA 01 DE FÍSICO-QUÍMICA – PROF. PEDRO MADEIRA (2022) 
 
Atenção ao quadro conclusivo: 
Condição Transformação 
H!
H6
 K!
K6
 
L!
L6
 
Isocórica 
 
Isotérmica 
 
Isobárica 
 
Qualquer 
 
 
 
 
 
 
RESOLVA AGORA 10 (ITA 1989) 
Consideremos um gás formado de moléculas todas 
iguais e que corresponda ao que se considera um gás 
ideal. Este gás é mantido num recipiente de volume 
constante. Dentre as afirmações abaixo, todas 
referentes ao efeito do aumento de temperatura, 
assinale a CORRETA, em relação ao caminho livre 
médio das moléculas e à freqüência das colisões entre 
as mesmas: 
 Caminho livre médio Freqüência de colisões 
A ( ) Inalterado Aumenta 
B ( ) Diminui Inalterada 
C ( ) Aumenta Aumenta 
D ( ) Inalterado Diminui 
E ( ) Diminui Aumenta 
PROF. PEDRO MADEIRA 
G) Velocidades de difusão e efusão 
gasosas. 
A velocidade de difusão é dada por espaço/tempo. 
A velocidade de efusão é dada por uma quantidade 
/tempo. 
Nos dois casos, a lei de Graham é válida. 
H78 =
!M1N+
√CJ;*'
 
 
OBSERVAÇÕES 
 
 
 
 
 
 
 
 
 
 
 
 
 
 
 
 
 
 
 
 
RESOLVA AGORA 11 (ITA 1992) 
Um recipiente A contém, 
inicialmente, uma mistura gasosa, 
comprimida, dos isótopos 20 e 22 
do Neônio. Este recipiente é 
envolvido completamente por 
outro, B, conforme a figura 
ilustrada abaixo. No inicio, o 
recipiente B estava 
completamente evacuado. Por um pequeno furo na 
parede de A, o gás escapa de A para B. Numa situação 
deste tipo, a concentração (em fração molar) do isótopo 
mais leve no gás remanescente dentro do recipiente A, 
em função do tempo, a partir do início do vazamento: 
A ( ) permanece constante. 
B ( ) vai diminuindo sempre. 
C ( ) vai aumentando sempre. 
D ( ) aumenta, passa por um máximo, retomando ao 
valor inicial. 
E ( ) diminui, passa por um mínimo, retomando ao 
valor inicial. 
OBSERVAÇÕES 
 
 
 
 
 
 
 
 
 
 
 
 
 
 
 
 
 
 
 
 
 
RESOLVA AGORA 12 (ITA 2019 – Q08 FÍSICA) 
Em um reservatório são armazenados 1 mol de gás hélio 
e 1 mol de gás oxigênio em equilíbrio térmico. Por meio 
de um orifício de dimensões muito menores que o 
comprimento livre médio das espécies gasosas, inicia-
se um vazamento de gás para o exterior. Sobre essa 
situação são feitas as seguintes afirmações: 
I. No interior do reservatório, os átomos de hélio têm, 
em média, energia cinética menor em comparação à 
das moléculas de oxigênio. 
II. No interior do reservatório, os átomos de hélio têm, 
em média, velocidade de translação maior em 
comparação à das moléculas de oxigênio. 
III. A porção do gás que vaza e a que permanece no 
interior do reservatório têm a mesma fração molar de 
hélio. 
2 2
1 1
T P=4 =4
T P
Þ
2 2
1 1
P V 1=4 =
P V 4
Þ
2 2
1 1
T V=4 =4
T V
Þ
FURO 
A 
 
B 
15 
 
 
 
APOSTILA 01 DE FÍSICO-QUÍMICA – PROF. PEDRO MADEIRA (2022) 
 
A ( ) Apenas a afirmação I é falsa. 
B ( ) Apenas a afirmação II é falsa. 
C ( ) Apenas a afirmação III é falsa. 
D ( ) Há mais de uma afirmação falsa. 
E ( ) Todas as afirmações são verdadeiras. 
 
 
 
 
 
 
 
 
 
 
 
 
 
 
 
 
 
 
 
 
 
 
 
 
 
 
 
 
 
 
 
 
 
 
 
 
 
 
 
 
 
 
 
 
 
 
 
 
 
 
 
TÓPICO 03 – GASES REAIS 
 
A) Interações Intermoleculares 
As interações intermoleculares podem ser avaliadas 
pelo fator de compressibilidade, Z: 
 
Se Z = 1 è o comportamento é ideal e as interações 
intermoleculares são nulas. 
Se Z > 1 è as interações de repulsão são 
predominantes de modo que o volume 
observado é maior do que o estimado pela 
equação dos gases ideais. 
Se Z < 1 è as interações de atração são 
predominantes. 
 
Graficamente, tem-se: 
 
OBSERVAÇÕES 
 
 
 
 
 
 
 
 
 
 
 
 
 
 
 
 
 
 
 
 
 
PROF. PEDRO MADEIRA 
real
ideal
V
Z = 
V
16 
 
 
 
APOSTILA 01 DE FÍSICO-QUÍMICA – PROF. PEDRO MADEIRA (2022) 
 
B) Isotermas Reais 
 
Observe o esquema das isotermas reais: 
 
 
PROF. PEDRO MADEIRA 
OBSERVAÇÕES 
 
 
 
 
 
 
 
 
 
 
 
 
 
 
 
 
 
 
 
 
 
 
 
 
 
 
 
 
 
 
 
C) A Equação de van der Waals 
Correções feitas por van der Waals à equação dos 
gases ideais: 
 
(V – nb) è volume de movimentação das moléculas 
gasosas. 
 
[a(n/V)2] è fator de diminuição da pressão devido às 
interações de atração. 
 
OBS.: A pressão do gás depende da freqüência de 
colisões e da força das colisões 
Assim, a equação de van der Waals é dada por: 
 
 
RESOLVA AGORA 13 (IME 2021) 
O modelo dos gases ideais, ou perfeitos, descreve bem 
o comportamento para a maioria dos casos, no entanto, 
foi necessário desenvolver modelos mais precisos 
dentre os quais se destaca a equação de Van der Waals. 
Deduza a equação de Van der Waals, assumindo que o 
volume da partícula/molécula não seja desprezível e 
existam interações entre as partículas moléculas. 
Considere o seguinte: 
• V é o volume do recipiente do gás; 
• B é o volume total ocupado pelas moléculas do gás; 
• As forças de atração são praticamente nulas no seio 
da mistura do gás; e 
• Próximo às paredes do recipiente, as moléculas são 
atraídas ao centro com uma força proporcional ao 
quadrado da concentração do gás, o que reduz a 
intensidade dos impactos nas paredes do recipiente. 
 
 
 
 
 
 
 
 
 
 
 
 
 
 
 
 
 
 
 
 
 
 
 
 
( ) ( ) [ ]
2 2
nRT n n
p = - a ou p+a V - bn = nRT
V-nb V V
é ù
ê ú
ë û
17 
 
 
 
APOSTILA 01 DE FÍSICO-QUÍMICA – PROF. PEDRO MADEIRA (2022) 
 
RESOLVA AGORA 14 
a) De acordo com a equação de van der Waals, o 
volume crítico é VC = 3b e a pressão crítica é PC = 
(a/27b2). Calcule a temperatura crítica e o fator de 
compressibilidade previsto no ponto crítico. 
b) Encontre a expressão do princípio dos estados 
correspondentes para um gás de van der Waals. Esta 
expressão relaciona a pressão reduzida (PR) com as 
outras variáveis reduzidas (TR e VR). Dados: PR = 
P/PC; TR = T/TC; VR = V/Vc; 
 
 
 
 
 
 
 
 
 
 
 
 
 
 
 
 
 
 
 
 
 
 
 
 
 
 
 
 
 
 
 
 
 
 
 
 
 
 
 
 
 
 
 
 
 
 
 
 
 
 
 
RESOLVA AGORA 15 (ITA 1998) 
A figura a seguir mostra de forma esquemática três 
isotermas, pressão versus volume, para o caso de um 
gás ideal. Trace isotermas análogas para o caso de um 
gás real que, por compressão, acaba totalmente 
liquefeito. No seu gráfico deve ficar claro, para cada 
isoterma, quais são os pontos que correspondem ao 
início e ao fim da liquefação em função da redução do 
volume. 
 
 
 
 
 
 
 
 
 
 
 
 
 
 
 
 
 
 
 
PROF. PEDRO MADEIRA 
D) Isotermas de van der Waals e a 
correção de Maxwell 
 
 
 
 
 
 
 
 
 
 
A reta paralela correspondente à pressão de vapor do 
líquido deve ser traçada de tal forma que as áreas A1 e 
A2 sejam iguais (critério de Maxwell). 
PROF. PEDRO MADEIRA 
18 
 
 
 
APOSTILA 01 DE FÍSICO-QUÍMICA – PROF. PEDRO MADEIRA (2022) 
 
E) Tabelas 
Tabela 1: Constantes Críticas de gases 
 Pc / atm Vc / 
(cm3mol-1) TC / K ZC TB / K 
He 2,26 57,76 5,21 0,305 22,64 
Ne 26,86 41,74 44,44 0,307 122,1 
Ar 48,00 75,25 150,72 0,292 411,5 
Kr 54,27 92,24 209,39 0,291 575,0 
Xe 58,0 118,8 289,75 0,290 768,0 
F2 55 144 
Cl2 76,1 124 417,2 0,276 
Br2 102 135 5840,287 
H2 12,8 64,99 33,23 0,305 110,0 
N2 33,54 90,10 126,3 0,292 327,2 
O2 50,14 78,0 154,8 0,308 405,9 
CH4 45,6 98,7 190,6 0,288 510 
C2H4 50,50 124 283,1 0,270 
C2H6 48,20 148 305,4 0,285 
C6H6 48,6 260 562,7 0,274 
CO2 72,85 94,0 304,2 0,274 714,8 
H2O 218,3 55,3 647,4 0,227 
NH3 111,3 72,5 405,5 0,242 
HCl 81,5 81,0 324,7 0,248 
HBr 84,0 363,0 
HI 80,8 423,2 
PROF. PEDRO MADEIRA 
 
 
 
 
 
 
 
 
Tabela 2: Constantes de Van der 
Waals 
 a 
(L2.atm/mol) 
b 
(L/mol) 
He 0,03412 0,02370 
Ne 0,21070 0,01709 
H2 0,24440 0,02661 
O2 1,36000 0,03183 
N2 1,39000 0,03913 
Cl2 6,49300 0,05622 
CO 1,48500 0,03985 
NO 1,34000 0,02789 
CO2 3,59200 0,04267 
H2O 5,46400 0,03049 
NH3 4,17000 0,03707 
CH4 2,25300 0,04278 
C2H2 4,39000 0,05136 
C2H4 4,47100 0,05714 
C2H6 5,48900 0,06380 
CH3OH 9,52300 0,06702 
19 
 
 
 
APOSTILA 01 DE FÍSICO-QUÍMICA – PROF. PEDRO MADEIRA (2022) 
 
 
CAPÍTULO 01 – O ESTUDO DOS GASES 
EXERCÍCIOS 
 
TÓPICO 01 – GASES IDEAIS 
 
SEÇÃO VESTIBULARES 
 
1. (UFC 1998) Observe a reação da nitroglicerina 
Quando a nitroglicerina, C3H5N3O9, explode, todos os 
produtos são gases. 
Utilizando a equação da reação dada abaixo e os dados 
a seguir apresentados, calcule o volume total de gases, 
em litros, produzido nas condições normais de 
temperatura e pressão, quando 454g de nitroglicerina 
explodem. 
 
Reação: 
4C3H5N3O9(!) ® 12CO2(g)+6N2(g)+10H2O(g)+O2(g) 
Dados: Massa molar de C3H5N3O9 = 227g 
 Volume molar nas CNTP = 22,4L 
Marque a opção que apresenta o cálculo correto do 
volume solicitado. 
A) 22,4L B) 44,8L C) 156,8L 
D) 324,8L E) 649,6L 
 
2. (UFC 1996) Observe o funcionamento do air bag 
A indústria automobilística tem utilizado um novo 
dispositivo de segurança, instalado nos carros, que é 
constituído por um balão inflável (air bag), o qual, após 
impacto do veículo, infla em quarenta milisegundos pela 
injeção de nitrogênio gasoso. Este gás é originado do 
composto NaN3(s), armazenado no balão, o qual se 
decompõe através da reação: 
2NaN3(s) ® 2Na(s) + 3N2(g) 
Calcule a quantidade de NaN3(s) necessária para gerar 
o volume de 50L à temperatura de 27 ºC e pressão de 
2 atm. 
 
3. (Fac. Albert Einstein) A reforma do biogás 
O biogás, majoritariamente constituído por metano é 
uma mistura gasosa obtida a partir da degradação da 
matéria orgânica. Essa mistura, quando descartada na 
atmosfera, causa inúmeros danos ao meio ambiente. O 
processo denominado reforma a vapor do biogás, 
representado na equação, produz gás hidrogênio, uma 
fonte alternativa aos combustíveis fósseis. 
CH4(g) + H2O(g) → CO(g) + 3 H2(g) 
DH = 206 kJ/mol 
Considerando R = 0,08 atm.L/mol.K, a quantidade 
mínima de energia que deve se fornecida na reforma a 
vapor com quantidade suficiente de metano para 
produção de 100 L de gás hidrogênio, armazenados a 
300 K e 3,6 atm, é igual a 
a) 512 kJ. 
b) 1536 kJ. 
c) 1030 kJ. 
d) 206 kJ. 
e) 2060 kJ. 
 
4. (UFC 1998) Observe o poluente 
O monóxido de carbono é um dos poluentes do ar 
presente, especialmente, em zona urbana. A pressão do 
monóxido de carbono de 0,004 atm, no ar, resulta em 
morte, em pouco tempo. Um carro ligado, porém parado, 
pode produzir, entre outros gases, 0,60 mol de monóxido 
de carbono por minuto. Se uma garagem a 27oC tem 
volume de 4,1 x 104 L, em quanto tempo, na garagem 
fechada, atinge-se a concentração letal de CO? 
Obs: Considere que a pressão, na garagem, permanece 
constante e que não há monóxido de carbono presente, 
inicialmente. (Constante dos gases = 0,082 L.atm/mol.k) 
 
5. (UCS 2020) Combustível e comburente 
A dimetil-hidrazina e o tetróxido de dinitrogênio formam 
uma mistura autoignitora para propulsores de foguetes 
espaciais. Essa combinação de combustíveis foi 
utilizada em alguns motores dos módulos espaciais que 
pousaram na Lua durante as missões Apollo na década 
de 1970. A reação química que ocorre entre esses dois 
compostos pode ser representada, simplificadamente, 
por meio da equação descrita abaixo: 
 
C2H8N2(liq) + N2O4(liq) → N2(g) + H2O(g) + CO2(g) 
 
Admitindo que o tetróxido de dinitrogênio esteja em 
excesso e que o rendimento da reação seja 100%, pode-
se concluir que o volume total de gases liberado nas 
CNTP, a partir de 240 g de dimetil-hidrazina, é de 
a) 572,8 L. 
b) 658,6 L. 
c) 724,2 L. 
d) 806,4 L. 
e) 930,0 L. 
 
6. (UFC 2001) 
Ao desejar identificar o conteúdo de um cilindro 
contendo um gás monoatômico puro, um estudante de 
Química coletou uma amostra desse gás e determinou 
sua densidade, d = 5,38 g/L, nas seguintes condições de 
temperatura e pressão: 15oC e 0,97 atm. Com base 
nestas informações, e assumindo o modelo do gás ideal: 
A) Calcular a massa molar do gás. 
B) Identificar o gás. 
 
7. (UECE – 2003) 
O gás metano, conhecido há mais de 2000 anos pelos 
chineses que o utilizavam em processos de 
mumificação, é o composto mais simples da química 
orgânica. Imagine que 6,4 kg de gás metano e 25,6 kg 
de um gás X ocupam volumes iguais na mesma pressão 
e temperatura. O mol do gás X é: 
A) 8g B) 32g C) 48g D) 64g 
 
20 
 
 
 
APOSTILA 01 DE FÍSICO-QUÍMICA – PROF. PEDRO MADEIRA (2022) 
 
8. (UECE 2006) Tem algo errado neste enunciado 
Através da eletrólise de uma solução de sais, obteve-se 
no cátodo uma mistura de gases com a seguinte 
composição em massa: 67% de cloro; 28% de bromo e 
5% de oxigênio. A porcentagem volumétrica do cloro na 
mistura é, aproximadamente: 
A) 67% B) 72% C) 74% D) 76% 
 
9. (UECE 2007) Recipientes conectados 
Um frasco de 250 mL contém neônio a uma pressão de 
0,65 atm. Um outro frasco de 450 mL contém argônio a 
uma pressão de 1,25 atm. Os gases são misturados a 
partir da abertura de uma válvula na conexão que liga os 
dois recipientes. Considerando o volume da conexão 
desprezível e, ainda, o sistema mantido a uma 
temperatura constante, a pressão final da mistura de 
gases é, aproximadamente, 
A) 1,03 atm B) 1,90 atm C) 2,06 atm D) 2,80 atm 
 
10. (FAMERP 2021) Recipientes conectados 
A figura ilustra uma montagem experimental composta 
por três recipientes contendo gases puros à mesma 
temperatura e separados por válvulas. 
 
 
 
Em determinado instante as válvulas são abertas, 
permitindo que as moléculas gasosas possam se 
difundir pelos recipientes até que seja atingido o 
equilíbrio. A temperatura permanece constante durante 
todo o processo. 
 
a) Classifique o sistema quanto ao número de fases 
após a abertura das válvulas. Considerando que a 
velocidade de difusão é inversamente proporcional à 
raiz quadrada da densidade dos gases, qual dos 
gases deve se difundir com a maior velocidade? 
b) Calcule a pressão parcial do gás carbônico na mistura 
após o equilíbrio. Organize os gases em ordem 
crescente de número de moléculas existentes no 
sistema. 
 
11. (CEFET/CE 2003) 
Num reator isotérmico de volume constante V, são 
adicionados n mols de NOBr(g). Se a pressão inicial é p0 
e a conversão (a) do NOBr é 100%, de acordo com a 
equação química não balanceada abaixo, podemos 
afirmar que a relação entre a pressão final (p) e a 
pressão inicial (p0) é igual a: (g = gás) 
NOBr(g) à NO(g) + Br2(g) 
A) 1 B) 1,5 C) 2 D) 2,5 E) 3 
Prof. Pedro Madeira 
 
12. (CEFET/CE 2003) 
Um balão meteorológico esférico tem um raio de 1m, 
quando está no nível do mar a 20ºC, e se expande para 
um raio de 3m, quando elevado a sua altitude máxima 
na temperatura de –20ºC. Admitindo comportamento 
ideal dos gases no balão, podemos afirmar que a 
pressão, medida em atm, dentro do balão, na sua 
altitude máxima, é igual a: 
A) 1,2 x 10–2 B) 2,2 x 10–2 C) 3,2 x10–2 
D) 4,2 x 10–2 E) 5,2 x10–2 
 
13. (CEFET/CE 2004) 
Considere dois recipientes de volumes iguais, A e B, 
contendo gases ideais onde a razão entre os números 
de moléculas nA/nB é igual a 3, e a razão entre as 
temperaturasabsolutas TB/TA é igual a 2. Nestas 
condições, podemos afirmar corretamente que a razão 
entre as pressões pA / pB é igual a: 
A) 1 B) 2 C) 7/2 D) 3 E) 3/2 
 
14. (CEFET/CE 2004) 
Um recipiente de 11L contém 20g de neônio e certa 
massa de hidrogênio. A densidade da mistura (Ne + H2) 
é igual a 0,002 g/cm3 a 0ºC. A pressão exercida, em atm, 
pela mistura é igual a: 
A) 1,1 B) 2,1 C) 4,1 D) 6,1 E) 8,1 
PROF. PEDRO MADEIRA 
15. (CEFET/CE 2005) 
Um gás natural tem a seguinte composição (em 
percentagem molar): 
CH4 (metano) 80% C2H6 (etano) 20% 
Pede-se: a) A composição percentual em volume. 
b) A composição percentual em massa. 
c) O número de metros cúbicos que serão ocupados por 
13,9 kg da mistura nas CNTP. 
 
16. (CEFET/CE 2006) 
Sobre as leis das transformações gasosas, pede-se: 
a) Esboce a Lei de Boyle num diagrama p versus 1/V e 
determine o coeficiente angular da reta obtida. 
b) Esboce a lei das isóbaras (Charles-Gay Lussac) num 
diagrama V versus T e determine o coeficiente 
angular da reta obtida . 
 
17. (UNESP 2021) metal + ácido = sal + H2 
Um professor de química fez uma demonstração em que 
produziu hidrogênio gasoso pela reação, a quente, de 
ácido clorídrico com 6,75g de alumínio sólido sob forma 
de folhas amassadas. 
 
A equação que representa essa reação é: 
 
Al(s) + 3HCl(aq) → AlCl3(aq) + 3/2 H2(g) 
 
Adote para a constante universal do gases o valor R = 8 
J/(mol.K). Supondo que todo o gás hidrogênio produzido 
21 
 
 
 
APOSTILA 01 DE FÍSICO-QUÍMICA – PROF. PEDRO MADEIRA (2022) 
 
nessa reação seja armazenado a uma temperatura 
constante de 27ºC em um recipiente rígido de volume 10 
L, a quantidade de hidrogênio produzida nessas 
condições ficaria submetida a uma pressão de 
a) 6x104 N/m2. b) 8x104 N/m2. c) 5x104 N/m2. 
d) 9x104 N/m2. e) 4x104 N/m2. 
 
18. (UNICAMP 2021) Adsorção de gases 
Um dos grandes desafios para a consolidação de uso do 
hidrogênio como combustível é seu armazenamento 
seguro e em grande quantidade. O hidrogênio pode ser 
armazenado puro, como gás ou líquido. Atualmente, 
parece mais adequado armazenar o hidrogênio na forma 
de hidretos metálicos ou adsorvido em materiais 
porosos nanoestruturados. Para que o armazenamento 
seja considerado eficiente, o material deve apresentar 
capacidade de armazenamento máxima em pressão 
constante e boa reversibilidade; ou seja, o 
armazenamento (adsorção) e a liberação (dessorção) 
devem ocorrer em condições similares. Essas 
características do armazenamento podem ser 
observadas em um gráfico denominado “isoterma de 
adsorção”, que é uma curva de composição de 
hidrogênio no material (C, kg de H2 / kg de material) em 
função da pressão. 
 
 
a) A figura mostra a isoterma de três materiais que 
poderiam ser empregados para armazenar H2. Qual 
curva (A, B ou C) representa o melhor material para 
se armazenar o hidrogênio? Justifique sua escolha. 
 
b) Um carro com motor a combustão interna consome 
24 kg de gasolina (d=700 kg/m3) ou 8 kg de hidrogênio 
para percorrer uma distância de 400 km, adsorvido 
em um material intermetálico do tipo Mg2Ni. 
Considerando que a massa e o volume de um carro 
médio são aproximadamente de 6m3 e 1000 kg, 
respectivamente, uma possível desvantagem desta 
tecnologia alternativa estaria relacionada à massa ou 
ao volume relativamente ocupado pelo Mg2Ni? 
Justifique. 
 
Dados do Mg2Ni: capacidade de armazenamento de H2 
= 3,6 kg de H2 por 100kg de Mg2Ni; densidade = 3400 
kg/m3. 
19. (UNICAMP 1996) 
O gás carbônico, CO2, é pouco solúvel em água. Esse 
processo de dissolução pode ser representado pela 
equação: 
CO2(g) + H2O(l) = HCO3
–(aq) + H+(aq) 
Essa dissolução é muito aumentada quando se adiciona 
NaOH na água. Para se determinar a quantidade de CO2 
em uma mistura desse gás com o gás nobre neônio, foi 
realizado um experimento. O esquema abaixo mostra o 
experimento e o resultado observado. A proveta está 
graduada em mililitros (mL). 
 
 
Sabendo que não houve variação de temperatura 
durante o experimento e considerando desprezíveis a 
solubilidade do gás neônio em água e a pressão de 
vapor da água nessas condições: 
a) como a presença de NaOH aumenta a dissolução do 
gás carbônico na água? 
b) calcule a pressão parcial do CO2 na mistura inicial, 
sabendo que a pressão ambiente é de 90kPa 
(quilopascal). 
 
20. (UNICAMP 2020) 
O CO2 dissolvido em bebidas carbonatadas, como 
refrigerantes e cervejas, é o responsável pela formação 
da espuma nessas bebidas e pelo aumento da pressão 
interna das garrafas, tornando-a superior à pressão 
atmosférica. O volume de gás no “pescoço” de uma 
garrafa com uma bebida carbonatada a 7ºC é igual a 24 
mL, e a pressão no interior da garrafa é de 2,8x105 Pa. 
Trate o gás do “pescoço” da garrafa como um gás 
perfeito. Considere que a constante universal dos gases 
é de aproximadamente 8 J/mol.K. Calcule o número de 
moles de gás no “pescoço” da garrafa é igual a 
a) 1,2x105 
b) 3,0x103 
c) 1,2x10–1 
d) 3,0x10–3 
 
21. (UNICAMP 1997) Observe o air bag 
Com a intenção de proteger o motorista e o passageiro 
de lesões corporais mais graves, em muitos países já é 
obrigatório, em automóveis, o dispositivo chamado de 
“air bag”. Em caso de acidente um microprocessador 
desencadeia uma série de reações químicas que 
22 
 
 
 
APOSTILA 01 DE FÍSICO-QUÍMICA – PROF. PEDRO MADEIRA (2022) 
 
liberam uma certa quantidade de nitrogênio, N2(g), que 
infla rapidamente um balão plástico situado à frente dos 
ocupantes do automóvel. As reações químicas que 
ocorrem nesse processo estão representadas pelas 
seguintes equações: 
 
(Equação 1) 
2 NaN3(s) = 2 Na(s) + 3 N2(g) 
(Equação 2) 
10 Na(s) + 2 KNO3(s) = 5 Na2O(s) + K2O(s) + N2(g) 
(Equação 3) 
K2O(s) + Na2O(s) + SiO2(s) = silicato alcalino (vidro) 
 
No caso de acionamento do sistema de segurança 
descrito, supondo que o volume do saco plástico, 
quando totalmente inflado, seja de 70 litros e que, 
inicialmente, houvesse 2,0 moles de NaN3 e 2,0 moles 
de KNO3: 
a) Qual será a pressão do gás (em kPa), dentro do 
balão, quando este estiver totalmente inflado? 
Considere a temperatura como sendo 27oC. 
b) Supondo-se que o processo envolvesse apenas as 
reações representadas pelas equações 1 e 2, qual 
seria a massa total de substâncias sólidas restantes 
no sistema? 
 
22. (UNICAMP 1997) Recipientes conectados 
O esquema abaixo representa um dispositivo para se 
estudar o comportamento de um gás ideal. Inicialmente, 
no frasco 1, é colocado um gás à pressão de 1 
atmosfera, ficando sob 
vácuo os frascos 2 e 3 . 
Abre-se, em seguida, a 
torneira entre os frascos 1 
e 2 até que se estabeleça o 
equilíbrio. Fecha-se, então, 
esta torneira e abre-se a 
torneira entre os frascos 1 
e 3. O volume do frasco 1 é 
9 vezes maior do que o do 
frasco 2 e o do 3 é 9 vezes 
maior que o do 1. 
a) Feito o procedimento 
acima descrito, em que frasco haverá menor 
quantidade de moléculas do gás? Justifique. 
b) Sendo p2 a pressão final no frasco 2 e p3 a pressão 
final no frasco 3 qual será o valor da relação p2/p3, 
ao final do experimento? 
Justifique. 
Observação: Desprezar o volume dos tubos das 
conexões. 
 
23. (UNICAMP 1998) 
Os polímeros são formados pela união de um grande 
número de unidades básicas, denominadas 
monômeros. Um dos polímeros mais utilizados é o 
polietileno, que é produzido a partir da reação de 
polimerização do gás etileno, que se pode indicar como: 
j CH2=CH2(g) à (-CH2-CH2-)j(s) sendo j um 
valor médio. 
Para a fabricação de um balde, foram utilizados 280 g 
de polietileno com j = 10.000. 
a) Calcule o volume de etileno, a 25oC e 1 bar, 
necessário para produzir o referido balde. Considere 
que o gás seja ideal. 
b) Se um balde de mesma massa e praticamente de 
mesmo tamanho fosse produzido a partir de 
polietileno com j = 20.000, o volume de etileno 
utilizado seria maior? Justifique sua resposta. 
 
24. (UNICAMP 2000) Hidróxido absorveCO2 
A Apolo 13, uma nave tripulada, não pôde concluir sua 
missão de pousar no solo lunar devido a uma explosão 
num tanque de oxigênio líquido. Esse fato desencadeou 
uma série de problemas que necessitaram de soluções 
rápidas e criativas. Um desses problemas foi o acúmulo 
de gás carbônico no módulo espacial. Para reduzir o teor 
desse gás na cabine da nave, foi improvisado um filtro 
com hidróxido de lítio que, por reação química, removia 
o gás carbônico formado. 
a) Escreva a equação química que justifica o uso do 
hidróxido de lítio como absorvedor desse gás. 
b) Qual seria a massa de hidróxido de lítio necessária 
para remover totalmente o gás carbônico presente, 
considerando-o a uma pressão parcial igual a 2% da 
pressão ambiente total de 1,0 atm, estando a cabine 
à temperatura de 20ºC e supondo-se um volume 
interno de 60 m3? 
Prof. Pedro Madeira 
 
25. (UNICAMP 2002) 
Observando o local do incêndio, nossos heróis 
perceberam que aquele não era o lugar ideal para 
guardar nem medicamentos, nem reagentes destinados 
ao laboratório de análises da empresa. Apesar disso, o 
local era considerado o mais seguro e, como também 
era refrigerado, fora o escolhido. Destruição geral! Como 
a explosão fora seguida de incêndio e de outras 
explosões, o teto e as janelas foram destruídos, e a 
chuva, apesar de ajudar a extinguir o fogo, também 
causou estragos. Examinando com cuidado o local, 
Rango observou várias garrafas e garrafões quebrados 
além de uma estante metálica caída e uma geladeira 
destruída... Preso aos cacos de um garrafão de 5 litros, 
pôde ler num rótulo: “Éter etílico”. O volume do 
almoxarifado foi estimado em 82 metros cúbicos. – E se 
o éter de 5 garrafões, contendo 4 kg de éter, cada um, 
houvesse se evaporado naquela sala?... – perguntou-
se Rango. 
a) Considerando o conteúdo de cinco garrafões, qual a 
pressão parcial aproximada do éter (C4H10O) que 
evaporou no almoxarifado, supondo que ele tivesse 
se distribuído uniformemente e considerando as 
propriedades de gás ideal? 
b) Se apenas 10 % do vapor de éter tivesse se 
queimado, qual a energia liberada em joules? 
Dado: DH = – 2530 kJ.mol–1 (combustão do éter). 
Temperatura = 27oC. 
 
26. (UNICAMP 2005) NO é vasodilatador 
O óxido nítrico (NO) é um gás que, produzido por uma 
célula, regula o funcionamento de outras células, 
configurando-se como um princípio sinalizador em 
sistemas biológicos. Essa descoberta não só conferiu o 
23 
 
 
 
APOSTILA 01 DE FÍSICO-QUÍMICA – PROF. PEDRO MADEIRA (2022) 
 
Prêmio Nobel de Medicina em 1998 para Ignaro, 
Furchgott e Murad, como também abriu as portas para 
muitos progressos científicos nesta área, inclusive no 
desenvolvimento do ViagraÒ. Como fármaco, a 
produção do NO começa com a reação entre SO2, ácido 
nítrico e água, originando, além desse gás, o ácido 
sulfúrico. Como produto final, o NO é comercializado em 
cilindros de 16 litros, diluído em N2. 
A concentração máxima é de 0,08 % em massa. Este 
cilindro chega a fornecer cerca de 2400 litros de gás a 
25ºC e 1 atmosfera. 
a) Escreva a equação química da reação de produção 
do NO. 
b) Qual é a massa aproximada de NO contida no 
cilindro a que se refere o texto da questão? 
 
27. (UNICAMP 2006) 
Algumas misturas gasosas podem ser importantes em 
ambientes hospitalares, assim como na prática de 
esportes, como mergulho autônomo a grandes 
profundidades. Uma dessas misturas, denominada 
Trimix, contém 16% de oxigênio, 24% de hélio e 60% de 
nitrogênio (porcentagem em volume). Suponha um 
cilindro de Trimix mantido à temperatura ambiente e a 
uma pressão de 9000 kPa. 
a) Escreva as fórmulas dos gases da mistura. 
b) Qual é a pressão parcial do hélio no cilindro? Mostre 
os cálculos. 
c) Qual é a massa molar média da mistura? Mostre os 
cálculos. 
PROF. PEDRO MADEIRA 
 
28. (UNICAMP 2000) Uma questão de lógica 
Recentemente a Prefeitura de São Paulo ameaçava 
fechar as portas de um centro comercial por causa do 
excesso de gás metano em seu subsolo. O 
empreendimento foi construído nos anos 1980 sobre um 
lixão e, segundo a CETESB, o gás metano poderia subir 
à superfície e, eventualmente, causar explosões. 
a) Uma propriedade que garante a ascensão do metano 
na atmosfera é a sua densidade. Considerando que 
os gases se comportam como ideais, e que a massa 
molar média do ar atmosférico é de 28,8 g mol–1, 
justifique esse comportamento do metano em relação 
ao ar atmosférico. 
b) Na época do acontecimento, veiculou-se na imprensa 
que, “numa mistura com o ar, se o metano se encontra 
dentro de um determinado percentual (5% a 15% em 
volume quando em ar ambiente com 21% de 
oxigênio) e existe uma faísca ou iniciador, a explosão 
irá ocorrer”. Partindo-se do ar atmosférico e de 
metano gasoso, seria possível obter a mistura com a 
composição acima mencionada, pela simples mistura 
desses gases? Justifique. 
 
29. (UNICAMP 2013) Decomposição do H2O2 é exo 
Na década de 1960, desenvolveu-se um foguete 
individual denominado “Bell Rocket Belt", que fez grande 
sucesso na abertura das Olimpíadas de 1984. 
 
 
Simplificadamente, esse foguete funciona à base da 
decomposição de peróxido de hidrogênio contido no 
compartimento 2, onde ele é estável. Abrindo-se a 
válvula 3, o peróxido de hidrogênio passa para o 
compartimento 4, onde há um catalisador. Nesse 
compartimento, o peróxido se decompõe muito 
rapidamente, de acordo com a equação abaixo: 
 
H2O2(l) à H2O(g) + ½ O2(g) DH = – 54 kJ.mol–1 
 
Com base nessas informações, responda: 
a) No funcionamento do dispositivo há liberação ou 
absorção de energia? Justifique. 
b) Considerando a decomposição total de 68 
quilogramas de peróxido de hidrogênio contidos no 
dispositivo, quantos metros cúbicos de gases são 
produzidos? Leve em conta que nas condições de 
uso do dispositivo o volume molar gasoso é de 0,075 
m3 mol–1. 
 
30. (FUVEST 2000) Observe o Heliox 
Os humanos estão acostumados a respirar ar com 
pressão parcial de O2 próxima de 2,1x104 Pa, que 
corresponde, no ar, a uma porcentagem (em volume) 
desse gás igual a 21%. No entanto, podem se adaptar a 
uma pressão parcial de O2 na faixa de (1 a 6)x104 Pa, 
mas não conseguem sobreviver se forçados a respirar 
O2 fora desses limites. 
a) Um piloto de uma aeronave, em uma cabine não 
pressurizada, voando a uma altitude de 12 km, onde 
a pressão atmosférica é de 2,2 x 104 Pa, poderá 
sobreviver se a cabine for alimentada por O2 puro? 
Explique. 
b) Um mergulhador no mar, a uma profundidade de 40 
m, está sujeito a uma pressão cinco vezes maior do 
que na superfície. Para que possa sobreviver, ele 
deve respirar uma mistura de gás He com O2, em 
proporção adequada. Qual deve ser a porcentagem 
de O2, nessa mistura, para que o mergulhador respire 
um "ar" com a mesma pressão parcial de O2 existente 
no ar da superfície, ou seja, 2,1 x 104 Pa? Justifique. 
Obs.: O He substitui com vantagem o N2. 
 
31. (FUVEST 2004) 
Para demonstrar a combustão de substâncias em 
oxigênio puro, este gás pode ser gerado a partir de água 
sanitária e água oxigenada, que contêm, 
respectivamente, hipoclorito de sódio e peróxido de 
24 
 
 
 
APOSTILA 01 DE FÍSICO-QUÍMICA – PROF. PEDRO MADEIRA (2022) 
 
hidrogênio. A reação que ocorre pode ser representada 
por 
NaClO + H2O2 à NaCl + H2O + O2(g) 
É assim que, num frasco, coloca-se certo volume de 
água oxigenada e acrescenta-se, aos poucos, certo 
volume de água sanitária. Observa-se forte 
efervescência. Ao final da adição, tampa-se o frasco 
com um pedaço de papelão. Em seguida, palha de aço, 
presa a um fio de cobre, é aquecida em uma chama até 
ficar em brasa. O frasco com oxigênio é destampado e, 
rapidamente, a palha de aço rubra é nele inserida. 
Então, observa-se luminosidade branca intensa, com 
partículas de ferro incandescentes espalhando-se pelo 
frasco. 
a) Calcule o volume de água sanitária quando se usa, no 
experimento, um frasco de volume adequado, 
sabendo-se que deve ser gerado, nas condições 
ambiente, um volume de 500mL de oxigênio, volume 
este suficiente para expulsar o ar e preencher o 
frasco. 
b) Explique por que, ao ar atmosférico, o ferro fica 
apenas vermelho rubro, mas queima rapidamente, 
quando exposto a oxigênio puro. 
Dados: volume molar do oxigênio nas condições 
ambiente........25,0 L/mol 
densidade da água sanitária..............................1,0 g/mL 
composição da água sanitária: 2,13 g de Cl, na forma de 
hipoclorito, em 100 g de solução aquosa. 
Prof. Pedro Madeira 
 
32. (FUVEST 2008) 
Foram misturados 2,00 L de um alcano de m átomos de 
carbono por molécula e 2,00 L de outro alcano de n 
átomos de carbono por molécula, ambos gasosos. 
Esses alcanos podem ser quaisquer dois dentre os 
seguintes: metano, etano, propano ou butano. Na 
combustão completa dessa mistura gasosa, foram 
consumidos 23,00 L de oxigênio. Todos os volumes 
foram medidos nas mesmas condições de pressão e 
temperatura. 
a) Escreva a equação da combustão completa de um 
alcano de n átomos de carbono por molécula. 
Para identificar os dois alcanos que foram misturados, 
conforme indicado acima, é preciso considerar a lei de 
Avogadro, que relaciona o volume de um gás com seu 
número de moléculas. 
b) Escreva o enunciado dessa lei. 
c) Identifique os dois alcanos. Explique como chegou a 
essa conclusão. 
 
33. (FUVEST 2012) 
 
A um recipiente, contendo solução aquosa de ácido 
sulfúrico, foi adicionada uma massa m de carbonato de 
sódio. Imediatamente após a adição desse sal, foi 
adaptado, à boca do recipiente, um cilindro de raio r, no 
interior do qual um êmbolo, de massa desprezível, pode 
se deslocar sem atrito. Após algum tempo, o carbonato 
de sódio foi totalmente consumido, e o gás liberado 
moveu o êmbolo para cima. Nessa transformação, o 
ácido sulfúrico era o reagente em excesso. 
 
a) Escreva a equação química balanceada que 
representa a transformação que ocorreu dentro do 
recipiente. 
b) O experimento descrito foi repetido utilizando-se 
carbonato de potássio em lugar de carbonato de 
sódio. A massa de carbonato de potássio utilizada 
nesse segundo experimento também foi m. A altura 
atingida pelo êmbolo foi a mesma nos dois 
experimentos? Explique. (Considere desprezível a 
variação de temperatura no sistema). 
c) Escreva a expressão matemática que relaciona a 
altura x, atingida pelo êmbolo, com a massa m de 
carbonato de sódio. Para isso, considere que 
- a solubilidade do gás, na solução, é desprezível, e 
não há perda de gás para a atmosfera; 
- nas condições do experimento, o gás formado se 
comporta como um gás ideal, cujo volume é dado por 
V = nRT/P, em que: 
P = pressão do gás 
n = quantidade de matéria do gás (em mol) 
R = constante universal dos gases 
T = temperatura do gás (em K) 
Observação: Use a abreviatura MM para representar 
a massa molar do carbonato de sódio. 
 
34. (FUVEST 2019) Como o milho vira pipoca 
Um grão de milho de pipoca, visto a olho nu, apresenta 
duas regiões distintas, representadas por A e B na 
figura. Em A, ocorre o tecido acumulador de amido, 
usado, pela planta, para nutrir o embrião. Em B, os 
tecidos vegetais possuem maior teor de água. Ao ser 
aquecida, parte da água transforma-se em vapor, 
aumentando a pressão interna do grão. Quando a 
temperatura atinge 177ºC, a pressão se torna suficiente 
para romper o grão, que vira uma pipoca. 
25 
 
 
 
APOSTILA 01 DE FÍSICO-QUÍMICA – PROF. PEDRO MADEIRA (2022) 
 
 
 
 
Um estudo feito por um grupo de pesquisadores 
determinou que o interior do grão tem 4,5 mg de água 
da qual, no momento imediatamente anterior ao seu 
rompimento, apenas 9% está na fase vapor, atuando 
como um gás ideal e ocupando 0,1 mL. Dessa forma, foi 
possível calcular a pressão Pfinal no momento 
imediatamente anterior ao rompimento do grão. 
 
A associação correta entre região do milho e Pfinal é dada 
por: 
 
a) A = endosperma e Pfinal = 8,3 atm. 
b) B = endosperma e Pfinal = 5,9 atm. 
c) A = xilema e Pfinal = 22,1 atm. 
d) B = xilema e Pfinal = 5,9 atm. 
e) B = endosperma e Pfinal = 92,0 atm. 
 
35. (UNICAMP 2017) 
Bebidas gaseificadas apresentam o inconveniente de 
perderem a graça depois de abertas. A pressão do CO2 
no interior de uma garrafa de refrigerante, antes de ser 
aberta, gira em torno de 3,5 atm, e é sabido que, depois 
de aberta, ele não apresenta as mesmas características 
iniciais. Considere uma garrafa de refrigerante de 
litros, sendo aberta e fechada a cada horas, retirando-
se de seu interior 250 mL de refrigerante de cada vez. 
Nessas condições, pode-se afirmar corretamente que, 
dos gráficos a seguir, o que mais se aproxima do 
comportamento da pressão dentro da garrafa, em 
função do tempo é o 
 
A) 
 
B) 
 
C) 
 
 
D) 
 
36. (UECE 2017) Coleta de gás sobre água 
No laboratório de química, onde é comum recolher-se 
um gás pelo deslocamento de água, foram coletados 
400 mL de gás oxigênio a 25ºC e 1atm de pressão. 
Sabendo-se que a pressão de vapor da água na mesma 
temperatura é 0,03 atm, é correto afirmar que o volume 
de oxigênio seco obtido nas mesmas condições de 
temperatura e pressão é 
a) 328,0 mL 
b) 388,0 mL 
c) 368,0 mL 
d) 354,0 mL 
 
37. (UECE 2016) Catalisador do processo Haber 
Usado como catalisador no processo Haber, como 
agente de contraste em ressonância magnética e em 
camada protetora de aço contra ferrugem, o óxido 
ferroso-férrico é obtido pela reação entre o ferro metálico 
e o vapor d’água que produz também hidrogênio 
molecular. Ao fazer reagir 840g de ferro metálico, obtém-
se um volume de hidrogênio medido a 127ºC e 5 atm de 
pressão correspondente a 
A) 87,46 L. 
B) 131,20 L. 
C) 57,06 L. 
D) 43,73 L. 
 
2
4
 
26 
 
 
 
APOSTILA 01 DE FÍSICO-QUÍMICA – PROF. PEDRO MADEIRA (2022) 
 
38. (UFJF 2015) 
A lei dos gases ideais pode ser utilizada para determinar 
a massa molar de uma substância. Sabendo-se que a 
densidade (d) do enxofre na forma gasosa, na 
temperatura de 500ºC e pressão de 0,888 atm, é 3,710 
g/L, é CORRETO dizer que a fórmula da molécula de 
enxofre nessas condições é: 
A) S2 
B) S4 
C) S6 
D) S8 
E) S9 
 
39. (UEM 2015) 
Uma garrafa metálica aberta, de volume interno de 
(um) litro, é colocada em um sistema onde sua 
temperatura pode ser alterada (aquecida ou resfriada), 
sem que seu volume interno se altere. Assinale a(s) 
afirmação(ões) correta(s) sobre esse sistema, 
inicialmente colocado na temperatura de 27ºC, nos 
experimentos descritos abaixo, realizados ao nível do 
mar, onde a pressão atmosférica é 1,0 atm, ou na cidade 
de La Paz, na Bolívia, onde a pressão atmosférica é de 
0,75 atm. 
 
01) Tanto ao nível do mar como em La Paz, constatou-
se que a quantidade de gás dentro da garrafa 
diminui com o seu aquecimento. 
02) Ao se aquecer a garrafa ao nível do mar até 127ºC, 
a quantidade de ar dentro da garrafa será 
aproximadamente igual à quantidade de ar dentro da 
garrafa colocada em La Paz na temperatura de 
27ºC. 
04) Tanto ao nível do mar como em La Paz, ao se 
aquecer a garrafa até a temperatura de 250ºC, 
tampá-la e resfriá-la à temperatura ambiente, a 
pressão do gás no interior da garrafa será menor do 
que a pressão atmosférica. 
08) Na temperatura de 0oC, o número de moléculas de 
ar no interior da garrafa colocada ao nível do mar ou 
colocada na cidade de La Paz é idêntico. 
16) O número de moléculas de ar dentro da garrafa a 
27ºC dividido pelo número de moléculas de ar dentro 
da garrafa a 227ºC não será o mesmo para 
experimentos feitos ao nível do mar ou em La Paz. 
Prof. Pedro Madeira 
 
40. (UFG 2014) Gases sobre água 
Em um experimento, 90cm3 de um gás são injetados em 
uma proveta submersa, de modo que o nível do gás em 
seu interior tenha a mesma altura que a água da cuba, 
conforme esquema apresentado a seguir. O 
experimento ocorre a 29ºC. A massa do gás injetado é 
de 203 mg. 
 
 
Dados: Pressão de vapor da água a 29ºC = 30 mmHg 
 
Considerando o exposto, determine a massa molar do 
gás em questão e escreva a fórmulaestrutural plana de 
um dos isômeros do gás. 
 
41. (UFG 2012) 
Considere o esquema apresentado a seguir, em que um 
experimento é executado do seguinte modo: um ovo 
cozido e sem casca, colocado sobre o bocal de uma 
garrafa à temperatura ambiente, não passa para seu 
interior em virtude de seu diâmetro ser levemente maior 
que o do bocal, conforme desenho A. Em seguida o ovo 
é retirado e a garrafa é aquecida à conforme 
desenho B. Com a garrafa ainda aquecida, o ovo é 
recolocado sobre o bocal da garrafa e, durante o 
processo de resfriamento da garrafa, ele passa para seu 
interior conforme desenho C. 
 
 
Explique o fenômeno que ocorre no experimento descrito 
e justifique por que o ovo, após o resfriamento, passa 
pelo bocal da garrafa. 
 
42. (UESPI 2012) Observe o Heliox 
Uma criança com severa infecção nos brônquios 
apresenta problemas respiratórios, e o médico 
administra “heliox”, uma mistura de oxigênio e hélio com 
90,0% em massa de O2. Se a pressão atmosférica é 
igual a 1 atm, calcule a pressão parcial de oxigênio que 
foi administrada à criança. 
a) 0,53 atm 
b) 0,60 atm 
c) 0,69 atm 
d) 0,75 atm 
e) 0,82 atm 
 
43. (UEM 2012) 
Para as situações (I) e (II) expressas abaixo, na mesma 
altitude, e o dado fornecido a seguir, considerando uma 
bexiga de borracha deformável e de massa desprezível, 
hermeticamente fechada, contendo 2,0 g de gás hélio 
(supondo que seja um gás ideal), inicialmente a 25°C, 
que pode explodir quando atingido o dobro de sua 
capacidade volumιtrica inicial, assinale o que for correto. 
 
Situações: 
 
1
60 °C,
27 
 
 
 
APOSTILA 01 DE FÍSICO-QUÍMICA – PROF. PEDRO MADEIRA (2022) 
 
I. A bexiga permanece em repouso sobre um piso 
plano e horizontal, cuja área de contato entre a 
bexiga e o piso é 1,0 cm2 e a pressão no interior da 
bexiga é de 2,0 atm. 
II. Com a situação descrita em (I), é colocado sobre a 
bexiga um corpo de massa M. A área de contato 
entre a bexiga e o piso se torna igual a 10 cm2 e é 
exatamente igual à área de contato entre o corpo e a 
bexiga. Considere que a face do corpo de massa M 
que toca a bexiga é plana e possui área sempre 
maior do que a área de contato entre o corpo e a 
bexiga. 
01) Na situação II, seria possível calcular a massa M do 
corpo, se soubéssemos também a pressão interna 
na bexiga e a pressão atmosférica (ambiente). 
02) Ao aumentar-se a temperatura do sistema na 
situação I para 51ºC, a bexiga irá explodir. 
04) Ao colocar-se o corpo de massa M sobre a bexiga, 
mantendo-se o sistema a 25ºC, sua pressão interior 
deverá aumentar em virtude do aumento do volume 
do gás. 
08) O volume ocupado pelo gás hélio na situação I é, 
aproximadamente, de 6,1 litros. 
16) Na situação II, a pressão exercida pelo sistema 
corpo+bexiga sobre o piso é dependente da 
pressão atmosférica no local do experimento. 
 
44. (UEM 2011) 
O manômetro é um aparelho que serve para medir a 
pressão de um gás. Ele consiste em um tubo em forma 
de U, aberto nas duas extremidades, contendo em seu 
interior Hg, conforme ilustrado na figura abaixo. Uma das 
extremidades está conectada à válvula de saída de gás 
do botijão. Com relação ao funcionamento desse 
manômetro, analise as alternativas abaixo e assinale o 
que for correto. 
 
 
01) Se a pressão atmosférica no local da medida é 710 
mmHg, tem-se que a pressão do gás é 2.610 mmHg. 
02) Se essa medida fosse realizada ao nível do mar e no 
topo do monte Everest e, em ambos os casos, a uma 
temperatura de -5 °C, os valores encontrados para a 
pressão do gás seriam diferentes. 
04) Se, nas condições normais de temperatura e 
pressão (CNTP), substituíssemos o Hg por água, os 
valores da altura da coluna no tubo em U seriam 
diferentes, pois a água possui densidade menor que 
a do Hg. 
08) A elevação ou diminuição da temperatura de todo o 
sistema (botijão e manômetro) não alterará a 
diferença entre os níveis do Hg no tubo. 
16) Supondo que a massa de 13 kg de gás butano 
contido no botijão fosse substituída por 13 kg de gás 
hidrogênio na mesma temperatura, não haveria 
alteração entre os níveis de Hg no tubo. 
 
45. (UNICAMP 2019) 
Episódios recentes de erupções vulcânicas têm trazido 
consequências trágicas para a sociedade e para o meio 
ambiente. Ativo desde 1983, o Vulcão Kilauea 
apresentou, em 2018, a sua maior erupção já registrada. 
Quase ao mesmo tempo, foi a vez do Vulcão Fuego da 
Guatemala mostrar sua força. No Kilauea não houve 
explosões, ao contrário do que ocorreu no Fuego. Os 
especialistas afirmam que a ocorrência de uma erupção 
explosiva depende da concentração e do tipo de gases 
dissolvidos no magma, como SO2, HF e HCl, além de 
vapor de água e CO2 aprisionados. A figura a seguir dá 
informações sobre a relação entre quantidades (em mol) 
de SO2, HF e HCl no magma de três vulcões distintos. 
 
De acordo com a figura, em relação às quantidades de 
gases dissolvidos no magma, é correto afirmar que as 
concentrações de SO2 são maiores que as de HF e de 
HCl. 
A) nos três vulcões e, neles, HF e HCl são 
aproximadamente iguais. 
B) em apenas dois vulcões e, neles, HF e HCl são 
aproximadamente iguais. 
C) nos três vulcões, mas em apenas dois deles HF e HCl 
são aproximadamente iguais. 
D) em apenas dois vulcões, mas nos três vulcões HF e 
HCl são aproximadamente iguais. 
28 
 
 
 
APOSTILA 01 DE FÍSICO-QUÍMICA – PROF. PEDRO MADEIRA (2022) 
 
 
46. (UFRGS 2018) Decomposição do HNO3 
A decomposição térmica do ácido nítrico na presença de 
luz libera NO2 de acordo com a seguinte reação (não 
balanceada). 
HNO3(aq) → H2O(ℓ) + NO2(g) + O2(g) 
Assinale a alternativa que apresenta o volume de gás 
liberado, nas CNTP, quando 6,3g de HNO3 são 
decompostos termicamente. 
A) 2,24 L 
B) 2,80 L 
C) 4,48 L 
D) 6,30 L 
E) 22,4 L 
 
47. (UFSC 2018) 
O verão é a estação na qual, ao menos em países de 
clima tropical e subtropical, faz-se uso significativo de 
condicionadores de ar para ampliar o “conforto térmico” 
em ambientes fechados. Você sabe como funciona um 
condicionador de ar? O sistema é baseado em ciclos de 
compressão e expansão de um gás refrigerante, 
tipicamente formado por substâncias como CHClF2 e 
CHF3, que flui por um sistema fechado. A representação 
esquemática abaixo ilustra simplificadamente o 
processo. 
 
 
Disponível em: 
<https://cen.acs.org/articles/95/i33/Periodic-graphics-
chemistry-air-conditioning.html>. [Adaptado]. Acesso 
em: 19 ago. 2017. 
 
Com base no exposto acima, é correto afirmar que: 
01) no compressor, representado na etapa 1, o 
aumento da pressão sobre o gás faz com que a 
temperatura diminua. 
02) no processo de expansão, representado na etapa 
3, o gás refrigerante tem sua temperatura reduzida. 
04) no condensador, representado na etapa 2, o gás 
refrigerante no estado gasoso é convertido em um 
sólido. 
08) a variação de temperatura que ocorre durante a 
expansão (etapa 3) independe do volume do 
dispositivo no qual a expansão é induzida. 
16) os processos de expansão e compressão 
dependem do vapor de água no sistema, já que o 
gás refrigerante é um composto iônico gasoso e, 
portanto, não está sujeito a variações de volume. 
32) as variações de pressão que ocorrem nos 
processos de expansão e compressão dependem 
da quantidade de gás refrigerante no sistema. 
 
48. (UNICAMP 2017) Fermento em pó químico 
Um teste caseiro para saber se um fermento químico 
ainda se apresenta em condições de bom uso consiste 
em introduzir uma amostra sólida desse fermento em um 
pouco de água e observar o que acontece. Se o 
fermento estiver bom, ocorre uma boa efervescência; 
caso contrário, ele está ruim. Considere uma mistura 
sólida que contém os íons dihidrogenofosfato, H2PO4
– e 
hidrogenocarbonato, HCO3
–. 
 
a) Considerando que o teste descrito anteriormente 
indica que a mistura sólida pode ser de um fermento 
que está bom, escreva a equação química que 
justifica esse resultado. 
b) Tendo em vista quea embalagem do produto informa 
que 18 g desse fermento químico devem liberar, no 
mínimo, 1,45x10–3 m3 de gases a 298K e 93.000 Pa 
determine a mínima massa de hidrogenocarbonato 
de sódio que o fabricante deve colocar em 18 gramas 
do produto. 
 
49. (UCS 2021) Os Boranos são muito loucos 
A figura abaixo ilustra uma série de boranos, ou seja, 
compostos químicos formados exclusivamente por boro 
e hidrogênio. Suponha que uma amostra de 1,0g de um 
desses compostos ocupe um volume de 820 cm3, a 
357ºC e 1 atm. 
 
 
 
 
 
Com base nessas informações, é possível concluir que 
o composto em questão é o 
a) B4H10 b) B5H9 c) B4H11 
d) B6H10 e) B10H14 
 
 
29 
 
 
 
APOSTILA 01 DE FÍSICO-QUÍMICA – PROF. PEDRO MADEIRA (2022) 
 
SEÇÃO ITA / IME 
 
50. (ITA 1980 – Q13) 
Dois balões esféricos de mesmo volume são unidos por 
um tubo de volume desprezível, provido de torneira. 
Inicialmente o balão A contém 1,00 mol de um gás ideal 
e em B há vácuo. 
 
Os dois balões são mantidos às temperaturas indicados 
no desenho acima. A torneira é aberta durante certo 
tempo. Voltando a fechá-la, verifica-se que a pressão em 
B é 0,81 do valor da pressão em A. Quanto do gás deve 
ter sobrado no balão A? 
A ( ) 0,20 mol 
B ( ) 0,40 mol 
C ( ) 0,50 mol 
D ( ) 0,60 mol 
E ( ) 0,80 mol 
 
51. (ITA 1981 – Q02) 
Considere a reação representada pela equação: 
Na(c) + 1/2 Cl2(g) à NaCl(c) 
executada em recipiente de 22,4 litros, empregando 1,0 
mol de cada um dos reagentes. Desejam-se 
representar: 
I. as variações dos números de mols dos reagentes e 
do produto, do início ao fim da reação; e 
II. a variação da pressão gasosa dentro do recipiente, a 
0oC, do início ao fim da reação. 
Dispõe-se dos seguintes gráficos onde a escala de 
tempo é arbitrária: 
 
 
 
 
A melhor combinação de variações a representar e 
gráficos representativos é: 
A ( ) I-a e II-c 
B ( ) I-a e II-d 
C ( ) I-a e II-e 
D ( ) I-b e II-d 
E ( ) I-b e II-e 
 
52. (ITA 1981 – Q09) 
Um cilindro de gás de 50 litros contém N2 e O2 na 
proporção molar de 4 para 1; a pressão no interior do 
cilindro é de 2,0 atm e a temperatura é mantida a 31oC. 
Nessa temperatura injeta-se monóxido de carbono no 
cilindro até que a pressão suba para 2,5 atm. Sabe-se 
que não há reação entre os gases e que o volume do 
cilindro não se altera. 
É CORRETO afirmar-se que, após a injeção do 
monóxido de carbono: 
A ( ) o número de mols de O2 é 1,0 
B ( ) os números de mols de O2 e de CO são iguais 
C ( ) a concentração de O2 na mistura final é de 20% 
em volume 
D ( ) a concentração de CO na mistura final é de 25% 
em volume 
E ( ) a concentração de N2 na mistura final é de 64% 
em volume 
 
53. (ITA 1983 – Q11) 
Num equipamento adequado para permitir adição de 
soluções, assim como coleta e medida de volume de 
gases, fez-se a seguinte experiência: 
Após colocar neste equipamento 100 cm3 de uma 
solução aquosa contendo 1,06 g de carbonato de sódio 
por litro de solução, adiciona-se um excesso de solução 
de ácido clorídrico. 
Admitindo que, nesta experiência, todo o gás que 
pudesse ser produzido pela reação entre as duas 
soluções foi de fato coletado, qual o volume medido, em 
cm3, sabendo-se que a experiência foi realizada na 
temperatura de 27oC e pressão de 750 mmHg? 
A ( ) 0,10 x 22,4 
B ( ) 0,10 x 24,9 
C ( ) 100 x 0,0821 
D ( ) 0,20 x 62,3 
E ( ) 0,40 x 62,3 
 
54. (ITA 1984 – Q04) 
Num saco plástico de paredes flexíveis, constituído com 
filme de polietileno, são introduzidos 8,0 litros de hélio e 
2,0 litros de metano, medidos nas condições normais de 
temperatura e pressão. Este saco está num laboratório 
onde a temperatura é 25oC e a pressão é 700 mmHg. 
Com relação a esta mistura, nas condições do 
laboratório, assinale a única afirmação FALSA: 
A ( ) A fração molar de metano na mistura é 0,20. 
B ( ) A concentração de metano na mistura é 20% (V). 
C ( ) A pressão parcial de metano na mistura é 140 
mmHg. 
D ( ) A concentração de metano na mistura é 50% 
(massa). 
E ( ) A concentração de metano na mistura é 3,2 g de 
CH4/22,4 L mistura. 
Prof. Pedro Madeira 
 
30 
 
 
 
APOSTILA 01 DE FÍSICO-QUÍMICA – PROF. PEDRO MADEIRA (2022) 
 
55. (ITA 1984 – Q11) 
No processo de Mond de purificação do níquel, o metal 
impuro é tratado com CO, segundo a equação: 
 
O volume de CO medido nas CNTP, necessário para 
purificar 0,500 g de níquel contendo 25% de impurezas 
inertes será 
A ( ) (22,4 x 0,25 x 0,500 / 58,7) litros. 
B ( ) (22,4 x 4 x 0,500 x 58,7) litros. 
C ( ) (22,4 x 4 x 0,500 / 0,75 x 58,7) litros. 
D ( ) (22,4 x 0,75 x 0,500 / 58,7) litros. 
E ( ) (22,4 x 4 x 1,5 x 58,7) litros. 
 
56. (ITA 1985 – Q05) 
Todas as afirmações desta questão referem-se a 1,00 
cm3 de hidrogênio líquido, cuja densidade vale 0,070 
g.cm–3 a 13,4 K. 
Qual das afirmações abaixo é FALSA? 
A ( ) O líquido contém 0,035 mol de moléculas H2. 
B ( ) O líquido contém 2,1 x 1022 moléculas de H2. 
C ( ) O gás obtido por vaporização desse líquido ocupa 
o volume de 0,57 L a 25oC e pressão de 1,5 
atmosferas. 
D ( ) O gás obtido por vaporização desse líquido 
apresenta densidade de 6,2 x 10–2 g.L–1 a 25oC e 
pressão de 1,5 atmosfera. 
E ( ) O gás obtido por vaporização desse líquido 
exercerá uma pressão de 8,6 atmosferas à 
temperatura de 25oC e volume de 0,10 L. 
 
PERGUNTA 
Calcule a densidade do hidrogênio gasoso a 25oC e 
pressão de 1,5 atmosferas. 
PROF. PEDRO MADEIRA 
57. (ITA 1987 – Q18) 
Uma amostra de óxido de crômio (III) contaminada com 
impureza inerte é reduzida com hidrogênio de acordo 
com a seguinte equação: 
Cr2O3(c) + 3 H2(g) à 2 Cr(c) + 3 H2O(g) 
O volume de H2 medido nas CNTP, necessário para 
purificar 5,0 g de óxido de crômio (III) contendo 15% de 
impurezas inertes será: 
A ( ) ((0,15 x 5,0 x 3 x 22,4) / 152) litros. 
B ( ) ((0,85 x 5,0 x 3 x 22,4) / 152) litros. 
C ( ) ((0,15 x 5,0 x 3 x 22,4) / 104) litros. 
D ( ) ((0,85 x 5,0 x 3 x 22,4) / 104) litros. 
E ( ) ((0,15 x 5,0 x 3 x 22,4) / 104) litros. 
 
58. (ITA 1988 – Q27) 
Num saco de plástico flexível e não permeável a gases, 
inicialmente vazio, são introduzidos sucessivamente, 
50,0 cm3 de N2, 20,0 cm3 de O2 e 30,0 cm3 de CO2, todos 
medidos nas CNTP. Considere as afirmações seguintes, 
relativas às concentrações nesta solução gasosa 
mantida nas CNTP. 
I. A solução contém 50,0% de N2, 20,0% de O2 e 
30,0% de CO2, todas estas porcentagens em 
volume. 
II. A solução contém 50,0% de N2, 20,0% de O2 e 
30,0% de CO2, todas estas porcentagens em 
massa. 
III. As frações molares de N2, O2 e CO2 são, 
respectivamente, 0,500; 0,200 e 0,300. 
IV. A solução é 0,500/22,4 molar em N2; 0,200/22,4 
molar em O2 e 0,300/22,4 molar em CO2. 
Destas afirmações estão corretas apenas: 
A ( ) I e III B ( ) I e IV C ( ) II e IV 
D ( ) I, III e IV E ( ) II, III e IV. 
 
59. (ITA 1989 – Q30) 
Num garrafão de 3,5 L de capacidade, contendo 1,5 L 
de solução 1,0 molar de ácido sulfúrico, introduzem-se 
32,7 g de aparas de zinco; fecha-se rapidamente com 
rolha de borracha. Supondo que a temperatura do 
ambiente onde esta perigosa experiência está sendo 
feita seja de 20oC, o incremento máximo de pressão 
interna (DP) do frasco será de 
A ( ) 0,41 atm B ( ) 3,4 atm C ( ) 5,6 atm 
D ( ) 6,0 atm E ( ) 12,0 atm. 
 
60. (ITA 1990 – Q19) 
Considere os dois recipientes cilíndricos, 1 e 2, providos 
de manômetro e interligados por um tubo com torneira, 
de volume desprezível, conforme figura abaixo. 
 
O primeiro índice, nas grandezas abordadas, se refere 
ao recipiente 1 ou 2. O segundo índice, i ou f, refere-se 
respectivamente, ao que ocorre inicialmente, antes de 
abrir a torneira e ao que ocorre no estado final, depois 
de a torneira permanecer aberta muito tempo. Em face 
destas informações podemos afirmar que: 
A ( ) P1,f = .
2
3
/P2,f B ( ) n1,f =	n2,f 
C ( ) n1,f = .
2
3
/n2,f D ( ) n1,f = .
6
3
/ 	n2,i 
E ( ) P1,f=	P2,f = .
2
3
/P2,i 
 
61. (ITA 1991 – Q04) 
Um recipiente de aço de volume V1, contém ar 
comprimido na pressão P1. Um segundo recipiente de 
aço de volume V2, contém ar menos comprimido na 
pressão P2. Ambos os cilindros estão na pressão 
ambiente. Caso sejam interligados por uma tubulação 
de volume desprezível, a pressão final em ambos os 
cilindros será igual a: 
A ( ) (V1P1+V2P2)/(V1 + V2) 
B ( ) (V1P2+V2P1)/(V1 + V2) 
C ( ) (V1P1+V2P2)/(P1 + P2) 
D ( ) (V1P2+V2P1)/(P1 + P2) 
E ( ) (P1/V1 + P2/V2).V1V2 
 
Ni
CO
Ni(CO)4
31 
 
 
 
APOSTILA 01 DE FÍSICO-QUÍMICA – PROF. PEDRO MADEIRA (2022) 
 
62. (ITA 1991 – Q11) 
No início de uma experiência a bureta 
do lado esquerdo da figura estava 
completamente cheia de água. Pela 
torneira da bureta foi admitido 
nitrogênio gasoso puro. Após a 
admissão do nitrogênio o nível de 
água na bureta ficou na mesma altura 
que no recipiente do lado direito da 
figura. A pressão ambiente externa 
era 760mmHg e a temperatura era 
25oC. Nesta temperatura a pressão de 
vapor da água é 24 mmHg. 
Assinale a afirmação falsa a respeito do volume gasoso 
dentro da bureta após o estabelecimento do equilíbrio: 
A ( ) 736 / 760 das moléculas correspondem a N2. 
B ( ) A pressão total é 784 mm Hg. 
C ( ) A pressão parcial do vapor de água é 24 mm Hg. 
D ( ) A pressão parcial do nitrogênio é 736 mm Hg. 
E ( ) A fração molar das moléculas de água é 24 / 760. 
Prof. Pedro Madeira 
63. (ITA 1991 – Q25) 
Considere o volume de 5,0 litros de uma mistura gasosa 
contendo 20% (V/V) do isótopo 40 do Argônio e 80% 
(V/V) do isótopo 20 de Neônio. Na temperatura de 273ºC 
a mistura exerce a pressão de 20 atm. A quantidade (em 
mol) de Argônio nesta mistura é: 
A ( ) 
 
B ( ) 
 
C ( ) 
 
D ( ) E ( ) 
 
64. (ITA 1993 – Q03) 
Dois balões de vidro, A e B, de mesmo volume contêm 
ar úmido. Em ambos os balões a pressão e a 
temperatura são as mesmas, a única diferença sendo 
que no balão A, a umidade relativa do ar é de 70% 
enquanto que no balão B ela é de apenas 10%. Em 
relação ao conteúdo destes dois balões é ERRADO 
afirmar que: 
A ( ) Os dois balões contêm o mesmo número de 
moléculas. 
B ( ) Os dois balões contêm a mesma quantidade de 
gás, expressa em mol. 
C ( ) No balão B há maior massa de nitrogênio. 
D ( ) No balão A há maior massa total de gás. 
E ( ) A quantidade (mol) e a massa (grama) de vapor 
de água são maiores no balão A. 
 
65. (ITA 1993 – Q15) 
1,31 g de uma mistura de limalhas de cobre e zinco 
reagiu com excesso de solução de ácido clorídrico, 
numa aparelhagem adequada, produzindo gás 
hidrogênio. Este gás, depois de seco, ocupou um 
volume de 269 m! sob pressão de 0,90 atm e a 300 K 
(que corresponde a 1,10 x 273 K). A fração de massa do 
zinco nesta mistura é: 
A ( ) 0,13 B ( ) 0,25 C ( ) 0,50 
D ( ) 0,75 E ( ) 1,00 
66. (ITA 1994 – Q19) 
Por ocasião do jogo Brasil versus Bolívia, disputado em 
La Paz, um comentarista esportivo afirmou que: “Um dos 
maiores problemas que os jogadores da seleção 
brasileira de futebol terão de enfrentar é o fato de o teor 
de oxigênio no ar, em La Paz, ser cerca de 40% menor 
do que aquele ao nível do mar.” Lembrando que a 
concentração do oxigênio no ar, ao nível do mar, é 
aproximadamente 20% (v/v) e supondo que no dia em 
que o comentarista fez esta afirmação a pressão 
atmosférica, em La Paz, fosse igual a, 
aproximadamente, 450 mm Hg, qual das opções a 
seguir contém a afirmação que mais se aproxima daquilo 
que o comentarista poderia ter dito? 
A ( ) A concentração do oxigênio no ar é cerca de 12% 
(v/v). 
B ( ) A fração molar do oxigênio no ar é cerca de 0,12. 
C ( ) A pressão parcial do oxigênio no ar é 
aproximadamente expressa por (0,20 x 760 
mmHg x 0,60). 
D ( ) A pressão parcial do oxigênio no ar é cerca de 152 
mmHg. 
E ( ) A pressão parcial do oxigênio no ar é 
aproximadamente expressa por (0,20 x 760 mm 
Hg x 0,40). 
 
67. (ITA 1994 – Q21) 
Um composto de Grignard é obtido a partir de 13,7 g de 
um brometo de alquila. Esse composto por hidrólise 
forma um hidrocarboneto que ocupa 2,5 L, medido a 
32°C e pressão de 1,0 atmosfera. O hidrocarboneto 
pode ser: 
A ( ) Propano B ( ) Isobutano 
C ( ) 2-Metilbutano D ( ) Isoctano 
E ( ) 2,5-dimetil-hexano 
 
68. (ITA 1995 – Q02) 
A concentração de O2 na atmosfera ao nível do mar é 
20,9% em volume. Assinale a opção que contém a 
afirmação falsa. 
A ( ) Um litro de ar contém 0,209 ℓ de O2. 
B ( ) Um mol de ar contém 0,209 mol de O2. 
C ( ) Um volume molar de ar à CNTP contém 6,7 g de 
O2. 
D ( ) A concentração de O2 no ar é 20,9% em massa. 
E ( ) A concentração de O2 expressa como uma 
relação de volume ou uma relação de mol não se 
altera, se a temperatura ou a pressão são 
modificadas. 
 
69. (ITA 1995 – Q14) 
Considere a queima completa de vapores das quatro 
seguintes substâncias: metano, etano, metanol, e 
etanol. Os volumes de ar necessários para queima de l 
litro de cada um destes vapores, todos à mesma pressão 
e temperatura, são respectivamente, V1, V2, V3 e V4. 
Assinale a alternativa que apresenta a comparação 
correta entre os volumes de ar utilizados na combustão. 
A ( ) V2 > V4 > V1 > V3 B ( ) V2 > V1 > V4 > V3 
C ( ) V4 > V2 > V3 > V1 D ( ) V4 > V3 > V2 > V1 
E ( ) V4 = V2 > V3 = V1 
 
22,4
2,0
22,4
10
22,4
20
22,4
50
22,4
100
 
760 mmHg 
32 
 
 
 
APOSTILA 01 DE FÍSICO-QUÍMICA – PROF. PEDRO MADEIRA (2022) 
 
70. (ITA 1996 – Q24) 
Três recipientes iguais de 4 litros de capacidade, 
chamados de 1, 2 e 3, mantidos na mesma temperatura, 
contêm 180 ml de água. A cada um destes recipientes 
se junta, respectivamente, 0,10 mol de cada uma das 
seguintes substâncias: óxido de cálcio, cálcio metálico e 
hidreto de cálcio. Após a introdução do respectivo sólido, 
cada frasco é bem fechado. Atingido o equilíbrio e 
descartada a hipótese de ocorrência de explosão, a 
pressão final dentro de cada recipiente pode ser 
colocada na seguinte ordem: 
A ( ) p1 = p2 = p3 B ( ) p1 < p2 < p3 
C ( ) p1 < p2 ≈ p3 D ( ) p1 ≈ p2 < p3 
E ( ) p1 > p2 ≈ p3 
 
71. (ITA 1996 – Q25) 
Considere um recipiente de paredes reforçadas (volume 
fixo) provido de torneira, manômetro e de um dispositivo 
para produção de faíscas análogo à “vela de ignição” em 
motores de automóveis. No fundo do recipiente também 
é colocado um dissecante granulado (p. ex. sílica gel). 
Neste recipiente, previamente evacuado, se introduz 
uma mistura de hidrogênio e nitrogênio gasosos até que 
a pressão dentro dele atinja o valor de 0,70 atm, a 
temperatura sendo mantida em 20°C. 
O problema é descobrir a proporção de H2 e N2 nesta 
mistura inicial. Para isso se junta excesso de O2 à 
mistura, já no recipiente, até que a pressão passe ao 
valor de 1,00 atm. Em seguida se faz saltar uma faísca 
através da mistura. Assim, a temperatura e a pressão 
sobem transitoriamente. Deixando a mistura voltar à 
temperatura de 20°C, notamos que o manômetro acusa 
uma pressão de 0,85 atm. (lembrar que a água formada 
é absorvida pelo dissecante, não exercendo pressão 
parcial significativa). 
Das informações podemos concluir que a fração molar 
do hidrogênio na mistura inicial de H2 e N2 era igual a: 
A ( ) 0,07 B ( ) 0,11 C ( ) 0,14 
D ( ) 0,70 E ( ) 1,00 
 
72. (ITA 1996 – P08) 
Uma garrafa de refrigerante, com capacidade de 2,0 
litros, contém 1,0 litro de uma solução aquosa 0,30 molar 
de HCl e é mantida na temperatura de 25°C. 
Introduzindo um pedaço de zinco metálico nesta garrafa 
e fechando a tampa, a pressão no interior da garrafa irá 
aumentar gradualmente. A questão é calcular a massa 
(em gramas) de zinco a ser introduzida para que a 
pressão aumente de 1,0 para 2,0 atm, a temperatura 
sendo mantida em 25°C. Escreva a equação química 
balanceada da reação envolvida e indique os cálculos 
realizados. Para os cálculos, despreze tanto a pressão 
de vapor da solução quanto a solubilidade do gás 
formado.73. (ITA 1997 – Q10) 
Três recipientes fechados, providos de êmbolos móveis, 
contêm a mesma quantidade (mol) do único gás 
especificado: N2 no recipiente 1; CO no recipiente 2 e 
CO2 no recipiente 3. Considerando a temperatura 
medida em kelvin e a pressão em atm, são feitas as 
afirmações: 
I. Se a pressão e a temperatura forem as mesmas, as 
massas específicas dos gases nos recipientes 1 e 2 
serão praticamente iguais. 
II. Se a pressão e a temperatura forem as mesmas, as 
massas específicas dos gases nos recipientes 2 e 3 
serão praticamente iguais. 
III. Se a temperatura for a mesma, mas a pressão no 
interior do recipiente 1 for o duplo da pressão no 
recipiente 2, a massa específica do gás no 
recipiente 1 será praticamente o duplo da massa 
específica do gás no recipiente 2. 
IV. Se a temperatura for a mesma, mas a pressão no 
interior do recipiente 3 for o duplo da pressão no 
recipiente 2, a massa específica do gás no 
recipiente 3 será maior do que o duplo da massa 
específica do gás no recipiente 2. 
V. Se a pressão for a mesma, mas a temperatura do 
recipiente 1 for o duplo da temperatura no recipiente 
2, a massa específica do gás no recipiente 1 será 
praticamente o duplo da massa específica do gás no 
recipiente 2. 
Estão CORRETAS apenas: 
A ( ) I, III e IV B ( ) I e II C ( ) I e V 
D ( ) II e V E ( ) III e IV 
 
74. (ITA 1998 – P02) 
Numa experiência de eletrólise da água formam-se 
3,00g de H2(g). Calcule o volume ocupado por esta 
massa de hidrogênio, suposta isenta de umidade, na 
temperatura de 300K e sob a pressão de 684 mmHg 
(0,90 x 760 mmHg). 
 
75. (ITA 2000 – Q07) 
Considere as afirmações abaixo relativas ao 
aquecimento de um mol de gás N2 contido em um 
cilindro provido de um pistão móvel sem atrito: 
I. A massa específica do gás permanece constante. 
II. A energia cinética média das moléculas aumenta. 
III. A massa do gás permanece a mesma. 
IV. O produto pressão x volume permanece constante. 
Das afirmações feitas, estão CORRETAS 
A ( ) apenas I, II e III B ( ) apenas I e IV 
C ( ) apenas II e III D ( ) apenas II, III e IV 
E ( ) todas. 
Prof. Pedro Madeira 
76. (ITA 2001 – Q10) 
Um cilindro provido de um pistão móvel, sem atrito, 
contém um gás ideal. Qual dos gráficos abaixo 
representa, qualitativamente, o comportamento 
CORRETO do sistema quando a pressão (P) e/ou o 
volume (V) são modificados, sendo mantida constante a 
temperatura (T)? 
A ( ) B ( ) C ( ) 
 
 
 
 
33 
 
 
 
APOSTILA 01 DE FÍSICO-QUÍMICA – PROF. PEDRO MADEIRA (2022) 
 
D ( ) E ( ) 
 
 
 
77. (ITA 2003 – Q16) 
Dois compartimentos, 1 e 2, têm volumes iguais e estão 
separados por uma membrana de paládio, permeável 
apenas à passagem de hidrogênio. Inicialmente, o 
compartimento 1 contém hidrogênio puro (gasoso) na 
pressão PH2, puro = 1 atm, enquanto que o compartimento 
2 contém uma mistura de hidrogênio e nitrogênio, ambos 
no estado gasoso, com pressão total Pmist = (PH2 + PN2) 
= 1 atm. Após o equilíbrio termodinâmico entre os dois 
compartimentos ter sido atingido, é CORRETO afirmar 
que: 
A ( ) PH2, puro = 0 B ( ) PH2, puro = PN2, mist 
C ( ) PH2, puro = Pmist D ( ) PH2, puro = PH2, mist 
E ( ) Pcompartimento 2 = 2 atm 
 
78. (ITA 2003 – Q23) 
Determine a massa específica do ar úmido, a 25oC e 
pressão de 1 atm, quando a umidade relativa do ar for 
igual a 60%. Nessa temperatura, a pressão de vapor 
saturante da água é igual a 23,8 mmHg. Assuma que o 
ar seco é constituído por N2(g) e O2(g) e que as 
concentrações dessas espécies no ar seco são iguais a 
79 e 21% (v/v), respectivamente. 
 
79. (ITA 2004 – Q16) 
Uma massa de 180 g de zinco metálico é adicionada a 
um erlenmeyer contendo solução aquosa de ácido 
clorídrico. Ocorre reação com liberação de gás que é 
totalmente coletado em um Balão A, de volume igual a 
2 L. Terminada a reação, restam 49 g de zinco metálico 
no erlenmeyer. A seguir, por meio de um tubo provido de 
torneira, de volumes desprezíveis, o Balão A é 
conectado a um Balão B, de volume igual a 4 L, que 
contém gás nitrogênio sob pressão de 3 atm. Considere 
que a temperatura é igual em ambos os balões e que 
esta é mantida constante durante todo o experimento. 
Abrindo-se a torneira do tubo de conexão entre os dois 
balões, ocorre a mistura dos dois gases. Após 
estabelecido o equilíbrio, a pressão nos dois balões 
pode ser expressa em função da constante dos gases 
(R) e da temperatura absoluta (T) por 
 
A ( ) 1
2
RT B ( ) 1
3
RT + 2 C ( ) 3
2
RT 
 
B ( ) 1
3
RT + 2 
 
E ( )	RT + 3 
 
80. (ITA 2005 – Q15) 
A 25°C, uma mistura de metano e propano ocupa um 
volume (V), sob uma pressão total de 0,080 atm. 
Quando é realizada a combustão completa desta 
mistura e apenas dióxido de carbono é coletado, verifica-
se que a pressão desse gás é de 0,12 atm, quando este 
ocupa o mesmo volume (V) e está sob a mesma 
temperatura da mistura original. Admitindo que os gases 
têm comportamento ideal, assinale a opção que contém 
o valor CORRETO da concentração, em fração em mols, 
do gás metano na mistura original. 
A ( ) 0,01 B ( ) 0,25 C ( ) 0,50 
D ( ) 0,75 E ( ) 1,00 
 
81. (ITA 2006 – Q14) 
Um recipiente fechado, mantido a volume e temperatura 
constantes, contém a espécie química X no estado 
gasoso a pressão inicial P0. Esta espécie decompõe-se 
em Y e Z de acordo com a seguinte equação química: 
 
X(g) à 2 Y(g) + 1/2 Z(g). 
 
Admita que X, Y e Z tenham comportamento de gases 
ideais. Assinale a opção que apresenta a expressão 
CORRETA da pressão (P) no interior do recipiente em 
função do andamento da reação, em termos da fração a 
de moléculas de X que reagiram. 
A ( ) P = [1 + (1/2) a] P0 B ( ) P = [1 + (2/2) a] P0 
C ( ) P = [1 + (3/2) a] P0 D ( ) P = [1 + (4/2) a] P0 
E ( ) P = [1 + (5/2) a] P0 
 
82. (ITA 2008 – Q24) 
Em um laboratório, a 20°C e utilizando um sistema 
adequado, H2(g) foi obtido através da reação entre uma 
amostra de uma liga de 0,3 g de magnésio e um litro de 
uma solução aquosa 0,1 mol.L–1 em HCl. Um manômetro 
indicou que a pressão no interior do recipiente que 
contém o H2(g) era de 756,7 Torr. Sabendo-se que a 
pressão de vapor d’água a 20°C é 17,54 Torr e o volume 
de H2(g) obtido foi 0,200 L, determine a pureza da 
amostra da liga de magnésio (massa de magnésio x 
100/massa total da amostra), considerando que 
somente o magnésio reaja com o HCl. 
 
83. (ITA 2008 – Q27) 
Uma chapa de ferro é colocada dentro de um 
reservatório contendo solução aquosa de ácido 
clorídrico. Após um certo tempo observa-se a dissolução 
do ferro e formação de bolhas gasosas sobre a 
superfície metálica. Uma bolha gasosa, de massa 
constante e perfeitamente esférica, é formada sobre a 
superfície do metal a 2,0 metros de profundidade. 
Calcule: 
a) o volume máximo dessa bolha de gás que se 
expandiu até atingir a superfície do líquido, 
admitindo-se que a temperatura é mantida constante 
e igual a 25oC e que a base do reservatório está 
posicionada ao nível do mar. 
b) a massa de gás contida no volume em expansão da 
bolha. 
Sabe-se que no processo corrosivo que originou a 
formação da bolha de gás foram consumidos 3,0 x 1015 
átomos de ferro. 
Dado: massa específica da solução aquosa de HCl é 
igual a 1020 kg.m–3 na temperatura de 25oC. 
 
34 
 
 
 
APOSTILA 01 DE FÍSICO-QUÍMICA – PROF. PEDRO MADEIRA (2022) 
 
84. (ITA 2010 – Q12) 
Um vaso de pressão com volume interno de 250 cm3 
contém gás nitrogênio (N2) quimicamente puro, 
submetido à temperatura constante de 250°C e pressão 
total de 2,0 atm. Assumindo que o N2 se comporta como 
gás ideal, assinale a opção CORRETA que apresenta os 
respectivos valores numéricos do número de moléculas 
e da massa específica, em kg m–3, desse gás quando 
exposto às condições de pressão e temperatura 
apresentadas. 
A ( ) 3,7 x 1021 e 1,1 B ( ) 4,2 x 1021 e 1,4 
C ( ) 5,9 x 1021 e 1,4 D ( ) 7,2 x 1021 e 1,3 
E ( ) 8,7 x 1021 e 1,3 
 
85. (ITA 2014 – Q11) 
Considere um mol de um gás que se comporta 
idealmente, contidoem um cilindro indeformável provido 
de pistão de massa desprezível, que se move sem atrito. 
Com relação a este sistema, são feitas as seguintes 
afirmações: 
I. Se o gás for resfriado contra pressão externa 
constante, o sistema contrai-se. 
II. Se pressão for exercida sobre o pistão, a velocidade 
média das moléculas do gás aumenta. 
III. Se o sistema for aquecido a volume constante, a 
velocidade média das moléculas aumenta, 
independentemente da natureza do gás. 
IV. A velocidade média das moléculas será maior se o 
gás for o xenônio e menor se for o argônio. 
Das afirmações acima, está(ão) ERRADA(S) apenas 
A ( ) I e II. B ( ) I, III e IV. C ( ) II e III. 
D ( ) II e IV. E ( ) IV. 
Prof. Pedro Madeira 
86. (ITA 2014 – Q24) 
Escreva a reação de combustão completa de um 
hidrocarboneto genérico (CaHb) com ar atmosférico. 
Considere a presença do nitrogênio gasoso no balanço 
estequiométrico da reação e expresse os coeficientes 
estequiométricos dessa reação em função de α e β. 
 
87. (ITA 2014 – Q27) 
Considere uma mistura gasosa constituída de C3H8, CO 
e CH4. A combustão, em excesso de oxigênio, de 50 mL 
dessa mistura gasosa forneceu 70 mL de CO2 (g). 
Determine o valor numérico do percentual de C3H8 na 
mistura gasosa. 
 
88. (ITA 2015 – Q20) 
Contribuíram de forma direta para o desenvolvimento 
do conceito de pressão atmosférica 
A ( ) Friedrich August Kekulé e John Dalton. 
B ( ) Michael Faraday e Fritz Haber. 
C ( ) Galilei Galilei e Evangelista Torricelli. 
D ( ) Jons Jacob Berelius e Eduard Buchner. 
E ( ) Robert Bunsen e Henry Louis Le Chatelier 
 
89. (ITA 2015 – Q21) 
3,64 gramas de fosfeto de cálcio foram adicioados a uma 
certa quantidade de água. Após a reação completa, todo 
o produto gasoso formado foi recolhido em um recipiente 
de 8,2 mL. Calcule o valor numérico da pressão, em atm, 
exercida pelo produto gasoso a 27ºC. 
90. (ITA 2016 – Q08) 
Uma amostra de 4,4 g de um gás ocupa um volume de 
3,1 L a 10ºC e 566 mmHg. Assinale a alternativa que 
apresenta a razão entre as massas específicas dste gás 
e a do hidrogênio gasoso nas mesmas condições de 
pressão e temperatura. 
A ( ) 2,2 
B ( ) 4,4 
C ( ) 10 
D ( ) 22 
E ( ) 44 
 
91. (ITA 2017 – Q03) 
Um frasco fechado contém dois gases cujo 
comportamento é considerado ideal: hidrogênio 
molecular e monóxido de nitrogênio. Sabendo que a 
pressão parcial do monóxido de nitrogênio é igual a 3/5 
da pressão parcial do hidrogênio molecular, e que a 
massa total da mistura é de 20g, assinale a alternativa 
que fornece a porcentagem em massa do hidrogênio 
molecular na mistura gasosa. 
A ( ) 4% B ( ) 6% C ( ) 8% 
D ( ) 10% E ( ) 12% 
 
92. (ITA 2017 – Q24) 
Após inalar ar na superfície, uma pessoa mergulha até 
uma profundidade de 200 m, em apneia, sem exalar. 
Desconsiderando as trocas gasosas que ocorrem nos 
alvéolos pulmonares, calcule a pressão parcial do 
nitrogênio e do oxigênio do ar contido no pulmão do 
mergulhador. 
 
93. (ITA 2018 – Q05) 
Um recipiente de 240L de capacidade contém uma 
mistura dos gases ideais hidrogênio e dióxido de 
carbono, a 27ºC. Sabendo que a pressão parcial do 
dióxido de carbono é três vezes menor que a pressão 
parcial do hidrogênio e que a pressão total da mistura é 
de 0,82 atm, assinale a alternativa que apresenta, 
respectivamente, as massas de hidrogênio e de dióxido 
de carbono no recipiente. 
A ( ) 2g e 44g 
B ( ) 6g e 44g 
C ( ) 8g e 88g 
D ( ) 12g e 88g 
E ( ) 16g e 44g 
 
94. (ITA 2018 – Q07) 
Considere um recipiente de 320 L, ao qual são 
adicionados gases ideais nas seguintes condições: 
I. Hélio: 30.000 cm3 a 760 cmHg e 27ºC 
II. Monóxido de carbono: 250 L a 1.140 mmHg e –23oC 
III. Monóxido de nitrogênio: 2m3 a 0,273 atm e 0oC 
 
Sabendo que a pressão total da mistura gasosa é 4,5 
atm, assinale a opção que apresenta a pressão parcial 
do hélio na mistura gasosa. 
A ( ) 0,1 atm 
B ( ) 0,2 atm 
C ( ) 0,5 atm 
D ( ) 1,0 atm 
E ( ) 2,0 atm 
35 
 
 
 
APOSTILA 01 DE FÍSICO-QUÍMICA – PROF. PEDRO MADEIRA (2022) 
 
95. (ITA 2019 – Q60) 
Uma amostra de gás contém 80% de metano, 10% de 
etano, 5% de propano e 5% de nitrogênio, em volume. 
Considerando que todos os átomos de carbono na 
amostra de gás são convertidos em butadieno com 
100% de rendimento, assinale a opção que apresenta a 
massa de butadieno obtido a partir de 100 g do gás. 
A ( ) 50 g 
B ( ) 60 g 
C ( ) 70 g 
D ( ) 80 g 
E ( ) 90 g 
 
96. (ITA 2020 – Q62) 
Um reator com 200 L de capacidade, possui uma mistura 
de dióxido de nitrogênio e monóxido de carbono a 400 
K, cujo comportamento pode ser considerado ideal. Os 
gases reagem entre si para formar dióxido de carbono e 
monóxido de nitrogênio. A pressão total no reator é igual 
a 32,8 atm e, no início da reação, a pressão parcial do 
monóxido de carbono é três vezes maior que a do 
dióxido de nitrogênio. As massas iniciais de dióxido de 
nitrogênio e de monóxido de carbono são, 
respectivamente, 
A ( ) 1,5kg e 4,2kg. 
B ( ) 1,5kg e 4,5kg. 
C ( ) 1,5kg e 6,6kg. 
D ( ) 2,3kg e 4,2kg. 
E ( ) 2,3kg e 6,6kg. 
 
97. (ITA 2020 – Q68) 
Considerando que o ar é composto aproximadamente 
de 21% de O2 e 79% de N2 em volume, tem-se que a 
razão molar ar/combustível da combustão completa de 
um determinado alcano é igual a 59,5. A partir desse 
dado, assinale a alternativa que corresponde à soma 
dos coeficientes estequiométricos de todas as 
substâncias presentes nessa reação. 
A ( ) 30,5 
B ( ) 55,5 
C ( ) 82,0 
D ( ) 112,0 
E ( ) 124,5 
 
98. (IME 1993 – Q01) 
Uma determinada reação química gera um produto 
gasoso, do qual foi coletada uma amostra para análise. 
Verificou-se que a amostra, pesando 0,32g, ocupa 
492cm3 a 27oC e 1atm de pressão, obedece à lei dos 
gases ideais e é formada por 75% em peso de carbono 
e 25% em peso de hidrogênio. Determine: 
a) Qual o peso molecular deste gás e 
b) Qual a sua fórmula molecular mínima? 
Prof. Pedro Madeira 
 
99. (IME 1995 – Q02) 
Mistura-se um fluxo de ar seco com vapor de água para 
se obter ar úmido com 2,0%, em volume, de umidade. 
Admitindo o comportamento ideal dos gases e a massa 
molecular média do ar seco como 28,96 g/mol, calcule a 
massa específica do ar úmido a 14,25oC e 1,00 x 105 Pa. 
100. (IME 1997 – Q01) 
Para a determinação do poder calorífico de uma 
amostra, devemos encher uma bomba calorimétrica de 
volume 4,0x10-4 m3 com oxigênio até atingirmos uma 
pressão manométrica de 2,0x106 Pa. 
Na preparação da bomba calorimétrica para a análise, 
utilizamos o oxigênio de um cilindro com volume de 0,01 
m3, a uma pressão manométrica de 1,0x107 Pa. 
Admitindo que apenas 80% do conteúdo de oxigênio do 
cilindro seja efetivamente utilizado, e que devemos 
realizar 20 testes por semana, determine a duração, em 
semanas, do cilindro de oxigênio utilizado para encher a 
bomba calorimétrica, considerando que os gases 
tenham comportamento ideal. 
 
101. (IME 1998 – Q05) 
Uma determinada quantidade de nitrogênio (N2) ocupa 
um recipiente com volume de 10 litros a uma 
temperatura de 127oC e a uma pressão de 4,92 atm. 
Adiciona-se ao nitrogênio 9,03 x 1023 moléculas de 
oxigênio (O2). Sabendo-se que a pressão final de 
equilíbrio do sistema é de 6,15 atm, calcule a 
temperatura final de equilíbrio. 
R = 0,082 atm.L/mol.K 
 
102. (IME 1999 – Q04) 
Borbulha-se oxigênio através de uma coluna de água e, 
em seguida, coletam-se 100 cm3 do gás úmido a 23ºC e 
1,06 atm. Sabendo que a pressão de vapor da água a 
23ºC pode ser considerada igual a 0,03 atm, calcule o 
volume coletado de oxigênio seco nas CNTP. 
 
103. (IME 2004 – Q05) 
Na figura abaixo, o cilindro A de volume VA contém um 
gás inicialmente a uma pressão P0 e encontra-se 
conectado, através de uma tubulação dotada de uma 
válvula (1), a um vaso menor B de volume VB, repleto do 
mesmo gás a uma pressão p tal que P0 > p > Patm. Abre-
se a válvula 1 até que a pressão fique equalizada nos 
dois vasos, após o que, fecha-se esta válvula e abre-se 
a válvula 2 ate que apressão do vaso menor B retorne 
ao seu valor inicial p, completando um ciclo de operação. 
Sabendo-se que o sistema é mantido a uma temperatura 
constante T, pede-se uma expressão para a pressão do 
vaso A após N ciclos. 
 
 
 
104. (IME 2005 – Q09) 
No equipamento esquematizado na figura abaixo, as 
torneiras A, B e C estão inicialmente fechadas. O 
compartimento 1 de volume 2,00 L contém oxigênio sob 
pressão de 1,80 atm. O compartimento 2 contém 
nitrogênio. O compartimento 3 de volume 1,00 L contém 
36 
 
 
 
APOSTILA 01 DE FÍSICO-QUÍMICA – PROF. PEDRO MADEIRA (2022) 
 
nitrogênio e uma certa quantidade de sódio metálico. 
Executam-se, então, isotermicamente, as três 
operações descritas a seguir: 
1ª) mantendo a torneira A fechada, abrem-se B e C e 
faz-se o vácuo nos recipientes 2 e 3, sem alterar a 
quantidade de sódio existente em 3; 
2ª) fecham-se B e C e abre-se A, constatando que, 
após atingir o equilíbrio, o manômetro M1 indica 
uma pressão de 1,20 atm; 
3ª) fecha-se A e abre-se B, verificando que, atingido o 
equilíbrio, o manômetro M2 indica uma pressão de 
0,300 atm. 
Finalmente, fecha-se a torneira B e eleva-se a 
temperatura do recipiente 3 até 77,0oC, quando então, a 
pressão indicada por M2 é de 0,400 atm. 
Calcule a massa inicial de sódio, considerando que, 
antes da elevação da temperatura, todo o sódio se 
transformara em óxido de sódio, e que os volumes das 
tubulações e dos sólidos (sódio e seu óxido) são 
desprezíveis. 
 
 
105. (IME 2006 – Q03) 
O gás obtido pela completa decomposição térmica de 
uma amostra de carbonato de cálcio com 50,0% de 
pureza é recolhido em um recipiente de 300 mL a 
27,0ºC. Sabendo-se que a pressão no recipiente é de 
1,66 MPa, determine: 
a) a massa de gás produzido, admitindo que seu 
comportamento seja ideal; 
b) a massa da amostra utilizada. 
 
106. (IME 2007 – Q01) 
Determine o volume de cloro obtido, a 27,0ºC e 738 
mmHg, pela ação de excesso de ácido clorídrico 
concentrado sobre 30,7 g de pirolusita com 85,0% em 
peso de MnO2. Considere o cloro com comportamento 
ideal. 
 
107. (IME 2008 – Q07) 
Uma amostra de 0,512 
g de uma liga metálica 
Al-Zn reage com HCl, 
recolhendo-se o gás 
formado. Após a total 
dissolução da amostra, 
o gás recolhido é seco, 
resfriado e submetido a 
um processo de 
compressão 
representado pela reta AB no diagrama P-V. Sabendo 
que a temperatura máxima ao longo do processo de 
compressão é 298K, determine o teor de alumínio nesta 
amostra. Considere que o gás se comporta idealmente. 
 
108. (IME 2010 – Q32) 
Em um recipiente fechado queima-se propano com 80% 
da quantidade estequiométrica de ar. Admitindo que não 
haja hidrocarbonetos após a combustão, que todos os 
produtos da reação estejam na fase gasosa e que a 
composição volumétrica do ar seja de uma parte de O2 
para quatro partes de N2, calcule a porcentagem molar 
de CO2 no recipiente após a combustão (considere 
comportamento ideal para os gases). 
(A) 4,35% (B) 4,76% (C) 5,26% 
(D) 8,70% (E) 14,28% 
Prof. Pedro Madeira 
 
109. (IME 2010 – Q39) – RESOLVA AGORA 
As alternativas abaixo representam processos 
hipotéticos envolvendo 2 mols de um gás ideal, contidos 
em um conjunto cilindro-pistão. Assinale a alternativa 
que apresenta mais de três estados (V, T) nos quais a 
pressão é máxima: 
 
(A) 
 
(B) 
 
 
(C) 
 
 
 V(Litros) 
 
 
 15 
 
 12,5 
 
 10 
 300 390 480 T(K) 
 V(Litros) 
 
 
 12 
 
 
 
 9 
 300 330 450 480 T(K) 
 V(Litros) 
 
 
 15 
 
 
 
 10 
 320 480 T(K) 
P(atm) 
 0,25 0,40 V (L) 
0,90 
 
 
 
0,60 
B 
A 
37 
 
 
 
APOSTILA 01 DE FÍSICO-QUÍMICA – PROF. PEDRO MADEIRA (2022) 
 
(D) 
 
 
 (E) 
 
 
110. (IME 2010 – Q40) 
Um sistema fechado e sem fronteiras móveis contém 
uma determinada massa gasosa inerte. Sabe-se que, 
após aquecimento, o sistema registra um aumento de 
5% na pressão e de 15ºC na temperatura (considere que 
o gás se comporta idealmente). A respeito do valor da 
temperatura inicial, pode-se dizer que: 
(A) é igual ou inferior a 30ºC. 
(B) é superior a 30ºC e inferior a 300ºC. 
(C) é igual ou superior a 300ºC. 
(D) somente pode ser calculado conhecendo-se o 
volume e a massa de gás. 
(E) somente pode ser calculado conhecendo-se o 
volume, a massa e a pressão inicial do gás. 
 
111. (IME 2011 – Q31) 
Um recipiente de paredes rígidas, contendo apenas ar, 
aberto para a atmosfera, é aquecido de 27ºC a 127ºC. 
Calcule a percentagem mássica de ar que saiu do 
recipiente, quando atingido o equilíbrio final. 
(A) 79% 
(B) 75% 
(C) 30% 
(D) 25% 
(E) 21% 
 
112. (IME 2011 – Q40) 
Um gás ideal sofre uma mudança de estado ilustrada 
pelos gráficos I e II abaixo. 
Dentre as alternativas abaixo, assinale aquela que se 
ajusta aos gráficos acima. 
(A) a é o volume, b é a temperatura, d é a pressão e o 
processo é uma expansão a temperatura constante. 
(B) d é a temperatura, b é a pressão, a é o volume e o 
processo é uma compressão. 
(C) a é o volume, b é a pressão, d é a temperatura e o 
processo é um resfriamento isobárico. 
(D) a é o volume, b é a temperatura, d é a pressão e o 
processo é uma compressão isotérmica. 
(E) a é a pressão, b é o volume, d é a temperatura e o 
processo é um aquecimento isobárico. 
 
 
113. (IME 2012 – Q05) 
Na figura, uma solução concentrada de HCl, contida em 
A, é gotejada sobre zinco sólido em B. Um dos produtos 
dessa reação escoa para C, onde é completamente 
consumido na reação com o vapor de uma substância 
simples, cujo elemento pertence à família 17. O produto 
da reação ocorrida em C é um gás incolor. A válvula V 
permite somente o escoamento no sentido de B para C. 
O recipiente C possui volume de 1,0 L, é mantido a 
100°C durante todo o processo e contém inicialmente 
0,05 mol da substância simples supracitada. 
Observações: 
* os volumes das conexões e tubulações devem ser 
desconsiderados; 
* a substância presente inicialmente em C é um líquido 
marrom-avermelhado à temperatura ambiente. 
Determine: 
a) as reações que ocorrem em B e C, identificando o 
estado físico de cada uma das substâncias 
envolvidas. 
b) o número máximo de mols do produto da reação em 
B que pode escoar para C, sem que a pressão neste 
exceda 2,0 atm, se a extremidade D for fechada. 
 
 V(Litros) 
 
 
 12 
 
 
 
 9 
 300 330 450 480 T(K) 
 V(Litros) 
 
 
 15 
 
 
 
 10 
300 450 T(K) 
38 
 
 
 
APOSTILA 01 DE FÍSICO-QUÍMICA – PROF. PEDRO MADEIRA (2022) 
 
114. (IME 2013 – Q06) 
Um tubo vertical graduado, dotado de um êmbolo de 
peso não desprezível e sem atrito e de um dispositivo 
elétrico para produzir centelhamento, contém uma 
mistura gasosa composta de amônia (NH3) e fosfina 
(PH3) em equilíbrio térmico. Introduz-se, então, um 
volume de oxigênio gasoso que contém apenas a massa 
necessária para a oxidação estequiométrica dos 
reagentes presentes. Após a estabilização à 
temperatura original, o deslocamento do êmbolo indica 
um aumento de volume de 150 cm3. Provoca-se o 
centelhamento elétrico e, após o término da reação de 
combustão e o retorno à temperatura inicial, identifica-
se um volume parcial de 20,0 cm3 de nitrogênio gasoso. 
Considerando que os únicos produtos reacionais 
nitrogenado e fosforado são, respectivamente, 
nitrogênio gasoso e pentóxido de difósforo, determine o 
volume da mistura original, antes da introdução do O2. 
 
115. (IME 2014 – Q04) 
O TNT (2,4,6-trinitrotolueno) é um composto químico 
com propriedades combustíveis e explosivas. Em 
condições específicas e controladas, m gramas de TNT 
entram em combustão completa em presença de ar 
estequiométrico sem detonar ou explodir. Os produtos 
dessa reação foram coletados e transferidospara um 
sistema de captura de 820 L. Ao atingirem equilíbrio 
térmico com o ambiente (27oC), a pressão registrada no 
sistema de captura foi de 1,77 atm. Assumindo que a 
hipótese do gás ideal é válida, que o ar é uma mistura 
de N2 e O2 na proporção volumétrica de 4:1, que todo o 
nitrogênio existente nos produtos está na forma de uma 
única substância simples e que não existem produtos 
sólidos, determine o valor de m. 
 
116. (IME 2015 – Q01) 
Uma amostra de 1,264 g de Nitropenta, uma 
substância sólida explosiva cuja fórmula estrutural 
é dada abaixo, é detonada num vaso fechado 
resistente de 0,050 dm3 de volume interno, 
pressurizado com a quantidade estequiométrica de 
oxigênio puro, a 300 K, necessária para a 
combustão completa. Calcule a pressão inicial do 
vaso, considerando o comportamento dos gases 
como ideal. 
 
 
117. (IME 2017 – Q02) 
Uma amostra de magnésio metálico reage completa e 
estequiometricamente com uma mistura de oxigênio e 
nitrogênio em proporção molar 1:3, respectivamente, 
produzindo óxido de magnésio (sólido) e nitreto de 
magnésio (sólido). Em seguida, adiciona-se água em 
excesso aos produtos. Determine as massas de nitreto 
de magnésio e de magnésio, necessárias para liberar 
11,2 L de amônia nas CNTP, conforme o procedimento 
descrito. 
 
118. (IME 2018 – Q32) 
Um sistema fechado contendo um gás ideal no estado 1 
sofre as transformações α e β, conforme indicado na 
figura abaixo. 
 
 
Sabendo que a transformação α é isotérmica e β 
isobárica, indique o gráfico que representa os estados 
do sistema. 
 
 
 
 
Prof. Pedro Madeira 
119. (IME 2022 – Q39) 
Uma amostra de 390 g de sulfito de cálcio com 25% de 
impurezas, em massa, é atacada por ácido clorídrico 
concentrado em um meio reacional a 2 atm e 300 K. 
Considere comportamento ideal de gases. 
O
N
O
O
O
N
O O
O
N
O
O
O
N
O O
+
+ +
+
39 
 
 
 
APOSTILA 01 DE FÍSICO-QUÍMICA – PROF. PEDRO MADEIRA (2022) 
 
Pode-se afirmar que o volume, em litros, de anidrido 
sulforoso obtido pelo consumo completo do sulfito é: 
(A) 22,4 
(B) 30,0 
(C) 40,0 
(D) 54,6 
(E) 72,8 
 
TÓPICO 02 – TEORIA CINÉTICA DOS 
GASES IDEAIS 
 
SEÇÃO VESTIBULARES 
 
120. (UNICID 2016) 
Comprime-se um gás, à pressão constante de 1,0 atm, 
empurrando um êmbolo de modo que seu volume passe 
de 0,20 m3 para 0,10 m3. 
 
a) Nessa compressão, a energia interna desse gás 
aumenta ou diminui? Justifique sua resposta. 
b) Sabendo que a compressão foi realizada a 27ºC, 
calcule a pressão que deve ser aplicada para manter 
o mesmo volume de gás comprimido, à temperatura 
de 0oC. 
 
121. (ESPCEX 2021) 
Gases apresentam um fenômeno chamado de difusão 
de gases. 
Considere que, em determinadas condições de 
temperatura e pressão, a velocidade de difusão 
de 1 mol do gás hidrogênio seja de 28 km/min. Nestas 
mesmas condições a velocidade 
(em km/h) de 1 mol do gás metano é de 
a) 600 km/h 
b) 729 km/h 
c) 1211 km/h 
d) 422 km/h 
e) 785 km/h 
 
122. (UEM 2016) 
Assinale o que for correto. 
01) No que diz respeito à interpretação microscópica de 
uma transformação isobárica, pode-se dizer que o 
aumento da violência das colisões contra as paredes 
internas do recipiente, provocado pelo aumento de 
temperatura, é compensado pela diminuição da 
frequência com que as colisões ocorrem. 
02) Um balão de festas de 2 L, mantido a 21oC em um 
ambiente com ar condicionado, é levado para o 
exterior, onde a temperatura é 32oC. Admitindo-se 
que as variações de pressão possam ser 
desprezadas, o aumento de volume do balão será 
inferior a 10% do volume inicial. 
04) Considere a decomposição do CaCO3(s) em CaO(s) 
e CO2(g). Admitindo-se que 50,0g de CaCO3(s) 
tenham sido totalmente decompostos, a pressão do 
CO2(g) produzido será de aproximadamente 4,1 atm 
se este gás tiver sido coletado em um recipiente de 
3L e estabilizado a uma temperatura de 27ºC. 
08) Um gás real, sendo resfriado isobaricamente a 1atm, 
atingirá seu menor volume possível, no estado 
gasoso, a –273oC. 
16) Segundo a Lei de Amagat, o volume total de uma 
mistura gasosa é igual a soma dos volumes parciais 
de seus componentes. 
 
123. (UEM 2012) 
Considerando dois recipientes idênticos e 
hermeticamente fechados A e B, contendo as mesmas 
quantidades molares dos gases rarefeitos CO2 e H2, 
respectivamente, que possuem a mesma energia 
cinética média por molécula, assinale o que for correto. 
01) A soma da energia cinética média de todas as 
partículas constitui a energia interna dos gases 
contidos nos recipientes A e B. 
02) Quanto maior a energia cinética média das 
partículas, maior será a temperatura do gás. 
04) Se os gases contidos em A e B estiverem sob o 
mesmo nível de agitação térmica, a energia interna 
do gás em A será maior devido à sua massa molar 
maior. 
08) Como o CO2 possui uma massa molar maior que o 
H2, a pressão que ele exerce sobre as paredes do 
recipiente A é maior que a pressão que o H2 exerce 
sobre as paredes do recipiente B. 
16) A pressão manométrica exercida pelos gases 
contidos em A e B sobre as paredes dos respectivos 
recipientes independe da velocidade média ou da 
taxa de colisão das moléculas do gás com as 
paredes do recipiente. 
 
124. (UPE 2012) 
Dois chumaços de algodão, I e II, embebidos com 
soluções de ácido clorídrico, HCl, e amônia, NH3, 
respectivamente, são colocados nas extremidades de 
um tubo de vidro mantido fixo na horizontal por um 
suporte, conforme representação abaixo. Após um certo 
tempo, um anel branco, III, forma-se próximo ao 
chumaço de algodão I. 
 
 
Baseando-se nessas informações e no esquema 
experimental, analise as seguintes afirmações: 
 
I. O anel branco forma-se mais próximo do HCl, porque 
este é um ácido forte, e NH3 é uma base fraca. 
40 
 
 
 
APOSTILA 01 DE FÍSICO-QUÍMICA – PROF. PEDRO MADEIRA (2022) 
 
II. O anel branco formado é o NH4Cl sólido, resultado da 
reação química entre HCl e NH3 gasosos. 
III. O HCl é um gás mais leve que NH3, logo se 
movimenta mais lentamente, por isso o anel branco 
está mais próximo do ácido clorídrico. 
 
Está correto o que se afirma em 
a) II. b) III. c) I e II. d) I e III. e) II e III. 
 
125. (UECE 2008) 
A partir das pesquisas de Robert Boyle (1627- 1691), foi 
possível estabelecer a teoria cinético-molecular dos 
gases. Essa teoria afirma que 
A) todos os choques entre as partículas de um gás ideal 
e as paredes do recipiente são perfeitamente 
elásticos. 
B) nas mesmas condições de temperatura e pressão, as 
velocidades de difusão de dois gases são 
diretamente proporcionais às raízes quadradas de 
suas densidades. 
C) um mol de qualquer gás, nas condições padrões de 
temperatura e pressão (CPTP), ocupa um volume de 
22,4L. 
D) à temperatura constante, o volume de uma massa de 
gás é diretamente proporcional à sua pressão. 
 
126. (CEFET/CE 2006) 
Sobre o comportamento dos gases, é falsa a opção: 
A) Quando dois gases estão envolvidos numa reação, 
há uma relação simples entre seus volumes, medidos 
à mesma pressão e temperatura. 
B) As colisões entre as moléculas de um gás são 
perfeitamente elásticas, logo ocorrem sem perda de 
energia. 
C) Duas ampolas, uma contendo amônia (NH3) e a outra 
contendo gás sulfídrico (H2S – que tem cheiro de ovo 
podre), foram quebradas num canto de uma sala. 
Sabendo que a densidade da amônia é menor que a 
densidade do H2S, uma pessoa que está no centro 
dessa sala sentirá primeiro o cheiro de ovo podre. 
D) Um gás é composto por moléculas que estão 
separadas umas das outras por distâncias muito 
maiores do que suas próprias dimensões. As 
moléculas podem ser consideradas “pontos”, isto é, 
têm massa, mas o seu volume é desprezível. 
E) Todos os gases são semelhantes em um aspecto: a 
dependência do volume em relação à quantidade, à 
temperatura e à pressão. 
 
127. (CEFET/CE 2007) 
Sobre os gases, é falso afirmar que: 
A) Quando entram em contato com uma superfície, 
exercem pressão sobre ela,porque as moléculas 
gasosas estão em movimento constante. 
B) Ao contrário das moléculas na matéria condensada, 
as moléculas gasosas estão separadas por 
distâncias grandes comparadas com o seu volume. 
Em conseqüência, as densidades dos gases são 
muito baixas nas condições atmosféricas. 
C) As leis dos gases ajudam-nos a prever o seu 
comportamento, mas não explicam que acontece à 
escala molecular e que origina as modificações 
observadas no mundo macroscópico. 
D) Diminuindo o volume de uma dada quantidade de 
gás, aumenta a sua densidade numérica (número de 
moléculas por unidade de volume) e, 
conseqüentemente, diminui a freqüência de colisões, 
portanto a pressão de um gás é inversamente 
proporcional ao volume que ele ocupa. 
E) A teoria cinética, um método de descrever o 
comportamento das moléculas gasosas, baseia-se 
nas seguintes hipóteses: as moléculas gasosas 
estão separadas por distâncias muito maiores que 
suas próprias dimensões; possuem massa, mas têm 
volume desprezível, estão em movimento constante 
e colidem freqüentemente umas com as outras. 
 
128. (FUVEST 2012) 
 
Uma estudante de Química realizou um experimento 
para investigar as velocidades de difusão dos gases HCl 
e NH3. Para tanto, colocou, simultaneamente, dois 
chumaços de algodão nas extremidades de um tubo de 
vidro, como mostrado na figura acima. Um dos 
chumaços estava embebido de solução aquosa de 
HCl(g), e o outro, de solução aquosa de NH3(g). Cada 
um desses chumaços liberou o respectivo gás. No ponto 
de encontro dos gases, dentro do tubo, formou-se, após 
10 s, um anel de sólido branco (NH4Cl), distante 6,0 cm 
do chumaço que liberava HCl(g). 
 
a) Qual dos dois gases, desse experimento, tem maior 
velocidade de difusão? Explique. 
b) Quando o experimento foi repetido a uma temperatura 
mais alta, o anel de NH4Cl(s) se formou na mesma 
posição. O tempo necessário para a formação do 
anel, a essa nova temperatura, foi igual a, maior ou 
menor do que 10 s? Justifique. 
c) Com os dados do experimento descrito, e sabendo-se 
a massa molar de um dos dois gases, pode-se 
determinar a massa molar do outro. Para isso, utiliza-
se a expressão 
 
Considere que se queira determinar a massa molar 
do HCl. Caso o algodão embebido de solução aquosa 
de NH3 (g) seja colocado no tubo um pouco antes do 
algodão que libera HCl(g) (e não simultaneamente), 
como isso afetará o valor obtido para a massa molar 
do HCl? Explique. 
 
3
3
velocidade de difusão do NH (g) MM (HC )
velocidade de difusão do HC (g) MM (NH )
=
!
!
41 
 
 
 
APOSTILA 01 DE FÍSICO-QUÍMICA – PROF. PEDRO MADEIRA (2022) 
 
SEÇÃO ITA / IME 
 
129. (ITA 1983 – Q19) 
Nitrogênio gasoso, inicialmente na temperatura 
ambiente, é passado por um tubo mantido num forno. A 
vazão do gás é tão baixa que a pressão na saída 
(quente) é praticamente igual à da entrada (frio). 
Chamemos as vazões do gás (cm3/s) na entrada de v1 e 
na saída de v2. A densidade do gás (g/cm3) na entrada 
é designada por d1 e na saída por d2. Nas condições 
acima teremos que: 
A ( ) v1 < v2 ; d1 < d2 B ( ) v1 < v2 ; d1 > d2 
C ( ) v1 > v2 ; d1 < d2 D ( ) v1 > v2 ; d1 > d2 
E ( ) v1 = v2 ; d1 = d2 
 
PERGUNTA 
Partindo da lei geral dos gases perfeitos, deduza uma 
expressão que fornece o valor da densidade de um gás 
(d) em função da pressão (p), da temperatura (T) e de 
sua massa molar (M). 
 
130. (ITA 1989 – Q29) 
Consideremos um gás formado de moléculas todas 
iguais e que corresponda ao que se considera um gás 
ideal. Este gás é mantido num recipiente de volume 
constante. Dentre as afirmações abaixo, todas 
referentes ao efeito do aumento de temperatura, 
assinale a CORRETA, em relação ao caminho livre 
médio das moléculas e à freqüência das colisões entre 
as mesmas: 
 Caminho livre médio Freqüência de colisões 
A ( ) Inalterado Aumenta 
B ( ) Diminui Inalterada 
C ( ) Aumenta Aumenta 
D ( ) Inalterado Diminui 
E ( ) Diminui Aumenta 
Prof. Pedro Madeira 
 
131. (ITA – 1992 – Q19) 
Um recipiente A contém, 
inicialmente, uma mistura gasosa, 
comprimida, dos isótopos 20 e 22 
do Neônio. Este recipiente é 
envolvido completamente por 
outro, B, conforme a figura 
ilustrada abaixo. No inicio, o 
recipiente B estava completamente 
evacuado. Por um pequeno furo na parede de A, o gás 
escapa de A para B. Numa situação deste tipo, a 
concentração (em fração molar) do isótopo mais leve no 
gás remanescente dentro do recipiente A, em função do 
tempo, a partir do início do vazamento: 
A ( ) permanece constante. 
B ( ) vai diminuindo sempre. 
C ( ) vai aumentando sempre. 
D ( ) aumenta, passa por um máximo, retomando ao 
valor inicial. 
E ( ) diminui, passa por um mínimo, retomando ao valor 
inicial. 
 
132. (ITA 2006 – Q09) 
A figura mostra cinco curvas de distribuição de 
velocidade molecular para diferentes gases (I, II, III, IV e 
V) a uma dada temperatura. Assinale a opção que 
relaciona CORRETAMENTE a curva de distribuição de 
velocidade molecular a cada um dos gases. 
A ( ) I = H2, II = He, III = O2, IV = N2 e V = H2O. 
B ( ) I = O2, II = N2, III = H2O, IV = He e V = H2. 
C ( ) I = He, II = H2, III = N2, IV = O2 e V = H2O. 
D ( ) I = N2, II = O2, III = H2 , IV = H2O e V = He. 
E ( ) I = H2O, II = N2, III = O2 , IV = H2 e V = He. 
 
133. (ITA 2007 – Q30) 
Dois frascos, A e B, contêm soluções aquosas 
concentradas em HCl e NH3, respectivamente. Os 
frascos são mantidos aproximadamente a um metro de 
distância entre si, à mesma temperatura ambiente. 
Abertos os frascos, observa-se a formação de um 
aerossol branco entre os mesmos. Descreva o 
fenômeno e justifique por que o aerossol branco se 
forma em uma posição mais próxima a um dos frascos 
do que ao outro. 
 
134. (ITA 2009 – Q13) 
Assumindo um comportamento ideal dos gases, 
assinale a opção com a afirmação CORRETA. 
A ( ) De acordo com a Lei de Charles, o volume de um 
gás torna-se maior quanto menor for a sua 
temperatura. 
B ( ) Numa mistura de gases contendo somente 
moléculas de oxigênio e nitrogênio, a velocidade 
média das moléculas de oxigênio é menor do que 
as de nitrogênio. 
C ( ) Mantendo-se a pressão constante, ao aquecer um 
mol de gás nitrogênio sua densidade irá 
aumentar. 
D ( ) Volumes iguais dos gases metano e dióxido de 
carbono, nas mesmas condições de temperatura 
e pressão, apresentam as mesmas densidades. 
E ( ) Comprimindo-se um gás a temperatura constante, 
sua densidade deve diminuir. 
 
135. (ITA 2010 – Q13) 
Um recipiente contendo gás hidrogênio (H2) é mantido à 
temperatura constante de 0°C. Assumindo que, nessa 
condição, o H2 é um gás ideal e sabendo-se que a 
velocidade média das moléculas desse gás, nessa 
temperatura, é de 1,85 x 103 m s–1, assinale a alternativa 
CORRETA que apresenta o valor calculado da energia 
cinética média, em J, de uma única molécula de H2. 
A ( ) 3,1 x 10–24 B ( ) 5,7 x 10–24 C ( ) 3,1 x 10–21 
D ( ) 5,7 x 10–21 E ( ) 2,8 x 10–18 
FURO 
A 
 
B 
42 
 
 
 
APOSTILA 01 DE FÍSICO-QUÍMICA – PROF. PEDRO MADEIRA (2022) 
 
136. (ITA 2011 – Q20) 
Considere dois cilindros idênticos (C1 e C2), de paredes 
rígidas e indeformáveis, inicialmente evacuados. Os 
cilindros C1 e C2 são preenchidos, respectivamente, 
com O2(g) e Ne(g) até atingirem a pressão de 0,5 atm e 
temperatura de 50ºC. Supondo comportamento ideal 
dos gases, são feitas as seguintes afirmações: 
I. O cilindro C1 contém maior quantidade de matéria 
que o cilindro C2. 
II. A velocidade média das moléculas no cilindro C1 é 
maior que no cilindro C2. 
III. A densidade do gás no cilindro C1 é maior que a 
densidade do gás no cilindro C2 . 
IV. A distribuição de velocidades das moléculas contidas 
no cilindro C1 é maior que a das contidas no cilindro 
C2. 
Assinale a opção que apresenta a(s) afirmação(ões) 
CORRETA(S). 
A ( ) Apenas I e III. B ( ) Apenas I e IV. 
C ( ) Apenas II. D ( ) Apenas II e IV. 
E ( ) Apenas III. 
 
137. (ITA 2016 – Q15) 
Considerando um gás monoatômicoideal, assinale a 
opção que contém o gráfico que melhor representa 
como a energia cinética média (Ec) das partículas que 
compõem este gás varia em função da temperatura 
absoluta (T) deste gás 
 
A ( ) B ( ) 
 
 
 
 
 
 
 
 
C ( ) D ( ) 
 
 
 
 
 
 
 
 
E ( ) 
 
 
 
 
 
 
 
 
TÓPICO 03 – GASES REAIS 
 
SEÇÃO VESTIBULARES 
 
138. (UFC 2004) 
“AR EM TUBULAÇÃO FAZ CONTA DE ÁGUA 
DISPARAR” (Folha de São Paulo, 27 de agosto de 
2001). Esse fenômeno ocorre porque o ar ocupa 
rapidamente os espaços vazios nas tubulações de água. 
Quando o fornecimento é regularizado, a água empurra 
a solução gasosa acumulada nas tubulações fazendo o 
hidrômetro girar rapidamente. Sabendo que há uma 
pressão moderada na tubulação, analise as afirmativas 
I, II e III, e assinale a alternativa correta. 
I. O ar é constituído de uma solução gasosa real, cujos 
componentes nas CNTP experimentam interações 
de atração que o tornam mais denso, se comparado 
a uma mistura ideal de mesma composição. 
II. O ar ocupa rapidamente os espaços vazios nas 
tubulações devido a sua elevada densidade, uma vez 
que trata-se de uma mistura heterogênea. 
III. Deve-se esperar uma redução na velocidade de 
rotação do hidrômetro em dias frios. 
A) Somente I e II são verdadeiras. 
B) Somente II é verdadeira. 
C) Somente III é verdadeira. 
D) Somente I e III são verdadeiras. 
E) Somente II e III são verdadeiras. 
 
SEÇÃO ITA / IME 
 
139. (ITA 1982 – Q08) 
O cilindro provido de um pistão 
móvel, esquematizado ao lado, 
contém apenas H2O e é mantido 
sob temperatura constante igual 
a 25oC. Assinale a alternativa 
que melhor representa a 
variação do volume com a 
pressão aplicada, abrangendo o H2O desde 
completamente vaporizado até totalmente liquefeito. 
 
A ( ) B ( ) C ( ) 
 
 
 
D ( ) E ( ) 
 
 
 Ec 
 
 
 
 
 
(0,0) 
 T 
 Ec 
 
 
 
 
 
(0,0) 
 T 
 Ec 
 
 
 
 
 
(0,0) 
 T 
 Ec 
 
 
 
 
 
(0,0) 
 T 
 Ec 
 
 
 
 
 
(0,0) 
 T 
43 
 
 
 
APOSTILA 01 DE FÍSICO-QUÍMICA – PROF. PEDRO MADEIRA (2022) 
 
 
PERGUNTA 
Exponha as razões físicas responsáveis pelas 
inclinações da cada um dos três trechos do gráfico 
correto e diga que tipo de equação descreve cada um 
dos três trechos, explicando se a equação é exata ou 
aproximada. 
 
140. (ITA 1988 – Q10) 
Consideremos um recipiente de paredes inertes e 
indeformáveis. A capacidade desse recipiente é de 
aproximadamente 25 litros. Ele é provido de um 
manômetro absoluto e é mantido numa sala 
termostatada a 20oC. A única comunicação do recipiente 
com o exterior é feita através de um tubo provido de 
torneira. Inicialmente extraímos todo o ar contido no 
recipiente com o auxílio de uma bomba de vácuo. Feito 
isto, introduzimos no recipiente, contínua e lentamente, 
água pura (isenta de ar) até um total de 40 g de água. 
Qual dos gráficos a seguir descreve corretamente a 
variação da pressão no recipiente versus massa de água 
introduzida após evacuação prévia do recipiente? 
 
A ( ) B ( ) C ( ) 
 
 
D ( ) E ( ) 
 
Prof. Pedro Madeira 
 
141. (ITA 1989 – Q33) 
Num grande cilindro provido de torneira e pistão com 
êmbolo, conforme figura abaixo, foi introduzido um 
pouco de água líquida, tomando o cuidado de não deixar 
entrar ar. 
 
 
Após a admissão da porção de água, a torneira foi 
fechada. Variando o volume, por movimento lento do 
pistão, mantendo a temperatura, no interior do cilindro, 
igual a 20oC, o gráfico de pressão no cilindro versus 
volume, corresponde a: 
A ( ) B ( ) C ( ) 
 
D ( ) E ( ) 
 
 
142. (ITA 1993 – Q10) 
O cilindro de um pistão móvel, esquematizado abaixo, 
contém apenas etanol puro e é mantido sob temperatura 
constante de 20°C. Assinale a alternativa que melhor 
representa a variação do volume (v) com a pressão (p) 
aplicada, abrangendo etanol desde completamente 
vaporizado até totalmente liquefeito. 
 
 
 
 
 
 
 
A ( ) B ( ) C ( ) 
 
 
D ( ) E ( ) 
 
 
143. (ITA 1998 – P10) 
A figura a seguir mostra de forma esquemática três 
isotermas, pressão versus volume, para o caso de um 
gás ideal. Trace isotermas análogas para o caso de um 
gás real que, por compressão, acaba totalmente 
liquefeito. No seu gráfico deve ficar claro, para cada 
isoterma, quais são os pontos que correspondem ao 
início e ao fim da liquefação em função da redução do 
volume. 
m 
 P 
0 
m 
 P 
0 
m 
 P 
0 
m 
 P 
0 
m 
 P 
0 
V 
 P 
0 V 
 P 
0 V 
 P 
0 
V 
 P 
0 V 
 P 
0 
P 
V 
 P 
 V 
 P 
 V 
 
P 
V 
 P 
 V 
 
44 
 
 
 
APOSTILA 01 DE FÍSICO-QUÍMICA – PROF. PEDRO MADEIRA (2022) 
 
 
 
144. (ITA 2012 – Q09) 
Considere volumes iguais dos gases NH3, CH4 e O2 nas 
CNTP. Assinale a opção que apresenta o(s) gás(es) que 
se comporta(m) idealmente. 
A ( ) Apenas NH3 
B ( ) Apenas CH4 
C ( ) Apenas O2 
D ( ) Apenas NH3 e CH4 
E ( ) Apenas CH4 e O2 
 
145. (IME 2001 – Q03) 
A equação do gás ideal só pode ser aplicada para gases 
reais em determinadas condições especiais de 
temperatura e pressão. Na maioria dos casos práticos é 
necessário empregar uma outra equação, como a de 
van der Waals. Considere um mol do gás hipotético A 
contido num recipiente hermético de 1,1 litros a 27ºC. 
Com auxílio da equação de van der Waals, determine o 
erro cometido no cálculo da pressão total do recipiente 
quando se considera o gás A como ideal. 
Dados: 
Constante universal dos gases: 
R=0,082atm.L.mol–1.K–1. 
Constantes da equação de van der Waals: 
a = 1,21atm.L2.mol–2 e b = 0,10L.mol–1. 
 
146. (IME 2009 – Q36) 
Assinale a alternativa correta. 
A) Um veículo de testes para redução de poluição 
ambiental, projetado para operar entre –40oC e 
50oC, emprega H2 e O2, os quais são estocados em 
tanques a 13 MPa. Pode-se afirmar que a lei dos 
gases ideais não é uma aproximação adequada 
para o comportamento dos gases no interior dos 
tanques. (Dado: 1 atm = 101,3 kPa). 
B) A pressão de vapor de um líquido independe da 
temperatura. 
C) Um recipiente de 500 mL, inicialmente fechado e 
contendo um líquido em equilíbrio com seu vapor, é 
aberto. Pode-se afirmar que a pressão de vapor do 
líquido aumentará. 
D) Na equação PV = nRT, o valor numérico de R é 
constante e independe do sistema de unidades 
empregado. 
E) De acordo com o princípio de Avogadro, pode-se 
afirmar que, dadas as condições de temperatura e 
pressão, o volume molar gasoso depende do gás 
considerado. 
 
147. (ITA 2018 – Q26) 
Uma dada reação (I), cujo calor liberado é 
desconhecido, é conduzida em um reator que utiliza um 
gás mantido a volume constante (V) como banho 
térmico. Outras duas reações (II e III) conduzidas em 
condições similares apresentam calor liberado a volume 
constante (QV) conforme apresentado na tabela abaixo: 
Reação Equação QV (kJ mol–1) 
I A + ½ B à D ? 
II A + B à C 400 
III D + ½ B à C 300 
 
Considere as deguintes informações sobre o gás do 
banho térmico, que tem comportamento não ideal e 
obedece à equação: 
 
!P+n
2a
V2 ' (V	-	nb)=nRT, 
 
Em que: a = 62,5 L2 atm mol–1; b = 0,4 L mol–1; n = 0,4 
mol; V = 10 L; capacidade calorífica molar a volume 
constante (CV,m) = 83,33 J K–1mol–1 ; temperatura inicial 
(Ti) = 300 K. 
 
a) Sabendo que 0,1 mol de A são utilizados na reação 
I, calcule o QV liberado nessa reação. 
b) Determine a temperatura final do banho térmico. 
c) Determine a pressão inicial e a pressão final do 
banho térmico. 
 
148. (IME 2021 – Q06) 
O modelo dos gases ideais, ou perfeitos, descreve bem 
o comportamento para a maioria dos casos, no entanto, 
foi necessário desenvolver modelos mais precisos 
dentre os quais se destaca a equação de Van der Waals. 
Deduza a equação de Van der Waals, assumindo que o 
volume da partícula/molécula não seja desprezível e 
existam interações entre as partículas moléculas. 
Considere o seguinte: 
 
• V é o volume do recipiente do gás; 
• B é o volume total ocupado pelas moléculas do gás; 
• As forças de atração são praticamentenulas no seio 
da mistura do gás; e 
• Próximo às paredes do recipiente, as moléculas são 
atraídas ao centro com uma força proporcional ao 
quadrado da concentração do gás, o que reduz a 
intensidade dos impactos nas paredes do recipiente. 
 
45 
 
 
 
APOSTILA 01 DE FÍSICO-QUÍMICA – PROF. PEDRO MADEIRA (2022) 
 
TÓPICO 04 – QUÍMICA DESCRITIVA DE 
GASES 
 
SEÇÃO VESTIBULARES 
 
149. (UPE 2014) 
A formulação de um determinado produto comercial 
contém, em massa, 58% de solvente e 40% de uma 
mistura gasosa formada por CH3(CH2)2CH3, 
(CH3)2CHCH3 e CH3CH2CH3, numa proporção de 65%, 
15% e 20%, respectivamente. 
Qual alternativa apresenta o produto que atende à 
descrição acima? 
a) Desodorante aerossol 
b) Extintor de incêndio 
c) Gás de cozinha 
d) Gás natural veicular – GNV 
e) Gás refrigerante de geladeira 
 
150. (UFPR 2013) 
Nos últimos dois anos, a imprensa divulgou notícias 
sobre o risco de explosão oferecido por condomínios de 
luxo e um Shopping Center de São Paulo. Os 
estabelecimentos foram construídos sobre antigos 
lixões. Nesses casos, o órgão responsável, ligado à 
Secretaria de Meio Ambiente, autuou os 
estabelecimentos, exigindo providências quanto à 
instalação de sistema de extração de gases. 
 
Em relação a esse risco, considere as seguintes 
afirmativas: 
 
1. O risco de explosão deve-se principalmente à 
presença de metano, produzido por micro-
organismos em condições anaeróbicas, na 
decomposição do material orgânico presente no 
lixão. 
2. Os gases oferecem risco de explosão porque reagem 
vigorosamente com agentes oxidantes fortes. 
3. O gás metano é facilmente detectado pelo odor 
característico. 
4. Os gases que oferecem risco de explosão 
apresentam alta densidade, formando lençóis nos 
compartimentos de subsolo, como garagens 
subterrâneas. 
 
Assinale a alternativa correta. 
a) Somente as afirmativas 2 e 3 são verdadeiras. 
b) Somente as afirmativas 1 e 2 são verdadeiras. 
c) Somente as afirmativas 2, 3 e 4 são verdadeiras. 
d) Somente as afirmativas 1 e 4 são verdadeiras. 
e) As afirmativas 1, 2, 3 e 4 são verdadeiras. 
 
151. (UERJ 2010) 
O oxigênio gasoso pode ser obtido em laboratório por 
meio da decomposição térmica do clorato de potássio. 
Em um experimento, o gás foi produzido em um frasco 
A e recolhido em um frasco B que, inicialmente, continha 
apenas água. Observe o esquema: 
 
 
Ao final do experimento, verificaram-se as seguintes 
medidas no interior do frasco B: 
• volume de gás recolhido: 123 mL 
• temperatura interna: 27 oC 
• pressão total no nível da água: 786,7 mmHg 
• pressão de vapor da água: 26,7 mmHg 
Determine a massa de oxigênio gasoso, em gramas, 
recolhida no frasco B, e apresente a equação química 
completa e balanceada correspondente a sua obtenção. 
Prof. Pedro Madeira 
 
152. (FUVEST 2008) 
No seguinte trecho (adaptado) de uma peça teatral de 
C. Djerassi e R. Hoffmann, as esposas de três químicos 
do século XVIII conversam sobre um experimento feito 
com uma mistura de gases. 
 
"SENHORA POHL - Uma vez o farmacêutico Scheele 
estava borbulhando [a mistura gasosa] através de uma 
espécie de água. 
MADAME LAVOISIER - Deve ter sido água de cal. 
SENHORA PRIESTLEY - A água ficou turva, não ficou? 
MADAME LAVOISIER - É o mesmo gás que 
expiramos... o gás que removemos com a passagem 
através da água de cal. 
SENHORA POHL - Depois ele me pediu que colocasse 
no gás remanescente um graveto já apagado, apenas 
em brasa numa das extremidades. Já estava 
escurecendo. 
SENHORA PRIESTLEY - E o graveto inflamou-se com 
uma chama brilhante... e permaneceu aceso!" 
Empregando símbolos e fórmulas atuais, podem-se 
representar os referidos componentes da mistura 
gasosa por: 
a) CO2 e O2 b) CO2 e H2 c) N2 e O2 
d) N2 e H2 e) CO e O2 
 
153. (FUVEST 2009) 
Cinco cilindros, A, B, C, D e E, contêm gases diferentes. 
Cada um contém apenas um dos seguintes gases: 
monóxido de carbono, dióxido de carbono, dióxido de 
enxofre, amônia e metano, não se sabendo, porém, qual 
gás está em qual cilindro. Com amostras dos gases, 
retiradas de cada cilindro, foram feitos os seguintes 
experimentos, a fim de identificá-los. 
 
I) Cada gás foi borbulhado em água, contendo algumas 
gotas de solução incolor de fenolftaleína. Apenas o 
do cilindro A produziu cor vermelha. 
46 
 
 
 
APOSTILA 01 DE FÍSICO-QUÍMICA – PROF. PEDRO MADEIRA (2022) 
 
II) O gás de cada cilindro foi borbulhado em água de cal. 
Apenas os gases dos cilindros C e D produziram 
precipitado. 
III) Os gases dos cilindros C e D foram borbulhados em 
uma solução aquosa ácida de permanganato de 
potássio, de coloração violeta. Apenas o gás do 
cilindro D descorou essa solução. 
IV) Os gases dos cilindros restantes (B e E) mostraram-
se combustíveis. Ao passar os produtos da 
combustão dos gases desses dois cilindros por um 
tubo contendo cloreto de cálcio anidro, houve 
aumento de massa desse tubo apenas no caso do 
gás do cilindro B. 
a) Identifique os gases contidos nos cilindros A, B, C, D 
e E. 
b) Escreva as equações químicas balanceadas das 
reações do item II. 
c) A reação que ocorre no item III é uma reação de 
precipitação, neutralização ou oxirredução? 
Explique, sem escrever a equação química, o que 
ocorre nessa transformação. 
Dados: Sais de cálcio pouco solúveis em água 
CaCO3 - Carbonato de cálcio 
CaSO3 - Sulfato de cálcio 
CaSO4 - Sulfato de cálcio 
CaC2O4 - Oxalato de cálcio 
O cloreto de cálcio anidro é usado para absorver água. 
 
154. (FUVEST 2009) 
Michael Faraday (1791-1867), em fragmento de "A 
história química de uma vela", assim descreve uma 
substância gasosa que preparou diante do público que 
assistia a sua conferência: "Podemos experimentar do 
jeito que quisermos, mas ela não pegará fogo, não 
deixará o pavio queimar e extinguirá a combustão de 
tudo. Não há nada que queime nela, em circunstâncias 
comuns. Não tem cheiro, pouco se dissolve na água, 
não forma solução aquosa ácida nem alcalina, e é tão 
indiferente a todos os órgãos do corpo humano quanto 
uma coisa pode ser. Então, diriam os senhores: 'Ela não 
é nada, não é digna de atenção da química. O que faz 
no ar?'" A substância gasosa descrita por Faraday é: 
a) H2 (g) b) CO2 (g) c) CO (g) 
d) N2 (g) e) NO2 (g) 
 
SEÇÃO ITA / IME 
 
155. (ITA 1981 – Q06) 
Nas condições ambientes, qual das substâncias abaixo 
é um gás incolor e inodoro quando puro e que, se muito 
comprimido, pode explodir? 
A ( ) SH2 
B ( ) NH3 
C ( ) C2H4 
D ( ) C2H2 
E ( ) LiH 
 
PERGUNTA 
Como e para que fins esse gás incolor, que pode 
explodir por compressão excessiva, é preparado e 
armazenado usualmente? 
 
156. (ITA 1994 – Q06) 
Ao colocar-se um pedaço de magnésio em uma solução 
de ácido clorídrico, verifica-se que ocorre aumento da 
temperatura e desprendimento de gás. O gás que se 
desprende é, sobretudo: 
A ( ) Hidrogênio 
B ( ) Vapor de água 
C ( ) Vapor de magnésio. 
D ( ) Mistura de vapores de magnésio e água. 
E ( ) Mistura de vapores de magnésio e hidrogênio. 
 
157. (ITA 2000 – Q20) 
Num tubo de ensaio dissolve-se açúcar em água e 
acrescenta-se uma porção de fermento biológico do tipo 
utilizado na fabricação de pães. Após certo tempo 
observa-se a liberação de gás nesta mistura. O 
borbulhamento deste gás em uma solução aquosa não 
saturada em Ba(OH)2 provoca, inicialmente, sua 
turvação. 
Esta desaparece com o borbulhamento prolongado do 
gás. A respeito das descrições feitas nestes 
experimentos são feitas as seguintes afirmações: 
I. O produto gasoso formado, e responsável pela 
turvação inicial da solução de Ba(OH)2, é o 
monóxido de carbono (CO). 
II. O produto gasoso formado, e responsável pela 
turvação inicial da solução de Ba(OH)2, é o etanol. 
III. A turvação inicial da solução de Ba(OH)2 é 
justificada pela precipitação do Ba(HCO3)2(c). 
IV. A turvação inicial da solução de Ba(OH)2 é 
justificada pela precipitação do Ba(OH)2(c). 
V. O desaparecimento da turvação inicial da solução 
de Ba(OH)2 é justificado pelareação química 
representada pela seguinte equação: Ba(OH)2(c) + 
HCO3
–(aq) ® BaCO3(aq) + H2O(l) + OH–(aq) 
Das informações acima estão ERRADAS: 
A ( ) apenas I e III. 
B ( ) apenas I e V. 
C ( ) apenas II e IV. 
D ( ) apenas II, IV e V. 
E ( ) todas. 
 
47 
 
 
 
APOSTILA 01 DE FÍSICO-QUÍMICA – PROF. PEDRO MADEIRA (2022) 
 
158. (ITA 2001 – Q06) 
Quando carbeto de alumínio (Al4C3) é adicionado em um 
béquer contendo água líquida a 25ºC, ocorre a formação 
de hidróxido de alumínio e a liberação de um gás. O gás 
formado é o 
A ( ) H2 B ( ) CO C ( ) CO2 
D ( ) CH4 E ( ) C2H2 
 
159. (ITA 2007 – Q07) 
Embrulhar frutas verdes em papel jornal favorece o seu 
processo de amadurecimento devido ao acúmulo de um 
composto gasoso produzido pelas frutas. Assinale a 
opção que indica o composto responsável por esse 
fenômeno. 
A ( ) Eteno. 
B ( ) Metano. 
C ( ) Dióxido de carbono. 
D ( ) Monóxido de carbono. 
E ( ) Amônia. 
 
160. (ITA 2010 – Q23) 
A nitroglicerina, C3H5(ONO2)3(!), é um óleo denso que 
detona se aquecido a 218ºC ou quando é submetido a 
um choque mecânico. Escreva a equação que 
representa a reação química do processo, sabendo que 
a reação de decomposição é completa, e explique 
porque a molécula é explosiva. 
 
161. (ITA 2013 – Q04) 
Na temperatura ambiente, hidróxido de potássio sólido 
reage com o cloreto de amônio sólido, com a liberação 
de um gás. Assinale a alternativa CORRETA para o gás 
liberado nesta reação. 
A ( ) Cl2 
B ( ) H2 
C ( ) HCl 
D ( ) NH3 
E ( ) O2 
Prof. Pedro Madeira 
 
QUESTÕES EXTRAS: OLIMPÍADA 
 
162. (USNCO 2011) 
Um cilindro de 5,00L evacuado foi carregado com 25,5g 
de NH3 e 36,5g de HCl. Calcule a pressão final a 85ºC 
após os dois compostos reagirem completamente. 
(A) 2,94 atm (B) 5,88 atm (C) 8,82 atm (D) 14,7 atm 
 
163. (USNCO 2011) 
A velocidade média molecular em uma amostra gasosa 
a 300K é 500 m/s. A temperatura deste gás é aumentada 
até que a velocidade média das moléculas fique 1000 
m/s. Qual é a nova temperatura? 
(A) 420 K (B) 573 K (C) 600 K (D) 1200 K 
 
164. (USNCO 2011) 
Um recipiente rígido foi preenchido com quantidades 
iguais de N2 e H2 gasosos a uma pressão total de 10,0 
atm. Os gases reagem produzindo amônia gasosa. Se a 
pressão total do gás diminui a uma velocidade de 0,20 
atm/s, qual é a taxa de diminuição da pressão parcial do 
N2 no recipiente? 
(A) 0,40 atm/s (B) 0,30 atm/s 
(C) 0,20 atm/s (D) 0,10 atm/s 
 
165. (USNCO 2012) 
Uma amostra de H2 coletada sobre H2O a 23ºC e sob 
pressão de 732 mmHg possui um volume de 245 mL. 
Qual será o volume do H2 seco a 0oC e 1 atm? 
Po
vap = 21 mmHg 
(A) 211 mL (B) 218 mL (C) 224 mL (D) 249 mL 
 
166. (USNCO 2012) 
Duas amostras gasosas, uma de argônio e outra de 
hélio, possuem a mesma pressão, temperatura e 
volume. Qual afirmativa é correta assumindo 
comportamento ideal? 
(A) A amostra de hélio contém mais átomos do que a 
amostra de argônio e os átomos de hélio possuem 
maior velocidade média. 
(B) As duas amostras possuem O mesmo número de 
átomos mas os átomos de Hélio possuem maior 
velocidade média. 
(C) As duas amostras possuem o mesmo número de 
átomos e ambos os tipos de átomos possuem a 
mesma velocidade média. 
(D) As duas amostras possui o mesmo número de 
átomos, porém o argônio possui maior velocidade 
média. 
 
167. (USNCO 2013) 
Qual substância é usada em equipamentos de auto-
respiração porque absorve CO2 e H2O e libera gás O2? 
(A) KO2 (B) Na2O2 (C) NaOH (D) Li2O 
 
168. (USNCO 2013) 
Em um experimento para verificar o valor do zero 
absoluto, um estudante foi instruído para medir o volume 
do hélio em uma seringa de 10 mL em intervalos de 10°C 
entre 0°C e 100°C. Ela foi orientada a plotar o volume 
versus temperatura e a extrapolar este gráfico até o 
volume zero e ler a temperatura resultante. Que 
modificação do procedimento experimental fornecerá o 
melhor valor para o zero absoluto? 
(A) corrigir cada volume medido para pressão de um 
atmosfera antes de plantar. 
(B) dobrar o número de valores temperatura-volume 
entre 0°C e 100°C. 
(C) usar um termômetro que possa medir a 
temperatura a ±0,10ºC entre 0°C e 100ºC. 
(D) pedir o volume do Hélio em uma seringa a -40°C e 
-80°C. 
 
169. (USNCO 2013) 
Uma amostra de gás medida a 20ºC e 4,0 atm foi 
aquecida até 40ºC sob volume constante. Qual(ais) 
afirmativa(s) é(são) verdadeira(s) após o aquecimento 
relativamente a sua situação inicial? 
I. A energia cinética molecular média aumentou. 
II. A velocidade média molecular não mudou. 
III. A pressão do gás foi aumentada para 8,0 atm. 
48 
 
 
 
APOSTILA 01 DE FÍSICO-QUÍMICA – PROF. PEDRO MADEIRA (2022) 
 
IV. O número de colisões moleculares por segundo não 
mudou. 
(A) apenas I (B) apenas I e IV 
(C) apenas II e III (D) apenas II e IV 
 
170. (USNCO 2013) 
Sob que condições o comportamento dos gases reais 
sofre mais desvia daquele previsto pela lei dos gases 
ideais? 
(A) baixa P, baixa T (B) alta P, baixa T 
(C) baixa P, alta T (D) alta P, alta T 
 
171. (USNCO 2013) 
Quando 0,25L de nitrogênio líquido (d = 0,807 g/mL) for 
vaporizado, qual volume o gás ocuparia a 25ºC e 5,00 
atm? 
(A) 71 L (B) 54 L (C) 35 L (D) 32 L 
 
172. (USNCO 2015) 
Uma mistura de 0,50 mol de gás H2 e 1,3 mol de gás Ar 
foi colocada em um recipiente com volume de 4,82L. Se 
a temperatura da mistura é 50,0oC, qual é a pressão de 
H2 na amostra? 
(A) 1,5 atm (B) 2,8 atm (C) 7,2 atm (D) 9,9 atm 
 
173. (USNCO 2017) 
Um gás com P = 615 mmHg está contido no tubo em U 
mostrado. Se h = 65 mm, qual é a pressão atmosférica, 
Patm? 
 
(A) 550 mmHg (B) 615 mmHg 
(C) 680 mmHg (D) 760 mmHg 
 
174. (USNCO 2017) 
Em ambas as extremidades de um longo tubo de vidro, 
amostras de gases foram introduzidas simultaneamente. 
Uma delas recebeu cloreto de hidrogênio gasoso 
enquanto a outra extremidade recebeu amônia gasosa. 
Onde no tubo o NH4Cl se formou? 
(A) No centro do tubo 
(B) Próximo a extremidade onde o cloreto de hidrogênio 
foi inserido 
(C) Próximo a extremidade onde a amônia foi inserida 
(D) Uniformemente em todas as posições posições do 
tubo 
 
175. (USNCO 2018) 
Acetona possui uma pressão de vapor de 0,307bar a 
25°C. Uma amostra de 0,100 mol de acetona foi 
adicionada um recipiente que continha 1,00 L de argônio 
gasoso a uma pressão de 1,00 bar a 25ºC. Volume do 
recipiente então aumentou para 4,00 L sendo mantida a 
mesma temperatura. 
Qual é a pressão no recipiente após a expansão? 
(A) 0,250 bar (B) 0,307 bar 
(C) 0,557 bar (D) 0,870 bar 
 
176. 
Dois frascos (A e B), de mesmo volume, estão cheios de 
gás submetidos à mesma temperatura. O frasco A 
contém 0,32 g de oxigênio (O2) e o frasco B contém 0,68 
g de um gás B, de fórmula molecular Z2Y. A pressão no 
frasco A é "x" e a pressão no frasco B é "2x". 
a) Identifique o gás B. 
b) Que frasco contém o maior número de moléculas? 
c) Compare os dois gases quanto à energia cinética 
média de suas moléculas. 
d) Compare os dois gases quanto à velocidade 
molecular média. 
 
177. 
Estima-se que a concentração de NO2 no ar atmosférico, 
em zonas industriais, seja da ordem de 0,021 ppm. 
a) calcule a pressão parcial de NO2, numa amostra de 
ar, quando a pressão atmosférica for de 0,98 atm. 
b) quantas moléculas de NO2 estarão presentes, 
nestas condições e na temperatura de 20� C, num 
aposento de 4,5 m x 4,3 m x 2,4 m ? 
c) escreva as equações químicas correspondentes aos 
seguintes enunciados: 
 c.1) o dióxido de nitrogênio dissolve-se em água, 
formando ácido nítrico e óxido nitroso; 
 c.2) a molécula de óxido nítrico sofre fotodissociação 
na atmosfera superior; 
 c.3) na estratosfera o óxido nítrico sofre oxidação 
pelo ozônio. 
 
178. 
Em um laboratório dedicado ao estudo da toxicidade de 
produtos químicos, foi estabelecido que: 
I) Para evitar danos à saúde, não se pode expor uma 
pessoa, por mais que oito horas, a uma atmosfera 
que contenha 10 ppm de HCN. 
II)A concentração letal de HCN no ar é de 300 mg/kg 
de ar (d=0,0012 g/cm3). 
Pergunta-se: 
a) Quantos miligramas de HCN por kg de ar 
corresponde a 10 ppm ? 
b) A que fração da dose letal corresponde 10 ppm ? 
c) Qual a massa de HCN que deve estar contida no ar, 
em um pequeno laboratório que mede 5m x 4m x 
2,2m, para atingir a concentração letal? 
 
179. 
Se a densidade de uma mistura de gases metano e 
propano, apresenta a mesma densidade que o gás 
etano, então, a proporção entre os volumes dos gases 
metano e propano nesta mistura é de: 
A) 1:1 B) 1:2 C) 1:3 D) 2:1 E) 3:1 
 
 
49 
 
 
 
APOSTILA 01 DE FÍSICO-QUÍMICA – PROF. PEDRO MADEIRA (2022) 
 
180. 
Dentre as amostras de gases citadas a seguir, todas 
ocupando o volume de 1 litro, assinale aquela que 
contém o maior número de átomos: 
A) Metano, a 300 K e 1 atm 
B) Neônio, a 273 K e 760 torr 
C) Oxigênio, a 27°C e 760 mmHg 
D) Monóxido de carbono, em CNTP 
E) Dióxido de carbono, a 0°C e 2 atm 
 
181. 
A maioria dos automóveis modernos é equipada com air 
bags que têm sido muito eficazes na redução de 
acidentes fatais no trânsito. Durante uma colisão frontal, 
o sensor do air bag envia um sinal elétrico que “detona” 
a rápida decomposição de um composto X, liberando 
uma grande quantidade de um certo gás. Dentro de um 
curto espaço de tempo o air bag infla, protegendo 
motorista e passageiros de um impacto frontal. 
1,0 g de amostra de X é decomposto liberando 507 mL 
(a 110 kPa e 18°C) de um gás A, que é um dos 
componentes do ar. A reação também produz um 
resíduo sólido. O tratamento deste resíduo com excesso 
de água produz 172 mL (em CNTP) de um gás B. 
a) Determine as fórmulas moleculares dos compostos 
A, B e X. Mostre os cálculos realizados para chegar 
à sua resposta. 
b) O composto X é esperado ser molecular ou iônico. 
c) Qual a estrutura de X no estado sólido? 
 
182. 
A análise química de um composto orgânico (um líquido 
incolor em CNTP) dá 14,40% de hidrogênio (por massa). 
Quando 0,870 g deste composto é queimado 
completamente na presença de oxigênio e os produtos 
da combustão são borbulhados através de um excesso 
de água de cal, são formados 6,20 g de um precipitado. 
A gravidade específica dos vapores do composto acima 
citado, em relação ao ar seco, é aproximadamente 2,5 
vezes a do eteno. 
a) Determine a fórmula empírica do composto orgânico 
sob investigação; 
b) Determine a fórmula molecular e desenhe todas as 
possíveis estruturas isoméricas para esse 
composto; 
c) Dê o nome IUPAC para um par de isômeros que 
pertença a diferentes classes; 
d) Que volume de ar (em CNTP) contendo 20,8% de 
oxigênio (por volume) é requerido para combustão 
de 10,4 g desse composto orgânico? 
 
183. 
11,2 dm3 (CNTP) de uma mistura de propano (C3H8) e 
butano (C4H10) foram queimadas na presença de 
excesso de oxigênio. Todo o dióxido de carbono obtido 
foi passado através de uma solução de NaOH, obtendo-
se 95,4 g de carbonato de sódio e 84 g de bicarbonato 
de sódio: 
a) Escreva as equações das reações de combustão do 
propano e do butano. 
b) Escreva as equações das reações de formação de 
carbonato e bicarbonato de sódio. 
c) Calcule a quantidade de CO2 desprendida na 
combustão da mistura de propano e butano. 
d) Calcule as massas de propano e butano na mistura. 
 
184. 
Um frasco de volume 5,00 L foi evacuado e 43,78g de 
N2O4 foram admitidos. Em –196oC, este composto é um 
sólido incolor. A amostra foi aquecida até 25oC e no 
processo, N2O4 se vaporiza e se dissocia parcialmente 
para formar o gás NO2. A pressão cresce lentamente e 
se estabiliza em 2,96 atm. 
(a) escreva uma equação para a reação. 
(b) se o gás que está no frasco fosse exclusivamente 
N2O4, qual seria a pressão? 
(c) se todo o gás que está no frasco fosse NO2, qual 
seria a pressão? 
(d) quais são as frações molares de N2O4 e NO2 quando 
a pressão se estabiliza em 2,96 atm? 
Prof. Pedro Madeira 
 
185. (IChO) 
a) Mostre com base na equação de van der Waals que 
i. em pressões suficientemente altas, Z > 1. Em altas 
temperaturas e baixas pressões, Z se aproxima do 
valor de um gás ideal. 
ii. em baixas temperaturas, Z pode ser menor do que 1. 
iii. para a = 0, Z aumenta linearmente com a pressão. 
 
b) Em uma certa temperatura, a variação de Z com P 
para o He e N2 é mostrada esquematicamente na 
seguinte figura. 
 
 a (bar L2 mol–2) b (L mol–1) 
He 3,46x10–2 2,38x10–2 
N2 1,37 3,87x10–2 
 
 
 Identifique o gráfico correspondente ao He e ao N2. 
 
c) Duas isotermas de van der Waals estão mostradas 
esquematicamente abaixo. Identifique aquela que 
corresponde a uma temperatura menor do que a 
temperatura crítica (Tc) do gás. 
 
 
 
 
 
 
 
 
 
50 
 
 
 
APOSTILA 01 DE FÍSICO-QUÍMICA – PROF. PEDRO MADEIRA (2022) 
 
 
 
d) Para uma dada pressão P, as três raízes da equação 
de van der Waals em V coincidem a uma certa 
temperatura T = Tc. Determine Tc em termos de a e 
b, e use o resultado para mostrar que N2 é liquefeito 
mais prontamente do que o He. 
 
Dica do professor: 
Derivando 1 vez (dp/dv) e igualando a zero, temos: 
 
*'9
(%9 − (,)!
=
C(-
%93
 
 
Derivando 2 vezes (d2p/dV2) e igualando a zero, temos: 
 
*'9
(%9 − (,)3
=
E(-
%9:
 
 
 
 
 
 
 
51 
 
 
 
APOSTILA 01 DE FÍSICO-QUÍMICA – PROF. PEDRO MADEIRA (2022) 
 
CAPÍTULO 01 – O ESTUDO DOS GASES 
GABARITO 
 
TÓPICO 01 
 
SEÇÃO VESTIBULARES 
 
1. OPÇÃO D 
 
2. n(NaN3) = 2,71 mol ou m(NaN3) = 176,15 g. 
 
3. OPÇÃO C 
 
4. t = 11 minutos. 
 
5. OPÇÃO D 
 
6. A) M = 131,5 g.mol–1 B) Xenônio. 
 
7. OPÇÃO D 
8. OPÇÃO C 
9. OPÇÃO A 
 
10. 
a) monofásico 
b) 0,90 atm; H2 < Ar < CO2 
 
11. OPÇÃO B 
12. OPÇÃO C 
13. OPÇÃO E 
14. OPÇÃO C 
 
15. 
A) X(CH4) = 0,80 ou 80%; X(C2H6) = 0,20 ou 20%B) 
P(CH4) = 68%; P(C2H6) = 32% C) V = 16,6 m3. 
 
16. A) e B) 
 
 
17. OPÇÃO D 
 
18. 
a) Curva que representa o melhor material para se 
armazenar o hidrogênio: curva B. 
 
Justificativa: 
O material A possui menor capacidade de 
armazenamento de H2, já os materiais B e C apresentam 
capacidades próximas, logo o material A é descartado. 
O material B apresenta uma menor distância entre as 
curvas de adsorção e dessorção, ou seja, neste caso a 
reversibilidade do processo de armazenamento e 
liberação de H2 é mais eficaz 
 
 
b) Uma possível desvantagem desta tecnologia 
alternativa estaria relacionada à massa ocupada pelo 
Mg2Ni. 
 
Conclusão: a porcentagem da massa do Mg2Ni é 9,25 
vezes maior do que a da gasolina, sendo isto uma 
enorme desvantagem. 
 
 
19. 
a) Íons OH – reagem com íons H+ da solução, 
deslocando o equilíbrio no sentido da dissolução do 
CO2. 
b) 40 kPa 
 
20. OPÇÃO D 
 
21. 
a) 113,83 kPa; b) 242,6g 
 
22. 
a) No frasco 1, existirá a menor quantidade de moléculas; b) 
P2/P3 = 10 
 
23. 
a) V(etileno) = 244 L; b) seria 224 L (o mesmo) 
 
24. 
a) CO2 + 2 LiOH à Li2CO3 + H2O; 
b) 2,4kg de LiOH 
 
25. 
a) 0,081 atm; b) 68378 kJ 
 
26. 
a) 3SO2 + 2HNO3 + 2H2O à 2NO + 3H2SO4 
b) 2,2g 
 
27. 
a) O2, He, N2; b) P(He) = 2160kPa; 
c) M = 22,9g/mol 
 
28. 
a) como a massa molar do metano é menor do que a 
massa molar média do ar, ele é menos denso e por isso 
ascende. 
P 
1 / V 
Aumento da 
temperatura 
P 
T 
Diminuição 
do volume 
tgq = nRT tga =nR/V 
52 
 
 
 
APOSTILA 01 DE FÍSICO-QUÍMICA – PROF. PEDRO MADEIRA (2022) 
 
b) Não seria possível obter a mistura com a composição 
acima mencionada pela simples mistura desses gases. 
O ar tem, aproximadamente, 21% em volume de gás 
oxigênio. Se o metano fosse misturado ao oxigênio, este 
apresentaria uma porcentagem menor que 21% na 
mistura. 
 
29. 
a) liberação de energia; b) V = 225 m3 
 
30. 
a) P(O2) = 2,2x104 Pa está dentro da faixa aceitável. 
b) 4,2% em volume 
 
31. 
a) 33,33 mL de água sanitária 
b) A combustão é mais rápida em oxigênio puro, pois no 
ar só há 21% de O2. 
 
32. 
a) CnH2n+2 + (1,5n+0,5) O2 à n CO2 + (n+1) H2O 
b) Nas mesmas condições de temperatura e pressão,volumes iguais de gases quaisquer contêm o mesmo 
número de moléculas (ou mols). 
c) propano e butano. 
 
33. 
a) 
Na2CO3(s)+H2SO4(aq) à H2O(l) + CO2(g) + Na2SO4(aq) 
b) Como a massa utilizada é a mesma e as substâncias 
possuem massas molares diferentes, o número de mol 
não é igual nos dois casos. Logo, a altura atingida pelo 
êmbolo não será a mesma, pois dependerá da 
quantidade de gás carbônico liberada. 
c) 
 
 
 
34. OPÇÃO A 
35. OPÇÃO B 
36. OPÇÃO B 
37. OPÇÃO B 
38. OPÇÃO D 
39. SOMA = 07 
 
40. M = 58 g/mol 
 
41. 
O ovo é empurrado para dentro da garrafa pela diferença 
de pressão existente. Quando a garrafa é aquecida, 
parte do gás escapa. Ao se retornar à temperatura 
original, a pressão é menor. 
 
 
42. OPÇÃO A 
43. SOMA = 25 
44. SOMA = 05 
45. OPÇÃO C 
46. OPÇÃO B 
47. SOMA = 34 
48. a) H2PO4– + HCO3– → H2O + CO2(g) + HPO42– 
b) 4,58 g 
 
49. OPÇÃO B 
 
 
PROF. PEDRO MADEIRA 
SEÇÃO ITA/IME 
 
50. OPÇÃO C 
51. OPÇÃO B 
52. OPÇÃO E 
53. OPÇÃO E 
54. OPÇÃO E 
55. QUESTÃO NULA 
56. OPÇÃO D 
57. OPÇÃO B 
58. OPÇÃO D 
59. OPÇÃO D 
60. OPÇÃO C 
61. OPÇÃO A 
62. OPÇÃO B 
63. OPÇÃO B 
64. OPÇÃO D 
65. OPÇÃO C 
66. OPÇÃO C 
67. OPÇÃO B 
68. OPÇÃO D 
69. OPÇÃO A 
70. OPÇÃO B 
71. OPÇÃO C 
 
72. 1 Zn(s) + 2 HCl(aq) à 1 ZnCl2(aq) + 1 H2O(l) 
 m(Zn) = 2,67 g 
 
73. OPÇÃO A 
 
74. V = 40,6 L 
 
75. OPÇÃO C 
76. OPÇÃO B (INCORRETA) 
77. OPÇÃO D 
 
78. d = 1,17 g.L–1. 
 
79. OPÇÃO D 
80. OPÇÃO D 
2
m R Tx
MM P r
´ ´
=
´ ´ ´π
53 
 
 
 
APOSTILA 01 DE FÍSICO-QUÍMICA – PROF. PEDRO MADEIRA (2022) 
 
81. OPÇÃO C 
 
82. P(Mg) = 65,5% 
 
83. A) V = 1,22 x 10–7 L B) m = 1,0 x 10–8 g 
 
84. OPÇÃO D 
85. OPÇÃO D 
 
86. 
CaHb+(a+b/4)O2+(4a+b)N2àaCO2+b/2H2O+(4a+b)N2 
 
87. X(C3H8) = 20% 
 
88. OPÇÃO C 
 
89. P = 120 atm 
 
90. OPÇÃO D 
91. OPÇÃO D 
 
92. P(O2) = 4,41 atm; P(N2) = 16,4 atm 
 
93. OPÇÃO D 
 
94. OPÇÃO D 
 
95. OPÇÃO D 
96. OPÇÃO D 
97. OPÇÃO E 
98. A) 16 u B) CH4 
 
99. d = 1,203 g.L–1 
 
100. t = 5 semanas. 
 
101. T = 250 K ou T = - 23oC 
 
102. V = 95,0 cm3 
 
103. (veja comentário) 
 
104. m(Na) = 2,56 g 
 
105. A) m(CO2) = 8,79 g B) m = 40 g 
 
106. V = 7,61 L 
 
107. Teor(Al) = 10,5% 
 
108. OPÇÃO A 
109. OPÇÃO E 
110. OPÇÃO A 
111. OPÇÃO D 
112. OPÇÃO E 
 
113. 
a) 2 HCl(conc) + Zn(s) à ZnCl2(aq) + H2(g) 
H2(g) + Br2(g) à 2 HBr(g) 
b) n(H2) = 0,015 mol 
 
114. V = 100 cm3. 
 
115. m = 454g deTNT 
 
116. p = 1,97 atm 
 
117. m(Mg) = 22g; m(Mg3N2) = 25g 
 
118. OPÇÃO C 
119. OPÇÃO B 
PROF. PEDRO MADEIRA 
TÓPICO 02 
 
SEÇÃO VESTIBULARES 
 
120. 
a) diminui a energia interna (pois diminui a temperatura). 
b) 0,91 atm 
 
121. OPÇÃO A 
 
 
122. SOMA 23 
123. SOMA 03 
124. OPÇÃO A 
125. OPÇÃO A 
126. OPÇÃO C 
127. OPÇÃO D 
128. 
a) NH3 pois possui menor massa molar 
b) o anel será formado em um tempo menor do que 10s 
c) Caso o algodão embebido de solução aquosa de NH3 
seja colocado no tubo um pouco antes do algodão que 
libera HCl(g) (e não simultaneamente) o anel de NH4Cl 
será formado a uma distância maior da extremidade do 
algodão embebido com NH3 dando a impressão de que 
a velocidade de difusão do HCl é menor do que a 
verdadeira. De acordo com a expressão matemática 
fornecida, quanto menor a velocidade de difusão, maior 
a massa molar. Consequentemente, a massa molar do 
HCl parecerá maior do que a verdadeira. 
PROF. PEDRO MADEIRA 
SEÇÃO ITA / IME 
 
129. OPÇÃO B 
130. OPÇÃO A 
131. OPÇÃO E 
132. OPÇÃO B 
 
( )-
N
A
N 0
A B
VP = P p + p
V +V
æ ö
ç ÷
è ø
54 
 
 
 
APOSTILA 01 DE FÍSICO-QUÍMICA – PROF. PEDRO MADEIRA (2022) 
 
133. 
NH3(g) + HCl(g) à NH4Cl(s); 
v(NH3) > v(HCl) 
 
H(NP3)
H(P5Q) = R
;(P5Q)
;(NP3)
≈ 2, IS 
 (Lei de Graham) 
 
134. OPÇÃO B 
135. OPÇÃO D 
136. OPÇÃO E 
137. OPÇÃO E 
 
PROF. PEDRO MADEIRA 
TÓPICO 03 
 
SEÇÃO VESTIBULARES 
 
138. OPÇÃO D 
PROF. PEDRO MADEIRA 
 
SEÇÃO ITA / IME 
 
139. OPÇÃO A 
140. OPÇÃO C 
141. OPÇÃO C 
142. OPÇÃO A 
143. 
 
Pontos A, B e C: início das liquefações em cada isoterma 
 
Pontos X, Y e Z: final das liquefações em cada isoterma 
 
144. OPÇÃO E 
 
145. Erro absoluto = - 1,2 atm; Erro relativo = - 5,1 % 
 
146. OPÇÃO A 
 
147. a) 10 kJ; b) 600K; Pi = 0,9 atm e PF = 1,9 atm. 
 
148. 
è Tendo em mente a equação do gás ideal, pV = nRT, 
pode-se incluir as 2 correções propostas por van der 
Waals. 
 
è A correção no volume consiste em subtrair do volume 
total do recipiente, o volume ocupado pelas moléculas 
do gás: 
%; = %− ,′4 
na qual Vb é o volume corrigido e b’ é uma constante 
característica de cada gás. 
 
è A pressão deve ser corrigida por um fator 
proporcional ao quadrado da concentração, de acordo 
com a diminuição da intensidade das colisões nas 
paredes do recipiente: 
= = =% − -.
(
%/
!
 
⟹=% = =+ -.
(
%/
!
 
na qual “-” é uma constante característica de cada gás, 
P é a pressão do gás e Pa é a pressão original do gás 
antes da correção, a que deve ser aplicada na equação 
do gás ideal. 
 
è Logo, aplicando na equação do gás ideal: 
U= + -.
(
%/
!
V (% − ,′4) = (*' 
ou 
= =
(*'
% − (, − -.
(
%/
!
 
 
Na qual “-” e “,” são constantes características de cada 
gás. 
PROF. PEDRO MADEIRA 
TÓPICO 04 
SEÇÃO VESTIBULARES 
 
149. OPÇÃO A 
150. OPÇÃO B 
151. 
a) 0,16g; b) KClO3(s) à KCl(s) + 3/2 O2(g) 
 
152. OPÇÃO A 
 
153. 
a) A = NH3; B = CH4; C = CO2; D = SO2; E = CO 
b) Ca(OH)2 + CO2 CaCO3↓ + H2O 
Ca(OH)2 + SO2 CaSO3↓ + H2O 
c) Teremos uma reação de oxi-redução. Nesta 
transformação o Nox do manganês Mn (+7) diminui. 
Concluímos que ocorre descoramento da solução. 
 
154. OPÇÃO D 
PROF. PEDRO MADEIRA 
SEÇÃO ITA / IME 
 
155. OPÇÃO D 
156. OPÇÃO A 
157. OPÇÃO E 
158. OPÇÃO D 
159. OPÇÃO A 
 
®
®
55 
 
 
 
APOSTILA 01 DE FÍSICO-QUÍMICA – PROF. PEDRO MADEIRA (2022) 
 
160. 
A nitroglicerina é uma substância líquida, incolor e 
oleosa, mas também altamente instável e explosiva. 
Basta um simples toque para que ocorra uma detonação 
espontânea. Como a molécula contém átomos de 
oxigênio, hidrogênio e carbono, quando ela explode 
libera novas moléculas (como O2, N2, H2 e CO2), mais 
estáveis, e com uma grande energia cinética, havendo 
uma conversão entre a energia de ligação para energia 
cinética. 
O poder dos explosivos deve-se ao fato de que o volume 
ocupado por um gás é muito maior do que o ocupado 
por um sólido ou por um líquido. 
A reação de decomposição da nitroglicerina é: 
4 C3H5(ONO2)3(!) à 6N2(g) + 12CO2(g) + 10 H2O(g) 
 + 1 O2(g) 
161. OPÇÃO D 
PROF. PEDRO MADEIRA 
 
QUESTÕES EXTRAS: OLIMPÍADA 
 
162. OPÇÃO A 
163. OPÇÃO D 
164. OPÇÃO D 
165. OPÇÃO A 
166. OPÇÃO B 
167. OPÇÃO A 
168. OPÇÃO D 
169. OPÇÃO A 
170. OPÇÃO B 
171. OPÇÃO C 
172. OPÇÃO B 
173. OPÇÃO C 
174. OPÇÃO B 
175. OPÇÃO C 
 
176. 
PARTE I 
A) M = 34 g.mol–1 
B) Frasco B (o dobro è Princípio de Avogadro) 
C) mesma Ec (mesma Temperatura è Teoria cinética 
dos gases ideiais) 
D) v(O2) > v(H2S); 
177. 
A) P(NO2) = 2,058 x 10–8 atm 
B) N = 2,39 x 1019 moléculas 
C) c.1) 2 NO2(g) + H2O(l) à HNO2(aq) + HNO3(aq) 
c.2) NO +hn à NO+ + e– 
c.3) NO + O3 à NO2 + O2 
 NO+ + e– à N + O 
 NO + hn à N + O 
PROF. PEDRO MADEIRA 
QUESTÃO COMENTADA 
103. (IME – 2004) RESOLUÇÃO FORMAL: 
i) Antes da abertura da válvula 1: 
 
Após N = 1: 
 
 
 
Após N = 2: 
 
 
 
Após N = N: 
 
ii) Chama-se e 
iii) OBSERVE A SOMA: 
 
iv) DIVINDO-SE A 1a EQUAÇÃO POR K, A 2a por K2, 
A N-ésima por KN: 
 
 
........eq.01
 
 
 
2 2
2 2
v(O ) M(H S) = = 1,03
v(H S) M(O )
RT
pV
n
RT
VP
n B
B
A
A == ;0
0,
( ) BA
B
BA
A
BA
BA
VV
pV
VV
VP
P
VV
RT
RT
pV
RT
VP
P
+
+
+
=Þ
+÷÷
ø
ö
çç
è
æ
+= 0
1
0
1
( ) BA
B
BA
A
BA
BA
VV
pV
VV
VP
P
VV
RT
RT
pV
RT
VP
P
+
+
+
=Þ
+÷÷
ø
ö
çç
è
æ
+= 1
2
1
2
RT
pV
n
RT
VP
n B
B
A
A == ;1
1,
BA
B
BA
AN
N VV
pV
VV
VP
P
+
+
+
= -1
K
VV
V
BA
A =
+
C
VV
pV
BA
B =
+
1 0
2 1
1 2
1
.
.
.
.
N N
N N
P P K C
P P K C
P P K C
P P K C
- -
-
= +
+
= +
+
+
= +
+
= +
!
1
0
2 1
2 2
1 2
1 2 1
1
1
N N
N N N
N N
N N N
P CP
K KP P C
KK K
P P C
K K K
P P C
K K K
- -
- - -
-
-
= +
+
= +
+
+
= +
+
= +
!
÷
ø
ö
ç
è
æ +++++=
- NNN
N
KKKK
CP
K
P 1111
120 !
56 
 
 
 
APOSTILA 01 DE FÍSICO-QUÍMICA – PROF. PEDRO MADEIRA (2022) 
 
 
v) Obs: Soma de uma PG de razão q = 1/K 
 e 1º termo a1 = 1/K: 
eq.02 
 
vi) SUBSTITUINDO A EQ.02 NA EQ.01 
 
 
 
 
 
 
PROF. PEDRO MADEIRA 
 
 
“Quem não perserverar nas pequenas tarefas, 
falhará nos grandes planos.” 
Provérbio Chinês 
 
 
 
 
 
( )1
1
11
1.11
1111
12 -
-
=
-
-
=÷
ø
ö
ç
è
æ ++++
- KK
K
K
KKK
KKKK N
NN
NN
!
( ) 1
1
1
1
00 -
-
+=Þ
-
-
+=
K
KCPKP
KK
KCP
K
P N
N
NN
N
N
N
Þ
-
+
-
´
+
+=Þ
1
1
0
BA
A
N
BA
BN
N
VV
V
K
VV
pV
PKP
BA
B
N
BA
BN
N
VV
V
K
VV
pV
PKP
+
-
-
´
+
+=
1
0
( )10 -´-=Þ NN
N KpPKP ( ) N
N KpPpP -+=Þ 0
( )
N
BA
A
N VV
V
pPpP ÷÷
ø
ö
çç
è
æ
+
´-+= 0
57 
 
 
 
APOSTILA 01 DE FÍSICO-QUÍMICA – PROF. PEDRO MADEIRA (2022) 
 
 
 
 
 
 
CAPÍTULO 02 – TERMODINÂMICA QUÍMICA 
 
CONTEÚDO 
 
TÓPICO 01: CONCEITOS FUNDAMENTAIS 
TÓPICO 02: PRIMEIRA LEI DA TERMODINÂMICA 
TÓPICO 03: SEGUNDA E TERCEIRA LEIS DA TERMODINÂMICA 
TÓPICO 04: FUNDAMENTOS DE TERMOQUÍMICA 
 
TÓPICO 01: CONCEITOS FUNDAMENTAIS 
 
A) Sistema x vizinhança 
O sistema é a porção do universo que se tem como 
objeto de análise. A vizinhança é tudo aquilo que não é 
o sistema. Eles são separados pela fronteira que é 
classificada de acordo com a capacidade de 
transferência de calor. 
PROF. PEDRO MADEIRA 
B) Tipos de Fronteira 
 
 
 
(a) permite a passagem de calor = diatérmica 
(b) não permite a troca de calor = adiabática 
PROF. PEDRO MADEIRA 
C) Tipos de Sistema 
 
 
 
(a) permite a troca de matéria e energia. Ex: garrafa de 
refrigerante aberta. 
(b) só permite a troca de energia. Ex: garrafa de 
refrigerante fechada. 
(c) não permite a troca nem de matéria nem de energia. 
Ex: frasco de Dewar. 
RESOLVA AGORA 01 (IChO PP 2012) 
Na natureza, os elementos radioativos de longa vida (Th 
e U) dão origem a isótopos radioativos de vida curta. Se 
o decaimento nuclear ocorrer em um sistema fechado, 
as atividades dos núcleos filhos se tornam iguais à 
atividade do núcleo-mãe. Isto indica que outros 
processos além do decaimento radioativo afetam a 
abundância dos núcleos-filho. 
Na água de um lago, a radioatividade do 222Rn dissolvido 
(t1/2 = 3,8 dias) é de 4,2 átomos/min.(100L). Todos os 
222Rn são produzidos a partir do 226Ra dissolvido (t1/2 = 
1600 anos), o qual tem uma atividade de 6,7 
átomos.min–1.(100L)–1. Isto implica que parte do 222Rn 
produzido está sendo perdido a partir do lago por um 
processo desconhecido. 
Baseado no conceito de sistema, explique porque a 
atividade do núcleo-filho não é igual à atividade do 
núcleo-mãe. 
 
 
 
 
 
 
 
 
 
 
 
 
 
 
 
 
 
 
 
 
 
 
PROF. PEDRO MADEIRA 
58 
 
 
 
APOSTILA 01 DE FÍSICO-QUÍMICA – PROF. PEDRO MADEIRA (2022) 
 
D) Processo endotérmico x Processo 
exotérmico 
 
 
 
 
 
 
 
 
 
 
 
 
 
 
 
 
 
 
 
 
 
 
 
 
 
OBSERVAÇÕES 
 
 
 
 
 
 
 
 
 
 
 
 
 
 
 
 
 
 
 
 
 
 
 
 
 
 
 
 
 
 
 
 
 
 
 
 
 
 
 
 
 
 
 
RESOLVA AGORA 02 (IME 2009) 
Foram introduzidos 10 mols de uma substância X no 
interior de um conjunto cilindro-pistão adiabático, sujeito 
a uma pressão constante de 1atm. X reage espontânea 
e irreversivelmente segundo a reação: 
 X(s) à 2 Y(g) DH = – 200 cal 
Considere que a temperatura no início da reação é 300 
K e que as capacidades caloríficas molares das 
substâncias X e Y são constantes e iguais a 5,0 cal.mol–
1.K–1 e 1,0 cal.mol–1.K–1, respectivamente. O volume final 
do conjunto cilindro-pistão é (Dado: R = 0,082 
atm.L.mol–1.K–1) 
A ( ) 410,0 L 
B ( ) 492,0 L 
C ( ) 508,4 L 
D ( ) 656,0 L 
E ( ) 820,0 L 
EXO
 tempo
ENDO
 tempo
59 
 
 
 
APOSTILA 01 DE FÍSICO-QUÍMICA – PROF. PEDRO MADEIRA (2022) 
 
 
 
 
 
 
 
 
 
 
 
 
 
 
 
 
 
 
 
 
 
 
RESOLVA AGORA 03 (IME 2010) 
A transformação isovolumétrica de um gás triatômico 
hipotético A3 em outro diatômico A2 envolve a liberação 
de 54 kJ/mol de A3. A capacidade calorífica molar a 
volume constante do gás A2 é de 30 J/mol.K. Após a 
transformação isocórica de todo A3 em A2, determine o 
aumento percentual de pressão em um recipiente 
isolado contendo o gás A3 a 27ºC. Considere que a 
capacidade calorífica molar a volume constante do gás 
A2 não varia com a temperatura e que os gases se 
comportam idealmente. 
 
 
 
 
 
 
 
 
 
 
 
 
 
 
 
 
 
 
 
 
 
 
 
 
PROF. PEDRO MADEIRA 
 
RESOLVA AGORA 04 (IME 2021 – Q08) 
Um motor de 6 cilindros e volume total de 5.700 cm3, 
utilizado em viaturas leves e blindadas, consome 0,5 g 
do combustível gasoso de composição média C8H18, em 
cada cilindro, por segundo de operação. 
 
 
 
 
 
 
 
Considerações: 
 
• o ciclo termodinâmico do motor compreende o 
funcionamento em 4 tempos: admissão, compra 
assim combustão e exaustão (escape); 
• o motor executa 10 ciclos por segundo, ou seja, a 
mistura de ar e combustível enche o cilindros e 
depois é comprimida 10 vezes por segundo; 
• 20,0% da quantidade de combustível sofre 
combustão incompleta, sendo convertida em CO(g); 
• 80,0% da quantidade de combustível sofre 
combustão completa, sendo convertida em CO2(g); 
• a mistura de ar e combustível comporta-se como gás 
ideal; 
• as capacidades caloríficas molares são 
independentes da temperatura; e 
• as entalpias de formação a 25ºC. 
 
Determine: 
a) a vazão da entrada de ar no motor, em m3/s; e 
b) a composição percentual molar dos produtos e a 
temperatura de combustão, em K. 
 
FOLHA DE DADOS 
• Composição percentual do ar atmosférico = 79,0% de N2(g) 
e 21,0% de O2(g) 
 
Entalpias-padrão de formação a 25ºC: 
Substância 
Química C8H18(g) H2O(g) H2O(l) CO2(g) CO(g) 
DHf
o 
(kJ.mol–1) -208,45 -241,82 -285,83 -393,51 -110,53 
 
Capacidade calorífica molar: 
Substância 
Química N2(g) O2(g) H2O(g) CO2(g) CO(g) 
"#! 
(J.K–1 .mol–1) 29,13 29,36 33,58 37,11 29,14 
 
 
 
 
 
 
 
 
ar e combustível motor 6 cilindros produtos 
trabalho 
60 
 
 
 
APOSTILA 01 DE FÍSICO-QUÍMICA – PROF. PEDRO MADEIRA (2022) 
 
 
 
 
 
 
 
 
 
 
 
 
 
 
 
 
 
 
 
 
 
 
 
 
 
 
 
 
 
 
 
 
 
 
 
 
 
 
 
 
 
 
 
 
 
 
 
 
 
 
 
 
 
 
 
 
 
 
 
 
 
 
 
E) Trabalho, calor e energia 
• Fazer trabalho é equivalente a elevar um peso em 
algum lugar nas vizinhanças. 
• A energia de um sistema é a sua capacidade de 
efetuar trabalho. 
• Calor à gradiente de temperatura. 
• Interpretação molecular: calor x trabalho 
 
 
PROF. PEDRO MADEIRA 
OBSERVAÇÕES 
 
 
 
 
 
 
 
 
F) Funções de estado x funções de linha 
(caminho) 
 
Energia interna, entalpia, entropia, energias livre (Gibbs 
e Helmholtz) = dependem só do estado. 
 
Calor e trabalho = dependem do caminho. 
PROF. PEDRO MADEIRA 
OBSERVAÇÕES 
 
 
 
 
 
 
 
 
 
 
 
61 
 
 
 
APOSTILA 01 DE FÍSICO-QUÍMICA – PROF. PEDRO MADEIRA (2022) 
 
RESOLVA AGORA 05 (ITA 2022 – Q67) 
Considere as seguintes afirmações sobre processos 
termodinâmicos, que podem ocorrer em uma ou mais 
etapas, em que DT se refere à variação de temperatura 
entre os estados inicial e final: 
I. Um processo termodinâmico é definido pelo estado 
final e estado inicial do sistema. 
II. DT é sempre nula em um processo isotérmico. 
III. A troca de calor envolvida em um processo 
isotérmico deve ser nula (q=0). 
IV. Todo processo em que DT = 0 é um processo 
isotérmico. 
V. DT = 0 para todo processo em sistema isolado. 
 
Assinale a opção que contém as afirmações 
ERRADAS: 
A ( ) Apenas I, II e IV 
B ( ) Apenas I, III, IV e V 
C ( ) Apenas I, III e V 
D ( ) Apenas II e IV 
E ( ) III e V 
 
G) A Distribuição de Boltzmann e a 
constante de Boltzmann 
 
 
 
 
 
 
OBSERVAÇÕES 
 
 
 
 
 
 
 
 
 
 
 
 
 
 
 
 
 
 
 
 
 
 
 
 
 
 
 
 
 
 
 
 
 
 
 
 
 
PROF. PEDRO MADEIRA 
• Distribuição de Maxwell-Boltzmann 
(velocidades gasosas). 
• Distribuição barométrica (distribuição 
gravimétrica). 
• Equação de Arrhenius. 
 
23 1; 1,38 10 . ;
i
i
E kT
i
AE kT
i
N e k J K R N k
N e
-
--
-= = × =
å
62 
 
 
 
APOSTILA 01 DE FÍSICO-QUÍMICA – PROF. PEDRO MADEIRA (2022) 
 
TÓPICO 02: 
PRIMEIRA LEI DA TERMODINÂMICA 
 
A) Energia Interna 
• Energia Interna (U): soma das energias cinética e 
potencial que compõem o sistema. 
• U é função de estado e propriedade extensiva. 
• Unidades de energia: J, eV, cal. 
 
OBSERVAÇÕES 
 
 
 
 
 
 
 
 
 
 
 
 
 
 
 
 
 
 
 
 
 
 
 
 
 
 
 
 
 
 
 
 
 
 
 
 
 
 
 
 
 
 
 
 
 
 
 
 
 
PROF. PEDRO MADEIRA 
B) Teorema da Equipartição de Energia 
(Mecânica clássica) 
• Se então Ek = 3/2 kT (gás 
monoatômico). 
• Na prática, o teorema se aplica somente para 
translação e rotação moleculares. 
• Um = Um(0) + 3/2 RT. 
• Uma vez que Ep (interação) = 0, então a energia 
interna de um gás perfeito não depende do volume. 
 
OBSERVAÇÕES 
 
 
 
 
 
 
 
 
 
 
 
 
 
 
 
 
 
 
 
 
 
 
 
 
 
 
 
 
 
 
 
 
 
 
 
 
 
 
 
 
 
 
 
 
 
 
PROF. PEDRO MADEIRA 
( )2 2 2
x y z
1E = m v +v +v ,k 2
63 
 
 
 
APOSTILA 01 DE FÍSICO-QUÍMICA – PROF. PEDRO MADEIRA (2022) 
 
C) A conservação da energia 
• A energia interna de um sistema isolado é constante. 
• Até hoje não se conseguiu se construir um “moto-
perpétuo de primeira espécie”. 
• PLT: DU = q + w (convenção aquisitiva) 
 
OBSERVAÇÕES 
 
 
 
 
 
 
 
 
 
 
 
 
 
 
 
 
 
 
 
 
 
 
 
 
 
 
PROF. PEDRO MADEIRA 
D) Trabalho de expansão volumétrica 
 
 
 
 
 
 
 
 
 
 
 
 
 
 
OBSERVAÇÕES 
 
 
 
 
 
 
 
 
 
 
 
 
 
 
 
 
 
RESOLVA AGORA 06 (ITA 2008) 
Dois cilindros (I e II) são providos de pistões, cujas 
massas são desprezíveis e se deslocam sem atrito. Um 
mol de um gás ideal é confinado em cada um dos 
cilindros I e II. São realizados, posteriormente, dois tipos 
de expansão, descritos a seguir: 
 
a) No cilindro I, é realizada uma expansão isotérmica à 
temperatura T, de um volume V até um volume 2V, 
contra uma pressão externa constante P. 
b) No cilindro II, é realizada uma expansão adiabática, 
de um volume V até um volume 2V, contra uma 
pressão externa constante P. 
 
Determine os módulos das seguintes 
grandezas: variação da energia interna, 
calor trocado e trabalho realizado para os 
dois tipos de expansão. 
 
 
 
 
 
 
 
PROF. PEDRO MADEIRA 
CASOS ESPECÍFICOS DE EXPANSÃO 
 
• Expansão Livre (pex = 0): 
 w = 0 
 
• Expansão contra pressão constante: 
 w = – pex.DV 
 
• Expansão reversível (pex = p): 
 
Isotérmica: w = – nRT ln(VF/Vo) 
Adiabática: w = – n.Cv.DT 
 
 
64 
 
 
 
APOSTILA 01 DE FÍSICO-QUÍMICA – PROF. PEDRO MADEIRA (2022) 
 
E) Expansão Reversível x Expansão Irreversível 
Observe os diagramas de James-Watt: 
 
 
 
OBSERVAÇÕES 
 
1º CASO: 
 
 
 
 
 
 
2º CASO: 
 
 
 
 
 
 
 
3º CASO: 
 
 
 
65 
 
 
 
APOSTILA 01 DE FÍSICO-QUÍMICA – PROF. PEDRO MADEIRA (2022) 
 
F) Entalpia 
• H = U + pV (por definição) è DH = qp 
 (p constante, sem trabalho extra). 
• H é função de estado e propriedade extensiva. 
 
 
OBSERVAÇÕES 
 
 
 
 
 
 
 
 
 
 
 
 
 
 
 
 
 
 
 
PROF. PEDRO MADEIRA 
G) Calorimetria 
 
 
 
 
 
 
 
 
 
 
 
 
 
 
 
 
 
 
 
 
 
 
 
 
 
 
 
 
 
 
 
 
 
 
 
 
RESOLVA AGORA 07 (IME 2009) 
O valor experimental para o calor liberado na queima de 
benzeno líquido a 25oC, com formação de dióxido de 
carbono e água líquida, é 780 kcal/mol. A combustão é 
feita em uma bomba calorimétrica a volume constante. 
Considerando comportamento ideal para os gases 
formados e R = 2,0 cal/mol.K, determine: 
a) o calor padrão de combustão do benzeno a 25oC; 
b) se o calor calculado no item anterior é maior ou 
menor quando a água é formada no estado gasoso. 
Justifique sua resposta. 
 
 
 
 
 
 
 
 
 
 
 
 
 
 
PROF. PEDRO MADEIRA 
H) Capacidades Caloríficas 
Capacidade calorífica a volume constante, Cv 
• Cv é propriedade extensiva. 
• Cv,m é propriedade intensiva. 
 Ex.: Cv,m = 12,47 J K–1mol–1 (gases monoatômicos) 
• OBS: capacidade calorífica específica. 
 Ex.: c = 4 J K–1g–1 (H2O) 
• OBS: capacidade calorífica infinita = transição de 
fases. 
• dU = CV dT à DU = CV DT à qV = CV DT (intervalo 
pequeno de temperatura à CV constante). 
BOMBA CALORIMÉTRICA 
(V = constante) 
 
• dU = dq + dwexp + dwe 
• dU = dq (V constante, sem we) 
 DU = qV 
• Calorimetria 
Bomba calorimétrica (adiabática) 
q = CDT; C = constante do calorímetro. 
 
CALORÍMETRO 
ISOBÁRICO 
• Calorímetro de chama 
adiabático 
(combustão). 
• Para reações 
envolvendo sólidos e 
líquidos, DU é 
praticamente igual ao 
DH. 
• Para reações 
envolvendo gases: 
• DH = DU + PDV = DU 
+ DngasesRT 
 
66 
 
 
 
APOSTILA 01 DE FÍSICO-QUÍMICA – PROF. PEDRO MADEIRA (2022) 
 
• Gráficos típicos. (Em geral, quanto menor a 
temperatura, menor Cv): 
 
 
Capacidade calorífica a pressão constante, CP 
• dH = CP dT à DH = CP DT à qP = CP DT (intervalo 
pequeno de temperatura à CP constante). 
• Cp,m = a + bT + cT–2 (a,b,c são parâmetros 
empíricos). 
• devido ao trabalho de expansão, o aquecimento é 
menor a pressão constante do que a volume 
constante. 
CP > CV 
Relação de Meyer: 
CP – CV = nR (gás perfeito) 
 
 
 
 
OBSERVAÇÕES 
 
 
 
 
 
 
 
 
 
 
 
 
 
 
 
 
 
 
 
 
 
 
 
 
 
 
 
 
 
 
 
 
 
 
 
 
 
 
RESOLVA AGORA 08 (ITA 2009) 
Nos gráficos abaixo, cada eixo representa uma 
propriedade termodinâmica de um gás que se comporta 
idealmente. Com relação a estes gráficos, é CORRETO 
afirmar que 
 
A ( ) I pode representar a curva de pressão versus 
volume. 
B ( ) II pode representar a curva de pressão versus 
inverso do volume. 
C ( ) II pode representar a curva de capacidade 
calorífica versus temperatura. 
D ( ) III pode representar a curva de energia interna 
versus temperatura. 
E ( ) III pode representar a curva de entalpia versus o 
produto da pressão pelo volume. 
PROF. PEDRO MADEIRA 
P
P
HC =
T
¶æ ö
ç ÷¶è ø
V
V
UC =
T
¶æ ö
ç ÷¶è ø
67 
 
 
 
APOSTILA 01 DE FÍSICO-QUÍMICA – PROF. PEDRO MADEIRA (2022) 
 
 
 
 
 
 
 
 
 
 
 
 
 
 
I) Transformação Adiabática / Politrópica 
• q + w = DU à DU = wad 
• Como U é função de estado, DU = CV DT 
• wad = CV DT 
• Outras relações: 
 e 
 
• Gráfico T x V para a transformação adiabática: 
 
• Adiabática x Isoterma: 
 
 
Prof. Pedro Madeira 
 
RESOLVA AGORA 09 
Deduza a expressão para o trabalho de uma expansão 
adiabática reversível de um gás com coeficiente de 
Poisson igual a g. O volume e a pressão inicial são Vo e 
Po e o volume e a pressão final é VF e PF. 
 
 
 
 
 
 
 
 
 
 
 
 
 
OBSERVAÇÕES: 
 
 
 
 
 
 
 
 
 
 
 
 
 
 
 
 
 
 
 
 
 
 
 
 
 
 
 
 
 
 
 
 
 
 
 
 
 
 
 
 
 
VC R
oF
o F
VT =
T V
æ ö
ç ÷
è ø
o o F FP V =P Vg g
68 
 
 
 
APOSTILA 01 DE FÍSICO-QUÍMICA – PROF. PEDRO MADEIRA (2022) 
 
TÓPICO 03 – SEGUNDA E TERCEIRA LEIS 
DA TERMODINÂMICA 
 
A) O Enunciado de Kelvin 
“Não é possível um processo que tenha como único 
resultado a absorção de calor de um reservatório térmico 
e a sua completa conversão em trabalho” 
OBSERVAÇÕES 
 
 
 
 
 
 
 
 
 
 
 
 
 
 
 
 
 
 
 
 
 
 
 
 
 
 
 
 
 
 
 
PROF. PEDRO MADEIRA 
B) O sentido da transformação espontânea 
• Observações experimentais: 
à Uma bola sobre uma superfície quente jamais 
principia, espontaneamente, a pular sobre ela. 
à Um gás não se contrai espontaneamente. 
à Um corpo não fica espontaneamente mais quente 
do que as suas vizinhanças. 
• Justificativa: As mudanças espontâneas são sempre 
acompanhadas pela dispersão mais desordenada da 
energia. A localização de movimentos caóticos num 
único movimento ordenado é extremamente 
IMPROVÁVEL. 
PROF. PEDRO MADEIRA 
C) Entropia 
• A primeira lei aproveita-se da energia interna para 
identificar as mudanças permitidas; a segunda lei usa 
a entropia para identificar as mudanças espontâneas 
entre as mudanças permitidas. 
• A entropia de um sistema isolado aumenta numa 
mudança espontânea: DStot = DSsis + DSviz ≥ 0 
 
Definição Termodinâmica: 
• O calor proporciona um aumento do movimento 
caótico das partículas das vizinhanças do sistema. 
 
• A vizinhança pode ser encarada como um 
reservatório a volume constante (dqviz = dUviz)ou a 
pressão constante (dqviz = dHviz). Em ambos os 
casos, uma função de estado. Logo, 
 
 
 
 
OBSERVAÇÕES 
 
 
 
 
 
 
 
 
 
 
 
 
 
 
 
 
 
 
 
 
 
 
 
 
 
 
 
 
 
 
 
 
 
 
 
 
 
 
 
 
 
 
 
 
 
PROF. PEDRO MADEIRA 
ò=DÞ=
f
i
revrev
T
dqS
T
dqdS
viz viz
viz viz
viz viz
dq q
dS S
T T
= ÞD =
69 
 
 
 
APOSTILA 01 DE FÍSICO-QUÍMICA – PROF. PEDRO MADEIRA (2022) 
 
D) A entropia como função de estado 
Ciclo de Carnot Reversível: 
1) Expansão isotérmica è DS = qh/Th 
2) Expansão adiabática è DS = 0 
3) Compressão isotérmica è DS = qc/Tc 
4) Compressão adiabática è DS = 0 
DSciclo = 0 
 
 
• A eficiência do ciclo de Carnot: 
 e = |w| / qQ 
 e = 1 + (qF / qQ) 
 e = 1 – (TF / TQ) 
Conclusão da Segunda Lei: todas as máquinas 
reversíveis têm a mesma eficiência, qualquer que seja o 
seu modo de operar. 
 
 
OBSERVAÇÕES 
 
 
 
 
 
 
 
 
 
 
 
 
 
 
 
 
 
 
 
 
 
 
 
 
 
 
 
 
 
 
 
 
 
 
 
 
 
 
 
 
 
 
RESOLVA AGORA 10 (ITA 2010) 
Uma maquina térmica opera segundo o ciclo JKLMJ 
mostrado no diagrama T-S da figura. 
 
Pode-se afirmar que 
A ( ) o processo JK corresponde a uma compressão 
isotérmica. 
B ( ) o trabalho realizado pela maquina em um ciclo é 
W = (T2 – T1)(S2 – S1). 
C ( ) o rendimento da maquina e dado por h = 1 – 
(T2/T1). 
D ( ) durante o processo LM uma quantidade de calor 
QLM = T1(S2 – S1) é absorvida pelo sistema. 
E ( ) outra maquina térmica que opere entre T2 e T1 
poderia eventualmente possuir um rendimento 
maior que a desta. 
PROF. PEDRO MADEIRA 
 
70 
 
 
 
APOSTILA 01 DE FÍSICO-QUÍMICA – PROF. PEDRO MADEIRA (2022) 
 
E) A Temperatura termodinâmica 
• T = (1 – e)Th 
• O Zero da escala ocorre quando a eficiência de uma 
máquina de Carnot é 1. (Definido por Kelvin) 
• Atribui-se ao ponto triplo da água o valor de 273,16 
K. Então, se uma máquina tem sua fonte quente no 
ponto triplo da água, a temperatura do sumidouro frio 
é encontrada pela determinação da eficiência da 
máquina. 
OBSERVAÇÕES 
 
 
 
 
 
 
 
 
 
 
 
 
 
 
 
 
PROF. PEDRO MADEIRA 
F) Variação de entropia de alguns 
processos 
• Transição de fase na temperatura de transição: 
 DtrsS = DtrsH / Ttrs 
 
OBS.: Regra de Trouton (empírica) = muitos líquidos têm 
DvapSo » 85 J/K.mol 
OBS2: DfusSo (metais) = R 
OBSERVAÇÕES: 
 
 
 
 
 
 
 
 
 
 
 
 
 
 
 
 
 
 
 
 
 
 
 
 
 
 
 
 
 
 
 
 
 
• Expansão isotérmica de um gás perfeito 
 DS = nR ln(VF / Vo) 
 
 
• Variação da entropia com a temperatura 
 
 
 
 
 
 
 
 
 
 
 
 
 
 
 
 
 
( ) ( ) ( ) ( )F F
o o
T T prev
F o F oT T
C dTdqS T = S T + S T = S T + 
T T
Þò ò
71 
 
 
 
APOSTILA 01 DE FÍSICO-QUÍMICA – PROF. PEDRO MADEIRA (2022) 
 
 
 
 
 
 
 
 
 
 
 
 
• Variação da entropia entre dois pontos: 
 
 
 
 
 
 
 
 
 
 
 
 
 
 
 
 
 
 
 
 
 
 
 
• A medição da entropia das substâncias 
 
 
 
 
 
 
 
No gráfico de CP/T versus T, as áreas hachuradas 
correspondem às variações de entropia de aquecimento 
da respectiva fase de agregação: 
 
 
 
Prof. Pedro Madeira 
OBSERVAÇÕES 
 
 
 
 
 
 
 
 
 
 
 
 
 
 
 
 
 
 
 
 
 
 
 
 
 
 
 
 
 
 
 
 
 
 
 
 
 
PROF. PEDRO MADEIRA 
f
eb
f eb
T p fus
0 f
T Tp peb
T Teb
C (s)dT Δ HS(T) = S(0) + + + 
T T
C (l)dT C (g)dTΔ H + + 
T T T
+
ò
ò ò
72 
 
 
 
APOSTILA 01 DE FÍSICO-QUÍMICA – PROF. PEDRO MADEIRA (2022) 
 
G) Terceira lei da Termodinâmica 
• O teorema do calor de Nernst: 
“A variação de entropia de qualquer transformação física 
ou química tende a zero quando a temperatura tende a 
zero: DS à 0 quando T à 0, admitindo-se que todas as 
substâncias envolvidas estão ordenadas perfeitamente” 
 
Exemplo: S(a) à S(b) 
DtrsS (369K) = (402 J.mol–1) / 369 K = 1,09 J.K–1.mol–1 
S(a,369K) = S(a,0K) + 37 J.K–1 .mol–1 (pelo Cp) 
S(b,369K) = S(b,0K) + 38 J.K–1 .mol–1 (pelo Cp) 
 S(a,OK) – S(b,0K) » 0 
• Conclusão: todos os cristais perfeitos a 0 K têm a 
mesma entropia. 
• Todos os cristais perfeitos em T = 0 K, têm entropia 
ZERO (fixado arbitrariamente) 
 
• Entropias da terceira lei = So(T) [J/K.mol] 
Gases: So(298,15K)/R 
1) Entropia x Massa 
 He = 15,1591; Ne = 17,5856; Ar = 18,6101 
 HF = 20,8872; HCl = 22,4653; HBr = 23,8872 
2) Entropia x Capacidade Calorífica (mesma massa) 
 Ne = 17,5856; HF = 20,8872; H2O = 22,6984 
3) Entropia x Simetria Molecular (mesma massa e 
mesma capacidade calorífica). 
 N2 = 23,0325; CO = 23,7607 
 NH3 = 23,173; CH4 = 22,389 
 
 
 
 
 
 
 
 
 
 
 
 
 
 
 
 
 
 
 
 
 
 
 
 
 
 
 
 
 
 
 
 
 
 
 
 
 
 
 
 
 
 
 
 
 
 
 
 
 
 
 
 
 
 
 
 
RESOLVA AGORA 11 
a) Suponha que há quatro formas diferentes da 
molécula CFClBrI se orientar no espaço em um 
cristal a zero Kelvin. Qual é a sua entropia residual 
molar? 
b) Calcule a entropia residual da mistura metaestável de 
orto- e para-hidrogênio (abundância de 25% de p-
H2), sabendo que as moléculas de para-hidrogênio 
atingem o menor estado rotacional J = 0 (1 
possibilidade energética), enquanto o orto-hidrogênio 
assume J = 1 (degenerescência 3). 
 
 
 
 
 
 
 
 
 
 
 
 
OBSERVAÇÕES: Entropias a 0K 
 
So(HCl) = 0 J/mol.K 
So(CO) = 4,6 J/mol.K 
So(N2O) = 6 J/mol.K 
So(H2) = 6,66 J/mol.K 
So(FClO3) = 10,1 J/mol.K 
 
 
 
 
 
 
 
73 
 
 
 
APOSTILA 01 DE FÍSICO-QUÍMICA – PROF. PEDRO MADEIRA (2022) 
 
 H) Espontaneidade e funções do sistema 
• Se V e T são constantes, uma transformação 
espontânea deve satisfazer a desigualdade: 
 
Se p e T são constantes, de forma análoga: 
Energias Livres: 
 > de Helmholtz è A = U – TS 
 > de Gibbs è G = H – TS 
 dA = dU – TdS (isotérmico) 
 dG = dH – TdS (isotérmico) 
 
• Critério para espontaneidade: 
 dAT,V £ 0 e dGT,p £ 0 
“A temperatura e pressão constantes, as reações 
químicas são espontâneas no sentido da diminuição da 
energia de Gibbs” 
OBS.: O significado de DG, além de indicar o sentido da 
transformação espontânea, consiste no valor do 
trabalho máximo possível diferente do de expansão. 
 
• Cálculo do DGo: 
 DGo = DHo – TDSo 
 
RESOLVA AGORA 12 (ITA 2006) 
Para cada um dos processos listados abaixo, indique se 
a variação de entropia será maior, menor ou igual a zero. 
Justifique suas respostas. 
a) N2 (g,1 atm,T = 300 K) → N2 (g,0,1 atm,T = 300 K) 
b) C (grafite) → C(diamante) 
c) solução supersaturada → solução saturada 
d) sólido amorfo → sólido cristalino 
e) N2 (g) → N2 (g, adsorvido em sílica) 
 
 
 
 
 
 
 
 
 
 
 
 
 
 
 
 
 
 
 
0dU dUdS - dS TdS dU dU - TdS 0
T T
³ Þ ³ Þ ³ Þ £
0dH dHdS - dS TdS dH dH - TdS 0
T T
³ Þ ³ Þ ³ Þ £
74 
 
 
 
APOSTILA 01 DE FÍSICO-QUÍMICA – PROF. PEDRO MADEIRA (2022) 
 
 
 
 
 
 
 
 
 
 
 
 
 
 
 
 
 
 
 
 
 
 
 
 
 
 
 
 
 
 
 
 
 
 
 
 
 
 
 
 
 
 
 
 
 
 
 
 
 
 
 
PROF. PEDRO MADEIRA 
TÓPICO 04 – TERMOQUÍMICA 
 
• É um ramo da Termodinâmica que estuda o calor 
envolvido nas reações químicas e nas transições de 
fase. 
• Reação Endotérmica: DH > 0 
 Reação Exotérmica: DH < 0. 
 
A) Variações de entalpia padrão 
• O estado padrão de uma substância, numa certa 
temperatura, é o da substância na sua forma pura 
sob pressão de 1 bar. 
• Ex.: H2O(l) à H2O(g) DvapHo (373K) = + 40,66 kJ.mol–
1. 
• OBS: A temperatura adotada para o registro dos 
dados termodinâmicos é 298,15 K ou (25,00oC). 
PROF. PEDRO MADEIRA 
 
B) Entalpias de transformações físicas 
• Entalpia padrão de transição. DtrsHo 
• Entalpia padrão de fusão. DfusHo 
• Entalpia padrão de vaporização. DvapHo 
• Entalpia padrão de sublimação. 
• DsubHo = DfusHo + DvapHo (medidos na mesma 
temperatura, pois H é função de estado). 
 
OBSERVAÇÕES 
 
 
 
 
 
 
 
 
 
 
 
 
 
 
 
 
 
 
 
 
 
 
 
 
 
 
PROF. PEDRO MADEIRA 
75 
 
 
 
APOSTILA 01 DE FÍSICO-QUÍMICA – PROF. PEDRO MADEIRA (2022) 
 
C) Entalpias de transformações químicas 
 
• Equação Termoquímica. 
CH4(g) + 2O2(g) à CO2(g) + 2 H2O(l) DHo = – 890 kJ 
 
Reagentes isolados, puros, nos estados padrões à 
produtos isolados, puros, nos estados padrões. 
OBS.: Exceto no caso de reações iônicas em solução, 
DmixH e DsepH são insignificantes em relação ao DrH. 
 
• Entalpia padrãode reação: 
 
OBSERVAÇÕES 
 
 
 
 
 
 
 
 
 
 
 
 
 
 
 
 
 
 
 
 
 
 
 
 
 
 
 
 
 
 
 
 
 
 
 
 
 
 
 
 
 
 
 
 
PROF. PEDRO MADEIRA 
 
D) Referencial zero de entalpia e Entalpias 
padrão de formação 
• Estado de referência de um elemento: estado mais 
estável, numa certa temperatura, sob pressão de 1 
bar. 
• OBS.: Alotropia. 
• O2(g); C(s,grafita); S8(s,rômbico). 
• Exceção: Fósforo branco é o estado de entalpia zero 
(apesar de ser o alótropo metaestável), pois é a 
forma mais reprodutível e fácil de se obter. 
 
OBSERVAÇÕES 
 
 
 
 
 
 
 
 
 
 
 
 
 
 
 
 
 
 
 
 
 
 
 
 
 
 
 
 
 
 
 
 
 
 
 
 
 
 
 
 
RESOLVA AGORA 13 (ITA 2008) 
Assinale a opção ERRADA que apresenta (em kJ/mol) a 
entalpia padrão de formação (ΔHf) da substância a 25oC. 
A ( ) ΔHf (H2(g)) = 0 B ( ) ΔHf (F2(g)) = 0 
C ( ) ΔHf (N2(g)) = 0 D ( ) ΔHf (Br2(g)) = 0 
E ( ) ΔHf (Cl2(g)) = 0 
o
r produtos reagentes
produtos reagentes
ΔH = H - Hu u× ×å å
76 
 
 
 
APOSTILA 01 DE FÍSICO-QUÍMICA – PROF. PEDRO MADEIRA (2022) 
 
E) A entalpia como função de estado 
Lei de Hess 
 
 
RESOLVA AGORA 14 (IME 2018) 
A reforma com vapor d`água, a temperaturas altas, é um 
método industrial para produção de hidrogênio a partir 
de metano . Calcule a entalpia de reação desse 
processo. 
Dados: 
i) Entalpias de combustão: 
C(grafite) ... ΔHo = -394 kJ/mol 
 H2(g) ... ΔHo = -286 kJ/mol (forma água líquida) 
 CH4(g) ... ΔHo = -890 kJ/mol (forma água líquida) 
ii) CO(g) + H2(g) → C(grafite) + H2O (g) 
ΔHo = -131 kJ/mol 
Entalpia de fusão do gelo: ΔHfus = 330 kJ·kg–1 
Capacidade calorífica específica média da água: 
CV = 4,2 kJ·kg–1·K–1 
 
 
 
 
 
 
 
 
 
 
 
 
 
 
 
 
 
 
 
 
 
 
 
 
 
 
RESOLVA AGORA 15 
Monte um ciclo de Born-Haber para o NaCl e calcule a 
sua energia reticular U. 
Dados para o Na (kJ/mol): DsubHo = 108; EI = 496 
Dados para o Cl (kJ/mol de Cl): DdissHo = 122; AE = 349 
Dados para o NaCl (kJ/mol): DfHo = - 411. 
 
 
 
 
 
 
 
 
 
 
 
 
 
 
 
 
 
 
 
 
 
 
 
 
 
 
 
 
 
 
 
 
 
 
 
 
 
 
 
 
 
 
 
 
 
PROF. PEDRO MADEIRA 
77 
 
 
 
APOSTILA 01 DE FÍSICO-QUÍMICA – PROF. PEDRO MADEIRA (2022) 
 
F) Métodos de cálculo de ΔH 
• A partir dos DHf
o 
 
 
 
• A partir das energias de ligação 
 
 
 
Observe as energias de ligação: 
 
HX BX3 AlX3 CX4 X2 
F 566 645 582 439 159 
Cl 431 444 427 347 243 
Br 366 368 360 276 193 
I 299 272 285 238 151 
 
 
Ligação Entalpia 
(kJ mol–1) Ligação Entalpia 
(kJ mol–1) 
H–H 436 C–O 360 
H–O 464 C=O 799 
N–N 163 C≡O 1074 
N=N 514 C–C 348 
N≡N 946 C–C 
(C6H6) 518 
C–H 413 C=C 612 
N–H 389 C≡C 837 
O=O 498 N–O 210 
O–O 134 N=O 630 
 
 
OBSERVAÇÕES 
 
 
 
 
 
 
 
 
 
 
 
 
 
 
 
 
 
 
 
 
 
 
 
 
 
 
 
 
 
 
 
 
 
 
 
 
 
 
 
 
 
 
 
 
 
 
 
 
 
 
 
 
 
 
 
 
 
 
 
 
 
 
 
 
 
o o o
r f produtos f reagentes
produtos reagentes
ΔH = H - Hu u×D ×Då å
o
r reagentes produtos
reagentes produtos
ΔH = EL - ELn n× ×å å
78 
 
 
 
APOSTILA 01 DE FÍSICO-QUÍMICA – PROF. PEDRO MADEIRA (2022) 
 
 
 
 
 
 
 
 
 
 
 
 
 
 
 
 
 
 
 
 
 
 
 
 
 
 
 
 
 
 
 
 
 
 
 
 
 
 
 
 
 
 
 
 
 
 
 
 
 
 
 
 
 
 
 
• Lei de Hess. 
 
• Contribuição de grupos termoquímicos. 
Exemplo: n-hexano(l). DvapH = 28,9 kJ.mol–1. 
 
Grupo DfHo / kJ.mol–1 
C(H)3(C) - 42,17 
C(H)2(C)2 - 20,7 
C(H)(C)3 - 6,91 
C(C)4 + 8,16 
 
DfH(método dos grupos de Benson) = - 196 kJ.mol–1 
DfH (experimental) = - 198,7 kJ.mol–1 
 
 
 
 
 
 
 
 
 
 
 
 
 
 
 
 
 
 
 
 
 
 
 
 
RESOLVA AGORA 16 (ITA 2001) 
A 25ºC e pressão de 1 atm, a queima completa de um 
mol de n-hexano produz dióxido de carbono e água no 
estado gasoso e libera 3883 kJ, enquanto que a queima 
completa da mesma quantidade de n-heptano produz as 
mesmas substâncias no estado gasoso e libera 4498 kJ. 
A) Escreva as equações químicas, balanceadas, para 
as reações de combustão em questão. 
B) Utilizando as informações fornecidas no enunciado 
desta questão, faça uma estimativa do valor do calor 
de combustão do n-decano. Deixe claro o raciocínio 
utilizando na estimativa realizada. 
C) Caso a água formada na reação de combustão 
estivesse no estado líquido, a quantidade de calor 
liberado seria MAIOR, MENOR OU IGUAL a 3883 
kJ? Por quê? 
 
79 
 
 
 
APOSTILA 01 DE FÍSICO-QUÍMICA – PROF. PEDRO MADEIRA (2022) 
 
 
 
 
 
 
 
 
 
 
 
 
 
 
 
 
 
 
 
 
 
 
 
 
 
 
 
 
 
 
 
 
 
 
RESOLVA AGORA 17 (IME 2010) 
Uma dada massa de óxido ferroso é aquecida a 1273 K 
e, em seguida, exposta a uma mistura gasosa de 
monóxido de carbono e hidrogênio. Desta forma, o óxido 
é reduzido a metal sem qualquer fornecimento adicional 
de energia. Admita que ocorra uma perda de calor para 
as circunvizinhanças de 4,2 kJ/mol de óxido reduzido. 
Calcule a razão mínima entre as pressões parciais de 
monóxido de carbono e de hidrogênio (pCO/pH2) na 
mistura gasosa inicial, de modo que o processo seja 
auto-sustentável. Despreze a decomposição da água. 
 
Calores de reação a 1273 K (kJ/mol) 
redução do óxido ferroso 265 
oxidação do hidrogênio – 250 
oxidação do monóxido de carbono – 282 
 
 
 
 
 
 
 
 
 
 
 
 
 
 
 
 
 
 
 
 
 
 
 
 
 
 
 
 
 
 
 
 
 
 
 
 
 
 
 
 
 
 
 
 
 
 
 
 
 
 
 
 
 
 
 
 
 
 
 
 
 
 
 
 
 
 
 
 
 
 
 
PROF. PEDRO MADEIRA 
80 
 
 
 
APOSTILA 01 DE FÍSICO-QUÍMICA – PROF. PEDRO MADEIRA (2022) 
 
G) Tipos específicos de reação 
 
Transição Processo Símbolo 
Transição fase a à fase b DtrsH 
Fusão s à ℓ DfusH 
Vaporização ℓ à g DvapH 
Sublimação s à g DsubH 
Misturação 
de fluidos puro à mistura DmixH 
Dissolução soluto à solução DsolH 
Hidratação X±(g) à X±(aq) DhidH 
Atomização espécies (s,ℓ,g) à 
átomos (g) DatH 
Ionização X(g)àX+(g) + e–(g) 
DionH 
(EI) 
Afinidade 
eletrônica X(g) + e–(g)à X–(g) DegH 
(AE) 
Reação reagentesàprodutos DrH 
Combustão composto(s,ℓ,g)+O2(g) 
à CO2(g) + H2O(ℓ,g) 
DcH 
Formação elementosàcomposto DfH 
Neutralização H+ + OH– à H2O DH 
 
 
OBSERVAÇÕES 
 
 
 
 
 
 
 
 
 
 
 
 
 
 
 
 
 
 
 
 
 
 
 
 
 
 
 
 
 
 
 
 
 
 
 
 
 
 
 
 
 
 
 
 
 
 
 
 
 
 
 
 
 
 
 
 
 
 
 
 
 
 
 
 
 
 
 
 
 
PROF. PEDRO MADEIRA 
 
81 
 
 
 
APOSTILA 01 DE FÍSICO-QUÍMICA – PROF. PEDRO MADEIRA (2022) 
 
RESOLVA AGORA 18 (ITA 2022 – Q07) 
Considere as seguintes informações: 
I. Primeira energia de ionização do cálcio: 
590 kJ.mol–1 
II. Segunda energia de ionização do cálcio: 
1145 kJ.mol–1 
III. Afinidade eletrônica do cloro: 
-340 kJ.mol–1 
IV. Entalpia de solubilização do cloreto de cálcio: 
-81 kJ.mol–1 
V. Entalpia de hidratação do íon de cálcio: 
-1579 kJ.mol–1 
V. Entalpia de hidratação do íon de cloro: 
-378 kJ.mol–1 
 
Com base nessas informações, responda os itens 
abaixo. 
a) Represente, na forma de equações químicas, as 
informações acima (I-VI). 
b) Equacione a reação de entalpia de rede do cloreto de 
cálcio a partir das equações I-VI, conforme a 
necessidade. 
c) Calcule o valor numérico da entalpia de rede do 
cloreto de cálcio (em kJ.mol–1). 
 
 
 
 
 
 
 
 
 
 
 
 
 
 
 
 
 
 
 
 
 
 
 
 
 
 
 
 
 
 
 
 
 
 
 
H) Casos Especiais 
 
Energia de hidratação de íons 
 
 DhidHo 
kJ/mol DhidHo 
kJ/mol DhidHo 
kJ/mol 
H+ - 1090 Mg2+ - 1920 OH– - 460 
Li+ - 520 Ca2+ - 1650 F– - 506 
Na+ - 405 Sr2+ - 1480 Cl– - 364 
K+ - 321 Ba2+ - 1360 Br– - 337 
Rb+ - 300 Fe2+ - 1950 I– - 296 
Cs+ - 277 Cu2+ - 2100 
 Zn2+ - 2050 
Ag+ - 464 Al3+ - 4690 
NH4
+ - 301 Fe3+ - 4430 
 
Energia de isômeros 
 
 
 DHo
f (equatorial) < DHo
f (axial) 
 
 
OBSERVAÇÕES 
 
 
 
 
 
 
 
 
 
 
 
 
 
 
 
 
 
 
 
 
 
 
 
82 
 
 
 
APOSTILA 01 DE FÍSICO-QUÍMICA – PROF. PEDRO MADEIRA (2022) 
 
 
Interconversão entre isômeros (Cinética) 
Processo 
necessário para 
interconverter 
enanciômeros ou 
diastereoisômeros 
Barreira 
de energia 
kcal/mol 
Velocidade 
de 
conversão a 
25oC 
Rompimento de 
ligações s 50 – 100 Nenhuma 
Rompimento de 
ligações p (rotação 
restrita em torno de 
ligações duplas) 
30 – 50 Nenhumaou 
muito pouca 
Rotação restrita em 
torno de ligações 
simples 
(impedimento 
estérico) 
15 – 30 Muito pouca 
ou pouca 
Inversal piramidal em 
aminas 10 – 13 Rápida 
Isomeria 
conformacional 
(rotação livre em 
torno de ligações 
simples) 
0 – 12 Rápida ou 
muito rápida 
 
 
 
 
 
 
 
 
 
 
 
 
 
 
 
 
 
 
 
 
 
 
 
 
RESOLVA AGORA 19 (FUVEST 2006) 
Considere os seguintes dados: 
 
 
a) Qual dos alcenos (A ou B) é o mais estável? 
Justifique. Neste caso, considere válido raciocinar 
com entalpia. 
 
A desidratação de álcoois, em presença de ácido, pode 
produzir uma mistura de alcenos, em que predomina o 
mais estável. 
b) A desidratação do álcool a seguir, em presença de 
ácido, produz cerca de 90% de um determinado 
alceno. Qual deve ser a fórmula estrutural desse 
alceno? Justifique. 
 
 
 
 
 
 
 
 
 
Energia de tensão nos cicloalcanos 
 
Ciclo-alcano 
[(CH2)n] n Tensão total 
(kcal/mol) 
Ciclo-propano 3 27,6 
Ciclo-butano 4 26,3 
Ciclo-pentano 5 6,5 
Ciclo-hexano 6 0,0 
Ciclo-heptano 7 6,4 
Ciclo-octano 8 10,0 
Ciclo-nonano 9 12,9 
Ciclo-decano 10 12,0 
Ciclo-pentadecano 15 1,5 
n-alcano de cadeia 
aberta n 0,0 
Fonte: Química orgânica – Allinger, Cava, De Johng 
 
OBSERVAÇÕES 
 
 
 
 
 
 
 
 
 
 
 
 
 
 
 
 
CH3OH
83 
 
 
 
APOSTILA 01 DE FÍSICO-QUÍMICA – PROF. PEDRO MADEIRA (2022) 
 
 
 
 
 
 
 
 
 
 
 
 
 
 
 
 
 
 
 
Energia de conjugação / ressonância 
RESOLVA AGORA 20 
As reações de hidrogenação do ciclohexeno, do 1,3-
ciclohexadieno e do benzeno conduzem ao ciclohexano 
com libertação de entalpia DH. Comente os valores 
observados experimentalmente, seguindo o roteiro a 
seguir: 
(a) Justifique a liberação de calor a partir das energias 
de ligação. 
(b) Explicando porque razão não se observa 
aditividade das entalpias com o número de ligações 
duplas. 
 
 
RESOLVA AGORA 21 (ATKINS) 
R. F. Curl, R. E. Smalley e H. W. Kroto receberam o 
prêmio Nobel em química em 1996 pela descoberta da 
molécula C60 com a forma de bola de futebol. A entalpia 
de combustão do C60 é -25937 kJ.mol–1 e sua entalpia 
de sublimação é + 233 kJ.mol–1. Existem 90 ligações em 
C60, das quais 60 são simples e 30 são duplas. O C60 é 
como o benzeno, em que há um conjunto de ligações 
múltiplas para as quais as estruturas ressonantes 
podem ser desenhadas. 
(a) Determine a entalpia de formação do C60 a partir de 
sua entalpia de combustão. 
(b) Calcule a entalpia de formação esperada do C60 a 
partir das entalpias de ligação, assumindo que as 
ligações duplas e simples possam ser isoladas. 
(c) O C60 é mais ou menos estável que o previsto com 
base no modelo de ligações isoladas? 
(d) Quantifique a resposta em (c) dividindo a diferença 
entre a entalpia de formação calculada dos dados 
de combustão e aqueles obtidos dos cálculos de 
entalpia do ligação por 60 para obter o valor por 
carbono. 
(e) Como o número em (d) se compara à energia de 
estabilização ressonante por carbono no benzeno (a 
energia de estabilização ressonante do benzeno é 
aproximadamente 150 kJ.mol–1)? 
(f) Por que esses números poderiam ser diferentes? A 
entalpia de atomização do C(graf) é +717 kJ.mol–1. 
Dados (kJ/mol): DHf°(CO2) = – 394; DHat°(grafite) = 717 
EL (C–C) = 348; EL (C=C) = 612 
 
 
 
 
 
 
 
 
 
 
 
 
 
 
 
 
 
 
 
 
 
 
 
 
 
 
 
 
 
 
 
 
 
 
 
 
 
 
 
 
 
Prof. Pedro Madeira 
+ H2 
+ 2 H2
+ 3 H2
DH (kcal/mol)
- 28,6
- 53,4
- 49,8
84 
 
 
 
APOSTILA 01 DE FÍSICO-QUÍMICA – PROF. PEDRO MADEIRA (2022) 
 
Comparação entre alótropos 
 
 DfHo 
kJ/mol 
DfGo 
kJ/mol 
Smo 
J/K/mol 
Cp,mo 
J/K.mol 
C(graf) 0 0 5,740 8,527 
C(diam) +1,895 +2,900 2,377 6,113 
S(a,romb) 0 0 31,80 22,64 
S(b,mono) +0,33 +0,1 32,6 23,6 
O2(g) 0 0 205,138 29,355 
O3(g) +142,7 +163,2 238,93 39,20 
P(verm) 22,8 
P4(bran) 164 
Observe as estruturas dos alótropos naturais do 
carbono: 
 
 
 c/2 = 3,35 Å 
 C–C = 1,42 Å 
 
Comparação entre isótopos do hidrogênio 
 
 DfHo 
kJ/mol 
DfGo 
kJ/mol 
Smo 
J/K.mol 
Cp,mo 
J/K.mol 
H2(g) 0 0 130,684 28,824 
D2(g) 0 0 144,96 29,20 
HD(g) + 0,138 - 1,464 143,80 29,196 
H2O(l) - 285,83 - 237,13 69,91 75,291 
D2O(l) - 294,60 - 243,44 75,94 84,35 
H2O(g) - 241,82 - 228,57 188,83 33,58 
D2O(g) - 249,60 - 234,54 198,34 34,27 
HDO(g) - 245,30 - 233,11 199,51 33,81 
HDO(l) - 289,89 - 241,86 79,29 
H2O2(l) - 187,78 - 120,35 109,6 89,1 
H+(aq) 0 0 0 0 
H+(g) 1536,20 
H(g) + 217,97 + 203,25 114,71 20,784 
 
Constante Física H2 D2 T2 
Massa atômica 
(uma) 1,0078 2,0141 3,0160 
Ponto de fusão 
(oC) - 259,0 - 254,3 - 252,4 
Ponto de ebulição 
(oC) - 252,6 - 249,3 - 248,0 
Comprimento de 
ligação (Å) 0,7414 0,7414 0,7414 
Calor de 
dissociação 
(kJ/mol) 
435,9 443,4 446,9 
Calor de fusão 
(kJ/mol) 0,117 0,197 0,250 
Calor de 
vaporização 
(kJ/mol) 
0,904 1,226 1,393 
 
 
 
 
 
 
 
 
 
 
 
 
 
 
 
 
 
 
 
 
 
 
 
Constante Física H2O D2O 
Ponto de fusão (oC) 0 3,82 
Ponto de ebulição (oC) 100 100,42 
Densidade a 20oC 0,917 1,017 
Temperatura da 
densidade máxima (oC) 4 11,6 
Kw a 25oC 1,0 x 10–14 3,0 x 10–15 
Constante dielétrica a 
20oC 82 80,5 
Solubilidade do NaCl a 
25oC (g/100g água) 35,9 30,5 
 
85 
 
 
 
APOSTILA 01 DE FÍSICO-QUÍMICA – PROF. PEDRO MADEIRA (2022) 
 
 
Família 1A 
 
 
Raio 
iônico 
(Å) 
Mobilidade 
iônica à 
diluição 
infinita 
Raio do 
íon 
hidratado 
(Å) 
Li+ 0,76 33,5 3,40 
Na+ 1,02 43,5 2,76 
K+ 1,38 64,5 2,32 
Rb+ 1,52 67,5 2,28 
Cs+ 1,67 68,0 2,28 
 
 
No de 
hidratação 
aproximado 
Parâmetros 
termodinâmicos da 
hidratação 
 DHo DSo DGo 
Li+ 25,3 - 544 - 134 - 506 
Na+ 16,6 - 435 - 100 - 406 
K+ 10,5 - 352 - 67 - 330 
Rb+ 10,0 - 326 - 54 - 310 
Cs+ 9,9 - 296 - 50 - 276 
 
 
Energia de 
coesão 
(entalpia de 
atomização) 
(kJ/mol) 
Ponto de 
Fusão 
(oC) 
Ponto de 
Ebulição 
(oC) 
Li 161 181 1347 
Na 108 98 881 
K 90 63 766 
Rb 82 39 688 
Cs 78 28,5 705 
 
PROF. PEDRO MADEIRA 
I)Dependência das entalpias com a 
temperatura Lei de Kirchoff 
 
OBSERVAÇÕES 
 
 
 
 
 
 
 
 
 
 
 
 
 
 
 
 
 
 
 
 
 
 
 
 
 
 
 
 
 
 
 
 
 
 
 
 
 
 
 
 
 
 
 
 
 
 
86 
 
 
 
APOSTILA 01 DE FÍSICO-QUÍMICA – PROF. PEDRO MADEIRA (2022) 
 
o o o
r p p,m p,m
produtos reagentes
Δ C = -C Cu u× ×å å
 
 
 
 
 
 
 
Prof. Pedro Madeira 
RESOLVA AGORA 22 (ITA 2002) 
A figura abaixo mostra como a capacidade calorífica, CP, 
de uma substância varia com a temperatura, sob 
pressão constante. 
 
 
Considerando as informações mostradas na figura 
acima, é ERRADO afirmar que 
A ( ) a substância em questão, no estado sólido, 
apresenta mais de uma estrutura 
 cristalina diferente. 
B ( ) a capacidade calorífica da substância no estado 
gasoso é menor do que aquela no estado 
líquido. 
C ( ) quer esteja a substância no estado sólido, líquido 
ou gasoso, sua capacidade calorífica 
aumenta com o aumento da temperatura. 
D ( ) caso a substância se mantenha no estado líquido 
em temperaturas inferiores a Tf, a capacidade 
calorífica da substância líquida é maior do que a 
capacidade calorífica da substância na fase 
sólida estável em temperaturas menores do que 
 Tf. 
E ( ) a variação de entalpia de uma reação envolvendo 
a substância em questão no estado líquido 
aumenta com o aumento da temperatura. 
 
 
 
 
 
 
 
 
RESOLVA AGORA 23 (IME 2015) 
Monóxido de carbono a 473 K é queimado, sob pressão 
atmosférica, com 90% em excesso de ar seco, em base 
molar, a 773 K. Os produtos da combustão abandonam 
a câmara de reação a 1273 K. Admita combustão 
completa e considere que 1 mol de ar é constituído por 
0,20 mol de oxigênio e 0,80 mol de nitrogênio. Calcule a 
quantidade de energia, em kJ, que é liberada no 
decorrer da reação, por mol de monóxido de carbono 
queimado. Considere que os gases apresentam 
comportamento ideal. 
 
DADOS: 
Calor de combustão do monóxido de carbono (a 298 K 
e 1 atm) = –283 kJ·mol–1 
T(K) = t(oC) + 273 
 
Substância CO CO2 O2 N2 
Cp médio 
(kJ/mol.K) 
0,03 0,04 0,03 0,03( ) ( ) 2
1
Tθ θ θ
r 2 r 1 r pT
Δ H T = Δ H T + Δ C dTò
87 
 
 
 
APOSTILA 01 DE FÍSICO-QUÍMICA – PROF. PEDRO MADEIRA (2022) 
 
 
 
 
 
 
 
 
 
 
 
 
 
 
 
 
 
 
 
 
 
 
 
 
 
 
 
 
 
 
 
 
 
 
 
 
 
 
 
 
 
 
 
 
 
 
 
 
 
 
 
 
 
 
 
 
 
 
 
 
 
 
 
 
 
 
 
 
 
 
 
 
 
 
 
 
 
 
 
 
 
 
 
 
 
 
 
 
 
 
 
 
 
 
 
 
 
 
 
 
 
 
 
 
 
 
 
 
 
 
 
 
 
 
 
 
 
 
 
 
 
 
 
 
 
88 
 
 
 
APOSTILA 01 DE FÍSICO-QUÍMICA – PROF. PEDRO MADEIRA (2022) 
 
J) Tabela de entalpias de formação e de energias de ligação 
Calores de formação (kJ/mol) a 25ºC e 1 atm 
AgBr(s) – 99,5 C2H2(g) + 226,7 H2O(l) – 285,8 NH4Cl(s) – 315,4 
AgCl(s) – 127,0 C2H4(g) + 52,3 H2O2(l) – 187,6 NH4NO3(s) – 365,1 
AgI(s) – 62,4 C2H6(g) – 84,7 H2S(g) – 20,2 NO(g) + 90,4 
Ag2O(s) – 30,6 C3H8(g) – 103,8 H2SO4(l) – 811,3 NO2(g) + 33,9 
Ag2S(s) – 31,8 n-C4H10(g) – 124,7 HgO(s) – 90,7 NiO(s) – 244,3 
Al2O3(s) – 1669,8 n-C5H12(l) – 173,1 HgS(s) – 58,2 PbBr2(s) – 277,0 
BaCl2(s) – 860,1 C2H5OH(l) – 277,6 KBr(s) – 392,2 PbCl2(s) – 359,2 
BaCO3(s) – 1218,8 CaO(s) – 239,3 KCl(s) – 453,9 PbO(s) – 217,9 
BaO(s) – 558,1 Cr2O3(s) – 1128,4 KClO4(s) – 391,4 PbO2(s) – 276,6 
BaSO4(s) – 1465,2 CuO(s) – 155,2 KF(s) – 562,6 Pb3O4(s) – 734,7 
CaCl2(s) – 795,0 Cu2O(s) – 166,7 MgCl2(s) – 641,8 PCl3(g) – 306,4 
CaCO3(s) – 1207,0 CuS(s) – 48,5 MgCO3(s) – 111,3 PCl5(g) – 398,9 
CaO(s) – 635,5 CuSO4(s) – 769,9 MgO(s) – 601,8 SiO2(s) – 859,4 
Ca(OH)2(s) – 986,6 Fe2O3(s) – 822,2 Mg(OH)2(s) – 924,7 SnCl2(s) – 349,8 
CaSO4(s) – 1432,7 Fe3O4(s) – 1120,9 MgSO4(s) – 1278,2 SnCl4(l) – 545,2 
CCl4(l) – 139,5 HBr(g) – 36,2 MnO(s) – 384,9 SnO(s) – 286,2 
CH4(g) – 74,8 HCl(g) – 92,3 MnO2(s) – 519,7 SnO2(s) – 580,7 
CHCl3(l) – 131,8 HF(g) – 268,6 NaCl(s) – 411,0 SO2(g) – 296,1 
CH3OH(l) – 238,6 HI(g) + 25,9 NaF(s) – 569,0 SO3(g) – 395,2 
CO(g) – 110,5 HNO3(l) – 173,2 NaOH(s) – 426,7 ZnO(s) – 348,0 
CO2(g) – 393,5 H2O(g) – 241,8 NH3(g) – 46,2 ZnS(s) – 202,9 
(Princípios de Química, Masterton, Editora LTC) 
 
Energias de ligação (kJ/mol) 
Br–Br 193 C–H 414 Cl–N 201 H–S 339 
Br–C 276 C–I 218 Cl–O 205 I–I 151 
Br–Cl 218 C–N 293 Cl–S 255 I–O 201 
Br–F 255 C=N 615 F–F 153 N–N 159 
Br–H 368 C≡N 890 F–H 565 N=N 418 
Br–I 180 C–O 351 F–I 277 N≡N 941 
Br–N 243 C=O 715 F–N 272 N–O 222 
Br–O 201 C≡O 1075 F–O 184 N=O 607 
Br–S 213 C–S 259 F–S 258 O–O 138 
C–C 347 C=S 477 H–H 436 O=O 498 
C=C 612 Cl–Cl 243 H–I 297 O–S 347 
C≡C 820 Cl–F 255 H–N 389 O=S 498 
C–Cl 331 Cl–H 431 H–O 464 S–S 226 
C–F 485 Cl–I 209 
 
89 
 
 
 
APOSTILA 01 DE FÍSICO-QUÍMICA – PROF. PEDRO MADEIRA (2022) 
 
CAPÍTULO 02 – TERMODINÂMICA QUÍMICA 
EXERCÍCIOS 
 
 
 
TÓPICO 01: CONCEITOS FUNDAMENTAIS 
 
SEÇÃO VESTIBULARES 
 
1. (FMJ 2021) Liga eutética Sn-Pb 
Ligas metálicas podem ser produzidas a partir da fusão 
dos metais puros e posterior mistura dos metais 
derretidos. Uma liga metálica muito utilizada no cotidiano 
é formada pela mistura de chumbo com estanho, na 
proporção de 37% de chumbo e 63% de estanho. Os 
gráficos mostram a curva de aquecimento dos metais 
isolados e da liga formada pela mistura dos metais. 
 
 
 
Considere que, para aquecer os metais até sua fusão, 
utiliza-se como combustível o metano. 
 
a) Com base na análise das curvas de aquecimento, dê 
o nome da mistura que constitui a liga Sn-Pb. 
b) Considerando que o calor de fusão do estanho é igual 
a 7 kJ/mol, e que as entalpias-padrão de CH4, CO2 e 
H2O são, respectivamente, –76 kJ/mol, –394 kJ/mol e 
–286 kJ/mol, calcule a massa de estanho, em gramas, 
que pode ser derretida pelo calor gerado na queima 
de 1 mol de metano. 
 
2. (FUVEST 2020) 
Equipamentos domésticos chamados de vaporizadores 
para roupa utilizam o vapor de água gerado por um 
sistema de resistências elétricas a partir de água líquida. 
Um equipamento com potência nominal de 1600 W foi 
utilizado para passar roupas por 20 minutos, 
consumindo 540 mL de água. Em relação ao gasto total 
de energia do equipamento, o gasto de energia utilizado 
apenas para vaporizar a água, após ela já ter atingido a 
temperatura de ebulição, equivale a, aproximadamente, 
Note e adote: 
Entalpia de vaporização da água a 100ºC = 40 kJ/mol; 
a) 0,04% 
b) 0,062% 
c) 4,6% 
d) 40% 
e) 62% 
 
3. (UNICAMP 2020) 
“O sal faz a água ferver mais rápido?” Essa é uma 
pergunta frequente na internet, mas não tente responder 
com os argumentos lá apresentados. Seria muito difícil 
responder à pergunta tal como está formulada, pois isso 
exigiria o conhecimento de vários parâmetros 
termodinâmicos e cinéticos no aquecimento desses 
líquidos. Do ponto de vista termodinâmico, entre tais 
parâmetros, caberia analisar os valores de calor 
específico e de temperatura de ebulição da solução em 
comparação com a água pura. Considerando massas 
iguais (água pura e solução), se apenas esses 
parâmetros fossem levados em consideração, a solução 
ferveria mais rapidamente se o seu calor específico 
fosse 
a) menor que o da água pura, observando-se ainda que 
a temperatura de ebulição da solução é menor. 
b) maior que o da água pura, observando-se ainda que 
a temperatura de ebulição da solução é menor. 
c) menor que o da água pura, observando-se, no 
entanto, que a temperatura de ebulição da solução é 
maior. 
d) maior que o da água pura, observando-se, no entanto, 
que a temperatura de ebulição da solução é maior. 
 
4. (UNICAMP 2018) Aluminotermia é exotérmica 
Em 12 de maio de 2017 o Metrô de São Paulo trocou 
240 metros de trilhos de uma de suas linhas, numa 
operação feita de madrugada, em apenas três horas. Na 
solda entre o trilho novo e o usado empregou-se uma 
reação química denominada térmita, que permite a 
obtenção de uma temperatura local de cerca de 2000ºC. 
A reação utilizada foi entre um óxido de ferro e o 
alumínio metálico. 
 
De acordo com essas informações, uma possível 
equação termoquímica do processo utilizado seria 
a) Fe2O3 + 2 Aℓ → 2 Fe + Aℓ2O3; DH = +852 kJ.mol–1 
b) FeO3 + Aℓ → Fe + AℓO3; DH = –852 kJ.mol–1 
c) FeO3 + Aℓ → Fe + AℓO3; DH = +852 kJ.mol–1 
d) Fe2O3 + 2 Aℓ → 2 Fe + Aℓ2O3; DH = –852 kJ.mol–1 
 
 
 
90 
 
 
 
APOSTILA 01 DE FÍSICO-QUÍMICA – PROF. PEDRO MADEIRA (2022) 
 
5. (UEMA 2015) 
Um técnico de laboratório de química, para destilar certa 
massa de água, usou um aquecedor elétrico para 
colocar em ebulição 80% dessa massa, pois o mesmo 
não pode funcionar a seco. Considere que essa massa 
estava a 20ºC e que levou 5 min para ferver a 100ºC. 
 
Adotando-se um regime estacionário e sem perda de 
energia, o calor de vaporização igual a 540 cal / g e o 
calor específico igual a 1 cal/goC calcule o tempo total 
programado pelo técnico para o desligamento do 
temporizador do aquecedor, considerando que o mesmo 
não tenha sofrido qualquer danificação. 
 
6. (UEM 2011) 
Sistemas domésticos de aquecimento de água estão 
cada vez mais presentes nos empreendimentos 
imobiliários. Esses sistemas são constituídos de uma 
unidade de aquecimento que utiliza a radiação solar 
como fonte de aquecimento e de um reservatório de 
água. Considere um sistema desse tipo com volume 
total de 500 litros, que seja capaz de aumentar a 
temperatura desse volume de água em 2 ºC a cada hora 
de exposição à luz solar. A temperatura inicial da água 
é de 23 ºC, e o sistema é exposto à luz solar das 8 às 18 
horas. 
 
Desprezando a possível troca de calor do sistema com 
o meio ambiente, assinale o que for correto. 
01) A temperatura da água às 18 horas é 43 ºC. 
02) A quantidade de calor recebido pelos 500 litros de 
água até as 18 horas é 1x107 cal. 
04) Se ao meio dia, metade do volume de água for 
retirado e imediatamente reposto com água a 23ºC, 
a temperatura de equilíbrio térmico é de 
aproximadamente 300 K. 
08) A capacidade térmica dos 500 litros de água é 1x105 
cal / oC. 
16) Às onze horas, a temperatura da água é inferior a 
35oF. 
 
7. (UFSCAR 2008) 
Após ter estudado calorimetria, um aluno decide 
construir um calorímetro usando uma lata de refrigerante 
e isopor. Da latinhade alumínio removeu parte da tampa 
superior. Em seguida, recortou anéis de isopor, de forma 
que estes se encaixassem na latinha recortada, 
envolvendo-a perfeitamente (Figura 1). 
Em seu livro didático, encontrou as seguintes 
informações (Figura 2): 
 
 
a) Determine a capacidade térmica desse calorímetro, 
sabendo que a massa da latinha após o recorte 
realizado era de 15 . 10-3 kg. 
b) Como a capacidade térmica do calorímetro era muito 
pequena, decidiu ignorar esse valor e então realizou 
uma previsão experimental para o seguinte problema: 
 
Determinar a temperatura que deve ter atingido um 
parafuso de ferro de 0,1 kg aquecido na chama de um 
fogão. 
Dentro do calorímetro, despejou 0,2 L de água. Após 
alguns minutos, constatou que a temperatura da água 
era de 19 °C. Aqueceu então o parafuso, colocando-o 
em seguida no interior do calorímetro. Atingido o 
equilíbrio térmico, mediu a temperatura do interior do 
calorímetro, obtendo 40 °C. Nessas condições, supondo 
que houvesse troca de calor apenas entre a água e o 
parafuso, determine aproximadamente a temperatura 
que este deve ter atingido sob o calor da chama do 
fogão. 
 
8. (PUCCAMP 1995) 
Um calorímetro de capacidade térmica 50 cal/oC contém 
520 g de gelo a 0oC. Injeta-se no calorímetro vapor de 
água a 120ºC, na quantidade necessária e suficiente 
para fundir totalmente o gelo. A massa de água, em 
gramas, que se forma no interior do calorímetro vale 
a) 520 b) 584 c) 589 d) 620 e) 700 
 
9. (UFC 2003) 
Dentre as diversas utilidades da água líquida, cita-se sua 
elevada capacidade de extinguir incêndios, que é 
decorrente, principalmente, do seu elevado calor de 
vaporização (DHvap= 9,72 kcal/mol, a 100oC). 
Assinale a alternativa correta. 
A) O calor de vaporização de uma substância resulta 
do rompimento das interações intermoleculares do 
estado gasoso. 
B) A evaporação de 18 g de água, a 100oC, libera 9.720 
calorias, acarretando o resfriamento da vizinhança. 
C) A evaporação de 1g de água, a 100oC, pode causar 
o resfriamento da vizinhança, correspondente a 540 
calorias. 
91 
 
 
 
APOSTILA 01 DE FÍSICO-QUÍMICA – PROF. PEDRO MADEIRA (2022) 
 
D) O elevado calor de vaporização da água resulta do 
fato de ela ocupar maior volume no estado líquido 
do que no estado sólido. 
E) A evaporação de um líquido, por se constituir em um 
fenômeno químico, envolve o rompimento de 
ligações químicas. 
 
10. (FUVEST 2012) Observe o gráfico 
Um aluno efetuou um experimento para avaliar o calor 
envolvido na reação de um ácido com uma base. Para 
isso, tomou 8 tubos de ensaio e a cada um deles 
adicionou 50 mL de uma mesma solução aquosa de 
 e diferentes volumes de água. Em seguida, 
acondicionou esses tubos em uma caixa de isopor, para 
minimizar trocas de calor com o ambiente. A cada um 
desses tubos, foram adaptados uma rolha e um 
termômetro para medir a temperatura máxima atingida 
pela respectiva solução, após o acréscimo rápido de 
volumes diferentes de uma mesma solução aquosa de 
NaOH. O volume final da mistura, em cada tubo, foi 
sempre 100 mL. Os resultados do experimento são 
apresentados na tabela. 
 
Tubo 
Volume 
de HCl 
(aq) 
(mL) 
Volume 
de H2O 
(mL) 
Volume 
de 
NaOH 
(aq) 
(mL) 
Temperatura 
máxima 
(ºC) 
1 50 50 0 23,0 
2 50 45 5 24,4 
3 50 40 10 25,8 
4 50 35 15 27,2 
5 50 30 20 28,6 
6 50 25 25 30,0 
7 50 20 30 30,0 
8 50 15 35 30,0 
a) Construa um gráfico, no quadriculado apresentado na 
página de resposta, que mostre como a temperatura 
máxima varia em função do volume de solução 
aquosa de NaOH acrescentado. 
b) A reação do ácido com a base libera ou absorve 
calor? Justifique sua resposta, considerando os 
dados da tabela. 
c) Calcule a concentração, em mol L–1, da solução 
aquosa de HCl, sabendo que a concentração da 
solução aquosa de NaOH utilizada era 2,0 mol L–1. 
 
11. (FUVEST 2013) Fotólise 
Em uma reação de síntese, induzida por luz vermelha de 
frequência f igual a 4,3X1014 Hz, ocorreu a formação de 
180 g de glicose. Determine 
a) o número N de mols de glicose produzido na reação; 
b) a energia E de um fóton de luz vermelha; 
c) o número mínimo n de fótons de luz vermelha 
necessário para a produção de 180 g de glicose; 
d) o volume V de oxigênio produzido na reação (CNTP). 
 
Note e adote: 
 
Constante de Planck: h = 6,6x10 – 34 J.s. Nessa reação 
são necessários 2800 kJ de energia para a formação de 
um mol de glicose. 
Prof. Pedro Madeira 
 
12. (FUVEST 2008) Método de Job 
Em um exame, para o preenchimento de uma vaga de 
químico, as seguintes fórmulas estruturais foram 
apresentadas ao candidato: 
 
A seguir, o examinador pediu ao candidato que 
determinasse, experimentalmente, o calor liberado ao 
fazer-se a mistura de volumes definidos de duas 
soluções aquosas, de mesma concentração, uma de 
hidróxido de sódio e outra de um dos três ácidos 
carboxílicos apresentados, sem revelar qual deles havia 
sido escolhido. Foi informado ao candidato que, quando 
o ácido e a base reagem na proporção estequiométrica, 
o calor liberado é máximo. 
Os resultados obtidos foram os seguintes: 
Volume da 
solução 
de base/mL 
0 15 30 35 40 45 50 
Volume da 
solução 
de ácido/mL 
50 35 20 15 10 5 0 
Calor liberado 
/J 0 700 1400 1500 1000 500 0 
 
Diante dos resultados obtidos, o examinador pediu ao 
candidato que determinasse qual dos ácidos havia sido 
utilizado no experimento. Para responder, o candidato 
construiu uma tabela e um gráfico do calor liberado 
versus Xbase, definido como: 
, equivalente a 
 
onde n = quantidade de ácido ou de base (em mol); 
V = volume da solução de ácido ou de base (em mL) 
 
a) Reproduza, na página ao lado, a tabela e o gráfico 
que devem ter sido obtidos pelo candidato. Pelos 
pontos do gráfico, podem ser traçadas duas retas, 
cujo cruzamento corresponde ao máximo calor 
liberado. 
b) Determine o valor de xbase que corresponde ao 
ponto de cruzamento das retas em seu gráfico. 
c) Qual foi o ácido escolhido pelo examinador? 
Explique. 
d) Indique qual é o reagente limitante para o 
experimento em que o calor liberado foi 1400 J e para 
aquele em que o calor liberado foi 1500 J. Explique. 
 
HC!
2 2 6 12 6 26H O 6CO energia C H O 6O ;+ + ® +
ácidobase
base
base VV
V
X
+
=
ácidobase
base
base nn
n
X
+
=
92 
 
 
 
APOSTILA 01 DE FÍSICO-QUÍMICA – PROF. PEDRO MADEIRA (2022) 
 
 
 
SEÇÃO ITA / IME 
 
13. (ITA 2011 – Q25) 
Em um frasco de vidro, uma certa quantidade de 
Ba(OH)2.8H2O(s) é adicionada uma quantidade, em 
excesso, de NH4NO3(s), ambos pulverizados. Quando 
os dois reagentes são misturados, observa-se a 
ocorrência de uma reação química. Imediatamente após 
a reação, o frasco é colocado sobre um bloco de 
madeira umedecido, permanecendo aderido a ele por 
um certo período de tempo. 
Escreva a equação química balanceada que representa 
a reação observada. Explique por que o frasco ficou 
aderido ao bloco de madeira, sabendo que o processo 
de dissolução em água do NH4NO3(s) é endotérmico. 
 
14. (IME 2014 – Q37) 
Uma mistura “A”, cuja composição percentual 
volumétrica é de 95% de água e 5% de álcool etílico, 
está contida no bécher 1. Uma mistura “B”, cuja 
composição percentual volumétrica é de 95% de água e 
5% de gasolina, está contida no bécher 2. Essas 
misturas são postas em repouso a 25oC e 1 atm, tempo 
suficiente para se estabelecer, em cada bécher, a 
situação de equilíbrio. Em seguida, aproximam-se 
chamas sobre as superfícies de ambas as misturas. O 
que ocorrerá? 
(A) Nada, ou seja, não ocorrerá combustão em 
nenhuma das superfícies devido à grande 
similaridade de polaridade e densidade entre os 
líquidos. 
(B) Nada, ou seja, não ocorrerá combustão em 
nenhuma das superfícies devido à grande diferença 
de polaridade e densidade entre os líquidos. 
(C) Ambas as superfícies entrarão em combustão, 
simultaneamente, devido à elevada diferença de 
polaridade e densidade entre os três líquidos. 
(D) Ocorrerá combustão somente sobre a superfícielíquida no bécher 1, devido à diferença de 
polaridade e densidade entre os líquidos. 
(E) Ocorrerá combustão somente sobre a superfície 
líquida no bécher 2, devido à diferença de 
polaridade e densidade entre os líquidos. 
 
15. (ITA 2014 – Q12) 
Considere três cubos maciços de 2 cm de aresta, 
constituídos, respectivamente, de Cr, Ni e Ti puros. Os 
três cubos são aquecidos até 80ºC e cada cubo é 
introduzido em um béquer contendo 50 g de água a 
10ºC. Com base nas informações constantes da tabela 
abaixo, assinale a opção que apresenta a relação 
CORRETA entre as temperaturas dos cubos, quando o 
conteúdo de cada béquer atingir o equilíbrio térmico. 
Substância Massa específica 
(g.cm–3) 
Calor específico 
(J.g–1.K–1) 
H2O 1,00 4,18 
Ti 4,54 0,52 
Cr 7,18 0,45 
Ni 8,90 0,44 
A ( ) TCr > TNi > TTi B ( ) TNi = TTi > TCr 
C ( ) TNi > TCr > TTi D ( ) TTi > TCr > TNi 
E ( ) TTi > TCr = TNi 
 
16. (ITA 2014 – Q15) 
São feitas as seguintes afirmações sobre o que Joule 
demonstrou em seus experimentos do século XIX: 
I. A relação entre calor e trabalho é fixa. 
II. Existe um equivalente mecânico do calor. 
III. O calor pode ser medido. 
Das afirmações acima, está(ão) CORRETA(S) apenas 
A ( ) I. 
B ( ) I, II e III. 
C ( ) I e III. 
D ( ) II. 
E ( ) II e III. 
 
17. (ITA 2018 – Q06) 
Deseja-se aquecer 586g de água pura da temperatura 
ambiente até 91ºC, em pressão ambiente. Utilizando um 
forno de micro-ondas convencional que emite radiação 
eletromagnética com frequência de 2,45 GHz e 
considerando a capacidade calorífica da água constante 
e igual a 4,18 Jg–1oC–1 , assinale a alternativa que 
apresenta o número aproximado de fótons necessário 
para realizar este aquecimento. 
A ( ) 3 x 1027 
B ( ) 4 x 1028 
C ( ) 1 x 1029 
D ( ) 5 x 1030 
E ( ) 2 x 1031 
 
93 
 
 
 
APOSTILA 01 DE FÍSICO-QUÍMICA – PROF. PEDRO MADEIRA (2022) 
 
18. (ITA 2022 – Q67) – RESOLVA AGORA 
Considere as seguintes afirmações sobre processos 
termodinâmicos, que podem ocorrer em uma ou mais 
etapas, em que DT se refere à variação de temperatura 
entre os estados inicial e final: 
 
I. Um processo termodinâmico é definido pelo estado 
final e estado inicial do sistema. 
II. DT é sempre nula em um processo isotérmico. 
III. A troca de calor envolvida em um processo 
isotérmico deve ser nula (q=0). 
IV. Todo processo em que DT = 0 é um processo 
isotérmico. 
V. DT = 0 para todo processo em sistema isolado. 
 
Assinale a opção que contém as afirmações ERRADAS: 
A ( ) Apenas I, II e IV 
B ( ) Apenas I, III, IV e V 
C ( ) Apenas I, III e V 
D ( ) Apenas II e IV 
E ( ) III e V 
 
TÓPICO 02: 
PRIMEIRA LEI DA TERMODINÂMICA 
 
SEÇÃO VESTIBULARES 
 
19. (UFG 2014) 
Em um recipiente com paredes perfeitamente 
condutoras de calor encontra-se uma solução altamente 
concentrada de ácido clorídrico à temperatura de 27oC e 
à pressão atmosférica. Certa quantidade de pó de 
magnésio é colocada na solução e, imediatamente 
depois, o recipiente é tampado com um pistão de massa 
desprezível, que fica em contato com a superfície do 
líquido e que pode deslizar sem atrito ao longo do 
recipiente. Quando a situação de equilíbrio é alcançada 
observa-se que o magnésio reagiu completamente com 
o ácido e que o pistão levantou-se em relação à 
superfície da solução devido à produção de gás. 
Sabendo que no processo todo o sistema realizou um 
trabalho de 240 J, e considerando o gás produzido como 
ideal, conclui-se que a massa, em gramas, de magnésio 
inicialmente colocada na solução foi: 
a) 0,243 
b) 0,486 
c) 0,729 
d) 1,215 
e) 2,430 
 
20. (UEM-PAS 2016) 
A formação de CO2(g), segundo a reação 
O2(g) + C(graf) à CO2(g), 
possui uma variação de entalpia ΔH = –395 kJ. Parte 
dessa energia foi utilizada para realizar o processo 
mostrado no diagrama pV no qual se têm 4 mols de um 
gás monoatômico ideal. 
 
Considerando as informações apresentadas, assinale o 
que for correto sobre esse diagrama pV. 
01) A temperatura inicial desse gás monoatômico ideal 
é de aproximadamente 500K. 
02) A temperatura final do gás monoatômico ideal é de 
aproximadamente 900K. 
04) A variação da energia interna do gás monoatômico 
ideal foi de aproximadamente 29,9 kJ. 
08) O trabalho realizado pelo sistema descrito no 
diagrama pV foi de 15kJ. 
16) O calor utilizado pelo sistema descrito no diagrama 
pV foi de aproximadamente 50% daquele gerado 
pela reação química do CO2(g). 
 
21. (UPE 2011) 
Um recipiente cilíndrico, de área de secção reta de 0,100 
m2 contém 20,0 g de gás hélio. Esse recipiente contém 
um êmbolo que pode se mover sem atrito. Uma fonte 
fornece calor ao recipiente a uma taxa constante. Num 
determinado instante, o gás sofre a transformação 
termodinâmica representada no diagrama PV abaixo, e 
o êmbolo se move com velocidade constante 
v=8,31x10–3 m/s Considere que o gás hélio se comporta 
como um gás monoatômico ideal. 
 
 
Depois de decorrido um intervalo de tempo de 25 s, 
analise as proposições a seguir e conclua. 
( ) A variação de temperatura do gás durante o 
processo foi ΔT = 50K. 
( ) O calor específico molar à pressão constante do 
hélio é Cp = 2,5R. 
( ) A energia adicionada ao hélio sob a forma de calor 
durante o processo foi Q = 375R. 
( ) A variação na energia interna do hélio durante o 
processo foi ΔE = 125R. 
( ) O trabalho realizado pelo hélio durante a 
transformação foi W = 250R. 
 
94 
 
 
 
APOSTILA 01 DE FÍSICO-QUÍMICA – PROF. PEDRO MADEIRA (2022) 
 
22. (UPE 2010) 
No diagrama PV, a seguir, está representada uma série 
de processos termodinâmicos. No processo ab, 250 J de 
calor são fornecidos ao sistema, e, no processo bd, 600 
J de calor são fornecidos ao sistema. 
 
Analise as afirmações que se seguem. 
I. O trabalho realizado no processo ab é nulo. 
II. A variação de energia interna no processo ab é 320 
J. 
III. A variação de energia interna no processo abd é 
610 J. 
IV. A variação de energia interna no processo acd é 
560 J. 
É CORRETO afirmar que apenas as(a) afirmações(ão) 
a) II e IV estão corretas. 
b) IV está correta. 
c) I e III estão corretas. 
d) III e IV estão corretas. 
e) II e III estão corretas. 
Prof. Pedro Madeira 
 
23. (UPE 2010) 
O diagrama PV para uma determinada amostra de gás 
está representado na figura a seguir. Se o sistema é 
levado do estado a para o estado b, ao longo do 
percurso acb, fornece-se a ele uma quantidade de calor 
igual a 100 cal, e ele realiza um trabalho de 40 cal. Se, 
por meio do percurso adb, o calor fornecido é de 72 cal, 
então o trabalho realizado vale em cal: 
 
a) 28 
b) 60 
c) 12 
d) 40 
e) 24 
 
 
 
 
 
 
 
24. (UFC 1998) 
Complete a afirmativa abaixo: 
A energia fornecida pelos alimentos é determinada em 
condições de temperatura e pressão constantes, 
constituindo-se numa medida de ______________. 
Assinale a opção escolhida para completar a afirmativa. 
A) Entalpia. B) Capacidade térmica. 
C) Calor específico. D) Calor latente. 
E) Energia interna. 
 
SEÇÃO ITA / IME 
 
25. (ITA 1992 – Q09) 
Dentre as opções seguintes assinale aquela que 
contém, afirmação FALSA relativa ao comportamento de 
gases. 
A ( ) Para uma mesma temperatura e pressão iniciais, 
o calor específico sob volume constante é maior 
do que sob pressão constante. 
B ( ) A energia cinética média das moléculas é 
diretamente proporcional à temperatura absoluta 
e independe da pressão. 
C ( ) Na mesma pressão e temperatura, ar úmido é 
menos denso que ar seco. 
D ( ) No equilíbrio, a concentração de um gás 
dissolvido num liquido é diretamente proporcional 
à pressão parcial do referido gás na fase gasosa 
sobre o líquido. 
E ( ) Na expressão yq = y0(1+ aq), relativa à dilatação 
isobárica de um gás, onde q é a temperatura em 
graus Celsius, foi notado que a = (1 / 273°C) 
independentemente da natureza química do gás. 
 
26. (ITA 1992 – Q13) 
Numa garrafa térmica, de capacidade caloríficadesprezível, são misturados um volume V1 de uma 
solução aquosa 2 molar de ácido clorídrico com um 
volume V2 de uma solução aquosa 1 molar de hidróxido 
de sódio. Antes da mistura, as duas soluções estavam 
na mesma temperatura. Em qual das misturas a seguir 
haverá maior aumento de temperatura? 
A ( ) V1 = 0,10 !; V2 = 0,20 ! 
B ( ) V1 = 0,20 !; V2 = 0,20 ! 
C ( ) V1 = 0,40 !; V2 = 0,40 ! 
D ( ) V1 = 0,20 !; V2 = 0,10 ! 
E ( ) V1 = 0,40 !; V2 = 0,20 ! 
 
PERGUNTA – Comparando as alternativas B e D, em 
qual dessas duas haverá maior aumento de 
ternperatura? Justifique a sua resposta. 
 
27. (ITA 1995 – Q07) 
Em um calorímetro adiabático, com capacidade térmica 
desprezível, são introduzidos, sob pressão constante de 
1 atm, um volume V1 de solução aquosa 1,0 molar de 
ácido clorídrico e um volume V2 de solução aquosa 1,0 
molar de hidróxido de sódio. A reação que ocorre é 
aquela representada pela equação química: 
H+(aq) + OH–(aq) ® H2O(l) 
As misturas efetuadas são as seguintes: 
95 
 
 
 
APOSTILA 01 DE FÍSICO-QUÍMICA – PROF. PEDRO MADEIRA (2022) 
 
I. V1 = 100 ml e V2 = 100 ml e observa-se um aumento 
de temperatura DT1. 
II. V1 = 50 ml e V2 = 150 ml e observa-se um aumento 
de temperatura DT2. 
III. V1 = 50 ml e V2 = 50 ml e observa-se um aumento de 
temperatura DT3. 
Com relação ao efeito térmico que se observa, é correto 
prever que: 
A ( ) DT1 @ DT3 > DT2 B ( ) DT1 > DT2 > DT3 
C ( ) DT1 > DT2 @ DT3 D ( ) DT1 > DT3 > DT2 
E ( ) DT1 @ DT3 @ DT2 
 
28. (ITA 2003 – Q13) 
Considere as seguintes comparações de calores 
específicos dos respectivos pares das substâncias 
indicadas. 
I. tetracloreto de carbono (ℓ, 25oC) > metanol(ℓ, 25oC) 
II. água pura (ℓ, -5oC) > água pura (s, -5oC) 
III. alumina (s, 25oC) > alumínio (s, 25oC) 
IV. isopor (s, 25oC) > vidro de janela (s, 25oC) 
Das comparações feitas, está(ao) CORRETA(S) 
A ( ) apenas I e II B ( ) apenas I, II e III 
C ( ) apenas II D ( ) apenas III e IV 
E ( ) apenas IV 
 
29. (ITA 2004 – Q05) 
Considere as reações representadas pelas seguintes 
equações químicas balanceadas: 
a) C2H5OH(ℓ) + O2(g) → 2 C(s) + 3 H2O(g); 
 ΔHI(T); ΔEI(T), 
b) C2H5OH(ℓ) + 2 O2(g) → 2 CO(g) + 3 H2O(l); 
 ΔHII(T); ΔEII(T), 
sendo ΔH(T) e ΔE(T), respectivamente, a variação da 
entalpia e da energia interna do sistema na temperatura 
T. Assuma que as reações acima são realizadas sob 
pressão constante, na temperatura T, e que a 
temperatura dos reagentes é igual à dos produtos. 
Considere que, para as reações representadas pelas 
equações acima, sejam feitas as seguintes 
comparações: 
I. |ΔEI| = |ΔEII|. 
II. |ΔHI| = |ΔHII|. 
III. |ΔHII| > |ΔEII|. 
IV. |ΔHI| < |ΔEI|. 
 
Das comparações acima, está(ão) CORRETA(S) 
A ( ) apenas I B ( ) apenas I e II 
C ( ) apenas II D ( ) apenas III 
E ( ) apenas IV 
 
30. (ITA 2005 – Q06) 
Um cilindro provido de um pistão móvel, que se desloca 
sem atrito, contém 3,2 g de gás hélio que ocupa um 
volume de 19,0 L sob pressão 1,2 x 105 N.m−2. Mantendo 
a pressão constante, a temperatura do gás é diminuída 
de 15 K e o volume ocupado pelo gás diminui para 18,2 
L. Sabendo que a capacidade calorífica molar do gás 
hélio à pressão constante é igual a 20,8 J K−1 mol−1, a 
variação da energia interna neste sistema é 
aproximadamente igual a 
A ( ) - 0,35 k J B ( ) - 0,25 k J 
C ( ) - 0,20 k J D ( ) - 0,15 k J 
E ( ) - 0,10 k J 
 
31. (ITA 2006 – Q19) 
Uma reação química hipotética é representada pela 
seguinte equação: X(g) + Y(g)→ 3Z(g). Considere que 
esta reação seja realizada em um cilindro provido de um 
pistão, de massa desprezível, que se desloca sem atrito, 
mantendo-se constantes a pressão em 1 atm e a 
temperatura em 25oC. Em relação a este sistema, são 
feitas as seguintes afirmações: 
I. O calor trocado na reação é igual à variação de 
entalpia. 
II. O trabalho realizado pelo sistema é igual a zero. 
III. A variação da energia interna é menor do que a 
variação da entalpia. 
IV. A variação da energia interna é igual a zero. 
V. A variação da energia livre de Gibbs é igual à 
variação de entalpia. 
Então, das afirmações acima, estão CORRETAS 
A ( ) apenas I, II e IV B ( ) apenas I e III 
C ( ) apenas II e V D ( ) apenas III e IV 
E ( ) apenas III, IV e V 
PROF. PEDRO MADEIRA 
 
32. (ITA 2007 – Q01) 
Amostras de massas iguais de duas substâncias, I e II, 
foram submetidas independentemente a um processo 
de aquecimento em atmosfera inerte e a pressão 
constante. O gráfico abaixo mostra a variação da 
temperatura em função do calor trocado entre cada uma 
das amostras e a vizinhança. 
 
Dados: ΔHf e ΔHv representam as variações de entalpia 
de fusão e de vaporização, respectivamente, e cp é o 
calor específico. 
Assinale a opção ERRADA em relação à comparação 
das grandezas termodinâmicas. 
A ( ) ΔHf (I) < ΔHf (II) B ( ) ΔHv (I) < ΔHv (II) 
C ( ) cp,I (s) < cp,II (s) D ( ) cp,II (g) < cp,I (g) 
E ( ) cp,II (ℓ) < cp,I (ℓ) 
Prof. Pedro Madeira 
 
33. (ITA 2007 – Q02) 
Um recipiente aberto contendo inicialmente 30 g de um 
líquido puro a 278 K, mantido à pressão constante de 1 
atm, é colocado sobre uma balança. A seguir, é imersa 
no líquido uma resistência elétrica de 3 Ω conectada, por 
meio de uma chave S, a uma fonte que fornece uma 
96 
 
 
 
APOSTILA 01 DE FÍSICO-QUÍMICA – PROF. PEDRO MADEIRA (2022) 
 
corrente elétrica constante de 2 A. No instante em que a 
chave S é fechada, dispara-se um cronômetro. Após 100 
s, a temperatura do líquido mantém-se constante a 330 
K e verifica-se que a massa do líquido começa a diminuir 
a uma velocidade constante de 0,015 g/s. Considere a 
massa molar do líquido igual a M. 
Assinale a opção que apresenta a variação de entalpia 
de vaporização (em J/mol) do líquido. 
A ( ) 500 M B ( ) 600 M C ( ) 700 M 
D ( ) 800 M E ( ) 900 M 
 
34. (ITA 2007 – Q03) 
Utilizando o enunciado da questão anterior, assinale a 
opção que apresenta o valor do trabalho em módulo (em 
kJ) realizado no processo de vaporização após 180 s de 
aquecimento na temperatura de 330 K. 
A ( ) 4,4 / M B ( ) 5,4 / M C ( ) 6,4 / M 
D ( ) 7,4 / M E ( ) 8,4 / M 
 
35. (ITA 2008 – Q13) 
300 gramas de gelo a 0oC foram adicionados a 400 
gramas de água a 55oC. Assinale a opção CORRETA 
para a temperatura final do sistema em condição 
adiabática. 
Dados: calor de fusão do gelo = 80 cal.g–1; calor 
específico do gelo = 0,50 cal.g–1.K–1; calor específico da 
água líquida = 1 cal.g–1.K–1. 
A ( ) – 4oC B ( ) – 3oC C ( ) 0oC 
D ( ) +3oC E ( ) +4oC 
 
36. (ITA 2008 – Q18) 
Considere que os quatro processos químicos, descritos 
a seguir nos itens I a IV, são realizados isobárica e 
isotermicamente: 
I. KNO3(s) à K+(aq) + NO3
–(aq) 
II. H2O(l) à H2O(g) 
III. C(grafita) à C(diamante) 
IV. 2 Na(s) + ½ O2(g) → Na2O(s) 
Qual das opções abaixo contém os processos químicos 
cuja variação de energia interna é nula? 
A ( ) Apenas I e II B ( ) Apenas I, II e III 
C ( ) Apenas II e III D ( ) Apenas III e IV 
E ( ) Nenhum processo 
 
37. (ITA 2008 – Q26) 
Dois cilindros (I e II) são providos de pistões, cujas 
 massas são desprezíveis e se deslocam sem atrito. Um 
mol de um gás ideal é confinado em cada um dos 
cilindros I e II. São realizados, posteriormente, dois tipos 
de expansão, descritos a seguir: 
 
a) No cilindro I, é realizada uma expansão isotérmica à 
temperatura T, de um volume V até um volume 2V, 
contra uma pressão externa constante P. 
b) No cilindro II, é realizada uma expansão adiabática, 
de um volume V até um volume 2V, contra uma 
pressão externa constante P. 
Determine os módulos das seguintes grandezas: 
variação da energia interna, calor trocado e trabalho 
realizado para os dois tipos de expansão. 
 
38. (ITA 2009 – Q08) 
O diagrama temperatura (T) versus volume (V) 
representa hipoteticamente as transformações pelas 
quais um gás ideal no estado 1 pode atingir o estado 3. 
Sendo DU a variação de energia interna e q a quantidade 
de calor trocadocom a vizinhança, assinale a opção com 
a afirmação ERRADA em relação às transformações 
termodinâmicas representadas no diagrama. 
 
A ( ) |DU12| = |q12| B ( ) |DU13| = |DU23| 
C ( ) |DU23| = |q23| D ( ) |DU23| > |DU12| 
E ( ) q23 > 0 
 
39. (ITA 2009 – Q18) 
Nos gráficos abaixo, cada eixo representa uma 
propriedade termodinâmica de um gás que se comporta 
idealmente. 
 
Com relação a estes gráficos, é CORRETO afirmar que 
A ( ) I pode representar a curva de pressão versus 
volume. 
B ( ) II pode representar a curva de pressão versus 
inverso do volume. 
C ( ) II pode representar a curva de capacidade 
calorífica versus temperatura. 
D ( ) III pode representar a curva de energia interna 
versus temperatura. 
E ( ) III pode representar a curva de entalpia versus o 
produto da pressão pelo volume. 
 
40. (ITA 2010 – Q01) 
A figura ao lado 
apresenta a curva 
de aquecimento 
de 100 g de uma 
substância pura 
genérica no 
estado sólido. 
Sabe-se que calor 
é fornecido a uma 
velocidade 
constante de 500 
cal min–1. Admite-se que não há perda de calor para o 
meio ambiente, que a pressão é de 1 atm durante toda 
a transformação e que a substância sólida apresenta 
apenas uma fase cristalina. Considere que sejam feitas 
as seguintes afirmações em relação aos estágios de 
aquecimento descritos na figura: 
I. No segmento PQ ocorre aumento da energia 
cinética das moléculas. 
97 
 
 
 
APOSTILA 01 DE FÍSICO-QUÍMICA – PROF. PEDRO MADEIRA (2022) 
 
II. No segmento QR ocorre aumento da energia 
potencial. 
III. O segmento QR é menor que o segmento ST porque 
o calor de fusão da substância é menor que o seu 
calor de vaporização. 
IV. O segmento RS tem inclinação menor que o 
segmento PQ porque o calor específico do sólido é 
maior que o calor específico do líquido. 
Das afirmações acima, está(ão) ERRADA(S): 
A ( ) apenas I. B ( ) apenas I, II e III. 
C ( ) apenas II e IV. D ( ) apenas III. 
E ( ) apenas IV. 
 
41. (ITA 2010 – Q19) 
Considere duas reações químicas, mantidas à 
temperatura e pressão ambientes, descritas pelas 
equações abaixo: 
I. H2(g) + 1/2 O2(g) à H2O(g) 
ΙΙ. H2(g) + 1/2 O2(g) à H2O(ℓ) 
Assinale a opção que apresenta a afirmação ERRADA 
sobre estas reações. 
A ( ) As reações I e II são exotérmicas. 
B ( ) Na reação I, o valor, em módulo, da variação de 
entalpia é menor que o da variação de energia 
interna. 
C ( ) O valor, em módulo, da variação de energia 
interna da reação I é menor que o da reação II. 
D ( ) O valor, em módulo, da variação de entalpia da 
reação I é menor que o da reação II. 
E ( ) A capacidade calorífica do produto da reação I é 
menor que a do produto da reação II. 
 
42. (ITA 2011 – Q10) 
São descritos abaixo dois experimentos, I e II, nos quais 
há sublimação completa de uma mesma quantidade de 
dióxido de carbono no estado sólido a 25ºC: 
I – O processo é realizado em um recipiente 
hermeticamente fechado, de paredes rígidas e 
indeformáveis. 
II – O processo é realizado em cilindro provido de um 
pistão, cuja massa é desprezível e se desloca sem 
atrito. 
A respeito da variação da energia interna do sistema 
(DU), calor (q) e trabalho (w), nos experimentos I e II, 
assinale a opção que contém a afirmação ERRADA. 
A ( ) qI > 0 B ( ) |wII| > |wI| C ( ) DUI > DUII 
D ( ) |wII| ≠ 0 E ( ) DUII = qII 
Prof. Pedro Madeira 
 
43. (ITA 2011 – Q14) 
Um sistema em equilíbrio é composto por no mol de um 
gás ideal a pressão Po, volume Vo, temperatura To e 
energia interna Uo. Partindo sempre deste sistema em 
equilíbrio, são realizados isoladamente os seguintes 
processos: 
I. Processo isobárico de To até To / 2. 
II. Processo isobárico de Vo até 2Vo. 
III. Processo isocórico de Po até Po / 2. 
IV. Processo isocórico de To até 2To. 
V. Processo isotérmico de Po até Po / 2. 
VI. Processo isotérmico de Vo até Vo / 2. 
Admitindo que uma nova condição de equilíbrio para 
esse sistema seja atingida em cada processo x (x = I, II, 
III, IV, V e VI), assinale a opção que contém a 
informação ERRADA. 
A ( ) UV = UVI / 2 B ( ) UVI = Uo C ( ) PIV = PVI 
D ( ) TII = 4 TIII E ( ) VI = VV / 4 
 
44. (ITA 2012 – Q21) 
A tabela mostra a variação de entalpia de formação nas 
condições-padrão a 25ºC de algumas substâncias. 
Calcule a variação da energia interna de formação, em 
kJ.mol–1, nas condições-padrão dos compostos 
tabelados. Mostre os cálculos realizados. 
 
Substância DHo
f (kJ.mol–1) 
AgCl(s) –127 
CaCO3(s) –1207 
H2O(l) –286 
H2S(g) –20 
NO2(g) +34 
 
45. (ITA 2013 – Q13) 
Assinale a opção que representa a afirmação 
CORRETA. 
A ( ) Um paciente com calor de 42ºC apresenta-se 
febril. 
B ( ) A adição de energia térmica à água líquida em 
ebulição sob pressão ambiente causa um 
aumento na sua capacidade calorífica. 
C ( ) Na temperatura de –4oC e pressão ambiente, 5g 
de água no estado líquido contêm uma 
quantidade de energia maior do que a de 5g de 
água no estado sólido. 
D ( ) A quantidade de energia necessária para aquecer 
5g de água de 20ºC até 25ºC é igual àquela 
necessária para aquecer 25g de água no mesmo 
intervalo de temperatura e pressão ambiente. 
E ( ) Sob pressão ambiente, a quantidade de energia 
necessária para aquecer massas iguais de 
alumínio (calor específico 0,89 J.g–1.K–1) e de 
ferro (calor específico 0,45 J.g–1.K–1), 
respectivamente, de um mesmo incremento de 
temperatura, DT, é aproximadamente igual. 
 
46. (ITA 2014 – Q29) 
Considere os seguintes dados: 
Entalpia de vaporização da água a 25 °C: 
DvapH = 44 kJ.mol–1 
Massa específica da água a 25 °C: rH2O = 1,0 g.cm–3 
Temperaturas de ebulição a 1 bar: 
Teb(H2O)=100ºC; Teb(H2S)= – 60ºC e Teb(H2Se)= –2ºC; 
Com base nestas informações: 
(a) determine o valor numérico da energia liberada, em 
J, durante a precipitação pluviométrica de 20 mm de 
chuva sobre uma área de (10 x 10) km2. 
(b) justifique, em termos moleculares, por que H2O 
apresenta Teb muito maior que outros calcogenetos 
de hidrogênio. 
(c) como se relaciona, em termos moleculares, a 
elevada Teb(H2O) com a quantidade de energia 
liberada durante uma precipitação pluviométrica? 
98 
 
 
 
APOSTILA 01 DE FÍSICO-QUÍMICA – PROF. PEDRO MADEIRA (2022) 
 
47. (ITA 2015 – Q17) 
São feitas as seguintes comparações sobre as 
capacidades caloríficas de diferentes substâncias puras, 
todas à temperatura ambiente: 
I. A capacidade calorífica da água é menor que a do 
peróxido de hidrogênio. 
II. A capacidade calorífica do bromo é menor que a do 
tetracloreto de carbono. 
III. A capacidade calorífica do metanol é menor que a 
do mercúrio. 
 
Assinale a opção que apresenta a(s) comparação(ões) 
CORRETA(S). 
A ( ) Apenas I 
B ( ) Apenas I e II 
C ( ) Apenas II 
D ( ) Apenas II e III 
E ( ) Apenas III 
 
48. (ITA 2016 – Q16) 
Considere a expansão de um gás ideal inicialmente 
contido em um recipiente de 1 L sob pressão de 10 atm. 
O processo de expansão pode ser realizado de duas 
maneiras diferentes, ambas à temperatura constante: 
I. Expansão em uma etapa, contra a pressão externa 
constante de 1 atm, levando o volume final do 
recipiente a 10 L. 
II. Expansão em duas etapas: na primeira, o gás 
expande contra a pressão externa constante de 5 
atm até atingir um volume de 2L; na segunda etapa, 
o gás expande contra uma pressão constante de 1 
atm atingido o volume final de 10 L. 
 
Com base nestas informações, assinale a proposição 
CORRETA. 
A ( ) O trabalho realizado pelo gás é igual nos dois 
processos de expansão. 
B ( ) O trabalho realizado no primeiro processo é 
metade do trabalho realizado no segundo 
processo. 
C ( ) A variação da energia interna do gás é igual em 
ambos os processos. 
D ( ) A variação da energia interna do gás no primeiro 
processo é metade da do segundo processo. 
E ( ) O calor trocado pelo gás é igual em ambos os 
processos. 
 
49. (ITA 2016 – Q28) 
A toda reação química corresponde uma variação de 
energia interna, DU, e uma variação de entalpia,DH. 
Explique em que condições DU tem valor igual ao de DH. 
 
50. (ITA 2017 – Q15) 
Um motor pulso-jato é uma máquina térmica que pode 
ser representada por um ciclo termodinâmico ideal de 
três etapas: 
I. Aquecimento isocórico (combustão). 
II. Expansão adiabática (liberação de gases). 
III. Compressão isobárica (rejeição de calor a pressão 
atmosférica). 
 
Considerando que essa máquina térmica opere com 
gases ideais, indique qual dos diagramas pressão 
versus volume a seguir representa o seu ciclo 
termodinâmico. 
 
A ( ) 
 
 
B ( ) 
 
C ( ) 
 
D ( ) 
 
 
99 
 
 
 
APOSTILA 01 DE FÍSICO-QUÍMICA – PROF. PEDRO MADEIRA (2022) 
 
 
E ( ) 
 
 
51. (ITA 2017 – Q29) 
Sobre um motor pulso jato como o apresentado na 
Questão anterior, considere verdadeiras as seguintes 
afirmações: 
I. A temperatura de fusão do material que compõe a 
câmara de combustão é 1500 K, e acima de 1200 
K o material do motor começa a sofrer desgaste 
considerável pelos gases de combustão; 
II. O material do motor resiste a pressões de até 30 
atm; 
III. O motor opera, em cada ciclo termodinâmico, com 
0,2 mol de uma mistura de gases com 
comportamento ideal, iniciando o ciclo em pressão 
atmosférica e a temperatura de 300 K. 
 
a) A partir destas informações e considerando que se 
deseja obter, de forma segura, o máximo de trabalho por 
ciclo, quais devem ser a pressão e a temperatura no 
ponto de intersecção entre os processos I e II do ciclo 
termodinâmico (vide Questão anterior)? 
b) Na mistura de gases que opera em cada ciclo há uma 
fração de combustível, o qual tem a reação de 
combustão dada por: 
CH4(g) + 2 O2(g) à CO2(g) + 2 H2O(g) QV = 45 kJ/g 
 
em que QV é o calor liberado a volume constante, por 
grama de metano. Considerando a capacidade calorífica 
molar a volume constante da mistura de gases igual a 
25 J·K–1·mol–1, qual é a massa de metano utilizada pelo 
ciclo projetado no item anterior? 
 
52. (ITA 2021 – Q08) 
Considere a reação de oxirredução não balanceada de 
um mol de sulfato de chumbo com ácido hipocloroso, a 
25ºC. 
1PbSO4(s) + HOCl(aq) → Cl2(g) + PbO2(s) + HSO4
–(aq) + H+(aq) 
Para esta reação, a variação de entalpia padrão é 
∆X<==+19,9 kJ. Sabe-se que o potencial de eletrodo 
padrão da espécie que sofre oxidação é +1,63 V e o da 
espécie que sofre redução é +1,61 V. 
a) Escreva as semirreações, a reação global 
balanceada e o potencial padrão da reação global. 
b) Determine a variação de energia interna da reação 
(∆Y<=), considerando comportamento ideal das 
espécies. 
c) Justifique termodinamicamente a diferença entre os 
valores de ∆X<= e ∆Y<= para a reação acima. 
53. (IME 1991 – Q09) 
A combustão completa de 0,436g de uma mistura de 
carvão, na forma alotrópica de grafite rômbico, realizada 
em atmosfera de oxigênio, elevou a temperatura de 2,00
ℓ d’água do calorímetro de 24,67oC para 25,40oC. 
Desprezando as perdas de calor para as partes 
metálicas do calorímetro, determine a percentagem de 
enxofre na mistura, sabendo que: 
a) a massa específica da água é 1,00 g/cm3; 
b) o calor específico da água é 1,00 cal/g oC; 
c) os calores padrões de formação do CO2 e do SO2 a 
25oC são: (∆Hfo)CO2 = – 94,1 kcal/mol; 
(∆Hfo)SO2 = – 71 kcal/mol 
Prof. Pedro Madeira 
 
54. (IME 1999 – Q05) 
Considerando que 100% do calor liberado na combustão 
de CH4 sejam utilizados para converter 100 kg de água 
a 100C em vapor a 100 0C, calcule o volume de metano 
consumido, medido nas CNTP, supondo que ele se 
comporte como um gás ideal. 
Dados: 
Constante universal dos gases (R) = 0,082 atm.l / mol.K 
Calor latente de vaporização da água = 2260 J / g; 
Calor específico da água = 4,2 J / g.0C 
Calor de combustão do metano = 890 kJ / mol 
 
55. (IME 2001 – Q08) 
Uma mistura de metano e ar atmosférico, a 298K e 1atm, 
entra em combustão num reservatório adiabático, 
consumindo completamente o metano. O processo 
ocorre a pressão constante e os produtos formados 
(CO2, H2O, N2 e O2) permanecem em fase gasosa. 
Calcule a temperatura final do sistema e a concentração 
molar final de vapor d’água, sabendo-se que a pressão 
inicial do CH4 é de 1/16atm e a do ar é de 15/16atm. 
Considere o ar atmosférico constituído somente por N2 
e O2 e o trabalho de expansão desprezível. 
Dados: 
Constante universal dos gases: R=0,082atm L.mol–1K–1 
Entalpia de formação a 298K: 
CO2(g) = - 98.050 cal/mol 
H2O(g) = - 57.800 cal/mol 
CH4(g) = - 17.900 cal/mol 
Variação de entalpia (HºT-Hº298K) em cal/mol: 
T (K) CO2(g) H2O(g) N2(g) O2(g) 
1.700 17.500 13.740 10.860 11.470 
2.000 21.900 17.260 13.420 14.150 
 
56. (IME 2002 – Q07) 
Uma amostra de 0,640 g de naftaleno sólido (C10H8) foi 
queimada num calorímetro de volume constante, 
produzindo somente dióxido de carbono e água. Após 
a reação, verificou-se um acréscimo de 2,4oC na 
temperatura do calorímetro. Sabendo-se que a 
capacidade calorífica do calorímetro era de 2.570 
cal/oC e considerando-se que a variação de pressão foi 
muito pequena, calcule a entalpia de formação do 
naftaleno. Dados: 
1) entalpia de formação do CO2 (g): - 94,1 kcal/mol 
2) entalpia de formação da água (l): - 68,3 kcal/mol 
100 
 
 
 
APOSTILA 01 DE FÍSICO-QUÍMICA – PROF. PEDRO MADEIRA (2022) 
 
57. (IME 2003 – Q06) 
O valor experimental para o calor liberado na queima de 
benzeno líquido a 25oC, com formação de dióxido de 
carbono e água líquida, é 780 kcal/mol. A combustão é 
feita em uma bomba calorimétrica a volume constante. 
Considerando comportamento ideal para os gases 
formados e R = 2,0 cal/mol.K, determine: 
a) o calor padrão de combustão do benzeno a 25oC; 
b) se o calor calculado no item anterior é maior ou 
menor quando a água é formada no estado gasoso. 
Justifique sua resposta. 
 
58. (IME 2005 – Q03) 
O consumo de água quente de uma casa é de 0,489 m3 
por dia. A água está disponível a 10,0oC e deve ser 
aquecida até 60,0oC pela queima de gás propano. 
Admitindo que não haja perda de calor para o ambiente 
e que a combustão seja completa, calcule o volume (em 
m3) necessário deste gás, medido a 25,0oC e 1,00 atm, 
para atender à demanda diária. 
DADOS: 
massa específica da água: 1,00 x 103 kg/m3 
calor específico da água: 1,00 kcal/kgoC 
calores de formação a 298 K a partir de seus elementos: 
C3H8(g) = - 25,0 kcal/mol H2O(g) = - 58,0 kcal/mol 
CO2(g) = - 94,0 kcal/mol 
R = 82,0 x 10-6 m3 atm / K mol 
 
59. (IME 2009 – Q35) 
Foram introduzidos 10 mols de uma substância X no 
interior de um conjunto cilindro-pistão adiabático, sujeito 
a uma pressão constante de 1atm. X reage espontânea 
e irreversivelmente segundo a reação: 
X(s) à 2 Y(g) DH = – 200 cal 
Considere que a temperatura no início da reação é 300 
K e que as capacidades caloríficas molares das 
substâncias X e Y são constantes e iguais a 5,0 cal.mol–
1.K–1 e 1,0 cal.mol–1.K–1, respectivamente. O volume final 
do conjunto cilindro-pistão é (Dado: R = 0,082 
atm.L.mol–1.K–1) 
A ( ) 410,0 L 
B ( ) 492,0 L 
C ( ) 508,4 L 
D ( ) 656,0 L 
E ( ) 820,0 L 
 
60. (IME 2010 – Q07) 
A transformação isovolumétrica de um gás triatômico 
hipotético A3 em outro diatômico A2 envolve a liberação 
de 54 kJ/mol de A3. A capacidade calorífica molar a 
volume constante do gás A2 é de 30 J/mol.K. Após a 
transformação isocórica de todo A3 em A2, determine o 
aumento percentual de pressão em um recipiente 
isolado contendo o gás A3 a 27ºC. Considere que a 
capacidade calorífica molar a volume constante do gás 
A2 não varia com a temperatura e que os gases se 
comportam idealmente. 
 
61. (IME 2012 – Q04) 
Na reação de formação de água líquida, a 1 atm e 298 
K, o módulo da variação da entropia é 39,0 cal.K–1. mol–
1 e o módulo da variação da energia livre de Gibbs é 
56.678 cal.mol–1. Considerando a combustão de 4,00 g 
de hidrogênio, a 1 atm e 298 K, calcule: 
a) a variação de energia interna na formação da água 
líquida; 
b) a variação de energia interna na formação da água 
gasosa; 
c) a variação de energia interna na vaporização de 1,00 
mol de água. 
Considere,ainda, que todos os gases envolvidos 
comportam-se idealmente e que: 
H2O(g) à H2(g) + ½ O2(g) ΔH = 57.800 cal 
 
62. (IME 2013 – Q08) 
Considere um recipiente adiabático conforme a 
ilustração abaixo, no qual 1000 g de uma solução 
aquosa de NaOH, a 30% em massa, e a uma 
temperatura inicial ti = 25oC, são diluídos a 20% em 
massa, com água à mesma temperatura. Calcule a 
temperatura tf da solução após a diluição. 
Dados: 
 - Para o sistema NaOH – água a 25oC: 
 a 30%: H = 104 J/g de solução; cP = 3,54 J/g.oC 
 a 20%: H = 76 J/g de solução; cP = 3,63 J/g.oC 
 - Calor específico da água líquida: cP = 4,18 J/g.oC 
 - Estado de referência para entalpia: 
 água líquida a 0oC. 
 
 
63. (IME 2017 – Q35) 
Um isótopo de cromo, de massa atômica 54, constitui 
53% da massa de um óxido formado exclusivamente 
pelo isótopo e por oxigênio. A partir dessa informação, 
pode-se estimar que a fórmula mínima do óxido e o calor 
específico do cromo-54 são: 
(A) CrO3 e 0,12 cal/(g.oC) 
(B) CrO3 e 0,18 cal/(g,oC) 
(C) Cr2O6 e 0,12 cal/(g.oC) 
(D) Cr2O3 e 0,16 cal/(g.oC) 
(E) Cr4O e 0,18 cal/(g.oC) 
Prof. Pedro Madeira 
 
101 
 
 
 
APOSTILA 01 DE FÍSICO-QUÍMICA – PROF. PEDRO MADEIRA (2022) 
 
TÓPICO 03: SEGUNDA E TERCEIRA LEIS 
DA TERMODINÂMICA 
 
SEÇÃO VESTIBULARES 
 
64. (UECE 2006) 
A partir de seus conhecimentos sobre entropia, marque 
a alternativa que apresenta uma reação onde a variação 
de entropia é negativa: 
A) CaCO3(s) à CaO(s) + CO2(g) 
B) Zn(s) + 2 H+(aq) à H2(g) + Zn2+(aq) 
C) 2 C2H6(g) + 7 O2(g) à 4 CO2(g) + 6 H2O(g) 
D) N2(g) + 3 H2(g) à 2 NH3(g) 
 
65. (UECE 2008) 
O conhecimento da energia livre é aplicado na indústria 
para a redução de gastos e otimização de alguns 
processos de produção. Considerando a reação: 
COCl2(g) CO(g) + Cl2(g) 
e os valores DH = – 108,28 kJ e DS = – 131,63 J/K a 
25oC, assinale a alternativa que indica a temperatura na 
qual a reação é espontânea. 
A) 549ºC B) 627ºC C) 727ºC D) 823ºC 
66. (CEFET 2003) 
Considere a reação abaixo a 1 atm para a qual se sabe 
que DH = + 22 Kcal e DS = + 3 cal/K: 
N2 (g) + O2(g) à 2NO(g) 
a) Calcule DG a 25ºC e 1 atm para a reação. 
b) A reação acima é espontânea em temperaturas 
baixas? Explique. 
c) Discuta o efeito das variações de temperatura sobre 
a posição do equilíbrio químico. 
 
67. (CEFET 2004) 
A geração de metano, a partir do metanol, pode ser 
representada pela reação química abaixo: 
CH3OH(l) ® CH4(g) + 1/2 O2(g) 
a) Esta reação é espontânea a 25ºC e 1 atm? 
b) Qual o sinal de ΔS para esta reação? Qual a sua 
grandeza? 
c) Acima de que temperatura esta reação é 
espontânea? 
d) Que pode impedir a produção de CH4 por essa 
reação? 
Dados: 1. Energia Livre de Formação (kcal/mol) a 25ºC 
e 1 atm. 
 Substância DGfº (kcal/mol) 
CH3OH(l) 
CH4(g) 
- 39,7 
- 12,1 
 
2. Calor de Formação (kcal/mol) a 25ºC e 1 atm. 
Substância DHfº (kcal/mol) 
CH3OH(l) 
CH4(g) 
- 57,0 
- 17,9 
 
68. (CEFET 2005) 
Estime a temperatura na qual o CuSO4 . 5 H2O sofre 
desidratação, formando o anidrido sólido. 
Composto DHf
o (kJ.mol–1) Sm
o(J.K.mol–1) 
CuSO4.5H2O(s) – 2279,7 300,4 
CuSO4(s) – 771,36 109 
H2O(g) – 241,82 188,83 
 
69. (CEFET 2006) 
À temperatura e à pressão constantes, a energia de 
Gibbs (G) constitui um critério de espontaneidade para 
os fenômenos físicos e químicos da natureza. A variação 
da energia de Gibbs, nesse caso, obedece à equação: 
ΔG = ΔH – TΔS 
onde as contribuições, para o valor de ΔG, são 
energética (ΔH) e entrópica (ΔS). Nesse sentido, 
responda aos itens abaixo: 
a) Qual é o valor de ΔG para a ebulição da H2O a 100ºC 
e 1,00 atm? E o sinal do ΔS para o processo? 
b) Quais os sinais de ΔG, ΔH e ΔS para a 
decomposição da H2O(l), em H2(g)
 
e O2(g)? 
c) A vaporização da H2O(l)
 
é um processo espontâneo? 
Explique em termos das variações de G, H e S. 
 
SEÇÃO ITA / IME 
 
70. (ITA 1999 – Q13) 
O processo de decomposição de peróxido de 
hidrogênio, H2O2, resulta na formação de água e 
oxigênio. Em relação a esse processo considere que 
sejam feitas as seguintes afirmações: 
I. Todas as moléculas de H2O2 são reduzidas. 
II. Todas as moléculas de H2O2 são oxidadas. 
III. A variação da energia livre de Gibbs é positiva. 
IV. Metade das moléculas de H2O2 é reduzida e a outra 
metade é oxidada. 
Qual das opções abaixo se refere à(s) afirmação(ões) 
CORRETA(S)? 
A ( ) I B ( ) II C ( ) III D ( ) IV E ( ) III e IV 
 
71. (ITA 2006 – Q26) 
Para cada um dos processos listados abaixo, indique se 
a variação de entropia será maior, menor ou igual a zero. 
Justifique suas respostas. 
a) N2 (g,1 atm,T = 300 K) → N2 (g,0,1 atm,T = 300 K) 
b) C (grafite) → C(diamante) 
c) solução supersaturada → solução saturada 
d) sólido amorfo → sólido cristalino 
e) N2 (g) → N2 (g, adsorvido em sílica) 
Prof. Pedro Madeira 
 
72. (ITA 2009 – Q02) 
No ciclo de Carnot, que trata do rendimento de uma 
máquina térmica ideal, estão presentes as seguintes 
transformações: 
A ( ) duas adiabáticas e duas isobáricas. 
B ( ) duas adiabáticas e duas isocóricas. 
C ( ) duas adiabáticas e duas isotérmicas. 
D ( ) duas isobáricas e duas isocóricas. 
E ( ) duas isocóricas e duas isotérmicas. 
 
102 
 
 
 
APOSTILA 01 DE FÍSICO-QUÍMICA – PROF. PEDRO MADEIRA (2022) 
 
73. (ITA 2012 – Q13) 
Considere as reações representadas pelas seguintes 
equações químicas: 
I. C(s) + 2 H2(g) à CH4(g) 
II. N2O(g) à N2(g) + 1/2 O2(g) 
III. 2 NI3(s) à N2(g) + 3 I2(g) 
IV. 2 O3(g) à 3 O2(g) 
Assinale a opção que apresenta a(s) reação(ões) 
química(s) na(s) qual(is) há uma variação negativa de 
entropia. 
A ( ) apenas I B ( ) apenas II e IV 
C ( ) apenas II e III e IV D ( ) apenas III 
E ( ) apenas IV 
 
74. (ITA 2013 – Q23) 
Em um gráfico de pressão versus volume, representa o 
diagrama do ciclo idealizado por Carnot (máquina 
térmica) para uma transformação cíclica, ininterrupta, e 
sem perdas de calor e de trabalho, e vice-versa. 
Identifique e denomine as quatro etapas dessa 
transformação cíclica. 
 
75. (ITA 2014 – Q04) 
Considere que 1 mol de uma substância sólida está em 
equilíbrio com seu respectivo líquido na temperatura de 
fusão de −183°C e a 1 atm. Sabendo que a variação de 
entalpia de fusão dessa substância é 6,0 kJ ∙ mol−1, 
assinale a opção que apresenta a variação de entropia, 
em J ∙ K−1∙mol−1. 
A ( ) −20 B ( ) −33 C ( ) +50 
D ( ) +67 E ( ) + 100 
 
76. (ITA 2015 – Q15) 
Para uma molécula diatômica, a energia potencial em 
função da distância internuclear é representada pela 
figura ao lado. As linhas horizontais representam os 
níveis de energia vibracional quanticamente permitidos 
para uma molécula diatômica. Uma amostra contendo 
um mol de moléculas diatômicas idênticas, na forma de 
um sólido cristalino, pode ser modelada como um 
conjunto de osciladores para os quais a energia 
potencial também pode ser representada 
qualitativamente pela figura. Em relação a este sólido 
cristalino, são feitas as seguintes proposições: 
 
I. À temperatura de 0 K, a maioria dos osciladores 
estará no estado vibracional fundamental, cujo 
número quântico vibracional, n, é igual a zero. 
II. À temperatura de 0 K, todos os osciladores estarão 
no estado vibracional fundamental, cujo número 
quântico vibracional, n, é igual a zero. 
III. O movimento vibracional cessa a 0 K. 
IV. O movimento vibracional não cessa a 0 K. 
V. O princípio de incerteza de Heisenberg será violado 
se o movimento vibracional cessar. 
Das proposições acima estão CORRETAS 
A ( ) apenas I e III. B ( ) apenas II e III. 
C ( ) apenas I, IV e V. D ( ) apenas II, IV e V. 
E ( ) apenas II, III e V. 
 
77. (ITA 2017 – Q11) 
Em relação às funções termodinâmicas de estado de um 
sistema, assinale a proposição ERRADA. 
A ( ) A variação de energia interna é nula na expansão 
de n mols de um gás ideal a temperatura 
constante. 
B ( ) A variação de energia interna é maior do que zero 
em um processo endotérmico a volume 
constante.C ( ) A variação de entalpia é nula em um processo de 
várias etapas em que os estados inicial e final são 
os mesmos. 
D ( ) A variação de entropia é maior do que zero em um 
processo endotérmico a pressão constante. 
E ( ) A variação de entropia é nula quando n mols de 
um gás ideal sofrem expansão livre contra 
pressão externa nula. 
 
78. (ITA 2018 – Q13) 
Um recipiente de paredes adiabáticas e de volume 
constante contém duas amostras de água pura 
separadas por uma parede também adiabática e de 
volume desprezível. Uma das amostras consiste em 54g 
de água a 25ºC e, a outra, em 126g a 75ºC. Considere 
que a parede que separa as amostras é retirada e que 
as amostras de água se misturam até atingir o equilíbrio. 
Sobre esse processo são feitas as seguintes afirmações: 
I. A temperatura da mistura no equilíbrio é de 323 K. 
II. A variação de entalpia no processo é nula. 
III. A variação de energia interna no processo é nula. 
IV. A variação de entropia no processo é nula. 
 
Assinale a opção que apresenta a(s) afirmação(ões) 
CORRETA(S) sobre a mistura das amostras de água. 
A ( ) Apenas I 
B ( ) Apenas I e II 
C ( ) Apenas II e III 
D ( ) Apenas III e IV 
E ( ) Apenas IV 
 
 
79. (ITA 2021 – Q63) 
Considere as seguintes proposições sobre processos 
termodinâmicos: 
 
I. A entropia permanece constante em um sistema 
fechado que sofre a ação de um processo 
reversível. 
103 
 
 
 
APOSTILA 01 DE FÍSICO-QUÍMICA – PROF. PEDRO MADEIRA (2022) 
 
II. A variação de entropia é nula dentro do sistema 
quando ele opera em um ciclo de Carnot. 
III. O valor absoluto da variação da energia interna de 
um gás ideal numa expansão reversível adiabática 
é maior que numa expansão reversível isotérmica. 
IV. Energia interna é uma propriedade cuja variação 
pode ser medida pelo trabalho adiabático realizado 
entre dois estados. 
 
Das afirmações acima, está(ão) ERRADA(S) apenas 
A ( ) I. 
B ( ) I,II e IV. 
C ( ) II e III. 
D ( ) III. 
E ( ) IV. 
 
80. (IME 1992 – Q07) 
Calcule o valor da variação da energia livre, a 25ºC, para 
a reação representada a seguir. 
2 Na2O2(s) + 2 H2O(l) à 4 NaOH(s) + O2(g) 
Substância Entalpia de formação a 25ºC 
kJ mol–1 
Sº a 25ºC 
J mol–1K–1 
H2O(l) – 286,0 69,69 
Na2O2(s) – 510,9 94,60 
NaOH(s) – 426,8 64,18 
O2(g) 0 205,00 
 
81. (IME 1993 – Q04) 
Calcule a mudança de energia interna, em kJ, para a 
reação de formação de dois moles de SOCl2 (g) a partir 
de S(g), O2 (g) e Cl2 (g) a 298K. 
Dados: ∆P8
1 S(g) = 277 kJ.mol–1 
 ∆P8
1 SOCl2(g) = – 210kJ.mol–1 
 
82. (IME 1994 – Q07) 
A variação da energia livre (DG) e a variação de entropia 
(DS), para a transformação do enxofre ortorrômbico em 
sua forma alotrópica monoclínica, são positivas nas 
CNTP. Responda: 
a) Qual das duas formas alotrópicas é mais estável a 
273 K e 101325 Pa; e 
b) Qual o sinal para a variação de entalpia ( DH) da 
transformação, também a 273 K e 101325 Pa? 
 
83. (IME 1996 – Q09) 
Uma fábrica, que produz cal (Ca(OH)2), necessita 
reduzir o custo da produção para se manter no mercado 
com preço competitivo para seu produto. A direção da 
fábrica solicitou ao departamento técnico o estudo da 
viabilidade de reduzir a temperatura do forno de 
calcinação de carbonato de cálcio, dos atuais 1500 K, 
para 800 K. Considerando apenas o aspecto 
termodinâmico, pergunta-se: o departamento técnico 
pode aceitar a nova temperatura de calcinação? Em 
caso, afirmativo, o departamento técnico pode fornecer 
uma outra temperatura de operação que proporcione 
maior economia? Em caso negativo, qual é a 
temperatura mais econômica para se operar o forno de 
calcinação? 
Dados: 
 
 DS° (J mol-1.K-1) DH° (kJ mol-1) 
CaCO3(s) 92,9 - 1206.9 
CaO(s) 39,8 - 635,1 
CO2(g) 213,6 - 393,5 
Observação: desconsidere a variação das propriedades 
com a temperatura. 
 
84. (IME 2007 – Q38) 
Considere os seguintes processos conduzidos a 25ºC 
e 1 atm: 
(1) 4Fe(s) + 3O2 (g) → 2Fe2O3(s) 
(2) H2O(s) → H2O(l) 
(3) CH4(g) + 2O2 (g) → CO2(g) + 2H2O(g) 
(4) Cu2S(s) → 2Cu(s) + S(s), com ΔG = + 86,2 kJ 
(5) S(s) + O2(g) → SO2(g), com ΔG = − 300,4 kJ 
(6) Cu2S(s) + O2(g) → 2Cu(s) + SO2(g) 
(7) 2NO(g) + O2(g) → 2NO2(g) 
Assinale a afirmativa correta. 
A ( ) Os processos (1), (4) e (5) não são espontâneos. 
B ( ) O processo (2) é exotérmico e apresenta 
variação de entropia positiva. 
C ( ) O processo (3) é endotérmico e apresenta 
variação de entropia negativa. 
D ( ) Os processos (2) e (7) apresentam variação de 
entropia positiva. 
E ( ) Os processos (1), (2) e (6) são espontâneos. 
 
85. (IME 2010 – Q36) 
Considere as supostas variações de entropia (ΔS) nos 
processos abaixo: 
I) cristalização do sal comum (ΔS > 0) 
II) sublimação da naftalina (naftaleno) (ΔS > 0) 
III) mistura de água e álcool (ΔS < 0) 
IV) ferro(s) ferro (l) (ΔS > 0) 
V) ar ar comprimido (ΔS < 0) 
As variações de entropia indicadas nos processos que 
estão corretas são: 
A) I, III e IV B) III, IV e V C) II, III e V 
D) I, II e IV E) II, IV e V 
 
86. (IME 2011 – Q36) 
A entalpia de fusão de uma determinada substância é 
200 kJ/kg, e seu ponto de fusão normal é 27oC. Após a 
solidificação de 3 kg do material, pode-se afirmar que a 
entropia desse sistema: 
A) diminuiu 2 kJ/K B) diminuiu 600 kJ/K 
C) não variou D) aumentou 2 kJ/K 
E) aumentou 600 kJ/K 
 
87. (IME 2013 – Q37) 
O dispositivo a seguir utiliza a radiação solar para 
quantificar variações em propriedades termodinâmicas. 
Este dispositivo é composto por uma lente convergente 
e por um porta-amostras. A lente possui área útil de 80,0 
cm2, absortividade (a) de 20% e transmissividade (t) de 
80%. O porta-amostras possui absortividade de 100% e 
¾¾ ®¾fusão
¾¾¾ ®¾compressão
104 
 
 
 
APOSTILA 01 DE FÍSICO-QUÍMICA – PROF. PEDRO MADEIRA (2022) 
 
volume variável, operando à pressão constante de 1,0 
atm. 
 
 
 
Em um procedimento experimental, injetou-se 0,100 mol 
de uma substância pura líquida no porta-amostras do 
dispositivo. Em seguida, mediu-se um tempo de 15,0 
min para a vaporização total da amostra, durante o qual 
a irradiação solar permaneceu constante e igual a 750 
W/m2. Nesse processo, a temperatura do porta-
amostras estabilizou-se em 351 K. No experimento, o 
calor sensível da amostra e a radiação emitida pelo 
porta-amostras são desprezíveis. Pode-se concluir que 
na vaporização total da substância, as variações de 
entalpia molar padrão e de entropia molar padrão são, 
respectivamente: 
(A) 4,32 kJ/mol e 12,3 J/(mol K) 
(B) 5,40 kJ/mol e 15,4 J/(mol K) 
(C) 43,2 kJ/mol e 123 J/(mol K) 
(D) 54,0 kJ/mol e 154 J/(mol K) 
(E) 31,6 kJ/mol e 90,0 J/(mol K) 
 
88. (IME 2014 – Q34) 
A variação de entropia de um sistema fechado 
constituído por um gás ideal, quando sofre uma 
transformação, pode ser calculada pela expressão 
genérica: 
 
em que os subscritos 1 e 2 representam dois estados 
quaisquer. Assinale a única afirmativa correta. 
(A) Se o estado inicial 1 é diferente do estado final 2, a 
variação da entropia do gás ideal não depende da 
quantidade de gás presente no sistema. 
(B) Se a mudança de estado é isotérmica, a variação 
da entropia é dada por . 
(C) Se o sistema realiza um processo cíclico, a 
variação de entropia é positiva. 
(D) Se a mudança de estado é isobárica, a variação de 
entropia é dada por . 
(E) Se a mudança de estado é isocórica, a variação da 
entropia do sistema é nula. 
 
89. (IME 2016 – Q38) 
Um sistema A transfere, naturalmente, uma determinada 
quantidade de energia, na forma de calor, para um 
sistema B, que envolve totalmente A. Assinale a única 
alternativa correta. 
(A) A entropia do Universo decrescerá. 
(B) A entropia do sistema A crescerá. 
(C) O aumento da entropia do sistema B será maior do 
que o decréscimo da entropia do sistema A. 
(D) O aumento da entropia do sistema B será menor do 
que o decréscimo da entropia do sistema A. 
(E) O aumento da entropia do sistema B será 
necessariamente igual ao decréscimoda entropia 
do sistema A. 
 
90. (IME 2016 – Q40) 
Um sistema é composto por dois balões idênticos 
resistentes, porém não inquebráveis, A e B, os quais 
estão conectados por meio de um tubo, também 
resistente, no qual se encontra uma válvula, tipo 
torneira. Este sistema encontra-se perfeitamente isolado 
termicamente do universo. Inicialmente as condições do 
sistema são as seguintes: temperatura constante; a 
válvula encontra-se fechada; o balão A contém um mol 
de um gás ideal monoatômico; e o balão B encontra-se 
perfeitamente evacuado. No tempo t = 0, a torneira é 
aberta repentinamente, permitindo que o gás ideal se 
expanda em direção ao balão B por um orifício pequeno. 
Indique qual das alternativas abaixo é a correta. 
(A) O balão B quebrar-se-á devido ao impacto do gás 
ideal, liberado bruscamente, contra sua parede. 
(B) O trabalho gerado pela expansão do gás aquecerá 
o sistema. 
(C) O gás em expansão absorverá calor da vizinhança 
fazendo o sistema se resfriar. 
(D) O valor da variação da energia interna ∆U da 
expansão será igual a zero. 
(E) Na expansão, a variação da energia interna ∆U do 
sistema será menor que zero. 
 
91. (IME 2016 – Q05) 
Um bloco de gelo a 0,00oC é colocado em contato com 
um recipiente fechado que contem vapor de água a 100 
oC e 1 atm. Após algum tempo, separa-se o bloco de 
gelo do recipiente fechado. Nesse instante observa-se 
que 25,0 g de gelo foram convertidos em água líquida a 
0,00oC, e que no recipiente fechado existem água 
líquida e vapor d’água em equilíbrio. Considerando que 
o bloco de gelo e o recipiente fechado formam um 
sistema e que só trocam calor entre si, calcule a variação 
de entropia do sistema. DADO: ∆X@ABã= = EEB	[\/[^ 
 
92. (IME 2018 – Q39) 
Considere as seguintes afirmativas: 
I – Uma reacao quimica a temperatura e pressao 
constantes sera espontanea se a variacao da 
energia livre de Gibbs (ΔG) for menor que zero. 
II – Em um sistema reacional onde a unica forma de 
trabalho observavel e o trabalho de expansao, a 
variação da entalpia (ΔH) e igual a quantidade de 
calor liberada ou absorvida pela reacao, a pressao 
constante. 
III – Para uma substancia simples que admite mais de 
uma forma alotropica, nao ha variacao de entalpia 
na conversao de uma forma em outra. 
2 2
1 1
T p
ΔS= nC ln nRlnp T p
-
2
1
p
ΔS= nC lnp p
- 
2
1
T
ΔS= nC lnp T
105 
 
 
 
APOSTILA 01 DE FÍSICO-QUÍMICA – PROF. PEDRO MADEIRA (2022) 
 
Sao corretas: 
(A) Somente I. (B) Somente II. (C) Somente III. 
(D) I e II. (E) I e III. 
 
93. (IME 2019 – Q33) 
Considere as reações abaixo: 
H2(g) + ½ O2(g) → H2O(ℓ) (I) 
H2(g) + ½ O2(g) → H2O(g) (II) 
Assinale a alternativa correta. 
(A) O decréscimo de entropia é menor na reação (I) do 
que na reação (II). 
(B) O acréscimo de entropia na reação (I) é maior do 
que na reação (II). 
(C) O decréscimo de entropia é menor na reação (II) do 
que na reação (I). 
(D) O acréscimo de entropia na reação (II) é maior do 
que na reação (I). 
(E) A variação de entropia é igual em ambas as 
reações. 
 
94. (IME 2019 – Q03) 
Mistura-se a água contida em dois recipientes, 
designados por A e B, de forma adiabática. Cada um 
contém a mesma massa m de água no estado líquido. 
Inicialmente, as temperaturas são T no recipiente A e T 
+ ΔT no recipiente B. Após a mistura, a água atinge a 
temperatura final de equilíbrio térmico. 
Mostre que a variação de entropia do processo de 
mistura é positiva. 
 
Dado: 
ΔS = m Cp ln D#
D$
 , onde T2 e T1 são duas temperaturas 
em dois estados diferentes do processo e cp é o calor 
específico da água, considerado constante. 
 
95. (IME 2020 – Q36) 
O astrônomo britânico Arthur Eddington cunhou o termo 
“seta do tempo” para distinguir uma direção no tempo 
nos fenômenos naturais, ou seja, o fato de que o estado 
2 de um sistema macroscópico ocorre após o estado 1. 
Podemos afirmar que o valor da entropia do estado 2 de 
um sistema fechado que evoluiu a partir do estado 1: 
(A) é igual ao valor da entropia do estado 1. 
(B) é menor que o valor da entropia do estado 1. 
(C) é maior que o valor da entropia do estado 1. 
(D) independe do valor da entropia do estado 1. 
(E) depende do caminho percorrido entre os estados. 
Prof. Pedro Madeira 
 
96. (IME 2022 – Q32) 
Uma reação entre dois líquidos A e B produz dois 
compostos gasosos C e D, de acordo com a 
estequiometria A + B → C + D. Se conduzida a pressão 
e temperatura constantes, pode-se afirmar que: 
(A) a reação será sempre espontânea, se for 
endotérmica. 
(B) a reação será sempre espontânea, se for 
exotérmica. 
(C) a reação será sempre espontânea, 
independentemente de ser exotérmica ou 
endotérmica. 
(D) a reação nunca será espontânea, 
independentemente de ser exotérmica ou 
endotérmica. 
(E) não há como prever a espontaneidade da reação, 
mesmo que informações adicionais sobre o calor de 
reação estejam disponíveis. 
 
97. (IME 2022 – Q03) 
Sob determinadas condições, a água pode ser super-
resfriada, ou seja, permanecer no estado líquido em 
temperaturas inferiores ao seu ponto de congelamento, 
em uma situação termodinamicamente instável. 
Considere um processo em que 5,0 mol de água super-
resfriada a –10ºC e 1,0 atm sejam convertidos em gelo 
à mesma temperatura. Determine a variação de 
entropia: 
a) do sistema; 
b) na vizinhança; e 
c) do universo. 
 
TÓPICO 04: 
FUNDAMENTOS DE TERMOQUÍMICA 
 
SEÇÃO VESTIBULARES 
 
98. (UFC 2001) A ligação tripla é muito forte 
A natureza atua na fixação do nitrogênio de diversas 
maneiras. Uma destas, que é responsável por cerca de 
somente 10% do processo natural total, é proveniente 
da ação da descarga elétrica dos raios sobre a massa 
atmosférica, que transforma o nitrogênio em óxido nítrico 
e, posteriormente, em dióxido de nitrogênio. O NO2, por 
sua vez, reage com a água das chuvas produzindo 
HNO3, que é, então, incorporado ao solo. 
 
Dadas as energias de ligação: 
N2 225 kcal/mol O2 118 kcal/mol 
NO 162 kcal/mol 
Assinale a alternativa correta. 
A) O processo descrito é acompanhado da formação 
seqüenciada de espécies de mais baixos estados 
de oxidação do nitrogênio. 
B) A fixação de nitrogênio é acompanhada de 
processos seqüenciados de redução, conduzindo à 
elevação do estado de oxidação do nitrogênio. 
C) Uma dificuldade admitida para a fixação do 
nitrogênio é a elevada quantidade de energia 
requerida para quebrar a tripla ligação N≡N. 
D) Somente com base nos valores das energias das 
ligações, espera-se que o processo de formação do 
NO seja termoquimicamente espontâneo. 
E) O processo descrito constitui-se de uma fonte 
natural de inibição da formação de chuvas ácidas, 
seguido de neutralização. 
 
99. (UFC 2007) A força dos retículos 
A energia de rede (U) para um composto iônico MX pode 
ser definida como a energia necessária para ocorrer a 
seguinte reação: MX(s) à M+(g) + X–(g) 
2 2 2
2 2 3
O ,h O H ON NO NO HNOn¾¾¾¾® ¾¾¾® ¾¾¾®
106 
 
 
 
APOSTILA 01 DE FÍSICO-QUÍMICA – PROF. PEDRO MADEIRA (2022) 
 
Considere os seguintes compostos: NaF, NaCl, CaF2, 
CaCl2, LiF e LiCl. Com base nas informações, assinale 
a alternativa correta. 
A) Todos os compostos apresentados são espécies 
apolares. 
B) A temperatura de fusão do LiCl é maior que a 
temperatura de fusão do LiF. 
C) A temperatura de fusão do NaF é menor que a 
temperatura de fusão do NaCl. 
D) O módulo da energia de rede do LiCl é maior que o 
módulo da energia de rede do LiF. 
E) O módulo da energia de rede do CaF2 é maior que o 
módulo da energia de rede do CaCl2. 
 
100. (UFC 1997) Hidrogenações são exotérmicas 
A reação de hidrogenação de compostos olefínicos é 
amplamente utilizada na fabricação de margarinas a 
partir de óleos vegetais. 
a) Determine o DHo de hidrogenação, sabendo-se que 
o DHº do C2H4(g) é 12,50 Kcal.mol–1, e DHº do 
C2H6(g) é –20,20 Kcal.mol–1. 
b) Utilizando-se de fórmulas estruturais, mostre a 
equação de hidrogenação catalítica (H2/Pt) do 
Eteno. 
 
101. (UFC 2001)Observe o gás d’água 
O gás d’água, uma importante mistura constituída de CO 
e H2, utilizada na indústria, é preparado através da 
passagem de um fluxo de água gasosa sobre carvão, a 
1000 oC. 
(I) C(s) + H2O(g) à CO(g) + H2(g) 
Dadas as seguintes informações: 
(II) C(s) + O2(g) à CO2(g); DHo = -393,5kJ 
(III) 2H2(g) + O2(g) à 2H2O(g); DHo = -483,6kJ 
(IV) 2CO(g) + O2(g) à 2CO2(g); DHo =-566,0kJ 
A) calcule o valor de DHo para a formação do gás d’água 
(I), e classifique a reação termoquimicamente. 
B) represente as estruturas de Lewis para os agentes 
oxidante e redutor da reação (I), somente os que 
constituem substâncias químicas compostas. 
 
102. (UFC 2003) Comparação entre combustíveis 
A quantidade de energia liberada na queima de 
combustíveis é denominada entalpia de combustão. As 
entalpias de combustão de algumas substâncias são 
dadas na tabela I. Para comparar a eficiência da 
combustão de diferentes combustíveis, são necessárias 
especificações de algumas condições. 
TABELA I 
Substância Entalpia de 
combustão (kJ.mol-1) 
Densidade 
(g/mL) 
Hidrogênio, H2 –286 0,07 (*) 
Metanol , 
CH3OH –726 0,79 
Etanol, 
C2H5OH –1367 0,80 
Octano, C8H18 –5470 0,70 
(*) calculada para o gás comprimido a 0 oC 
Identifique, dentre as substâncias relacionadas na 
tabela I, a que teria a maior eficiência nas seguintes 
situações: 
A) aplicações nas quais o uso da menor massa do 
combustível é o mais importante requerimento, por 
exemplo nos foguetes espaciais. Justifique. 
B) aplicações nas quais o uso do menor volume do 
combustível é o mais importante requerimento, por 
exemplo, nos veículos automotores. Justifique. 
 
103. (FUVEST 2019) 
O tricloreto de nitrogênio (NCℓ3), ou tricloramina, é um 
composto de aparência oleosa muito perigoso quando 
puro, pois se decompõe explosivamente formando N2 e 
Cℓ2. Ele era utilizado, até sua proibição na década de 
1940, para o branqueamento de farinhas comerciais, no 
chamado Processo Agene®. Atualmente, é conhecido 
como um dos subprodutos tóxicos indesejáveis 
formados no processo de desinfecção de piscinas. Ele 
se forma, por exemplo, pela reação do cloro usado nas 
piscinas com ureia proveniente da urina de nadadores 
que as utilizam. 
 
Pede-se: 
 
a) Represente a molécula do NCℓ3 utilizando fórmula de 
Lewis. 
b) Escreva a equação balanceada para a decomposição 
do NCℓ3 em N2 e Cℓ2. Qual é o número de oxidação do 
nitrogênio no reagente e no produto? 
c) Calcule a entalpia da reação de decomposição do 
NCℓ3, considerando os valores de energia de ligação 
fornecidos. A reação é endotérmica ou exotérmica? 
Demonstre seus cálculos e justifique sua resposta. 
 
Note e adote: 
 
Ligação 
Energia de 
ligação 
N≡N 940 
Cℓ–Cℓ 240 
N–Cℓ 200 
 
 
104. (UFC 2006) O processo Haber é exotérmico 
Dadas as reações: 
I. H2 (g) + Cl2 (g) à 2 HCl (g) 
II. N2 (g) + 3 H2 (g) à 2 NH3 (g) 
e as energias de ligação: 
Ligação Entalpia de ligação (kJ/mol) 
H–H 432 
NºN 942 
H–Cl 428 
Cl–Cl 240 
N–H 386 
 
A) Determine o DH para as reações I e II 
B) Baseado apenas nos valores de DH, qual das 
reações é mais favorável? 
Prof. Pedro Madeira 
 
105. (UFC 2008) Será que evapora tudo? 
Considere um recipiente hermeticamente fechado com 
capacidade de 1000 L e a uma temperatura de 27oC, 
(kJ mol)
107 
 
 
 
APOSTILA 01 DE FÍSICO-QUÍMICA – PROF. PEDRO MADEIRA (2022) 
 
onde é adicionado 1 L de água. Despreze os efeitos da 
temperatura sobre a densidade da água. 
Dados: densidade da água = 1g.mL–1; 
pressão de vapor da água a 27oC = 0,035 atm. 
A) Nessas condições, haverá a evaporação completa 
desta massa de água? Justifique numericamente a 
sua resposta, considerando gás com comportamento 
ideal. 
B) Sabendo que o calor de vaporização da água a 
100oC é 40,7 kJ.mol–1, qual deverá ser a quantidade 
de calor necessária para vaporizar 1 L de água? 
 
106. (UNESP 2022) Ciclo de Born-Haber clássico 
A variação de entalpia, associada à formação de um 
cristal iônico sólido a partir de seus íons no estado 
gasoso, é conhecida como energia reticular. Essa 
energia é difícil de ser medida diretamente, mas pode 
ser calculada de forma indireta, utilizando-se a Lei de 
Hess, a partir de outras transformações, cuja variação 
de entalpia é conhecida. Esse caminho para a 
determinação da energia reticular é conhecido como 
ciclo de Born-Haber. O diagrama a seguir mostra as 
etapas desse ciclo para o cloreto de sódio. 
 
 
 
Nesse diagrama, a sublimação do sódio metálico, a 
primeira energia de ionização do elemento sódio e a 
afinidade eletrônica do elemento cloro correspondem, 
respectivamente, aos valores de 
a) DH2, DH3 e DH4 
b) DH1, DH0 e DH5 
c) DH1, DH4 e DH3 
d) DH2, DH4 e DH3 
e) DH1, DH3 e DH4 
 
107. (UECE 2010) Um Born-Haber estranho 
O fluoreto de cálcio, CaF2, usado como fluxo na 
fabricação de ácido; como fundente, na obtenção de HF 
e esmaltação de utensílios de cozinha, pode ser 
preparado seguindo o ciclo de Born Haber a partir do 
cálcio e do gás flúor. Solicitou-se a um aluno para 
elaborar o ciclo, que, após concluído, apresentou um 
equivoco do aluno. 
 
Assinale a alternativa que mostra esse equívoco. 
A) Não são 2xDHAE, mas somente DHAE. 
B) Ao invés de Edis o correto são 2Edis. 
C) As posições de DEret e DHo
f foram trocadas entre si. 
D) Como os 2 elétrons do cálcio são retirados de uma 
única vez não deve ser usado a energia do 1o 
Potencial de Ionização. 
 
108. (CEFET 2003) Hidrazina é combustível de foguete 
Os foguetes espaciais utilizam como propelente uma 
mistura de hidrazina (N2H4), como combustível, e 
peróxido de hidrogênio (H2O2), como oxidante. Esses 
reagentes iniciam a reação pelo simples contato 
(reagentes hipergólicos). A reação em questão é: 
N2H4(ℓ) + 2H2O2(ℓ) ¾® N2(g) + 4H2O(g) 25ºC 
Calcule o calor liberado na reação, na temperatura 
considerada, sabendo que as entalpias padrão de 
formação são as seguintes: 
N2H4(ℓ) = + 12 Kcal/mol 
2H2O2(ℓ) = -46 Kcal/mol 
H2O(g) = -58 Kcal/mol 
 
109. (UNICAMP 1999) Hidrazina é combustível de 
foguete 
A hidrazina (H2N–NH2) tem sido utilizada como 
combustível em alguns motores de foguete. A reação de 
combustão que ocorre pode ser representada, 
simplificadamente, pela seguinte equação: 
H2N–NH2(g) + O2(g) = N2(g) + 2 H2O(g) 
A variação de entalpia dessa reação pode ser estimada 
a partir dos dados de entalpia das ligações químicas 
envolvidas. Para isso, considera-se uma absorção de 
energia quando a ligação é rompida, e uma liberação de 
energia quando a ligação é formada. A tabela abaixo 
apresenta dados de entalpia por mol de ligações 
rompidas. 
 DEret 
 Ca(s) + F2(g) CaF2(s) 
 
 Edis DHo
f 
DHo
sub 
 2xDHAE 
 2F–(g) 2F–(g) 
 DH(1oPI) + 
 Ca(g) Ca2+(g) 
 DH(2oPI) 
108 
 
 
 
APOSTILA 01 DE FÍSICO-QUÍMICA – PROF. PEDRO MADEIRA (2022) 
 
 
a) Calcule a variação de entalpia para a reação de 
combustão de um mol de hidrazina. 
b) Calcule a entalpia de formação da hidrazina sabendo-
se que a entalpia de formação da água no estado 
gasoso é de – 242 kJ mol–1. 
 
110. (UNICAMP 2000) 
Considere uma gasolina constituída apenas de etanol e 
de n-octano, com frações molares iguais. As entalpias 
de combustão do etanol e do n-octano são –1368 e –
5471 kJ/mol, respectivamente. A densidade dessa 
gasolina é 0,72 g/cm3 e a sua massa molar aparente, 
80,1 g/mol. 
a) Escreva a equação química que representa a 
combustão de um dos componentes dessa gasolina. 
b) Qual a energia liberada na combustão de 1,0 mol 
dessa gasolina? 
c) Qual a energia liberada na combustão de 1,0 litro 
dessa gasolina? 
 
111. (UNICAMP 2002) 
Examinando os copos com restos de café e de café com 
leite, Rango observa que apenas o de café apresenta 
impressões digitais,as quais coincidem com as do 
guarda. – Estranho! – disse ele. – Este outro copo não 
apresenta impressões! Talvez alguém usando luvas... – 
Ou talvez uma criança! – emendou Estrondosa. 
A observação de Estrondosa se baseou no fato de que 
a impressão digital de uma criança é composta 
principalmente por ácidos graxos (ácidos orgânicos) de 
cadeia contendo até 13 átomos de carbono, enquanto 
as dos adultos se compõem, principalmente, de ésteres 
contendo 32 átomos de carbono. O gráfico a seguir 
mostra a entalpia de sublimação de ésteres e de ácidos 
orgânicos em função do número de átomos de carbono 
na cadeia. 
 
 
a) Considerando o mesmo número de átomos de 
carbono na molécula, os ácidos apresentam maior 
entalpia de sublimação. Que tipo de interação entre 
suas moléculas poderia justificar esse fato? Explique. 
b) Determine a entalpia de sublimação do éster 
contendo 32 átomos de carbono, admitindo que as 
curvas se comportam do mesmo modo para 
moléculas contendo maior número de átomos de 
carbono. 
Prof. Pedro Madeira 
 
112. (UNICAMP 2005) 
Uma das grandes novidades em comunicação é a fibra 
óptica. Nesta, a luz é transmitida por grandes distâncias 
sem sofrer distorção ou grande atenuação. Para fabricar 
fibra óptica de quartzo, é necessário usar sílica de alta 
pureza, que é preparada industrialmente usando uma 
seqüência de reações cujas equações (não 
balanceadas) estão representadas a seguir: 
I – SiO2(s) + C(s) à Si(s) + CO2(g) 
II – Si(s) + Cl2(g) à SiCl4(g) 
III – SiCl4(g) + O2(g) à SiO2(s) + Cl2(g) 
a) Na obtenção de um tarugo de 300g de sílica pura, 
qual a quantidade de energia (em kJ) envolvida? 
Considere a condição padrão. 
Dados de entalpia padrão de formação em kJ.mol–1: 
SiO2(s) = – 910; CO2 = – 394; SiCl4(g) = – 657. 
b) Com a sílica produzida (densidade = 2,2 g.cm–3), foi 
feito um tarugo que, esticado, formou uma fibra de 
0,06 mm de diâmetro. Calcule o comprimento da fibra 
esticada, em metros. 
PROF. PEDRO MADEIRA 
 
113. (FUVEST 2003) Comparação de isômeros 
O 2-metilbutano pode ser obtido pela hidrogenação 
catalítica, em fase gasosa, de qualquer dos seguintes 
alcenos isoméricos: 
2-metil-2-buteno + H2 à 2-metilbutano; 
 ΔH1=–113 kJ/mol 
2-metil-1-buteno + H2 à 2-metilbutano 
 ΔH2 = –119 kJ/mol 
3-metil-1-buteno + H2 à 2-metilbutano 
 ΔH3 = –127 kJ/mol 
a) Complete o esquema da página ao lado com a 
fórmula estrutural de cada um dos alcenos que 
faltam. Além disso, ao lado de cada seta, coloque o 
respectivo ΔH de hidrogenação. 
b) Represente, em uma única equação e usando 
fórmulas moleculares, as reações de combustão 
completa dos três alcenos isoméricos. 
c) A combustão total de cada um desses alcenos 
também leva a uma variação negativa de entalpia. 
Essa variação é igual para esses três alcenos? 
Explique. 
109 
 
 
 
APOSTILA 01 DE FÍSICO-QUÍMICA – PROF. PEDRO MADEIRA (2022) 
 
 
 
114. (FUVEST 2004) 
O Veículo Lançador de Satélites brasileiro emprega, em 
seus propulsores, uma mistura de perclorato de amônio 
sólido (NH4ClO4) e alumínio em pó, junto com um 
polímero, para formar um combustível sólido. 
a) Na decomposição térmica do perclorato de amônio, 
na ausência de alumínio, formam-se quatro produtos. 
Um deles é a água e os outros três são substâncias 
simples diatômicas, duas das quais são 
componentes naturais do ar atmosférico. Escreva a 
equação balanceada que representa essa 
decomposição. 
b) Quando se dá a ignição do combustível sólido, todo 
o oxigênio liberado na decomposição térmica do 
perclorato de amônio reage com o alumínio, 
produzindo óxido de alumínio (Al2O3). Escreva a 
equação balanceada representativa das 
transformações que ocorrem pela ignição do 
combustível sólido. 
c) Para uma mesma quantidade de NH4ClO4, haverá 
uma diferença de calor liberado se sua 
decomposição for efetuada na presença ou na 
ausência de alumínio. Quanto calor a mais será 
liberado se 2 mols de NH4ClO4 forem decompostos 
na presença de alumínio? Mostre o cálculo. 
Dado: 
Calor de formação do óxido de alumínio = –1,68x103 
kJ/mol 
 
115. (FUVEST 2005) BOX de explosivos 
Define-se balanço de oxigênio de um explosivo, 
expresso em percentagem, como a massa de oxigênio 
faltante (sinal negativo) ou em excesso (sinal positivo), 
desse explosivo, para transformar todo o carbono, se 
houver, em gás carbônico e todo o hidrogênio, se 
houver, em água, dividida pela massa molar do 
explosivo e multiplicada por 100. O gráfico ao lado traz 
o calor liberado na decomposição de diversos 
explosivos, em função de seu balanço de oxigênio. 
Um desses explosivos é o tetranitrato de pentaeritritol 
(PETN, C5H8N4O12). A equação química da 
decomposição desse explosivo pode ser obtida, 
seguindo-se as seguintes regras: 
- Átomos de carbono são convertidos em monóxido de 
carbono. 
- Se sobrar oxigênio, hidrogênio é convertido em água. 
- Se ainda sobrar oxigênio, monóxido de carbono é 
convertido em dióxido de carbono. 
- Todo o nitrogênio é convertido em nitrogênio gasoso 
diatômico. 
a) Escreva a equação química balanceada para a 
decomposição do PETN. 
b) Calcule, para o PETN, o balanço de oxigênio. 
c) Calcule o ΔH de decomposição do PETN, utilizando 
as entalpias de formação das substâncias envolvidas 
nessa transformação. 
d) Que conclusão é possível tirar, do gráfico 
apresentado, relacionando calor liberado na 
decomposição de um explosivo e seu balanço de 
oxigênio? 
Massa Molar PETN = 316 g.mol–1. 
 
Substância PETN(s) CO2(g) CO(g) H2O(g) 
Entalpia 
de 
formação 
kJ mol–1 
–538 –394 –110 –242 
 
Prof. Pedro Madeira 
116. (ENEM PPL 2018) Ele nela, nunca ela nele 
Sobre a diluição do ácido sulfúrico em água, o químico 
e escritor Primo Levi afirma que, “está escrito em todos 
os tratados, é preciso operar às avessas, quer dizer, 
verter o ácido na água e não o contrário, senão aquele 
líquido oleoso de aspecto tão inócuo está sujeito a iras 
furibundas: sabem-no até os meninos do ginásio”. 
 
(furibundo: adj. furioso) 
 
LEVI, P. A tabela periódica. Rio de Janeiro: Relume-
Dumará, 1994 (adaptado). 
 
O alerta dado por Levi justifica-se porque a 
a) diluição do ácido libera muito calor. 
b) mistura de água e ácido é explosiva. 
c) água provoca a neutralização do ácido. 
d) mistura final de água e ácido separa-se em fases. 
e) água inibe a liberação dos vapores provenientes do 
ácido. 
 
SEÇÃO ITA / IME 
 
117. (ITA 1986 – Q16) 
Chamando de H1 a entalpia da mistura de 1 mol de 
C(diam.) + 1 mol de O2(g); chamando de H2 a entalpia 
da mistura de 1 mol de C(graf.) + 1 mol de O2(g); 
chamando de H3 a entalpia de 1 mol de CO2(g) e 
sabendo que 
110 
 
 
 
APOSTILA 01 DE FÍSICO-QUÍMICA – PROF. PEDRO MADEIRA (2022) 
 
(H3–H1)25oC = – 94,50 kcal e (H3–H2)25oC = – 94,05 kcal, 
são feitas as seguintes afirmações: 
I. A queima de 1 quilate de diamante libera mais calor 
do que a de 1 quilate de grafite. 
II. É impossível determinar os valores absolutos de H1, 
H2 e H3, embora seja fácil determinar os valores dos 
DH. 
III. Para a transformação 2 C(graf.) à 2 C(diam.) 
podemos concluir que DH = – 0,90 kcal. 
IV. Admitindo que o calor específico médio da água 
líquida seja 1 cal/(g.oC), o calor necessário para 
aquecer 31,4 kg de água da temperatura ambiente 
(25oC) até a ebulição (P = 1 atm) é praticamente 
igual ao liberado na queima de 3,0 g de C(graf.). 
V. Na presença de excesso de oxigênio, a queima do 
diamante resulta no mesmo produto que a queima 
do grafite. 
Das afirmações feitas é(são) FALSA(S) apenas: 
A ( ) I, II e V B ( ) I e III C ( ) II e V 
D ( ) III e IV E ( ) IV 
 
PERGUNTA. Qual o sentido físico de (H1–H2)? 
 
118. (ITA 1989 – Q24) 
Esta questão se refere à comparação do efeito térmico 
verificado ao se misturarem 100 cm3 de solução aquosa 
0,10 molar de cada um dos ácidos abaixo com 100 cm3 
de solução aquosa 0,10 molar de cada uma das bases 
abaixo. A tabela a seguir serve para deixar claro a 
notação empregada para designar os caloresdesprendidos. 
ácido 
 
base 
HCl HNO3 ácido 
acético 
NaOH | DH11 | | DH12 | | DH13 | 
KOH | DH21 | | DH22 | | DH23 | 
NH4OH | DH31 | | DH32 | | DH33 | 
 
Lembrando que o processo de dissociação de eletrólitos 
fracos é endotérmico, é CORRETO esperar que: 
 
A ( ) | DH33 | seja o maior dos | DH | citados. 
B ( ) | DH11 | = | DH13 | 
C ( ) | DH23 | = | DH33 | 
D ( ) | DH31 | = | DH32 | 
E ( ) | DH21 | > | DH22 | 
 
119. (ITA 1992 – Q05) 
Nitrato de amônio pode explodir porque a sua 
decomposição é exotérmica. Qual das opções a seguir 
contém a equação química, envolvendo este composto 
que representa a reação mais exotérmica? 
A ( ) NH4NO3(c) à 2 N(g) + 4 H(g) + 3 O(g) 
B ( ) NH4NO3(c) à NH3(g) + HNO3(g) 
C ( ) NH4NO3(c) à N2(g) + H2O(g) + H2O2(g) 
D ( ) NH4NO3(c) à N2(g) + 2 H2O(g) + 1/2 O2(g) 
E ( ) NH4NO3(c) à N2(g) + 2 H2(g) + 3/2 O2(g) 
 
120. (ITA 1992 – Q11) 
Assinale qual das reações a seguir é a mais 
endoenergética: 
A ( ) B2(g) à 2B(g) B ( ) C2(g) à 2C(g) 
C ( ) N2(g) à 2N(g) D ( ) O2(g) à 2O(g) 
E ( ) F2(g) à 2F(g) 
Prof. Pedro Madeira 
 
121. (ITA 1992 – P11) 
Relacione a energia de ligação entre átomos de uma 
molécula biatômica com comprimento de onda da luz 
capaz de provocar a sua clivagem homolítica. 
 
122. (ITA 1995 – Q28) 
Sob 1 atm e a 25°C, qual das reações abaixo 
equacionadas deve ser a mais exotérmica? 
A ( ) H2(g) + F2(g) ® 2 HF(g) 
B ( ) H2(g) + Cl2(g) ® 2 HCl(g) 
C ( ) H2(g) + I2(g) ® 2 HI(g) 
D ( ) Br2(g) + I2(g) ® 2 BrI(g) 
E ( ) Cl2(g) + Br2(g) ® 2 ClBr(g) 
 
123. (ITA 1996 – Q09) 
Considere as duas amostras seguintes, ambas puras e 
a 25°C e 1 atm: 
P à 1 litro de propano (g) 
B à 1 litro de butano (g) 
Em relação a estas duas amostras são feitas as 
afirmações seguintes: 
I. P é menos densa que B. 
II. A massa de carbono em B é maior que em P. 
III. O volume de oxigênio consumido na queima 
completa de B é maior que aquele consumido na 
queima completa de P. 
IV. O calor liberado na queima completa de B é maior 
que aquele liberado na queima completa de P. 
V. B contém um número total de átomos maior que P. 
VI. B e P são mais densas que o ar na mesma pressão 
e temperatura. 
Das afirmações acima são CORRETAS: 
A ( ) Todas B ( ) Nenhuma 
C ( ) Apenas I, II e III D ( ) Apenas I, III e V 
E ( ) Apenas II, IV e VI 
 
124. (ITA 1996 – Q30) 
Considere as informações contidas nas seguintes 
equações termoquímicas, todas referentes à 
temperatura de 25°C e pressão de uma atmosfera: 
1. H2O(ℓ) à H2O(g); 
 DH1 = 44,0 kJ / mol. 
2. CH3CH2OH(ℓ) à CH3CH2OH(g); 
 DH2 = 42,6 kJ / mol. 
3. CH3CH2OH(ℓ) + 7/2 O2(g) à 2 CO2(g) + 3 H2O(l); 
 DH3 = – 1366,8 kJ / mol. 
4. CH3CH2OH(ℓ) + 7/2 O2(g) à 2 CO2(g) + 3 H2O(g); 
 DH4 = ? 
5. CH3CH2OH(g) + 7/2 O2(g) à 2 CO2(g) + 3 H2O(l); 
 DH5 = ? 
111 
 
 
 
APOSTILA 01 DE FÍSICO-QUÍMICA – PROF. PEDRO MADEIRA (2022) 
 
6. CH3CH2OH(g) + 7/2 O2(g) à 2 CO2(g) + 3 H2O(g); 
 DH6 = ? 
Em relação ao exposto acima, é ERRADO afirmar que: 
A ( ) As reações representadas pelas equações 1 e 2 
são endotérmicas. 
B ( ) As reações representadas pelas equações 3, 4, 5 
e 6 são exotérmicas. 
C ( ) DH4 = – 1234,8 kJ / mol. 
D ( ) DH5 = – 1324,2 kJ / mol. 
E ( ) DH6 = – 1277,4 kJ / mol. 
 
125. (ITA 1997 – Q16) 
Considere as afirmações sobre os óxidos de nitrogênio 
NO, N2O e NO2: 
I. A formação destes óxidos, a partir de N2 e O2, é 
endotérmica. 
II. Os números de oxidação dos átomos de nitrogênio 
nos óxidos NO, N2O e NO2 são, respectivamente, 
+2, +1, e +4. 
III. O N2O é chamado de gás hilariante. 
IV. O NO é o anidrido do ácido nítrico. 
V. O NO2 é um gás colorido. 
Estão CORRETAS: 
A ( ) Apenas II, IV. B ( ) Apenas III e V. 
C ( ) Apenas I, II, III e V. D ( ) Apenas I, II, IV e V. 
E ( ) Todas. 
 
126. (ITA 1998 – Q18) 
Considere os valores das seguintes variações de 
entalpia (DH) para as reações químicas representadas 
pelas equações I e II, onde (graf) significa grafite. 
I. C(graf) + O2(g) à CO2(g); 
 DH (298 K; 1 atm) = – 393 kJ 
II. CO(g) + 1/2 O2(g) à CO2(g); 
 DH (298 K; 1 atm) = – 283 kJ 
Com base nestas informações e considerando que 
todos DH se referem à temperatura e pressão citadas 
acima, assinale a opção CORRETA: 
A ( ) C(graf) + 1/2 O2(g) à CO(g); DH = + 110 kJ 
B ( ) 2 C(graf) + O2(g) à 2CO(g); DH = – 110 kJ 
C ( ) 2 C(graf) + 1/2 O2(g) à C(graf) + CO(g); 
 DH = + 110 kJ 
D ( ) 2 C(graf) + 2 O2(g) à 2 CO(g) + O2(g); 
 DH = + 220 kJ 
E ( ) C(graf) + O2(g) à CO(g) + 1/2 O2(g); 
 DH = – 110 kJ 
Prof. Pedro Madeira 
 
127. (ITA 2000 – Q17) 
Na temperatura e pressão ambientes, a quantidade de 
calor liberada na combustão completa de 1,00 g de 
etanol (C2H5OH) é igual a 30 J. A combustão completa 
de igual massa de glicose (C6H12O6) libera 15 J. Com 
base nestas informações é CORRETO afirmar que: 
A ( ) a quantidade de calor liberada na queima de 1,00 
mol de etanol é igual a 2 vezes a quantidade de 
calor liberada na queima de 1,00 mol de glicose. 
B ( ) a quantidade de oxigênio necessária para queimar 
completamente 1,00 mol de etanol é igual a 2 
vezes aquela necessária para queimar a mesma 
quantidade de glicose. 
C ( ) a relação combustível / comburente para a queima 
completa de 1,00 mol de etanol é igual a 1/2 da 
mesma relação para a queima completa de 1,00 
mol de glicose. 
D ( ) a quantidade de calor liberada na queima de 
etanol será igual àquela liberada na queima de 
glicose quando a relação massa de etanol / massa 
de glicose queimada for igual a 1/2. 
E ( ) a quantidade de calor liberada na queima de 
etanol será igual àquela liberada na queima de 
glicose quando a relação mol de etanol / mol de 
glicose for igual a 1/2. 
 
128. (ITA 2001 – Q11) 
A figura abaixo mostra como a entalpia dos reagentes e 
dos produtos de uma reação química do tipo 
A(g) + B(g) ® C(g) varia com a temperatura. 
 
Levando em consideração as informações fornecidas 
nesta figura, e sabendo que a variação de entalpia (DH) 
é igual ao calor trocado pelo sistema à pressão 
constante, é ERRADO afirmar que 
A ( ) na temperatura T1 a reação ocorre com liberação 
de calor. 
B ( ) na temperatura T1 a capacidade calorífica dos 
reagentes é maior que a dos produtos. 
C ( ) no intervalo de temperatura compreendido entre 
T1 e T2, a reação ocorre com absorção de calor 
(DH > zero). 
D ( ) o DH, em módulo, da reação aumenta com o 
aumento de temperatura. 
E ( ) tanto a capacidade calorífica dos reagentes como 
a dos produtos aumenta com o aumento da 
temperatura. 
 
129. (ITA 2001 – Q29) 
A 25ºC e pressão de 1 atm, a queima completa de um 
mol de n-hexano produz dióxido de carbono e água no 
estado gasoso e libera 3883 kJ, enquanto que a queima 
completa da mesma quantidade de n-heptano produz as 
mesmas substâncias no estado gasoso e libera 4498 kJ. 
A) Escreva as equações químicas, balanceadas, para 
as reações de combustão em questão. 
B) Utilizando as informações fornecidas no enunciado 
desta questão, faça uma estimativa do valor do calor 
de combustão do n-decano. Deixe claro o raciocínio 
utilizando na estimativa realizada. 
C) Caso a água formada na reação de combustão 
estivesse no estado líquido, a quantidade de calor 
liberado seria MAIOR, MENOR OU IGUAL a 3383 
kJ? Por quê? 
112 
 
 
 
APOSTILA 01 DE FÍSICO-QUÍMICA – PROF. PEDRO MADEIRA (2022) 
 
130. (ITA 2002 – Q05) 
A figura abaixo mostra como a capacidade calorífica, CP, 
de uma substância varia com a temperatura, sob 
pressão constante. 
 
 
Considerando as informações mostradas na firgura 
acima, é ERRADO afirmar que 
A ( ) a substância em questão, no estado sólido, 
apresenta mais de uma estrutura cristalina 
diferente. 
B ( ) a capacidade calorífica da substância no estado 
gasoso é menor do que aquela no estado líquido. 
C ( ) quer esteja a substância no estado sólido, líquido 
ou gasoso, sua capacidade calorífica aumenta 
com o aumento da temperatura. 
D ( ) caso a substãncia se mantenha no estado líquidoem temperaturas inferiores a Tf, a capacidade 
calorífica da substância líquida é maior do que a 
capacidade calorífica da substância na fase sólida 
estável em temperaturas menores do que Tf. 
E ( ) a variação de entalpia de uma reação envolvendo 
a substância em questão no estado líquido 
aumenta com o aumento da temperatura. 
Prof. Pedro Madeira 
131. (ITA 2004 – Q01) 
Qual das opções a seguir apresenta a equação química 
balanceada para a reação de formação de óxido de ferro 
(II) sólido nas condições-padrão? 
A ( ) Fe(s) + Fe2O3(s) → 3FeO(s). 
B ( ) Fe(s) + 1/2 O2(g) → FeO(s). 
C ( ) Fe2O3(s) → 2FeO(s) + 1/2 O2(g). 
D ( ) Fe(s) + CO(g) → FeO(s) + C(graf). 
E ( ) Fe(s) + CO2(g) → FeO(s) + C(graf) + 1/2 O2(g). 
 
132. (ITA 2004 – Q26) 
O gráfico abaixo mostra a variação, com o tempo, da 
velocidade de troca de calor durante uma reação 
química. Admita que 1 moI de produto tenha se formado 
desde o início da reação até o tempo t = 11 min. 
Utilizando as informações contidas no gráfico, 
determine, de forma aproximada, o valor das 
quantidades abaixo, mostrando os cálculos realizados. 
a) Quantidade, em mols, de produto formado até t = 4 
min. 
b) Quantidade de calor, em kJ mol–1, liberada na reação 
até t = 11 min. 
 
133. (ITA 2005 – Q08) 
Assinale a opção que contém a substância cuja 
combustão, nas condições-padrão, libera maior 
quantidade de energia. 
A ( ) Benzeno B ( ) Ciclohexano 
C ( ) Ciclohexanona D ( ) Ciclohexeno 
E ( ) n-Hexano 
 
134. (ITA 2005 – Q21) 
Qualitativamente (sem fazer contas), como você explica 
o fato de a quantidade de calor trocado na vaporização 
de um mol de água no estado líquido ser muito maior do 
que o calor trocado na fusão da mesma quantidade de 
água no estado sólido? 
 
135. (ITA 2006 – Q05) 
Considere as seguintes afirmações a respeito da 
variação, em módulo, da entalpia (ΔH) e da energia 
interna (ΔU) das reações químicas, respectivamente 
representadas pelas equações químicas abaixo, cada 
uma mantida a temperatura e pressão constantes: 
I. H2O(g) + 1/2 O2(g) → H2O2(g); |DHI| > |DUI| 
II. 4 NH3(g) + N2(g) → 3 N2H4(g); |ΔHII| < |ΔUII| 
III. H2(g) + F2(g) → 2 HF(g); |ΔHIII| > |ΔUIII| 
IV. HCl(g) + 2 O2(g) → HClO4; |DHIV| < |DUIV| 
V. CaO(s) + 3 C(s) → CO(g) + CaC2(s); |DHV| > |DUV| 
Das afirmações acima, estão CORRETAS 
A ( ) apenas I, II e V B ( ) apenas I, III e IV 
C ( ) apenas II, IV e V D ( ) apenas III e V 
E ( ) todas 
 
136. (ITA 2006 – Q25) 
Uma substância A apresenta as seguintes propriedades: 
Temperatura de fusão a 1 atm = − 20oC 
Temperatura de ebulição a 1 atm = 85oC 
Variação de entalpia de fusão = 180 J g−1 
Variação de entalpia de vaporização = 500 J g−1 
Calor específico de A(s) = 1,0 J g−1 oC−1 
Calor específico de A(ℓ) = 2,5 J g−1 oC−1 
Calor específico de A(g) = 0,5 J g−1 oC−1 
À pressão de 1 atm, uma amostra sólida de 25 g da 
substância A é aquecida de −40oC até 100oC, a uma 
velocidade constante de 450 J min−1. Considere que 
todo calor fornecido é absorvido pela amostra. Construa 
o gráfico de temperatura (oC) versus tempo (min) para 
todo o processo de aquecimento considerado, indicando 
claramente as coordenadas dos pontos iniciais e finais 
de cada etapa do processo. Mostre os cálculos 
necessários. 
Prof. Pedro Madeira 
113 
 
 
 
APOSTILA 01 DE FÍSICO-QUÍMICA – PROF. PEDRO MADEIRA (2022) 
 
137. (ITA 2007 – Q14) 
Assinale a opção que indica a variação CORRETA de 
entalpia, em kJ/mol, da reação química a 298,15 K e 1 
bar, representada pela seguinte equação: 
C4H10(g) à C4H8(g) + H2(g). 
Dados eventualmente necessários: 
∆Hf
o`C4H8(g)a=	 − 11,4; ∆Hf
o`CO2(g)a=	 − 393,5; 
∆Hf
o`H2O(l)a= − 285,8; ∆Hc
o`C4H10(g)a= − 2.877,6, em 
que ∆Hf
o e ∆Hc
o, em kJ/mol, representam as variações de 
entalpia de formação e de combustão a 298,15 K e 1 
bar, respectivamente. 
A ( ) – 3.568,3 B ( ) – 2.186,9 
C ( ) + 2.186,9 D ( ) + 125,4 
E ( ) + 114,0 
 
138. (ITA 2008 – Q19) 
Assinale a opção ERRADA que apresenta (em kJ/mol) a 
entalpia padrão de formação (ΔHf) da substância a 25oC. 
A ( ) ΔHf (H2(g)) = 0 B ( ) ΔHf (F2(g)) = 0 
C ( ) ΔHf
 
(N2(g)) = 0 D ( ) ΔHf
 
(Br2(g)) = 0 
E ( ) ΔHf
 
(Cl2(g)) = 0 
 
139. (ITA 2009 – Q26) 
São dadas as seguintes informações: 
I. O polietileno é estável até aproximadamente 340oC. 
Acima de 350oC ele entra em combustão. 
II. Para reduzir ou retardar a propagação de chama em 
casos de incêndio, são adicionados retardantes de 
chama à formulação dos polímeros. 
III. O Al(OH)3
 
pode ser usado como retardante de 
chama. A aproximadamente 220oC, ele se 
decompõe, segundo a reação 2 Al(OH)3(s) à 
Al2O3(s) + H2O(g), cuja variação de entalpia (ΔH) 
envolvida é igual a 1170 J g–1. 
IV. Os três requisitos de combustão de um polímero 
são: calor de combustão, combustível e oxigênio. Os 
retardantes de chama interferem no fornecimento de 
um ou mais desses requisitos. 
Se Al(OH)3 for adicionado a polietileno, cite um dos 
requisitos de combustão que será influenciado por cada 
um dos parâmetros abaixo quando a temperatura 
próxima ao polietileno atingir 350oC. Justifique 
resumidamente sua resposta. 
a) Formação de Al2O3(s) 
b) Formação de H2O(g) 
c) ΔH de decomposição do Al(OH)3 
 
140. (ITA 2010 – Q16) 
Sabe-se que a 25°C as entalpias de combustão (em kJ 
mol−1) de grafita, gás hidrogênio e gás metano são, 
respectivamente: –393,5; –285,9 e –890,5. Assinale a 
alternativa que apresenta o valor CORRETO da entalpia 
da seguinte reação: C(grafita) + 2 H2(g) à CH4(g) 
A ( ) –211,1 kJ mol–1 B ( ) –74,8 kJ mol–1 
C ( ) 74,8 kJ mol–1 D ( ) 136,3 kJ mol–1 
E ( ) 211,1 kJ mol–1 
 
141. (ITA 2011 – Q05) 
Considere a energia liberada em 
I. combustão completa (estequiométrica) do octano e 
em 
II. célula de combustível de hidrogênio e oxigênio. 
Assinale a opção que apresenta a razão CORRETA 
entre a quantidade de energia liberada por átomo de 
hidrogênio na combustão do octano e na célula de 
combustível. 
Dados: Energia de ligação, em kJ.mol–1: 
C–C 347 H–H 436 
C–H 413 H–O 464 
C=O 803 O=O 498 
A ( ) 0,280 
B ( ) 1,18 
C ( ) 2,35 
D ( ) 10,5 
E ( ) 21,0 
 
142. (ITA 2012 – Q03) 
A reação de sulfonação do naftaleno ocorre por 
substituição eletrofílica nas posições a e b do composto 
orgânico, de acordo com o diagrama de coordenada de 
reação a 50ºC. 
 
Com base neste diagrama, são feitas as seguintes 
afirmações: 
I. A reação de sulfonação do naftaleno é endotérmica. 
II. A posição a do naftaleno é mais reativa do que a de 
b. 
III. O isômero b é mais estável que o isômero a. 
Das afirmações acima, está(ão) CORRETA(S) apenas 
A ( ) I B ( ) I e II C ( ) II D ( ) II e III E ( ) III 
 
143. (ITA 2012 – Q26) 
Considere a reação de combustão do composto X, de 
massa molar igual a 27,7 g.mol–1, representada pela 
seguinte equação química balanceada: 
X(g) + 3 O2(g) à Y(s) + 3 H2O(g); 
 DHo
c= –2035 kJ.mol–1 
Calcule o valor numérico, em kJ, da quantidade de calor 
liberado na combustão de: 
a) 1,0 x 103 g de X 
b) 1,0 x 102 mol de X 
c) 2,6 x 1022 moléculas de X 
d) uma mistura de 10,0 g de X e 10,0 g de O2 
 
144. (ITA 2013 – Q09) 
100 gramas de água líquida foram aquecidos utilizando 
o calor liberado na combustão completa de 0,25 gramas 
de etanol. Sabendo que a variação da temperatura da 
água foi de 12,5ºC, assinale a alternativa que apresenta 
o valor CORRETO para a entalpia molar de combustão 
do etanol. Considere que a capacidade calorífica da 
água é igual a 4,18 kJ.kg–1.oC–1 e que a energia 
114 
 
 
 
APOSTILA 01 DE FÍSICO-QUÍMICA – PROF. PEDRO MADEIRA (2022) 
 
liberada na combustão do etanol foi utilizada 
exclusivamente no aquecimento da água. 
A ( ) – 961 kJ B ( ) – 5,2 kJ 
C ( ) + 4,2 kJ D ( ) + 5,2 kJ 
E ( ) + 961 kJ 
 
145. (ITA 2015 – Q10) 
Para determinar a entalpia de vaporização do composto 
hipotético MX4(ℓ), o mesmo foi colocado num recipiente 
equipado com uma serpentina de aquecimento resistivo,a 80ºC e sob pressão de 1,0 bar. Para a manutenção da 
temperatura, foi utilizada uma fonte de 30 V com 
passagem de corrente de 900 mA durante 30s, tendo 
sido vaporizados 2,0g de MX4(ℓ). Sabendo que a massa 
molar desse composto é 200 g mol–1, assinale a opção 
que apresenta a entalpia molar de vaporização em kJ 
mol–1, a 80ºC. 
A ( ) 4,1 B ( ) 8,1 C ( ) 81 
D ( ) 405 E ( ) 810 
 
146. (ITA 2016 – Q13) 
Considere as entalpias padrão de formação dos 
seguintes compostos: 
 
 CH4(g) O2(g) CO2(g) H2O(g) 
DHfo/kJ.mol–1 –74,81 zero –393,51 –285,83 
 
Sabendo que a capacidade calorífica da água, à pressão 
constante, vale 75,9 J.mol–1 e que sua entalpia de 
vaporização é igual a 40,66 kJ.mol–1, assinale a 
alternativa que melhor corresponda ao número de mols 
de metano necessários para vaporizar 1 L de água pura, 
cuja temperatura inicial é 25ºC, ao nível do mar. 
A ( ) 1,0 B ( ) 2,0 C ( ) 2,9 
D ( ) 3,8 E ( ) 4,7 
Prof. Pedro Madeira 
 
147. (ITA 2016 – Q24) 
Dadas as informações: 
I. O poder calorífico de um combustível representa a 
quantidade de calor gerada na combustão por 
unidade de massa. 
II. O poder calorífico do H2(g) é aproximadamente 3 
vezes o da gasolina. 
III. O calor latente de ebulição do H2(ℓ) é desprezível 
frente ao poder calorífico do H2(g). 
IV. A massa específica do H2(ℓ) é de 0,071 g.cm–3 e a da 
gasolina é de 0,740 g.cm–3. 
 
Com base nestas informações, determine o valor 
numérico: 
a) da massa de 45 L de gasolina. 
b) do volume de H2(ℓ) que, ao sofrer combustão, fornece 
a mesma quantidade de calor liberada na combustão 
de 45 L de gasolina. 
c) do volume que o H2 ocuparia se estivesse na forma 
de gás, à pressão de 1 bar e a 25ºC. 
 
148. (ITA 2017 – Q27) 
Considere que a radiação de comprimento de onda igual 
a 427 nm seja usada no processo de fotossíntese para 
a produção de glicose. Suponha que esta radiação seja 
a única fonte de energia para este processo. Considere 
também que o valor da variação de entalpia padrão da 
reação de produção de glicose, a 
25ºC, seja igual a +2808 kJ·mol–1. 
 
a) Escreva a equação que representa a reação química 
de produção de um mol de glicose pelo processo de 
fotossíntese. 
b) Calcule a variação de entalpia envolvida na produção 
de uma molécula de glicose, via fotossíntese, a 25oC. 
c) Calcule a energia de um fóton de radiação com 
comprimento de onda de 427 nm. 
d) Quantos destes fótons (427 nm), no mínimo, são 
necessários para produzir uma molécula de glicose? 
 
149. (ITA 2018 – Q19) 
O perclorato de amônio (PA) é um dos componentes 
mais utilizados em propelentes de foguetes. Para 
aperfeiçoar seu desempenho, hidrogênio pode ser 
utilizado como aditivo. Considere dadas as entalpias de 
combustão destas espécies: ΔHc,PA = – 189 kJ mol–1; 
ΔHc,H2 = – 286 kJ mol–1. 
Com base nessas informações, assinale a opção que 
apresenta a equação linear da variação de entalpia de 
combustão da mistura de PA com H2 em função da 
quantidade de H2. 
A ( ) y = –0,48x + 189 
B ( ) y = –0,48x – 189 
C ( ) y = –0,48x + 208 
D ( ) y = –0,97x – 189 
E ( ) y = –0,97x – 208 
 
150. (ITA 2019 – Q01) 
Considere reações de combustão do etanol. 
a) Escreva a equação química balanceada para a 
reação com oxigênio puro. 
b) Escreva a equação química balanceada para a 
reação com ar atmosférico. 
c) Escreva a equação química balanceada para a 
reação com 50% da quantidade estequiométrica de 
ar atmosférico. 
d) Classifique as reações dos itens a), b) e c) em ordem 
crescente de variação de entalpia reacional. 
 
151. (ITA 2019 – Q02) 
Uma determinada quantidade de um composto A foi 
misturada a uma quantidade molar três vezes maior de 
um composto B, ou seja, A + 3B. Essa mistura foi 
submetida a dois experimentos de combustão (I e II) 
separadamente, observando-se: 
 
I. A combustão dessa mistura A + 3B liberou 550 kJ de 
energia. 
II. A combustão dessa mistura A + 3B, adicionada de 
um composto C em quantidade correspondente a 
25% em mol do total da nova mistura, liberou 814 kJ 
de energia. 
 
Considerando que os compostos A, B e C não reagem 
entre si, determine os valores numéricos 
a) da quantidade, em mol, de A, B e C. 
115 
 
 
 
APOSTILA 01 DE FÍSICO-QUÍMICA – PROF. PEDRO MADEIRA (2022) 
 
b) do calor de combustão, em kJ mol−1, do composto 
C, ΔHc(C). 
Dados: ΔHc(A) = −700 kJmol−1; ΔHc(B) = −500 kJ mol−1. 
 
152. (ITA 2019 – Q08) 
Considere as variações de entalpia de processo abaixo 
tabeladas. 
 
Processo ΔH (kJ.mol–1) 
Ionização do Nao 495,8 
Energia de ligação Cl–Cl 242,6 
Entalpia de vaporização do Nao 97,4 
Afinidade eletrônica do Cl -349 
Entalpia de rede do NaCl -787 
 
a) Esboce o diagrama de Born-Haber para a formação 
do NaCl(s) a partir de Na0(s) e Cl2(g) e calcule a 
variação de entalpia de formação do NaCl(s). 
b) Sabe-se que o valor absoluto (em módulo) da 
entalpia de rede do CaO(s) é maior do que a do 
NaCl(s). Explique por quê. 
 
153. (ITA 2020 – Q02) 
Os biodigestores possibilitam o reaproveitamento de 
detritos convertendo material orgânico em metano, que 
é utilizado como combustível em sistemas de geração 
de energia. Um laticínio utiliza a queima do metano para 
aquecer 1 m3/h de água, de 25ºC a 100ºC em uma 
caldeira que opera a 1 atm. Sabendo-se que 25% do 
calor produzido no processo é perdido e que, nessas 
condições, a combustão completa do metano produz 
água líquida, determine 
a) a entalpia molar da combustão do metano; 
b) a taxa de calor necessária para aquecer a água; 
c) a vazão de metano, em kg/h, que deve alimentar a 
caldeira. 
Dados: DHf
o(CH4(g)) = –17,9 kcal mol–1; 
DHf
o(CO2(g)) = –94,1 kcal mol–1; 
DHf
o(H2O(g)) = –57,9 kcal mol–1; 
DHeb
o(H2O(ℓ)) = –10,5 kcal mol–1; 
CP
o(H2O(ℓ)) = 1 cal g–1 oC–1; r(H2O(l)) = 1 g cm–3 
 
154. (ITA 2021 – Q04) 
O poder calorífico é um indicativo do potencial 
energético dos combustíveis, sendo que a diferença 
entre o poder calorífico superior (PCS) e o poder 
calorífico inferior (PCI) equivale à energia necessária A 
vaporização da água formada numa reação de 
combustão completa. Sabe-se que o PCS do metano é 
55 MJ.kg–1 e do etanol é 30 MJ.kg–1 e que a entalpia de 
vaporização da água é ∆Hvap,H2O = 44 kJ.mol–1. 
a) Calcule os valores do PCI do metano e do etanol, em 
kJ.mol–1. 
b) Sabendo que o gás natural é composto 
principalmente por metano e que os outros 
componentes possuem PCS muito inferiores ao 
deste gás, estime a porcentagem em massa de 
metano presente em um gás natural cujo PCS = 52 
MJ.kg–1. 
c) Explique por que o PCS do metano é muito superior 
ao do etanol. 
155. (ITA 2022 – Q57) 
Sobre a energia reticular, assinale alternativa ERRADA. 
A ( ) A energia reticular é a energia envolvida na 
formação do sólido cristalino quando o cristal 
sólido se forma a partir de íons separados no 
estado gasoso. 
B ( ) O ciclo de Born-Haber é um método usado para 
determinação do valor da energia reticular. 
C ( ) A energia reticular é função das cargas dos íons 
e da distância entre eles. 
D ( ) Um sólido que sofre mudança de estrutura 
cristalina mantém constante sua energia reticular. 
E ( ) A energia reticular representa a soma entre as 
forças de atração e repulsão eletrostática. 
 
156. (ITA 2022 – Q62) 
Sistemas compostos por água e tensoativos em 
diferentes proporções, depois de homogeneizados, 
passam por um processo termodinâmico quando 
atingem temperaturas em torno de 0°C. A variação de 
entalpia (DH) desse processo foi determinada para cada 
mistura em função da composição do sistema, conforme 
apresentado no gráfico. Considere que o ponto de fusão 
do tensoativo puro é menor que -20ºC e o calor latente 
de fusão da água pura é 334 J.g–1. Sobre esses sistemas 
são feitas as seguintes afirmações: 
 
 
I. O DH refere-se à transição de fase do tensoativo. 
II. O calor latente de fusão do tensoativo puro é -180 
J.g–1. 
III. Até 35% em massa de água pode se apresentar na 
forma associada à substância e não funde.IV. O DH é proporcional à quantidade de água não 
associada ao tensoativo. 
 
Com base no gráfico e nas informações do enunciado, 
assinale a opção que indica a(s) afirmação(ões) 
CORRETA(S). 
 
A ( ) Apenas I e II 
B ( ) Apenas I, II e IV 
C ( ) Apenas II 
D ( ) Apenas III e IV 
E ( ) Todas 
 
116 
 
 
 
APOSTILA 01 DE FÍSICO-QUÍMICA – PROF. PEDRO MADEIRA (2022) 
 
157. (ITA 2022 – Q07) – RESOLVA AGORA 
Considere as seguintes informações: 
I. Primeira energia de ionização do cálcio: 
590 kJ.mol–1 
II. Segunda energia de ionização do cálcio: 
1145 kJ.mol–1 
III. Afinidade eletrônica do cloro: 
-340 kJ.mol–1 
IV. Entalpia de solubilização do cloreto de cálcio: 
-81 kJ.mol–1 
V. Entalpia de hidratação do íon de cálcio: 
-1579 kJ.mol–1 
V. Entalpia de hidratação do íon de cloro: 
-378 kJ.mol–1 
 
Com base nessas informações, responda os itens 
abaixo. 
a) Represente, na forma de equações químicas, as 
informações acima (I-VI). 
b) Equacione a reação de entalpia de rede do cloreto de 
cálcio a partir das equações I-VI, conforme a 
necessidade. 
c) Calcule o valor numérico da entalpia de rede do 
cloreto de cálcio (em kJ.mol–1). 
158. (IME 1997 – Q03) 
Uma mistura de metano e etileno foi queimada em um 
recipiente, com volume constante de 3,0 litros, em 
presença de excesso de oxigênio, saturado em vapor 
d'água, de forma a que fosse obtida a combustão 
completa e para garantir que a água formada ficasse no 
estado líquido. 
A combustão foi realizada a 25oC, liberando 242,7 kcal, 
registrando-se uma redução na pressão de 16,3 atm. 
Determine o número de moles de metano e etileno 
presentes na mistura inicial. 
I) Entalpias de Formação (Ho
f ) 
H2O (líquida) = - 68,3 kcal / mol 
CO2 (gasoso) = - 94,1 kcal / mol 
CH4 (gasoso) = - 17,9 kcal / mol 
C2H4 (gasoso) = + 12,5 kcal / mol 
 
159. (IME 1998 – Q08) 
Metanol pode ser sintetizado diretamente a partir de 
monóxido de carbono e hidrogênio. Sabendo-se que os 
calores de combustão do monóxido de carbono e do 
metanol a 25oC, são respectivamente, –283,12 kJ/mol e 
–726,87 kJ/mol, calcule o calor de reação na formação 
de 2,0 g de metanol a 25oC, pela reação de 
hidrogenação direta do monóxido de carbono. 
DHf
0 (CO2) = – 393,70 kJ / mol 
DHf
0 (H2O) = – 281,79 kJ / mol 
 
160. (IME 2000 – Q03) 
Determine a massa de água que, com uma variação de 
temperatura de 30oC, fornece energia equivalente ao 
calor de formação de um mol de sulfeto de carbono 
sólido. 
Dados: 
calor de combustão do sulfeto de carbono = -265 
kcal/mol; 
calor de formação do gás sulfuroso = -71 kcal/mol; 
calor de formação do dióxido de carbono = -96 kcal/mol; 
capacidade calorífica da água líquida = 1,0 cal/g; 
peso molecular da água = 18. 
 
161. (IME 2008 – Q03) 
Nas combustões completas de x gramas de acetileno e 
de y gramas de benzeno são liberadas, 
respectivamente, Q1 kcal e Q2 kcal. Determine o calor 
liberado, em kcal, na formação de z gramas de benzeno 
a partir do acetileno. 
 
162. (IME 2010 – Q08) 
Uma dada massa de óxido ferroso é aquecida a 1273 K 
e, em seguida, exposta a uma mistura gasosa de 
monóxido de carbono e hidrogênio. Desta forma, o óxido 
é reduzido a metal sem qualquer fornecimento adicional 
de energia. Admita que ocorra uma perda de calor para 
as circunvizinhanças de 4,2 kJ/mol de óxido reduzido. 
Calcule a razão mínima entre as pressões parciais de 
monóxido de carbono e de hidrogênio (pCO/pH2) na 
mistura gasosa inicial, de modo que o processo seja 
auto-sustentável. Despreze a decomposição da água. 
Calores de reação a 1273 K (kJ/mol) 
redução do óxido ferroso 265 
oxidação do hidrogênio – 250 
oxidação do monóxido de carbono – 282 
 
163. (IME 2012 – Q32) 
Sobre a diferença entre sólido amorfo e sólido cristalino, 
pode-se afirmar o seguinte: 
(A) os sólidos amorfos não têm uma entalpia de fusão 
definida, enquanto os sólidos cristalinos têm. 
(B) sólido amorfo é aquele que pode sofrer sublimação, 
enquanto sólido cristalino não. 
(C) embora ambos possuam estrutura microscópica 
ordenada, os sólidos amorfos não possuem forma 
macroscópica definida. 
(D) os sólidos cristalinos têm como unidade formadora 
átomos, enquanto para os amorfos a unidade 
formadora são moléculas. 
(E) os sólidos cristalinos são sempre puros, enquanto 
os amorfos são sempre impuros. 
 
164. (IME 2012 – Q09) 
Em função do calor de formação do dióxido de carbono 
(ΔH°f,CO2); do calor de formação do vapor d’água 
(ΔH°f,H2O(g)); e do calor da combustão completa de 
uma mistura de metano e oxigênio, em proporção 
estequiométrica (ΔHr), deduza a expressão do calor de 
formação do metano (ΔH°f,CH4). 
Prof. Pedro Madeira 
 
165. (IME 2014 – Q05) 
1,00kg de carbonato de cálcio, na temperatura de 298K, 
é introduzido em um forno que opera a 101kPa. O forno 
é então aquecido até a temperatura Tc na qual ocorrerá 
a calcinação do carbonato de cálcio. Sabendo-se que o 
módulo da variação da energia livre de Gibbs da reação 
de calcinação à temperatura Tc é igual a 10,7kJ/mol, 
determine a temperatura de calcinação Tc e a 
quantidade de calor necessária à completa calcinação 
do carbonato. Despreze os efeitos de mistura e 
117 
 
 
 
APOSTILA 01 DE FÍSICO-QUÍMICA – PROF. PEDRO MADEIRA (2022) 
 
considere que, para o sistema reacional, aplicam-se as 
seguintes equações: 
 
Dados: 
Entalpias e entropias de formação a 298 K e 
capacidades caloríficas médias: 
Substância DHfo 
(kJ/mol) 
DSfo 
(J/mol.K) 
CP 
 (J/mol.K) 
CO2(g) –394 213 54,0 
CaO(s) –636 39,0 56,0 
CaCO3(s) –1207 94,0 110 
 
166. (IME 2015 – Q10) 
Monóxido de carbono a 473 K é queimado, sob pressão 
atmosférica, com 90% em excesso de ar seco, em base 
molar, a 773 K. Os produtos da combustão abandonam 
a câmara de reação a 1273 K. Admita combustão 
completa e considere que 1 mol de ar é constituído por 
0,20 mol de oxigênio e 0,80 mol de nitrogênio. Calcule a 
quantidade de energia, em kJ, que é liberada no 
decorrer da reação, por mol de monóxido de carbono 
queimado. Considere que os gases apresentam 
comportamento ideal. 
 
DADOS: 
Calor de combustão do monóxido de carbono (a 298 K 
e 1 atm) = –283 kJ·mol–1 
T(K) = t(oC) + 273 
 
Substância CO CO2 O2 N2 
Cp médio (kJ/mol.K) 0,03 0,04 0,03 0,03 
 
167. (IME 2017 – Q01) 
O oxigênio e o hidrogênio combinam-se, em células de 
combustível, produzindo água líquida e gerando 
corrente elétrica. O máximo trabalho elétrico útil que 
essas células produzem é dado por DGo = –237 x 103 
J.mol–1. Com base nos dados fornecidos, calcule o ponto 
de ebulição da água. Aproxime DH por DHo e DS por DSo. 
Dados termodinâmicos: 
O2(g) H2(g) H2O(l) H2O(g) 
So = 206 
J/mol.K 
So = 131 
J/mol.K 
So = 70,0 
J/mol.K 
So = 189 J/mol.K 
 DHo
f
 = –242x103 
J/mol 
 
168. (IME 2018 – Q03) 
A reforma com vapor d`água, a temperaturas altas, é um 
método industrial para produção de hidrogênio a partir 
de metano . Calcule a entalpia de reação desse 
processo. 
Dados: 
i) Entalpias de combustão: 
C(grafite) ... ΔHo = -394 kJ/mol 
 H2(g) ... ΔHo = -286 kJ/mol (forma água líquida) 
 CH4(g) ... ΔHo = -890 kJ/mol (forma água líquida) 
ii) CO(g) + H2(g) → C(grafite) + H2O (g) 
ΔHo = -131 kJ/mol 
Entalpia de fusão do gelo: ΔHfus = 330 kJ·kg–1 
Capacidade calorífica específica média da água: 
CV = 4,2 kJ·kg–1·K–1 
 
169. (IME 2020 – Q37) 
Uma medida quantitativa da estabilidade de um 
composto sólido iônico é a sua energia de rede, definida 
como a energia requerida para decompor 
completamente 1 mol desse composto nos seus íons em 
fase gasosa. Considere os seguintes dados: 
I. a entalpia padrão de formação do CaCl2 é - 790 
kJ.mol–1; 
II. a primeira energia de ionização do átomo de cálcio 
é 590 kJ.mol–1; 
III. a segunda energia de ionização do átomo de cálcio 
é 1146 kJ.mol–1; 
IV. a vaporização de um mol de Ca(s) consome 190 kJ; 
V. a energia de ligação do Cl2 é 242 kJ.mol–1; 
VI. a afinidade eletrônica do Cl é - 349 kJ.mol–1; 
 
Com base nessas informações, estima-se que a energia 
de rededo CaCl2, em kJ.mol–1, seja: 
(A) 790 
(B) 1029 
(C) 2070 
(D) 2260 
(E) 2609 
 
170. (IME 2020 – Q01) 
Calcule a variação de entalpia (em J) no processo de 
decomposição de 600 mg de nitroglicerina (C3H5N3O9) 
que produz nitrogênio, dióxido de carbono e oxigênio 
gasosos, além de água líquida. 
Dados: 
ΔHo
f (C3H5N3O9(ℓ)) = − 354 kJ/mol; 
ΔHo
f (H2O(ℓ)) = − 286 kJ/mol; 
ΔHo
f (CO2(g)) = − 394 kJ/mol. 
 
171. (IME 2021 – Q07) 
O RDX (ciclo-1,3,5-Trimetileno-2,4,6 trinitroamina) e o 
TNT (2-metil-1,3,5-trinitrobenzeno), quando misturados 
na proporção percentual 60/40 em massa, formam o 
“Composto B”. Considerando que cada munição contém 
2,5 kg de “Composto B”, inicialmente mantido a 25oC, 
determine a entalpia padrão teórica esperada na 
combustão completa de uma munição. 
 
DHo Formação H2O = - 286,2 kJ/mol 
DHo Formação CO2(g) = - 393,5 kJ/mol 
DHo Formação RDX = + 71 kJ/mol 
DHo Formação TNT = - 42 kJ/mol 
 
o
f P
o
f P
G H T S
H H c T
TS S c
T
• D = D - D
• D = D + D
D
• D = D +
118 
 
 
 
APOSTILA 01 DE FÍSICO-QUÍMICA – PROF. PEDRO MADEIRA (2022) 
 
172. (IME 2022 – Q08) – RESOLVA AGORA 
Um motor de 6 cilindros e volume total de 5.700 cm3, 
utilizado em viaturas leves e blindadas, consome 0,5 g 
do combustível gasoso de composição média C8H18, em 
cada cilindro, por segundo de operação. 
 
 
 
 
 
 
 
Considerações: 
 
• o ciclo termodinâmico do motor compreende o 
funcionamento em 4 tempos: admissão, compra 
assim combustão e exaustão (escape); 
• o motor executa 10 ciclos por segundo, ou seja, a 
mistura de ar e combustível enche o cilindros e 
depois é comprimida 10 vezes por segundo; 
• 20,0% da quantidade de combustível sofre 
combustão incompleta, sendo convertida em CO(g); 
• 80,0% da quantidade de combustível sofre 
combustão completa, sendo convertida em CO2(g); 
• a mistura de ar e combustível comporta-se como gás 
ideal; 
• as capacidades caloríficas molares são 
independentes da temperatura; e 
• as entalpias de formação a 25ºC. 
 
Determine: 
a) a vazão da entrada de ar no motor, em m3/s; e 
b) a composição percentual molar dos produtos e a 
temperatura de combustão, em K. 
 
FOLHA DE DADOS 
• Composição percentual do ar atmosférico = 79,0% de N2(g) 
e 21,0% de O2(g) 
 
Entalpias-padrão de formação a 25ºC: 
Substância 
Química C8H18(g) H2O(g) H2O(l) CO2(g) CO(g) 
DHf
o 
(kJ.mol–1) -208,45 -241,82 -285,83 -393,51 -110,53 
 
Capacidade calorífica molar: 
Substância 
Química N2(g) O2(g) H2O(g) CO2(g) CO(g) 
"#! 
(J.K–1 .mol–1) 29,13 29,36 33,58 37,11 29,14 
 
 
QUESTÕES EXTRAS: OLIMPÍADA 
 
173. 
Uréia, CO(NH2)2, reage com água produzindo dióxido de 
carbono e amônia. Os dados termodinâmicos para os 
possíveis reagentes e produtos são dados abaixo 
(negligencie a solubilidade do dióxido de carbono e da 
amônia em água líquida). 
Composto DHf
o (kJ/mol) So (J.K –1.mol –1) 
CO(NH2)2(s) – 333,51 104,60 
H2O(l) – 285,83 69,91 
H2O(g) – 241,82 188,83 
CO2(g) – 393,51 213,74 
NH3(g) – 46,11 192,45 
a) Considere a hidrólise de uréia com H2O (l) (Reação 
A) e com H2O (g) (Reação B), respectivamente. 
Calcule DH°, DS° e DG°, a 25 °C, para cada reação 
e especifique se a reação é espontânea ou não. 
b) Considerando que ambos, DH° e DS°, são 
independentes da temperatura, encontre a 
temperatura na qual a Reação A ocorrerá 
espontaneamente. 
c) Calcule Kp a 25 °C para cada reação, expressando 
esse valor em unidades apropriadas. 
 R = 8,314 J. K–1.mol–1. 
 
174. 
As energias-livres de formação do etileno (C2H4) e do 
etano (C2H6), a 25 oC e 1 atm são, respectivamente, 
+68,06 kJ/mol e –32,85 kJ/mol. Suas entalpias-padrão 
de formação são +52,23 kJ/mol e – 84,58 kJ/mol. 
 Pergunta-se: 
a) Para a redução do etileno a etano com H2, a 25oC e 
1 atm, determine a variação da energia livre e a 
variação da entropia. 
b) Como se modificaram a energia e a desordem do 
sistema? 
c) A reação é espontânea? Justifique. 
d) Como a temperatura modificaria a espontaneidade 
da reação? 
 
175. 
Alguns anos atrás, Texas City (Texas – USA) foi abalada 
por uma explosão de um depósito de nitrato de amônia, 
composto muito usado como fertilizante. Este composto, 
quando aquecido, pode decompor exotermicamente em 
N2O e água, conforme a equação: 
NH4NO3(s) à N2O(g) + 2 H2O(g) (equação 1) 
Se o calor liberado nesta reação ficar aprisionado, altas 
temperaturas serão atingidas, e assim, o NH4NO3 pode 
decompor explosivamente em N2, H2O e O2. 
2 NH4NO3(s) à 2 N2(g) + 4 H2O(g) + O2(g) (equação 2) 
Usando as informações fornecidas abaixo, responda: 
a) Qual o calor liberado (à pressão constante de 1 atm 
e à temperatura de 25 ºC) na primeira reação ? 
b) Se 8,00 kg de nitrato de amônio explodem (segunda 
reação), qual a quantidade de calor liberada (à 
pressão constante de 1 atm e à temperatura de 25 
ºC) ? 
c) Nitrato de amônia reage com alumínio em pó, 
produzindo Al2O3, segundo a reação: 
2 Al(s) + 3 NH4NO3(s) à 3 N2(g) + 6 H2O(g) + Al2O3(g) 
 Se 8,00 kg de nitrato de amônio são misturados com 
alumínio em pó, em excesso estequiométrico, qual a 
quantidade de calor produzida (à pressão 
constante)? 
Dados (valores aproximados): 
ar e combustível motor 6 cilindros produtos 
trabalho 
119 
 
 
 
APOSTILA 01 DE FÍSICO-QUÍMICA – PROF. PEDRO MADEIRA (2022) 
 
Entalpia de formação: DHfº , a 25 ºC (kJ/mol); 
Al2O3(g) = -1675,7; H2O(g) = -241,8 
NH4NO3(s) = -365,6; N2O(g) = 82,0 
 
176. 
PARTE I 
Para cada um dos seguintes processos, indique e 
justifique o sinal algébrico de DH0, DS0 e DG0: 
a) A "quebra" da molécula de água líquida em 
hidrogênio e oxigênio gasosos, processo que requer 
considerável quantidade de energia. 
b) A dissolução de pequena quantidade de NH4Cl em 
água. A solução torna-se bastante fria no processo. 
c) A explosão de dinamite, uma mistura de nitroglicerina 
(C3H5N3O9) e terra diatomácea. Essa decomposição 
explosiva origina produtos gasosos, tais como água, 
CO2 e outros. Muito calor é liberado neste processo 
d) A combustão de gasolina no motor de um carro, 
como exemplificado pela combustão do octano: 
2 C8H18(g) + 25 O2(g) à 16 CO2(g) + 18 H2O(g) + calor 
 
PARTE II 
Hidrogênio gasoso pode ser produzido a partir da reação 
entre carvão e vapor d'água, como mostra a reação: 
C(s) + H2O(g) à CO(g) + H2(g) 
a) Calcule o DG0 para esta reação a 25° C, 
considerando que C(s) é grafite. 
b) Calcule o kp para esta reação a 25° C. 
c) Esta reação ocorre espontaneamente sob estas 
condições? Se não, a que temperatura ela se tornará 
espontânea? 
Dados (valores aproximados): 
H2O(g) à DH0
f = -241,8 kJ/mol, DS0
f = +188,8 
J/K.mol e DG0
f = -228,6 kJ/mol (298,15K) 
CO(g) à DH0
f = -110,5 kJ/mol , DS0
f = +197,7 
J/K.mol e DG0
f = -137,2 kJ/mol (298,15K) 
 
177. (OBQ 2001) 
O álcool etílico líquido tem calor de vaporização igual a 
39,3 kJ/mol e as energias das ligações: C–C, C–O, O–H 
e O–O, em kJ/mol, são, respectivamente: 348; 356; 463 
e 493. 
a) Elabore um esquema (ciclo de Born-Haber) para 
indicar a obtenção de álcool etílico a partir de seus 
constituintes. 
b) Estes dados são suficientes para se calcular o DHf
o 
do álcool etílico? Justifique 
 
178. (OBQ 2005) 
Quando se queima uma amostra de 0,3212 g de glicose 
numa bomba calorimétrica, a volume constante, cuja 
constante calorimétrica é 641 JK–1, a elevação de 
temperatura é de 7,793 K. Pede-se: 
a) Calcule a energia interna padrão de combustão. 
b) Calcule a entalpia padrão de combustão 
c) Calcule a entalpia padrão de formação da glicose 
d) Que altura pode subir uma pessoa de 65 kg com a 
energia da combustão da amostra, admitindo que 
25% possam ser convertidos em trabalho? 
Dados: MGlicose = 180,16 g.mol–1 
ΔfHº (CO2(g)) = - 393,51 kJ mol–1 
ΔfHº (H2O(l)) = - 285,83 kJ mol–1 
Prof. Pedro Madeira 
179. (OBQ 2010) 
Considere os seguintes dados termodinâmicos: 
 
Substância ΔfHo
298 
(kJ.mol–1) 
So
298 
(J.K–1.mol–1) 
Cp
o
298 
(J.K–1.mol–
1) 
Glicose(s) 1273,2 211,9 218,6 
O2(g) 0 204,8 29,3 
CO2(g)393,1 213,4 37,1 
H2O(l) 285,6 69,8 75,2 
H2O(g) 241,6 188,5 33,5 
 
Responda as seguintes questões: 
a) Calcule o valor da variação de energia livre molar na 
temperatura de 298 K e 1 atm, para a reação de 
combustão completa para glicose. 
b) Calcule a variação de entropia molar para a reação 
na temperatura de 298 K e 1 atm e justifique com 
base na reação o valor positivo da mesma. 
c) Calcule o valor da variação de energia livre molar na 
temperatura de 400 K e 1 atm, para a reação de 
combustão completa para glicose. 
 
180. (OBQ 2011) 
Um experimento calorimétrico foi realizado para a 
determinação do calor de combustão completa do 
antraceno. Esta substância é um hidrocarboneto 
aromático sólido, de massa molar 178,23 g.mol–1, incolor 
mas que apresenta uma fluorescência azul quando 
irradiado por uma fonte de radiação ultravioleta. 
 
O calorímetro utilizado, uma bomba calorimétrica 
adiabática, é tem como componentes principais um vaso 
de reação construído com aço inoxidável com suas 
paredes espessas e rígidas para suportar uma pressão 
de oxigênio até 20 atm e um sistema elétrico de ignição 
das amostras a serem investigadas. Todo experimento 
calorimétrico realizado no interior deste calorímetro é 
feito sob o regime de volume constante. A alta pressão 
do gás oxigênio no vaso de reação é necessária para 
que seja assegurada a combustão completa das 
amostras estudadas. 
 
A reação de combustão neste equipamento é iniciada ao 
passar uma corrente elétrica através de um pequeno fio 
fino de ferro que é mantido em contato com a amostra. 
O vaso de reação é imerso em um banho de água que é 
mantido isolado das vizinhanças. O calorímetro utilizado 
é do tipo adiabático, isto é, todo o calor liberado pela 
reação permanece no calorímetro; nada é perdido para 
as vizinhanças. O calor liberado na reação de 
combustão é absorvido pela massa do banho de água e 
outras partes do calorímetro. Este processo causa um 
aumento na temperatura interna no calorímetro. O 
aumento da temperatura no interior deste calorímetro é 
medida por um termômetro sensível previamente 
adaptado ao sistema. 
 
120 
 
 
 
APOSTILA 01 DE FÍSICO-QUÍMICA – PROF. PEDRO MADEIRA (2022) 
 
Utilizando um calorímetro adiabático como descrito 
acima, uma amostra de 1,228 g de antraceno foi 
queimada em um calorímetro com um banho de água de 
2 L. a pressão de O2 no início da reação marcava 20 
atm. Um fio de ferro de 10,00 cm foi utilizado inicialmente 
na a ignição do sistema e, após o experimento, 0,65 cm 
intacto restante deste fio foi coletado. Neste experimento 
a temperatura medida no termômetro imerso no banho 
da água utilizada registrou uma variação do ser valor 
inicial de 23,74ºC até o valor final constante de 30,28ºC. 
 
Considerando os valores aproximados para a 
capacidade térmica específica e densidade da água 
como c(H2O,l) = 1,0 cal.g–1 grau–1 e ρ(H2O,l) = 1,0 g.ml–
1. A capacidade térmica do calorímetro adiabático 
utilizado vale Ccal = 881 J grau–1 e o calor de combustão 
linear por comprimento de fio de ferro vale Qcomb
linear (ε) 
= 0,1 J cm–1. 
 
Sabe-se também que 1 cal = 4,184 J e que a constante 
dos gases ideais tem o valor de R = 8,314 JK–1 mol–1. 
Utilizando os dados acima, determine a entalpia de 
combustão completa ΔHcomb do antraceno. 
 
181. (IChO) 
a) A quantity of 0.10 mol of an ideal gas A initially at 
22.2oC is expanded from 0.200 dm3 to 2.42 dm3. 
Calculate the values of work (w), heat (q), internal 
energy change (ΔU), entropy change of the system 
(ΔSsys), entropy change of the surroundings (ΔSsurr), 
and total entropy change (ΔSuniv) if the process is 
carried out isothermally and irreversibly against an 
external pressure of 1.00 atm. 
b) If 3.00 mol of A is condensed into liquid state and is 
mixed with 5.00 mol of liquid B, calculate the changes 
in entropy and Gibbs free energy upon such mixing at 
25.0oC. This mixture can be assumed to be ideal. 
 
182. (IChO) 
Students have made a device capable to operate in a 
mode that is close to the ideal Brayton cycle. This 
thermodynamic cycle has once been proposed for 
development of internal combustion engines. The device 
consists of a cylinder with 1 mole of helium fitted with a 
computer-controlled movable piston. A Peltier element 
which can heat or cool the gas is mounted in the cylinder 
wall. The device can operate in the following modes: 1) 
reversible adiabatic expansion or compression, 2) 
reversible isobaric cooling or heating. 
 
Through a number of cooling and compression steps, 
helium is going from the initial state with the pressure of 
1 bar and the temperature of 298 K into the final state 
with the pressure of 8 bar and the temperature of 298 K. 
(The total number of cooling and compression stages 
can be from two up to infinity). 
 
1. What is the minimum work that should be done on the 
gas for this? Compare this value to the work during a 
reversible isothermal compression. 
2. What is the maximum work that can be done on the 
gas in this process? 
3. Let the process be accomplished in three steps. At 
each step helium is first cooled and then compressed. 
At the end of each step the pressure increases twice 
and the temperature returns to the value of 298 K. 
What is the total heat removed from the gas by a 
Peltier element? 
 
Once the gas is compressed, it is returned to the initial 
state (1 bar and 298 K) in two stages (heating and 
expansion). 
 
4. What is the range of possible values of the formal 
efficiency η for the resulting cycle? η is the ratio of the 
useful work done by the gas to the amount of heat 
given to the gas during the heating stage. 
5. In one of the experiments, the gas has been 
compressed from 1 bar and 298 K to 8 bar and 298 K 
in several steps (like in question 3). At the end of each 
step the pressure is increased by x times and the 
temperature returns to 298 K. Then helium has been 
returned to the initial state in two stages – heating and 
expansion. Theoretical value of η for this cycle is 
0.379. How many steps were used? 
 
In fact, Peltier elements also consume electric energy 
during the cooling stage. Assume that they consume as 
much energy as is removed from the gas. 
 
6. What is the maximum possible efficiency of the 
considered cycle, taking into account energy 
consumption during cooling? 
 
Hint: in reversible adiabatic process for helium 
. Isochoric molar heat capacity of helium 
is 3/2R. 
 
183. (IChO) 
Two rigid containers in thermal equilibrium at 298 K 
connected by a valve are isolated from the surroundings. 
In one of the containers, 1.00 mol of He(g) and 0.50 mol 
of A(g) are present at 1.00 atm. In the other container, 
2.00 mol of Ar(g) and 0.50 mol of B2(g) are present at 
1.00 atm. 
a) Predict whether the entropy will increase or decrease 
when the valve separating the two containers is 
opened assuming that no chemical reaction takes 
place. 
b) Predict whether the entropy will increase or decrease, 
stating all factors that will have contribution, if a 
chemical reaction takes place according to the 
following equation when the valve separating the two 
containers is opened. 
A(g) + ½B2(g) → BA(g) ΔH°298 = - 99.0 kJ 
c) Assuming that all the gases present are ideal, 
calculate the final pressure at the end of the reaction. 
The total heat capacity of two containers is 547.0 
J/°C. 
 
184. (IChO) 
Given 10 liters of an ideal gas at 0oC and 10 atm, 
calculate the final volume and work done, under the 
5 3/pV const=
121 
 
 
 
APOSTILA 01 DE FÍSICO-QUÍMICA – PROF. PEDRO MADEIRA (2022) 
 
following three sets of condition, to a final pressure of 1 
atm 
1. Isothermal reversible expansion 
2. Adiabatic reversible expansion 
3. Irreversible adiabatic expansion carried out as 
follows: Assume the pressure is suddenly released to 
1 atm and the gas expands adiabatically at constant 
pressure. 
[Note that the molar heat capacity at constant volume is 
given by the relation: CV = 3R/2, whereR is the gas 
constant.] 
 
185. (IChO) 
Determine the work done by 1 mol of N2 gas when it 
expands reversibly and isothermally at 300 K from 1.00 
L to 10.0 L, treating it as a van der Waals gas. 
 
 
 
 
 
 
 
 
 
 
122 
 
 
 
APOSTILA 01 DE FÍSICO-QUÍMICA – PROF. PEDRO MADEIRA (2022) 
 
CAPÍTULO 02 – TERMODINÂMICA QUÍMICA 
GABARITO 
 
 
 
 
TÓPICO 01 
 
SEÇÃO VESTIBULARES 
 
1. 
a) eutética 
b) 15130 g 
 
2. OPÇÃO E 
 
3. OPÇÃO C 
4. OPÇÃO D 
 
5. 
t = 32 min 
 
6. SOMA 07 
 
7. 
a) C = 13,5 J / oC 
b) θ = 432ºC 
 
8. OPÇÃO B 
 
9. OPÇÃO C 
 
10. 
a) 
 
b) libera pois há aquecimento da água após a reação. 
c) 1,0 mol/L 
 
11. 
a) N = 1 mol de glicose 
b) E = 2,8x10 – 19 J 
c) n = 1025 fótons 
d) V = 134,4 L 
 
12. 
a) 
 
 
b) Xbase = 2/3 
c) ácido oxálico. Proporção de 2:1. 
d) è1400J: a base é limitante 
 è 1500J: o ácido é limitante. 
PROF. PEDRO MADEIRA 
 
SEÇÃO ITA / IME 
13. 
Reação: 
Ba(OH)2.8H2O(s) + 2 NH4NO3(s) 
 à 2 NH4OH(aq) + Ba(NO3)2(aq) + 8 H2O(l) 
Como a reação é endotérmica, retira calor do ambiente 
(incluindo da água que está na madeira), deste modo, 
como a reação é fortemente endotérmica, retira calor da 
água suficiente para causar sua solidificação, assim, o 
gelo formado une a madeira ao frasco de vidro. 
 
14. OPÇÃO E 
15. OPÇÃO C 
16. OPÇÃO B 
17. OPÇÃO C 
18. OPÇÃO B 
 
 
 
PROF. PEDRO MADEIRA 
123 
 
 
 
APOSTILA 01 DE FÍSICO-QUÍMICA – PROF. PEDRO MADEIRA (2022) 
 
TÓPICO 02 
 
SEÇÃO VESTIBULAR 
 
19. OPÇÃO E 
20. SOMA 06 
21. VVFFV 
22. OPÇÃO C 
23. OPÇÃO C 
24. OPÇÃO A 
PROF. PEDRO MADEIRA 
SEÇÃO ITA / IME 
 
25. OPÇÃO A 
26. OPÇÃO A 
Em B. Pois há o dobro de produto formado e uma massa 
de solução 1,5x maior. 
 
27. OPÇÃO A 
28. OPÇÃO C 
29. OPÇÃO E 
30. OPÇÃO D 
31. OPÇÃO B 
32. OPÇÃO B 
33. OPÇÃO D 
34. OPÇÃO D 
35. OPÇÃO C 
36. OPÇÃO E 
 
37. 
A) | w | = PV; | DU | = 0; | Q | = PV 
B) | w | = PV; | DU | = PV; | Q | = 0 
 
38. OPÇÃO A 
 
39. OPÇÃO C 
40. OPÇÃO E 
41. OPÇÃO B 
42. OPÇÃO E 
43. OPÇÃO A 
 
44. 
AgCl(s): DU = – 125,8 kJ/mol 
CaCO3(s): DU = – 1203,3 kJ/mol 
H2O(l): DU = – 282,3 kJ/mol 
H2S(g): DU = – 20 kJ/mol 
NO2(g): DU = – 35,2 kJ/mol 
 
45. OPÇÃO C 
 
46. 
a) 4,88x1015 J 
b) A molécula de água é a única que faz ponte de 
hidrogênio entre os calcogenetos, por esse motivo 
possui maior ponto de ebulição. 
c) Para que as pontes de hidrogênio sejam rompidas, 
uma grande quantidade de energia se faz necessária. A 
elevada Teb está relacionada com estas ligações de 
hidrogênio. Assim, como a precipitação pluviométrica é 
representada pela formação das ligações de hidrogênio, 
uma grande quantidade de energia é liberada. 
 
47. OPÇÃO B 
48. OPÇÃO C 
 
49. 
Como DU = DH – DngasesRT, o DU será igual ao DH 
quando o número de mol de gases nos reagentes for 
igual ao número de mol de gases nos produtos. 
 
50. OPÇÃO C 
 
51. 
a) P = 4 atm; T = 1200 K 
b) m = 0,10 g 
 
52. 
a) SRR: 2HOCℓ(aq) + 2e– + 2H+(aq) → Cℓ2(g) + 2H2O(ℓ) 
SRO: PbSO4(s) + 2H2O(ℓ) → PbO2(s) + 2e– + HSO4–(aq) + 3H+(aq) 
 
PbSO4(s) + 2HOCℓ(aq) → Cℓ2(g) + PbO2(s) + HSO4
–
(aq) + H+
(aq) 
 
DEo = 1,61 – 1,63 = – 0,02 V 
 
b) ∆Y<= = + 17,4 kJ 
c) uma vez que há a produção de gases na reação, 
ocorre trabalho de expansão volumétrica, consumindo 
parte da energia para tal processo. Logo, o saldo 
energético global (DU) é menor do que apenas o calor 
envolvido a pressão constante (DH). 
 
53. 
80% de enxofre 
 
54. 
V = 6,64 m3 
 
55. 
T = 1760 K; [H2O] = 8,7x10–4 mol.L–1 
 
56. 
DfHo (C10H8) = + 20,6 kcal / mol 
 
57. 
A) DcHo = – 780,89 kcal / mol 
B) DH = – 752 kcal / mol 
Se ocorre compressão qv > qp 
Se ocorre expansão qv < qp 
 
58. 
V = 1,22 m3. 
 
59. OPÇÃO D 
 
60. 
P =7,5xPo è aumento de 650% 
 
61. 
a) DU = – 134.812 cal 
b) DU = – 115.004 cal 
c) DU = 9.904 cal/mol 
 
124 
 
 
 
APOSTILA 01 DE FÍSICO-QUÍMICA – PROF. PEDRO MADEIRA (2022) 
 
62. 
TF = 32,8ºC 
 
63. OPÇÃO A 
PROF. PEDRO MADEIRA 
 
TÓPICO 03 
 
SEÇÃO VESTIBULARES 
 
64. OPÇÃO D 
65. OPÇÃO A 
 
66. 
A) DG = + 21,106 kcal B) Não; | TDS | < | DH | 
C) quanto maior a temperatura, mais a posição do 
equilíbrio está deslocada no sentido dos produtos. 
 
67. 
A) DGo = +27,6 kcal.mol–1, não espontânea. 
B) DSo = +38,6 cal.mol–1K–1. 
C) T > 1013 K; T > 740oC. 
D) Altas pressões. 
 
 
68. 
T > 397,5 K 
69. 
A) DG = 0; DS > 0. 
B) DH > 0; DS > 0; DG > 0. 
C) Espontânea acima de 100oC (1 atm) , pois DG < 0. 
 
PROF. PEDRO MADEIRA 
SEÇÃO ITA / IME 
 
70. OPÇÃO D 
 
71. 
A) DS > 0, mesma Ec, menor pressão. 
B) DS < 0, forças coesivas mais fortes no diamante. 
C) DS < 0, se houver formação de precipitado. 
DS < 0, se o sistema for apenas a solução (águas de 
solvatação diminuem). 
D) DS < 0, sólido mais organizado. 
E) DS < 0, diminuição do movimento molecular. 
 
72. OPÇÃO C 
73. OPÇÃO A 
 
74. 
 
A à B: Expansão isotérmica 
B à C: Expansão adiabática 
C à D: Compressão isotérmica 
D à A: Compressão adiabática 
 
75. OPÇÃO D 
76. OPÇÃO D 
77. OPÇÃO E 
78. OPÇÃO C 
79. OPÇÃO A 
 
80. 
DG = – 153,1 kJ/mol 
 
81. 
DU = – 966,6 kJ 
 
82. 
A) ortorrômbica 
B) DH > 0 
 
83. 
A reação é termodinamicamente favorável quando T > 
1111 K 
84. OPÇÃO E 
85. OPÇÃO E 
86. OPÇÃO A 
87. OPÇÃO C 
88. OPÇÃO D 
89. OPÇÃO C 
90. OPÇÃO D 
 
91. 
DS = 8,10 J/K 
 
92. OPÇÃO D 
93. OPÇÃO C 
 
94. 
è Calorimetria: 
mCp(T + ΔT – TF) = mCp (TF – T) 
 
TF = T + KL
!
 
è ΔSA = mCpQ( &MN,O∆&
&
 
 
è ΔSB = mCp	Q( &MN,O∆&
&MQ&
 
 
125 
 
 
 
APOSTILA 01 DE FÍSICO-QUÍMICA – PROF. PEDRO MADEIRA (2022) 
 
è ΔSTOT = ΔSA + ΔSB 
 
 ΔSTOT = mCp	Q( (&MN,O∆&)"
&∙(&MQ&)
 
 
ΔSTOT = mCp	Q( &"M&∙∆&MN,!O∆&"
&"M&∙∆& 
 
è Perceba que o numerador é maior do que o 
denominador. Logo, o ΔSTOT > 0. 
 
95. OPÇÃO D 
96. OPÇÃO B 
 
97. 
a) DSSISTEMA = -102,3 J.K–1 
b) DSVIZINHANÇA = +106,8 J.K–1 
c) DSUNIVERSO = +4,5 J.K–1 
 
PROF. PEDRO MADEIRA 
 
TÓPICO 04 
 
SEÇÃO VESTIBULARES 
 
98. OPÇÃO C 
99. OPÇÃO E 
 
100. 
A) DHo = – 32,70 kcal.mol–1 
B) 
 
 
101. 
A) DHo = 131,3 kJ (Endotérmica) 
B) O elemento C muda de estado de oxidação de zero 
para +2 na substância composta CO, portanto sofre uma 
oxidação, sendo classificado como agente redutor. Já o 
H muda de +1 na substância composta H2O para zero 
na substância simples H2, sofrendo uma redução, sendo 
a molécula H2O classificada como agente oxidante. 
 
102. 
A) O calor liberado por grama de substância será: 
H2 : q = – 143,0 kJ g–1 
CH3OH: q = – 22,69 kJ g–1 
C2H5OH: q = – 29,72 kJ g–1 
C8H18: q = – 47,98 kJ g–1 
 
A substância que libera a maior quantidade de energia 
por grama é o H2, apresentando, portanto, a maior 
eficiência nesta situação. 
 
B) A massa de 1 mL de cada substância será: 
H2 : 0,07 g; CH3OH: 0,79 g; 
C2H5OH: 0,80 g e C8H18: 0,70 g. 
Portanto, a energia liberada por mL de substância será: 
H2 : DH = – 10,01 kJ mL–1 
CH3OH: DH = – 17,93 kJ mL–1 
C2H5OH: DH = – 23,78 kJ mL–1 
C8H18: DH = – 33,59 kJ mL–1 
 
A substância que libera a maior quantidade de energia 
por mL é o C8H18 (octano), apresentando, portanto, a 
maior eficiência nesta situação. 
 
103. 
a) 
 
 
b) 2 NCℓ3 → N2 + 3 Cℓ2 
Reagente = +3; Produto = 0 
c) DH = – 230 kJ/mol; exotérmcia 
 
104. 
A) DHI = – 184 kJ e DHII = – 78 kJ 
B) Reação I 
 
105. 
A) ainda restarão 974,4 g de H2O na fase líquida. 
B) q = 2261 kJ. 
 
106. OPÇÃO E 
 
107. OPÇÃO C 
 
108. 
DHo = – 152 kcal.mol–1 
 
109. 
a) –585 kJ/mol; b) + 101 kJ/mol 
 
110. 
a) C8H18(v) + 25/2 O2(g) à 8CO2(g) + 9H2O(l) 
b) DH = – 3419,5 kJ/mol 
c) 30776 kJ 
 
111. 
a) ligação de hidrogênio 
b) 170 kJ/mol 
 
112. 
a) DH = – 1970 kJ/mol; b) 4,83x104 m 
 
113. 
a) 
 
 
b) C5H10 + 15/2 O2 à 5 CO2 + 5 H2O 
C C
H
H
H
H
+ H2
Pt
H C C
H H
H
HH
Cl
N Cl
Cl
126 
 
 
 
APOSTILA 01 DE FÍSICO-QUÍMICA – PROF. PEDRO MADEIRA (2022) 
 
c) DH’s diferentes pois as entalpias iniciais são 
diferentes e as finais são as mesmas em cada reação. 
 
114. 
a) 2NH4ClO4 à N2 + 2 O2 + Cl2 + 4 H2O 
b) 6NH4ClO4 + 8 Al à 3 N2 + 3Cl2 + 12H2O + 4Al2O3c) 2,24x103 kJ (calor liberado a mais) 
 
115. 
a) C5H8N4O12 à 2CO +3CO2 + 2N2 + 4H2O 
b) B.Ox = –10,12% 
c) DH = – 1832 kJ/mol 
d) quanto mais próximo de zero for o balanço de 
oxigênio, mais eficiente será a liberação de calor. 
PROF. PEDRO MADEIRA 
 
116. OPÇÃO A 
 
SEÇÃO ITA / IME 
 
117. OPÇÃO D 
Calor envolvido na transformação de 1 mol de C(grafite) 
em 1 mol de C(diamante) a pressão constante 
 
118. OPÇÃO D 
119. OPÇÃO D 
120. OPÇÃO C 
 
121. 
E = hc / l; Quanto maior a energia, menor o 
comprimento de onda. 
 
122. OPÇÃO A 
123. OPÇÃO A 
124. OPÇÃO D 
125. OPÇÃO C 
126. OPÇÃO E 
127. OPÇÃO D 
128. OPÇÃO C 
 
129. 
A) Combustão do n-hexano: 
1C6H14(l) + 19/2 O2(g) à 6 CO2(g) + 7 H2O(g) 
DH = – 3883 kJ/mol 
Combustão do n-heptano: 
1C7H16(l) + 11 O2(g) à 7 CO2(g) + 8 H2O(g) 
DH = – 4498 kJ/mol 
B) As moléculas de um hidrocarboneto linear podem ser 
entendidas como sendo CH3-(CH2)n-CH3. Portanto, a 
partir dos dados da questão, pode-se concluir que a 
adição de 1 grupo metileno (–CH2–) no n-hexano 
promove um aumento de 615 kJ/mol no calor liberado na 
combustão do n-heptano. 
Logo, do n-hexano para o n-decano há 4 grupos 
metileno a mais, o que acarretará um acréscimo do calor 
de combustão liberado em 4x615 kJ. Assim, temos: 
DcombHo (n-C10H22) = – 6343 kJ/mol 
C) O calor liberado seria maior, pois a água líquida tem 
uma entalpia menor do que a água gasosa. Uma vez que 
ela é produto da reação, a variação de entalpia (Hprodutos 
– Hreagentes) será mais negativa do que no caso da água 
gasosa. 
130. OPÇÃO E 
131. OPÇÃO B 
 
132. 
A) n = 0,06 mol 
B) q = 8,25 x 10–3 kJ 
 
133. OPÇÃO E 
 
134. 
No processo de fusão, deve haver grande absorção de 
energia a fim de que haja a ruptura das ligações de 
hidrogênio no gelo. Contudo, a formação de pontes de 
hidrogênio na fase líquida é acompanhada pela 
liberação de certa quantidade de calor. O saldo final de 
calor liberado é uma medida desses dois processos de 
absorção e liberação de calor. Já no estado vapor, as 
ligações de hidrogênio não são importantes (por 
exemplo, para um gás ideal não há interações 
intermoleculares), frente ao grande número e 
intensidade das mesmas na fase líquida. Portanto, o 
calor absorvido na ruptura das ligações de hidrogênio no 
líquido não tem uma contrapartida de calor liberado na 
fase vapor. Desta forma, DHvap > DHfus. 
 
135. OPÇÃO C 
136. 
 
 
137. OPÇÃO E 
138. OPÇÃO D 
139. 
A) O Al2O3 forma uma camada protetora sobre o 
polímero dificultando seu contato com o gás 
oxigênio (comburente). 
B) Da mesma forma, a H2O(g) também irá atuar 
dificultando tal contato. 
C) DH > 0 implica em absorção de calor. Portanto, parte 
do calo disponível para ativar a combustão será 
empregada na decomposição do hidróxido de 
alumínio. 
127 
 
 
 
APOSTILA 01 DE FÍSICO-QUÍMICA – PROF. PEDRO MADEIRA (2022) 
 
140. OPÇÃO B 
141. OPÇÃO C 
142. OPÇÃO D 
 
143. 
a) q = – 73.465 kJ b) q = – 203.500 kJ 
c) q = – 87,5 kJ d) q = – 211,64 kJ 
 
144. OPÇÃO A 
145. OPÇÃO C 
146. OPÇÃO C 
 
147. 
a) 33,3 kg 
b) 156,34 L 
c) 1,37x105 L 
148. 
a) 6 CO2(g) + 6 H2O(l) à C6H12O6(s) + 6 O2(g) 
b) Q = 4,664x10–21 kJ/molécula 
c) EFÓTON = 4,655x10–19 J 
d) n > 10; assim, no mínimo são 11 fótons. 
 
149. OPÇÃO D 
 
150. 
a) 1C2H6O(ℓ) + 3O2(g) → 2CO2(g) + 3H2O(ℓ) 
b) 1C2H6O(ℓ)+3O2(g)+ 12N2(g) → 2CO2(g) + 3H2O(ℓ) + 12N2(g) 
c) 
1C2H6O(ℓ)+3/2O2(g)+6N2(g)→3H2O(ℓ)+1CO(g)+1C(s)+6N2(g) 
d) as duas primeiras reações são combustões completas 
(são mais exotérmicas). A última é uma combustão 
incompleta (menos exotérmica). 
Logo, ΔH(a) = ΔH(b) < ΔH(c) 
 
151. 
a) nA = 0,25 mol; nB = 0,75 mol; nc = 1/3 mol 
b) ΔH = – 792 kJ.mol–1. 
 
152. 
a) 
 
 
ΔHf(NaCl) = - 421,5 kJ.mol–1 
b) O CaO possui íons de maiores cargas, o que contribui 
para que a sua entalpia de rede (em módulo) seja maior 
do que a do NaCl. 
 
153. 
Usando o DHeb
o = +10,5 kcal mol–1 (valor correto) 
a) ΔHo = − 213 kcal/mol 
b) 1,0x105 kcal/h 
c) 7,51 kg/h 
 
Usando o DHeb
o = –10,5 kcal mol–1 (valor dado) 
a) ΔHo = − 171 kcal/mol 
b) 1,0x105 kcal/h 
c) 9,36 kg/h 
 
154. 
a) PCI(metano) = 792 MJ.kg–1; 
PCI(etanol) = 1248 MJ.kg–1 
 
b) 94,5% de CH4 
 
c) Primeiramente, deve-se salientar que a oxidação que 
o carbono do metano sofre na combustão completa a 
CO2 é mais intensa do que a do etanol. Isto contribui 
para que o calor liberado por unidade de massa seja 
maior no caso do metano do que no caso do etanol. 
 
A entalpia molar de combustão do etanol é maior do que 
a do metano, o que se explica pela maior massa molar. 
Contudo, o acréscimo de calor não é proporcional ao 
acréscimo de massa pelo exposto anteriormente. Assim, 
ao se dividir pela massa molar, a PCS do metano se 
torna maior do que a PCS do etanol. 
 
 
155. OPÇÃO D 
156. OPÇÃO D 
 
157. 
a) 
I. Ca(g) → Ca+(g) + e– DH = 590 kJ.mol–1 
II. Ca+(g) → Ca2+(g) + e– DH = 1145 kJ.mol–1 
III. Cℓ(g) + e– → Cℓ–(g) DH = -340 kJ.mol–1 
IV. CaCℓ2(s) → Ca2+(aq) + 2Cℓ–(aq) DH = -81 kJ.mol–1 
V. Ca2+(g) → Ca2+(aq) DH = -1579 kJ.mol–1 
VI. Cℓ–(g) → Cℓ–(aq) DH = -378 kJ.mol–1 
 
b) 
CaCℓ2(s) → Ca2+(aq) + 2Cℓ–(aq) DH = -81 kJ.mol–1 
Ca2+(aq) → Ca2+(g) DH = +1579 kJ.mol–1 
2Cℓ–(aq) → 2Cℓ–(g) DH = 2x378 kJ.mol–1 
CaCℓ2(s) → Ca2+(g) + 2Cℓ–(g) DH = 2254 kJ.mol–1 
 
c) DH = 2254 kJ.mol–1 
 
158. 
n(CH4) = 0,76 mol; n(C2H4) = 0,24 mol 
 
159. 
1 CO + 2 H2 à 1 CH3OH DH = – 119,83 kJ/mol 
128 
 
 
 
APOSTILA 01 DE FÍSICO-QUÍMICA – PROF. PEDRO MADEIRA (2022) 
 
Q = - 7,49 kJ / 2,0 g de metanol 
 
160. 
m = 900g 
 
161. 
c = de ∙ .S"
T
− S&
U
/d kcal 
 
162. 
PCO / PH2 = 1,5 
 
163. OPÇÃO A 
 
164. 
ΔH°f,CH4 = 2x ΔH°f,H2O(g) + ΔH°f,CO2 – ΔHr 
 
165. 
T = 1188 K; Q = 2749 kJ 
 
166. 
Calor liberado para a vizinhança = 192,6 kJ/mol 
 
167. 
T = 369 K 
 
168. 
DH = + 207 kJ/mol 
 
169. OPÇÃO D 
 
170. 
DH = - 4,08x103 J/mol 
 
171. 
DH = - 29359 kJ/mol 
 
172. 
a) 0,0562 m3/s 
b) 73,4% N2; 12,0% H2O; 8,5% CO2; 4% O2; 2,1% CO 
TF = 2418 K 
 
PROF. PEDRO MADEIRA 
“Quem pergunta é bobo por cinco minutos; quem não 
pergunta é bobo para sempre.” 
Provérbio Chinês 
 
 
 
 
 
 
 
 
 
 
 
 
 
 
 
 
 
 
 
 
 
 
 
 
 
 
 
 
129 
 
 
 
APOSTILA 01 DE FÍSICO-QUÍMICA – PROF. PEDRO MADEIRA (2022) 
 
LISTA EXTRA – GASES 
 
01. 
Um estudante ligou um bulbo de vidro contendo gás 
neônio a um manômetro de tubo aberto e verificou que 
a pressão do gás é 0,890 atm. 
(a) Se a pressão atmosférica é 762 Torr, que diferença 
de altura entre os dois lados do mercúrio o estudante 
encontrou? 
(b) Que lado está mais alto, o lado do manômetro ligado 
ao bulbo ou o lado aberto à atmosfera? 
(c) Se o estudante se enganou e trocou as leituras dos 
lados do manômetro ao registrar os dados no 
caderno de laboratório, qual seria a pressão do bulbo 
que ele registrou? 
 
02. 
O gás de um composto fluorado de metano tem 
densidade 8,0 g.L–1, em 2,81 atm e em 300 K. 
(a) Qual é a massa molar do composto? 
(b) Qual é a fórmula do composto sabendo-se que ele é 
formado somente por C, H e F? 
(c) Qual é a densidade do gás em 1,00 atm e 298 K? 
 
03. 
Um composto usado na fabricação de saran é 24,7% C, 
2,1% H e 73,2% Cl em massa. O armazenamento de 
3,557g do composto em um recipiente de 755 mL, em 
0oC, eleva a pressão até 1,10 atm. Qual é a fórmula 
molecular do composto? 
 
04. 
O processo Haber de síntese da amônia é um dos 
processos industriais mais importantes para o bem-estar 
da humanidade. Ele é muito usado na produção de 
fertilizantes, polímeros e outros produtos. 
(a) Que volume de hidrogênio em 1,00 atm e 350oC 
deve ser usado para produzir 1,0 tonelada de NH3? 
(b) Que volume de hidrogênio seria necessário em (a) 
se o gás fosse fornecido em 3,76x102 atm e 250ºC? 
 
05. 
O dióxido de carbono e a água produzem, através de 
uma série de etapas enzimáticas do processo de 
fotossíntese, glicose e oxigênio, de acordo com a 
equação: 6 CO2(g) + 6 H2O(l) à C6H12O6(s) + 6 O2(g). 
Sabendo-se que a pressão parcial do dióxido de carbono 
na troposfera é 0,26 Torr e que a temperaturaé 25ºC, 
calcule o volume de ar necessário para produzir 10,0g 
de glicose. 
 
06. 
A pressão total de uma mistura dos gases dióxido de 
enxofre e nitrogênio, em 25ºC, em um recipiente de 500 
mL, é 1,09 atm. A mistura passa sobre óxido de cálcio 
em pó quente, que remove o dióxido de enxofre pela 
reação CaO(s) + SO2(g) à CaSO3(s), e é transferida 
para um recipiente de 150 mL, no qual a pressão é 1,09 
atm em 50ºC. 
(a) Qual era a pressão parcial de SO2 na mistura inicial? 
(b) Qual era a massa de SO2 na mistura inicial? 
07. 
Durante um experimento de eletrólise de água, o gás 
hidrogênio foi coletado em um dos eletrodos sob água 
em 20ºC e pressão externa igual a 756,7 Torr. A pressão 
de vapor da água em 20ºC é 17,54 Torr. O volume do 
gás era 0,220 L. 
(a) Qual é a pressão parcial do hidrogênio? 
(b) O outro produto da eletrólise da água é o gás 
oxigênio. Escreva uma equação balanceada para a 
eletrólise da água para dar H2 e O2. 
(c) Que massa de oxigênio foi produzida na reação? 
 
08. 
Um hidrocarboneto de fórmula empírica C2H3 levou 349s 
para efundir por uma rolha porosa. Nas mesmas 
condições de temperatura e pressão, são necessários 
210s para que ocorra a efusão da mesma quantidade de 
átomos de argônio. Determine a massa molar e a 
fórmula molecular do hidrocarboneto. 
 
09. 
Calcule a energia cinética molar (em joules) de uma 
amostra de gás criptônio em 
(a) 55,85ºC e 
(b) 54,85ºC 
(c) A diferença de energia molar entre as respostas de 
(a) e (b) é a energia por mol necessária para elevar a 
temperatura do gás criptônio de 1,00oC. Essa 
quantidade é conhecida como capacidade calorífica 
molar. Quanto ela vale? 
 
10. 
Considere a distribuição de velocidade de Maxwell. 
(a) A partir do gráfico, encontre o ponto que representa 
a velocidade mais provável das moléculas. 
(b) O que acontece com a percentagem de moléculas 
que têm a velocidade mais provável quando a 
temperatura aumenta? 
 
11. 
A pressão de uma amostra de fluoreto de hidrogênio é 
mais baixa do que a esperada e, com o aumento da 
temperatura, sobe mais depressa do que o predito pela 
lei dos gases ideais. Forneça uma explicação. 
 
12. 
Use a equação do gás ideal para calcula a pressão, em 
298 K, exercida por 1,00 mol de CO2(g) quando limitado 
ao volume de 
(a) 15,0 L; (b) 0,500 L; (c) 50,0 mL. 
Repita estes cálculos usando a equação de van der 
Waals. O que estes cálculos indicam sobre a precisão 
da dependência da pressão na lei dos gases ideais? 
DADOS: a = 3,592 atm.L2/mol2; b = 0,04267 L/mol 
 
13. 
(a) Calcule a pressão de CO2(g) confinado em um vaso 
de 1,00 L em temperatura constante (27ºC). Use a lei 
dos gases ideais e a equação de van der Waals no 
intervalo de 0,100 mol CO2 a 0,500 mol CO2, em 
incrementos de 0,100 mol. 
130 
 
 
 
APOSTILA 01 DE FÍSICO-QUÍMICA – PROF. PEDRO MADEIRA (2022) 
 
(b) Calcule o desvio percentual entre o valor ideal e o 
valo “real” (calculado pela equação de van der 
Waals) em cada ponto. 
(c) Nestas condições, que termo tem o maior efeito na 
pressão real de CO2, as atrações intermoleculares ou 
o volume molar? 
(d) Se considerarmos como ideais os gases para os 
quais a pressão observada difere menos de 5% do 
valor ideal, a que pressão o CO2 torna-se um gás 
“real”? 
 
14. 
A fumaça poluente fotoquímica é formada, em parte, 
pela ação da luz sobre o dióxido de nitrogênio e reação 
posterior com O2 para produzir O3. O comprimento da 
radiação absorvida pelo NO2 nesta reação é 197 nm. 
 NO2 + hn à NO + O 
 O + O2 à O3 + M ( M = N2 ou O2) 
(a) Desenhe a estrutura de Lewis de NO2 e faça um 
esquema de seus orbitais p. 
(b) Quando 1,07 mJ de energia são absorvido por 2,5 L 
de ar em 20ºC e 0,85 atm, todas as moléculas NO2 
da amostra se dissociam segundo a reação acima. 
Imagine que cada fóton absorvido resulte na 
dissociação (em NO e O) de uma molécula NO2. Qual 
é a proporção, em partes por milhão, de moléculas 
NO2 na amostra? Imagine que a amostra tem 
comportamento ideal. 
 
15. 
Suponha que 200 mL de cloreto de hidrogênio, em 690 
Torr e 20ºC, foram dissolvidos em 100 mL de água. A 
solução foi titulada até o ponto estequiométrico com 15,7 
mL de uma solução de hidróxido de sódio. Qual é a 
concentração molar do hidróxido de sódio em solução? 
 
16. 
Um frasco de volume 5,00 L foi evacuado e 43,78g de 
tetróxido de dinitrogênio, N2O4, foram admitidos. Em -
196ºC, este composto é um sólido incolor. A amostra foi 
aquecida até 25ºC e no processo, N2O4 se vaporiza e se 
dissocia parcialmente para formar o gás NO2. A pressão 
cresce lentamente e se estabiliza em 2,96 atm. 
(a) Escreva uma equação para a reação. 
(b) Se o gás que está no frasco fosse exclusivamente 
N2O4, qual seria a pressão? 
(c) Se todo o gás que está no frasco fosse NO2, qual 
seria a pressão? 
(d) Quais são as frações molares de N2O4 e NO2 quando 
da pressão se estabiliza em 2,96 atm? 
 
17. 
Determine a razão entre o número de moléculas de um 
gás que tem velocidade dez vezes maior do que a da 
velocidade quadrática média e o número de moléculas 
que tem velocidade igual à da velocidade quadrática 
média. Será que esta razão é independente da 
temperatura? Por quê? 
DADO: 
#(H) = IJ1
;
CJ*'7
3/!
H! ∙ 0#45"/'& 
18. 
Um estudante de pós-graduação tem de preparar gás 
fosgênio marcado isotopicamente, 13COCl2(g), para uso 
na síntese de um composto orgânico (todos os átomos 
C são carbono-13). O estudante colocou 3,59 atm de 
13CO(g) e 2,75 atm Cl2(g) em um cilindro de aço, em 
25ºC, selou o cilindro e o aqueceu até 227ºC. O cilindro 
foi mantido nesta temperatura elevada por uma semana 
em 227ºC, chegou a 9,75 atm. 
(a) Quais são as frações molares de 13CO, Cl2 e 13COCl2 
presentes no cilindro no fim da semana? 
(b) Qual é a densidade da mistura? 
 
19. 
O espalhamento de odores pelo ar é devido à difusão de 
moléculas de gás. Alguém abriu um frasco contendo 
octanoato de etila na extremidade norte de uma sala de 
5 m de comprimento e, simultaneamente, outra pessoa 
abriu um frasco contendo p-anisaldeído na extremidade 
sul da sala (C10H29O2) tem odor semelhante ao de frutas 
e o p-anisaldeído, odor semelhante ao da hortelã. A que 
distância (em metros) da extremidade norte deve estar 
uma pessoa para sentir primeiro o cheiro de hortelã? 
 
20. 
Uma amostra sólida, finamente pulverizada, de um óxido 
de ósmio (que funde em 40ºC e ferve em 130ºC), cuja 
massa é 1,509g, foi colocada em um cilindro dotado de 
um pistão móvel que pode se expandir contra a pressão 
atmosférica de 745 Torr. Imagine que a quantidade de 
ar residual inicialmente presente no cilindro é 
desprezível. Quando a amostra é aquecida até 200ºC, 
ocorre vaporização completa e o volume do cilindro se 
expande até 235 mL. Qual é a massa molar do óxido? 
Imaginando que a fórmula do óxido é OsOx, qual é o 
valor de x? 
 
21. 
(a) Os parâmetros de van der Waals do hélio são a = 
3,412x10–2 L2.atm.mol–2 e b = 2,370x10–2 L.mol–1. 
Calcule, a partir dos parâmetros de van der Waals, o 
volume aparente (em pm3) e o raio (em pm) de um 
átomo de hélio. 
(b) Estime o volume de um átomo de hélio na base do 
raio atômico. 
(c) Como estas quantidades se comparam? Será que 
elas deveriam ser iguais? Discuta. 
22. 
Um litro de gás cloro, em 1 atm e 298K, reage 
completamente com 1,00 L de gás nitrogênio e 2,00 L de 
gás oxigênio na mesma temperatura e mesma pressão. 
Forma-se um único produto gasoso, que enche um 
frasco de 2,00 L, em 1,00 atm e 298 K. Use estas 
informações para determinar as seguintes 
características do produto: 
(a) sua fórmula empírica; 
(b) sua fórmula molecular; 
(c) a fórmula de Lewis mais favorável com base em 
argumentos de carga formal (o átomo central é N); a 
forma da molécula. 
 
131 
 
 
 
APOSTILA 01 DE FÍSICO-QUÍMICA – PROF. PEDRO MADEIRA (2022) 
 
23. 
A equação de van der Waals pode ser rearranjada em 
uma equação cúbica: 
 
%3 + (1
*' + ,!
! 7%! + f
(!-
! g% −
(3-,
! = B 
 
(a) Use esta equação para calcular o volume ocupado 
por 0,505 mol de NH3(g) em 25ºC e95,0 atm. Os 
parâmetros de van der Waals do NH3 são a = 
4,225x10–2 L2.atm.mol–2 e b = 3,707x10–2 L.mol–1. 
(b) Que forças predominam, nesta temperatura e 
pressão, as atrativas ou as repulsivas? 
 
24. 
Um manômetro conectando dois frascos (rotulados A e 
B) contém um óleo com uma densidade de 0,847 g/cm3. 
O óleo no braço conectado ao frasco A está 74 cm mais 
alto que o óleo no braço conectado ao frasco B. O gás 
no frasco A tem uma pressão de 97,7 kPa (836 Torr). 
Qual é a pressão do gás no frasco B? 
 
25. 
Uma bomba de bicicleta possui um pistom de 75,0 cm 
de comprimento. Assumindo que, ao se puxar o êmbolo, 
o ar é introduzido a uma pressão de 101 kPa, de quanto 
deve ser esta êmbolo comprimido (em centímetros) para 
que a pressão do ar atinja 555 kPa, se a temperatura do 
ar permanecer constante? (Essa pressão corresponde, 
aproximadamente, à pressão de um pneu numa bicicleta 
de 10 marchas). 
 
26. 
Um gás é coletado sobre água até a pressão total interna 
de um frasco de 100 cm3 ser de 93,3 kPa a 25ºC. Calcule 
o volume do gás seco nas CNTP. 
 
27. 
Uma amostra de 0,2000g de um líquido com odor de 
peixe, contendo somente carbono, hidrogênio e 
nitrogênio, foi queimada e produziu 0,482g de CO2 e 
0,271g de H2O. Uma segunda amostra, pesando 
0,2500g, foi tratada de tal forma que todo o nitrogênio 
contido na substância foi convertido em N2. Este gás foi 
coletado, ocupando um volume de 42,3 cm3, a 26,5ºC e 
100,6 kPa. 
(a) Qual a porcentagem, em massa, de carbono, 
hidrogênio e nitrogênio no composto? 
(b) Qual a fórmula empírica do composto? 
 
28. 
O oxigênio gasoso, gerado na reação 
KClO3 à KCl + O2 
(não-balanceada), foi coletado sobre água a 30ºC em 
um vaso de 150 cm3 até a pressão total ser de 80,0 kPa. 
(a) Quantos gramas de o2 seco foram produzidos? 
(b) Quantos gramas do KClO3 foram consumidos na 
reação? 
 
 
 
29. 
Use a equação de van der Waals para calcular a pressão 
de 1,000 mol de C2H6 a 0oC em um volume de 22,400 
dm3. Compare com a pressão de um gás ideal sob estas 
mesmas condições. 
 
30. 
São coletados 280 cm3 de gás sobre água a 20ºC. O 
nível da água dentro do frasco coletor está 28,4 mm 
mais alto que o nível da água do lado de fora. A pressão 
atmosférica é de 102 kPa. Qual seria o volume de gás 
seco nas CNTP? 
 
31. 
O ozônio, O3, é uma espécie importante na cadeia de 
reações que conduzem à produção do nevoeiro. Em 
uma análise de ozônio, 2,0x104 dm3 de ar, nas CNTP, 
foram passados através de uma solução de NaI, onde o 
O3 sofre a reação 
 
O3 + 2 I– + H2O à O2 + I2 + 2 OH– 
O I2 formado foi titulado com 0,0100 M de Na2S2O3, com 
o qual reagiu: 
 
I2 + 2 S2O3
2– à 2 I– + S4O6
2– 
Na análise, foram necessários 0,042 cm3 da solução de 
Na2S2O3 para reagir completamente com todo o I2. 
 
(a) Calcule o número de moles de I2 que reagiram com a 
solução de S2O3
2– 
(b) Quantos moles de I2 foram produzidos na primeira 
reação? 
(c) Quantos moles de O3 estavam contidos nos 20000 
dm3 de ar? 
(d) Que volume o O3 ocuparia nas CNTP? 
(e) Qual é a concentração de O3, em partes por milhão 
em volume, na amostra de ar? 
 
GABARITO 
 
01. (a) 86 mmHg; 
(b) O lado direito do bulbo está mais alto; 
(c) 848 Torr. 
 
02. (a) 70 g.mol–1 ; 
(b) CHF3; (c) 2,9 g.L–1 
03. C2H2Cl2 
 
04. (a) 3,0x105 L; (b) 1,0x104 L 
 
05. 2,4x104 L 
06. (a) 0,787 atm; 
(b) 1,03 g 
 
07. (a) 739,2 Torr; 
(b) H2O(l) à H2(g) + ½ O2(g); 
(c) 0,142g 
 
08. 110g/mol; C8H12 
 
09. (a) 4103,2 J/mol; 
(b) 4090,7 J/mol; 
132 
 
 
 
APOSTILA 01 DE FÍSICO-QUÍMICA – PROF. PEDRO MADEIRA (2022) 
 
(c) 12,5 J/mol 
 
10. (a) A velocidade mais provável é a que corresponde 
ao máximo da curva de distribuição. 
(b) A percentagem decresce. 
 
11. Em temperaturas baixas, as ligações hidrogênio 
fazem com que as moléculas de HF se atraiam mais 
fortemente e a pressão cai. Quando a temperatura 
aumenta, as ligações hidrogênio se quebram e a 
pressão sobe mais depressa do que para um gás ideal. 
 
12. (a) 1,63; 1,62 atm. 
(b) 48,9; 38,9 atm. 
(c) 489; 1,88x103 atm. Em pressões baixas, a lei do gás 
ideal dá essencialmente o mesmo valor da equação de 
van der Waals, mas em pressões elevadas existem 
diferenças importantes. 
 
13. (a) e (b) 
n Pideal Pvan der Waals % de 
desvio 
0,100 2,46 2,44 0,8 
0,200 4,92 4,82 2,1 
0,300 7,38 7,15 3,2 
0,400 9,85 9,44 4,3 
0,500 12,31 11,67 5,5 
 
(c) Pvan der Waals < Pideal, o que sugere que as atrações 
intermoleculares são mais importantes. 
(d) em pressões superiores a cerca de 10 atm. 
 
14. 
(a) 
 
(b) 0,020 ppm 
 
15. 0,481 M 
 
16. (a) N2O4(g) à 2 NO2(g) 
(b) 2,33 atm; 
(c) 4,65 atm; 
(d) X(NO2) = 0,426; X(N2O4) = 0,574 
 
17. 
 
#(2BHh)
#(Hh) = 2BB ∙ 0#WW45X" !'&⁄ 
 
Não, a distribuição varia conforma a temperatura. 
 
18. 
(a) X(COCl2) = 0,090; X(CO) = 0,527; X(Cl2) = 0,383; 
(b) 7,20 g/L 
 
19. distância > 2,35 m 
 
20. 254 g/mol; x = 4 
 
21. (a) V = 3,936x107 pm3; r = 211 pm. 
(b) r = 128 pm; V = 8,78x106 pm3. 
(c) A diferença entre esses valores mostra que não 
existe definição simples para os limites de um átomo. O 
valor de van der Waals obtido da correção para o volume 
molar é consideravelmente maior do que o raio atômico, 
devido, talvez ao maior alcance e às interações fracas 
entre átomos. Lembre-se de que o valor da constante b 
de van der Waals é um parâmetro usado para obter um 
bom ajuste a uma curva e sua interpretação inclui outros 
fatores além do volume molar. 
 
22. (a) ClNO2; 
(b) ClNO2; 
(c) 
 
(d) trigonal planar. 1 
 
23. (a) V = 0,0979 L. 
(b) Videal = 0,130 L < VvdW; portanto, as forças atrativas 
dominam. 
 
24. 117,6 kPa 
 
25. 61,4 cm 
 
26. 81,5 cm3 
 
27. (a) 65,8% C; 15,2% H; 19,2% N 
(b) C4H11N 
 
28. (a) 0,144g 
(b) 0,368g 
 
29. 100,6 kPa (gás ideal, P = 101,3 kPa) 
 
30. 256 cm3 
 
 
 
 
O N O O N O
..
..
..
..
. ..
..
..
..
.....
O N O
Cl
O N O
Cl..
..
..
..
..
..
..
..
....
.. ....
.. ....
133 
 
 
 
APOSTILA 01 DE FÍSICO-QUÍMICA – PROF. PEDRO MADEIRA (2022) 
 
LISTA EXTRA – TERMODINÂMICA QUÍMICA 
 
01. 
Um gás em um cilindro foi colocado em um aquecedor e 
ganhou 5500 kJ de calor. Se o volume do cilindro 
aumentou de 345 mL para 1846 mL contra uma pressão 
atmosférica de 750 Torr durante o processo, qual é a 
variação de energia interna do gás no cilindro? 
 
02. 
Calcule o trabalho em cada um dos seguintes 
processos, começando com uma amostra de gás em um 
sistema com pistão com T = 305 K, P = 1,79 atm e V = 
4,29 L. 
(a) expansão irreversível contra a pressão externa 
constante de 1,00 atm, até o volume final 6,52 L; 
(b) expansão reversível isotérmica até o volume final 
6,52 L. 
 
03. 
Que gás tem maior capacidade calorífica molar, NO ou 
NO2? Por quê? 
 
04. 
Calcule o calor liberado por 5,025g de Kr(g), em 0,400 
atm, que esfria de 97,6ºC até 25,0oC, 
(a) em pressão constante e 
(b) em volume constante. 
Imagine que o criptônio comporta-se como um gás ideal. 
 
05. 
A contribuição dos modos vibracionais de alta 
temperatura para a capacidade calorífica molar de um 
sólido em volume constante é R para cada modo de 
vibração. Portanto, para um sólido atômico, a 
capacidade calorífica molar em volume constante é 
aproximadamente 3R. 
(a) A capacidade calorífica específica de um 
determinado sólido atômico é 0,392 J.K–1.g–1. O 
cloreto deste elemento (XCl2) é 52,7% cloro em 
massa. Identifique o elemento. 
(b) Esse elemento cristaliza em uma célula unitária 
cúbica de face centrada e seu raio atômico é 128 pm. 
Qual é a densidade desse sólido atômico? 
 
06. 
A entalpia de formação do trinitro tolueno (TNT) é -67 
kJ/mol e, a densidade, 1,65 g/cm3. Em princípio, ele 
poderia ser usado como combustível de foguetes, com 
os gases formados na decomposição saindo do foguete 
para dar o impulso necessário. Na prática, é claro, ele 
seria extremamente perigoso como combustível, porque 
é sensível ao choque. Explore seu potencial como 
combustível de foguete, calculando a densidade de 
entalpia (a entalpia liberada por litro) na reação 
4C7H5N3O6(s)+21O2(g)à 28CO2(g)+10H2O(g)+6N2(g) 
 
07. 
Dois estágios sucessivos da preparação industrial do 
ácido sulfúrico são a combustão do enxofre e a oxidação 
do dióxido de enxofre. A partir das entalpias padrão de 
reação 
S(s) + O2(g) à SO2(g) DHo = – 296,83 kJ 
2 S(s) + 3 O2(g) à 2 SO3(g) DHo = – 791,44 kJ 
Calcule a entalpia de reação da oxidação do dióxido de 
enxofre a Trióxido de enxofre na reação 
2 SO2(g) + O2(g) à 2 SO3(g) 
 
08. 
Determine a entalpia da reação de hidrogenação do 
etino a etano C2H2(g) + 2 H2(g) à C2H6(g), a partir dos 
seguintes dados: 
DHc
o(C2H2(g))=–1300 kJ/mol; 
DHc
o(C2H6(g)) = –1560 kJ/mol; 
DHc
o(H2(g)) = –286 kJ/mol. 
 
09. 
Complete a seguinte tabela (valores em kJ/mol) 
 
MX DHfo 
M(g) 
EI 
M 
DHfo 
X(g) 
AE 
X 
DHLo 
MX 
DHfo 
MX(s) 
NaCl 108 494 122 +349 787 ? 
KBr 89 418 97 +325 ? -394 
RbF ? 402 79 +328 774 -558 
 
10. 
Derive a lei de Kirchhoff para uma reação da forma A 
+ 2B à 3C + D, levando em conta a variação de entalpia 
molar de cada substância quando a temperatura 
aumenta de T1 até T2. 
 
11. 
A luz solar forte bombardeia a Terra com cerca de 1 
kJ/m2 em 1 s. Calcule a massa máxima de etanol puro 
que poderia ser vaporizada, em 10 min, em um bécher 
deixado sob luz solar forte, imaginando que a área 
superficial do etanol é 50 cm2 e que todo o calor é usado 
para a vaporização e não para aumentar a temperatura. 
DHvap = 38,6 kJ/mol 
 
12. 
Anilina, C6H5NH2(l) é um derivado do benzeno em que 
um átomo de hidrogênio foi substituído por um grupo 
NH2. 
(a) Escreva a equação balanceada da combustão da 
anilina. 
(b) Qual é a massa de cada produto quando 0,1754g de 
anilina são queimados em excesso de oxigênio? 
(c) Se o calorímetro de bomba no qual essa reação foi 
feia tinha um volume de 355 mL, qual foi a pressão 
mínima de oxigênio, em 23ºC, usado para garantir a 
combustão completa? Considere desprezível o 
volume da anilina. 
 
13. 
Será que a produção de gás de água (um combustível 
industrial barato de baixo grau calorífico) é um processo 
exotérmico ou endotérmico? A reação é: 
C(s) + H2O(g) à CO(g) + H2(g) 
14. 
Imagine que 50,0 mL de 0,500 M NaOH(aq) e 50,0 mL 
de 0,500 M HNO3(aq), inicialmente em 18,6ºC, são 
misturados e agitados em um calorímetro cuja 
134 
 
 
 
APOSTILA 01 DE FÍSICO-QUÍMICA – PROF. PEDRO MADEIRA (2022) 
 
capacidade calorífica é 525,0 J/oC quando vazio. A 
temperatura da mistura subiu para 20,0oC. 
(a) Qual é a variação de entalpia da reação de 
neutralização? 
(b) Qual é a variação de entalpia da neutralização em 
kJ/mol HNO3? 
Considere a capacidade específica de cada solução 
como sendo 4,184 J/g.oC 
 
15. 
Qual seria mais exotérmica, a oxidação do 1,3,5-
trinitrobenzeno, C6H3(NO2)3, ou a oxidação do 1,3,5-
triaminobenzeno, C6H3(NH2)3? Justifique sua resposta. 
 
16. 
Durante exercícios físicos, as gorduras reagem com 
água para produzir ácidos graxos. Estes são, então, 
convertidos em água e dióxido de carbono, uma reação 
que libera energia. O organismo usa essa energia para 
suas atividades. Um ácido graxo típico é o ácido láurico, 
CH3(CH2)10COOH, que tem o mesmo número de átomos 
de carbono da sacarose C12H22O11. 
DHf
o(CH3(CH2)10COOH) = – 774,6 kJ/mol e 
DHf
o(C12H22O11) = – 2222 kJ/mol 
(a) Que massa de sacarose é necessária para produzir 
a mesma energia que 15,0 g de ácido láurico? 
(b) Por que é mais eficiente armazenar energia na forma 
de gordura do que na forma de carboidratos? 
 
17. 
Os ácidos maléico e fumárico são dois isômeros cis e 
trans, respectivamente, cuja fórmula química é C4H4O4. 
Eles são insumos importantes para a indústria química. 
(a) Calcule a entalpia da isomerização cis-trans. 
(b) Que isômero tem a entalpia padrão de formação 
menor? 
(c) Na reação de combustão desses compostos, o mais 
negativo é DUc
 ou DHc? 
DHc(ácido maléico) = – 1355,2 kJ/mol 
DHc(ácido fumárico) = – 1334,7 kJ/mol 
 
18. 
Um técnico conduz a reação 2SO2(g) + O2(g) à 
2SO3(g), em 25ºC e 1,00 atm, em um cilindro dotado de 
um pistão, em pressão constante. No início, estão no 
cilindro 0,030 mol SO2 e 0,030 mol O2. O técnico 
adiciona, então, um catalisador para iniciar a reação. 
(a) Calcule o volume do cilindro que contém os gases 
antes do começo da reação. 
(b) Qual é o reagente limitante? 
(c) Imagine que a reação se completa e que a 
temperatura e a pressão permanecem constantes. 
Qual é o volume final do cilindro? 
(d) Qual é o trabalho executado? Ele é feito contra ou a 
favor do sistema? 
(e) Qual é a entalpia trocada? Ela deixa ou entra no 
sistema? 
(f) Leve em conta as respostas das partes (d) e (e) para 
calcular a variação de energia da reação. 
 
GABARITO 
 
01. +5500 kJ 02.a) -226 J b) -326 J 
 
03. NO2. Moléculas mais complexas têm mais vibrações, 
nas quais é possível armazenar energia. 
 
04.a) – 90,6 J; b) – 54,4 J 
 
05. a) Cu; b) 8,90 g/cm3 
 
06. +23,9x103 kJ/L 
 
07. –197,78 kJ 
 
08. –312 kJ/mol 
 
09. (a) –412 kJ/mol; (b) 673 kJ/mol; (c) +63 kJ/mol 
 
10. Se aumentarmos a temperatura de uma substância 
de T1 até T2, a entalpia da substância cresce de H1 até 
H2, em que H2 – H1 = Cp(T2 – T1). Portanto, a entalpia de 
qualquer reagente ou produto na temperatura final está 
relacionada à entalpia na temperatura inicial por H2 = H1 
+ Cp(T2 – T1). Essa expressão se aplica a todas as 
substâncias que tomam parte na reação; logo, deduz-se 
a lei de Kirchhoff a partir daí. 
 
11. 3g 
 
12. 
(a)C6H5NH2(l)+31/4O2(g)à6CO2(g)+7/2H2O(l)+1/2N2(g) 
(b) m(CO2) = 0,4873g, m(H2O) = 0,1188g, 
m(N2) = 0,02639g; (c) 0,999 atm 
 
13. endotérmico 
 
14. (a) – 1,33 kJ; (b) – 53,2 kJ/mol 
 
15. As equações de combustão mostram que as 
diferenças são (1) o consumo de 18/4 mols de O2(g) e 
(2) a produção de mais 6 mols de H2O(l) na combustão 
da anilina. Como DHf
o de O2(g) é zero, a diferença total 
é a produção de mais 3 mols de H2O(l), – 857,5 kJ. 
 
16. (a) 33,5g 
(b) A massa molar da sacarose é 71% maior do que a 
do ácido láurico, mas a sacarose produz 23% menos 
energia na conversão em dióxido de carbono do que o 
ácido láurico. 
 
17. (a) –20,5 kJ/mol; (b) ácido fumárico; (c) DU. 
 
18. (a) 1,5 L; (b) SO2; (c) 1,1 L; 
(d) + 40 J, contra o sistema; 
(e) – 2970 J, calor deixa o sistema; (f) – 2930 J 
 
 
 
 
 
 
 
 1 18 
1 
1 H 
1,01 
–259,3 
–252,9 2 
 
13 14 15 16 17 
2 He 
4,00 
 
–268,9 
2 
3 Li 
6,94 
180,5 
1342 
4 Be 
9,01 
1287 
2471 
 Número atômico, Símbolo 
Massa Atômica Média (u) 
Ponto de Fusão (oC) 
Ponto de Ebulição (oC) 
 5 B 
10,81 
2075 
4000 
6 C 
12,01 
4440 
 
7 N 
14,01 
–210,0 
–195,79 
8 O 
16,00 
–218,8 
–182,9 
9 F 
19,00 
–219,7 
–188,1 
10 Ne 
20,18 
–248,6 
–246,1 
3 
11 Na 
22,99 
97,80 
883 
12 Mg 
24,30 
650 
1090 3 4 5 6 7 8 9 10 11 12 
13 Al 
26,98 
660,32 
2519 
14 Si 
28,09 
1414 
3265 
15 P 
30,97 
44,15 
280,5 
16 S 
32,01 
115,2 
444,6 
17 Cl 
35,45 
–101,5 
–34,04 
18 Ar 
39,95 
–189,3 
–185,8 
4 
19 K 
39,10 
63,38 
759 
20 Ca 
40,08 
842 
1484 
21 Sc 
44,96 
1541 
2836 
22 Ti 
47,87 
1668 
3287 
23 V 
50,94 
1910 
3407 
24 Cr 
52,00 
1907 
2671 
25 Mn 
54,94 
1246 
2061 
26 Fe 
55,85 
1538 
2861 
27 Co 
58,93 
1495 
2927 
28 Ni 
58,69 
1455 
2913 
29 Cu 
63,46 
1085 
2562 
30 Zn 
65,39 
419,5 
907 
31 Ga 
69,72 
29,76 
2204 
32 Ge 
72,61 
938,2 
2833 
33 As 
74,92 
 
614 
34 Se 
78,96 
221 
685 
35 Br 
79,90 
–7,2 
58,8 
36 Kr 
83,80 
–157,4 
–153,2 
5 
37 Rb 
85,47 
39,31 
688 
38 Sr 
87,62 
777 
1382 
39 Y 
88,91 
1522 
3345 
40 Zr 
91,22 
1855 
4409 
41 Nb 
92,91 
2477 
4744 
42 Mo 
95,94 
2623 
4639 
43 Tc 
(98) 
2157 
4265 
44 Ru 
101,07 
2334 
4150 
45 Rh 
102,91 
1964 
3695 
46 Pd 
106,42 
1555 
2963 
47 Ag 
107,87 
961,8 
2162 
48 Cd 
112,4 
321,1 
767 
49 In 
114,82 
156,6 
2072 
50 Sn 
118,71 
231,9 
2602 
51 Sb 
121,76 
630,6 
1587 
52 Te 
127,60 
449,5 
988 
53 I 
126,90 
113,7 
184,4 
54 Xe 
131,3 
–111,7–108,0 
6 
55 Cs 
132,91 
28,44 
671 
56 Ba 
137,33 
727 
1897 
57 La 
138,91 
918 
3464 
72 Hf 
178,49 
2233 
4603 
73 Ta 
180,95 
3017 
5458 
74 W 
183,84 
3422 
5555 
75 Re 
186,21 
3186 
5596 
76 Os 
190,23 
3033 
5012 
77 Ir 
192,22 
2446 
4428 
78 Pt 
195,08 
1768 
3825 
79 Au 
196,97 
1064 
2856 
80 Hg 
200,59 
 –38,83 
356,7 
81 Tl 
204,38 
302 
1473 
82 Pb 
207,2 
327,5 
1749 
83 Bi 
208,98 
271,4 
1564 
84 Po 
(209) 
254 
962 
85 At 
(210) 
302 
 
86 Rn 
(222) 
–71 
–61,7 
7 
87 Fr 
(223) 
27 
 
88 Ra 
(226) 
7000 
89 Ac 
(227) 
1051 
3198 
104 Rf 
(267) 
 
 
105 Db 
(268) 
 
 
106 Sg 
(269) 
 
 
107 Bh 
(270) 
 
 
108 Hs 
(269) 
 
 
109 Mt 
(278) 
 
 
110 Ds 
(281) 
 
 
111 Rg 
(281) 
 
 
112 Cn 
(285) 
 
 
113 Nh 
(286) 
 
114 Fl 
(289) 
 
115 Mc 
(288) 
 
116 Lv 
(293) 
 
117 Ts 
(294) 
 
118 Og 
(294) 
 PROF. PEDRO MADEIRA 
 
Série dos 
LANTANÍDEOS 
58 Ce 
140,12 
798 
3443 
59 Pr 
140,91 
931 
3443 
60 Nd 
144,24 
1021 
3074 
61 Pm 
(145) 
1042 
3000 
62 Sm 
150,36 
1074 
1794 
63 Eu 
151,96 
822 
1529 
64 Gd 
157,25 
1313 
3273 
65 Tb 
158,93 
1356 
3230 
66 Dy 
162,50 
1412 
2567 
67 Ho 
164,93 
1474 
2700 
68 Er 
167,26 
1529 
2868 
69 Tm 
168,93 
1545 
1950 
70 Yb 
173,04 
819 
1196 
71 Lu 
174,97 
1663 
3402 
 
 
Série dos 
ACTNÍDEOS 
90 Th 
232,04 
1750 
4788 
91 Pa 
231,04 
1572 
 
92 U 
238,03 
1135 
4131 
93 Np 
(237) 
644 
94 Pu 
(244) 
640 
3228 
95 Am 
(243) 
1176 
2011 
96 Cm 
(247) 
1345 
3100 
97 Bk 
(247) 
1050 
98 Cf 
(251) 
900 
99 Es 
(252) 
860 
100 Fm 
(257) 
1527 
101 Md 
(258) 
827 
102No 
(259) 
827 
103 Lr 
(262) 
1627

Mais conteúdos dessa disciplina